Вы находитесь на странице: 1из 166

Dental [fu£6e

ORAL PATHOLOGY 8 MEDICINE


I. REFERENCE BOOKS TAKEN:
1. A TEXTBOOK OF ORAL PATHOLOGY by SHAFER-6th EDITION
2. BURKET'$ ORALMEDICINE by GREENBERG & GLICK-11 th & 12th EDITIONS
3. BURKET'$ ORALMEDICINE by GREENBERG & GLICK-10th EDITION
4. A TEXTBOOK OF ORAL PATHOLOGY by SHAFER - 7th EDITION

1. DEVELOPMENTAL DISTURBANCES
1. A union of the roots of adjacent teeth through the 6. Which of the following features are of naso alveolar cyst
cementum is referred to as a) An ovoid shaped radiolucency above the lateral incisor
a) Concrescence b) Fusion and canine teeth
c) Gemination d) None of the above b) Erosion of base above lateral incisor and canine teeth
(AIPG -05, PGI -02) c) An inverted funnel shaped radiolucent lesion above the
2. The term dilaceration refers to roots of lateral incisor and canine teeth
a) A deformity of a tooth consisting of a sharp bend in the root d) A pear shaped radiolucent lesion between roots of
b) Abrasions on two surfaces of a single tooth lateralincisor and canine teeth.
c) A root or tooth that is split into two (MAN -97,98)
d) A tooth that is fractured at two or more places 7. Facial edema, cheilitis granulomatosa and a fissured
(MAN -94) tongue characterize which of the following syndromes?
3. Peg-shaped incisors which taper towards the incisal edge a) Frey b) Melkerson-Rosenthal
are typically seen in all of the following conditions EXCEPT c) Treacher Collins d) None of the above
a) Congenital syphilis b) Rickets (MAN -97, AP- 2013)
c) Anhidrotic ectodermal dysplasia 8. Which of the following is a soft tissue cyst which do not
d) Supernumerary teeth produce any radiographic changes
(MAN -94, AIIMS-93) a) Nasolabial cyst b) Nasopalatine cyst
4. In erythema migrans,. which papilla of tongue are absent: c) Mid alveolar cyst d) Palatine cyst
a) Fungiform. b) Foliate. (MAN -97, AIIMS -94)
c) Filliform. d) Circumvallate 9. The syndrome which consists of cleft palate, micrognathia
(AIPG -04) and glossoptosis is known as
5. Globulomaxillary cyst is a) Marfan's syndrome b) Crouzon's syndrome
a) Soft tissue cyst present often between maxillary c) Paget's disease d) PierreRobin syndrome
Lateral incisor and cuspid teeth. (MAN -2K)
b) Often present between incisor and cuspid teeth but is a 10. Developing Ameloblasts are effected by the ingestion of
bone cyst high fluoride content water resulting in
c) A cyst present between the midline of the palate a) Turners tooth b) Hutchinson's teeth
d) A cy�L presenl in Lhe incisive canal c) Mollleu enamel tl) Muon's molar�
(MAN-97) (MAN -2K)

1) A 2) A 3) B 4) C 5) B 6) B 7) B 8) A 9) D 10) C
r ORAL PATHOLOGY & MEDICINE
11. Teeth that erupt with in 30 days of birth are called 21. The most common supernumerary tooth form is
a) Natal teeth b) Neonatal teeth a) Tuberculated b) Conical
c) Primary teeth d) Prenatal teeth c) Screw shaped d) Incisor shaped
(MAN-98) (AP -2K)
12. A permanent tooth with a local hypoplastic deformity in 22. A patient notices a well demarcated area of depapillation
a crown is called on his tongue which has been there for as long as he can
a) Turner's tooth b) Taurodontism remember. The most probable diagnosis:
c) Enameloma d) Ghost teeth a) Median rhomboid glossitis
(MAN -2001) b) Geographic tongue
13. True ankyloglossia occurs as a result of c) Black hairy tongue d) Moellers glossitis
a) Union between tongue and floor of mouth/ventral (AIIMS-98, 01)
surface of tongue 23. Fordyce's spots are:
b) Absence of lingual frenum a) Fat tissue embedded in buccal mucosa
c) Lingual frenum attached to the tip of tongue b) Red spots
d) Short lingual frenum c) Present on the cheek mucosa lateral to angle of the mouth
(MAN-2001) d) All of the above
14. True generalized mi crodontia is characterized by (AIPG -92, AIIMS -90)
a) Small teeth with large jaws 24. Most common missing tooth in the permanent dentition is:
b) Small teeth with small jaws a) Maxillary canine b) Maxillary first molar
c) Smaller teeth than the normal c) Mandibular second premolar
d) Large teeth with small jaws d) Mandibular first molar
(MAN-2001) (PGI -2K)
15. Gemination of teeth occur due to 25. Hypoplastic defects in permanent central and lateral
a) Division of a single tooth bud after calcification incisors are likely to result due to severe illness or other
b) Division of a single tooth bud before calcification factors during:
c) Fusion of two teeth before calcification a) First nine months of life
d) Fusion of two teeth after calcification b) First two years of life
(MAN -2001) c) First month of life d) Two or three years of life
16. A deep pit lined by enamel seen in the lingual surface of (AIPG -95)
Maxillary lateral incisor is most likely to be 26. Ectopic sebaceous glands in the mouth are called:
a) Dens in dente b) Enamel Hypoplasia a) Linea alba buccalis b) Heck's disease
c) Talon's cusp d) Enamel Pearl c) Lingual varices d) Fordyce spots.
(AP-10) (KAR -2003)
17. Ghost teeth is seen in which of the following 27. Gardner syndrome does not include:
a) Dens is dent b) Regional odontodysplasia a) Osteomas b) Epidermoid cysts
c) Dentin dysplasia d) None of the above c) Osteosarcoma d) Impacted permanent teeth
(MAN -2002) (AIPG -93, AIIMS -91,99)
18. Which of the following lesions does not give blood on 28. Lingual tonsils arise:
aspiration a) As developmental anomalies
a) Heamangioma b) Static bone cyst b) From carcinomatous transformation
c) Central giant cell granuloma c) As a result of hyperplasia
d) Aneurysmal bone cyst d) Due to repeated trauma in the area
(MAN -2002) (AIPG -94, 97)
19. Absence of pulp chambers is seen in 29. Dentinogenesis imperfecta is:
a) Craniofacial dysostosis b) Dentinogenesis imperfecta a) Autosomal dominant b) Autosomal recessive
c) Amelogenesis imperfecta c) Sex linked recessive d) Not a inheritable trait
d) Any of the above (AIPG -95)
(MAN -97) 30. Clinical evidence of dentinogenesis imperfecta is
20. Dentinogenesis imperfecta differs from amelogenesis a) Defective enamel and dentine
imperfecta in that, the former is: b) Defective dentine and obliterated pulp chamber
a) A hereditary disturbance c) Increased rate of caries d) Oligodontia
b) The result of excessive fluoride ingestion (AIPG-94)
c) The result of faulty enamel matrix formation 31. Bohn's nodules are
d) Characterized by calcification of pulp chambers and the a) Cystic swellings in neonates
root canals of the teeth b) Cysts associated with soft palate
(AIPG- 2002) c) Cysts of gingiva in growing children
d) Warts on the tongue
(KAR -98, COMEDK-05)

11) B 12) A 13) A 14) C 15) B 16) A 17) B 18) B 19) B 20) D 21) B 22) A 23) C
24) C 25) A 26) D 27) C 28) A 29) A 30) B 31) A
Dental 9'ul-6e
32. False about anodontia: c) Mand. central incisor d) Mand. first premolar
a) May involve both the deciduous and the permanent (AIIMS -95)
dentition 43. Taurodontism is characterized by:
b) In false anodontia tooth doesn't undergo full development a) Hypercementosis
c) May involve a single tooth b) Elongated wide pulp canals and short roots
d) In total anodontia all teeth are missing c) Obliterated pulp chambers with secondary dentine
(AIPG -2002) deposition
33. Turners tooth is seen in d) Rootless teeth with thin shell of enamel
a) enamel hypoplasia due to hypocalcemia (AIPG-06)
b) enamel hypoplasia due to birth injuries 44. Mottled enamel is due to:
c) enamel hypoplasia due to congenital syphilis a) Vitamin A deficiency b) Excess of fluoride
d) enamel hypoplasia due to local infection or local trauma c) Vitamin D deficiency d) Teratogens
(KAR -98) (AIPG-99)
34. All are true about supernumerary tooth except: 45. Which of the following is a Fissural cyst is
a) May have resemblance to normal teeth a) Dentigerous cyst b) Primordial cyst
b) Disto molars doesn't resemble any other tooth c) Nasopalatine cyst d) Radicular cyst
c) Mesiodens is the most common supernumerary tooth (AP-2K)
d) More common in mandible 46. Bifid tongue, a congenital anomaly occurs due to non­
(AIPG -2002) fusion of:
35. Dental anomaly of teeth associated with defective bone a) Tuberculum impar and lateral lingual swellings
formation is seen in b) Hypobranchial eminence and tuberculum impar
a) amelogenesis imperfecta c) The two lateral lingual swellings
b) dentinogenesis im1Perfecta d) Some of the above
c) Odontodysplasia d) Osteitis deformans (AIIMS-89)
(KAR -90) 47. Anodontia affects the growth of:
36. Fusion of teeth is more common in- a) Maxilla b) Mandible
a) Primary dentition. b) Permanent dentition c) Alveolar bone d) Cranium
c) Mixed dentition. d) None of the above. (AIIMS-2K)
{KAR -99) 48. Delayed eruption of at least part of the dentition is a
3 7. Multiple osteomas, multiple polyposis, supernumerary recognized feature of all of the following except:
teeth are found in __syndrome a) Rickets b) Congenital hyperthyroidism
a) Reiter's b) Peutz - jeghers c) Cleidocranial dysplasia d) Cherubism
c) Gardner's d) Behcet's (AIIMS-93, KAR -2K)
(AP -01, 03,99) 49. Geographic tongue, all are true except:
38. Prolonged administration of broad spectrum antibiotics a) Lesions may be Bilaterally symmetrical on tongue
results in the formation of: b) No treatment is required
a) Black hairy tongue b) Median rhomboid glossitis c) Is a Precancerous condition
c) Geographic tongue d) Fissured tongue d) May be related to emotional stress
(KAR -2K) {AIPG -2002)
39. Hairy tongue is characterized by 50. Which of the following dental sequel is likely in child
a) Hypertrophy of fungiform papillae with a history of generalized growth failure {failure to
b) Hypertrophy of foliate papillae thrive) in the first 6 months of life:
c) Hypertrophy of filiform papillae. a) Retrusive maxilla b) Enamel hypoplasia
d) Hypertrophy of circumvallate papillae. c) Retrusive Mandible d) Dentinogenesis imperfecta
(KAR-2002) (AIPG -2001)
40. A 4-year old child has less number of teeth and lateral 51. A 4-year-old child has a normal complement of primary
incisors with bifurcated roots with two root canals is called: teeth, but they are gray and exhibit extensive occlusal
a) Dilaceration b) Concrescence and incisal wear. Radiographic examination indicates
c) Fusion d) Gemination extensive deposits of secondary dentin in these teeth.
(PGI-2K) Most likely this condition is:
41. Naso Labial cyst is thought to arise from a) Neonatal hypoplasia b) Amelogenesis imperfecta
a) Remnants of cell rests of serrae c) Cleidocranial dysplasia d) Dentinogenesis imperfecta
b) Remnants of cell rests of mallasez (AIIMS- 92, 99,03)
c) Remnants of cell rests of embryonic lacrimal duct 52. A developmental abnormality characterized by the
d) Maxillary sinus lining epithelium presence of fewer than the usual number of teeth is
(KAR -03) a) Anodontia b) Oligodontia
42. Microdontia is most commonly seen affecting: c) Microdontia d) Dens in dente
a) Max. lateral incisor b) Mand. second premolar (AIPG -98, 03)

32) B 33) D 34) 0 35) B 36) A 37) C 38) A 39) C 40) C 41) C 42) A 43) B 44) B
45) C 46) C 47) C 48) B 49) C 50) B 51) D 52) B
53.
r
Median rhomboid glossitis is associated with:
ORAL PATHOLOGY & MEDICINE
d) Permanent mandibular anteriors
a) Oral cancer b) Fungal infection (PGI -95)
c) Leukoplakia d) Burning sensation of tongue 65. Shell teeth are more common in this variant of
(AIIMS -99) dentinogenesis imperfecta:
54. Most commonly submerged tooth is a) Type I b) Type III
a) Mand. primary l't molar b) Mand. primary 2nd molar c) Type II d) Type III and I
c) Maxi. primary pt molar d) Maxi. primary 2nd molar (KAR -01)
(AP -99) 66. Complete obliteration of pulp is seen in all except:
55. Epstein pearls are cysts that arise from: a) Type I dentinogenesis imperfecta
a) Squamous tissue of the mucosa b) Type II dentinogenesis imperfecta
b) Connective tissue of the mucosa c) Type III dentinogenesis imperfecta
c) Rests of malassez d) Dental lamina d) Dentin dysplasia
(AIPG -2003) (AIIMS -92)
56. A dens in dente is usually caused by: 67. Which of the following is unlikely to cause enamel hypoplasia?
a) An abnormal proliferation of pulp tissue a) Rickets b) Fluoride
b) Denticle formation within the pulp tissue c) Congenital syphilis d) Cleidocranial dysostosis
c) A deep invagination of the enamel organ during formation (AIIMS-01, KAR-01)
d) A supernumerary tooth bud enclaved within a normal tooth. 68. Mulberry molars are characteristic features of:
(AIPG -2003) a) Severe fluorosis.
57. In children the most frequently missing permanent b) Trauma at the time of birth
teeth are: c) Congenital syphilis.
a) First premolars b) Second premolars d) Due to chronic suppurative abscess in over lying gingival
c) Max. lateral incis.ors d) Mandibular lateral incisors tissue.
(KAR -2001) (AIPG -04)
58. Hutchinson's incisors are present in: 69. Talon's cusp is characteristic of which syndrome
a) Congenital syphilis b) Tertiary syphilis a) Edward's syndrome b) Klinefelter's syndrome
c) Secondary syphiliis d) Acquired syphilis c) Rubinstein -Taybi syndrome
(AIPG -91) d) Down's syndrome
59. A 15-year-old boy shows an inverted pear shaped (COMEDK -04)
radiolucency between the upper central incisors. The teeth 70. Thistle-tube appearance of pulp chamber is a feature of:
are normal in all aspects. The most likely diagnosis is: a) Coronal dentin dysplasia
a) Globulomaxillary cyst b) Nasopalatine cyst b) Regional odontodysplasia
c) Aneurysmal cyst d) Dentigerous cyst c) Dentinogenesis imperfecta
(COMED-14) d) Amelogenesis imperfecta
60. Odontodysplasia is most common in: (AIPG -04)
a) Mandibular premolar b) Mandibular canine 71. A bony hard asymptomatic swelling found on the
c) Mandibular third molar d) Maxillary central incisor midline of the hard palate, that appears radiopaque on a
(AIIMS -96) radiograph is most likely a
61. Epstein pearls are: a) Odontoma b) Myxoma
a) Gingival cyst of newborn c) Boney cyst d) Torus palatinus
b) Gingival cyst of adult (KCET-09)
c) Enamel pearls d) Epithelial rests 72. Which of the following conditions is characterized by
(AIIMS -95) abnormally large pulp chambers?
62. Which of the following is not hereditary: a) Amelogenesis imperfecta
a) Amelogenesis imperfecta b) Regional odontodysplasia
b) Cleidocranial dysostosis c) Dentinogenesis imperfecta
c) Regional odontodysplasia d) Dentinal dysplasia type I
d) Dentinogenesis imperfecta (APPSC -99)
(PGI -99) 73. In Treacher collin's syndrome there is:
63. The 2nd common most supernumerary teeth is a) Upward sloping of the palpebral fissure
a) Mesiodens b) Distal to 3rd molar in maxilla b) Poorly developed or absence of malar bones
c) Distal to mandibular 3 rd molar c) Progenia and mandibular prognathism
d) Para molars d) No loss of hearing
(PGI -95) (AIPG-05)
64. Turner's hypoplasia most commonly affects: 74. Pierre Robin syndrome is associated with:
a) Deciduous maxillary anteriors a) Micrognathia b) Cleft of the lip and plate
b) Deciduous mancli bular anteriors c) Tetrology of fallot cl) Syndactally
c) Permanent maxmary anteriors (AIPG-05)

53) B 54) B 55) D 56) C 57) C 58) A 59) A 60) D 61) A 62) C 63) B 64) C 65) C
66) B 67) D 68) C 69) C 70) A 71) D 72) B 73) B 74) A
Dental 9'ul-6e
75. Which of the following teeth is most likely to be 86. In hypodontia, the most commonly affected tooth is
congenitally missing? a) Permanent third molar b) Permanent second premolar
a) Maxi. central incisor b) Mandibular canine c) Permanent lateral incisor d) Permanent Canine
c) Mand. second prennolar d) Maxillary first premolar (COMEDK-14)
(AIPG-05) 87. The torus mandibularis most commonly seen in the region of:
76. Which of the following dentitions shows the highest a) Incisor region b) Canine region
frequency of ocurrence of supernumerary teeth ? c) Molar region d) Premolar region
a) Maxillary deciduous dentition (AIPG-06)
b) Maxillary permanent dentition 88. Submerged teeth are
c) Mandibular deciduous dentition a) Ankylosed teeth b) Unerupted teeth
d) Mandibular permanent dentition c) Impacted teeth d) Intruded teeth
(CAI-05) (COMEDK-07)
77. The inheritance pattern of dentinogenesis imperfecta is 89. The cyst located at the junction of medial nasal process,
a) Homozygous b) Autosomal dominant lateral nasal process and maxillary process is:
c) Recessive d) X-linked recessive a) Globulomaxillary cyst b) Median palatine cyst
(KAR-06} c) Nasopalatine cyst d) Nasoalveolar cyst
78. Heck's disease is another name for (AIPG-06, 10)
a) Focal epithelial hyperplasia 90. The most common congenital defect of the face and jaws is
b) Fibromatosis gingiva a) Macrostomia b) Fetal alcohol syndrome
c) Oral melanotic macule c) Cleft lip and palate d) Ectodermal dysplasia
d) Hereditary intestinal polyposis syndrome (COMEDK-07)
(COMEDK-06) 91. The cyst which is found within the bone at the junction
79. Large pulp chambers are characteristic of all the of the globular process, the lateral nasal process &
following conditions except maxillary process is:
a) Shell teeth b) Tauraodontisam a) Naso-alveolar cyst b) Globulomaxillary cyst
c) Dentin dysplasia d) Dentinogenesis imperfecta c) Naso palatine cyst d) Mid palatine cyst
(COMEDK -06) (AIIMS-07)
80. Dense in dente is most commonly seen in: 92. The histological appearance of "lava following around
a) Premolars b) Paramolars boulders in dentin dysplasia" suggests
c) Lateral incisors d) Maxillary canine a) Attempt to repair the defective dentin
(AIPG-06) b) Abrupt arrest to dentin formation in crown
81. Taurodontism is usually seen in: c) Abnormal dentin formation in a disorganized fashion
a) Mesiodens b) Incisor with talon/cusp d) Cascades of dentin to form root
c) Mandibular first molar d) Maxillary premolars (KCET-07)
(AIPG-06) 93. A 4 year old child with one tooth less than the normal
82. Puetz-Jegher syndrome is characterised by dentition shows a large tooth with two crowns and two
a) Deafness b) Multiple supernumerary teeth root canals, the anamoly is :
c) Multiple intestinal polyps a) Dilaceration b) Fusion
d) Scleroderma c) Gemination d) Concrescence
(KAR-06) (AP-08)
83. Which one of the following is the cause of dilacerations? 94. Which of the following is most common developmental cyst
a) Trauma to the tooth germ during root development a) Nasopalatine cyst b) Naso-alveolar cyst
b) Abnormal displacement of tooth germ during root development c) Globulomaxillary cyst d) Median palatal cyst
c) Abnormal proliferation of enamel epithelium during (AIIMS-08)
tooth development 95. A 6 year old patient with extra cusp on maxillary central
d) Abnormal displacement of ameloblasts during tooth formation incisor is associated with all except.
(AIPG-06) a) Mohr's syndrome
84. PARULIS is an inflammatory enlargement seen in b) Sturge-Weber Syndrome
a) End of sinus tract b) Extraction socket c) Rubinstein Taybi Syndrome
c) Due to irritation from calculus / over hanging restoration d) Proteus Syndrome
d) None of the above (AIIMS-09)
(AP-06) 96. Taurodontism is seen in
85. Dentinal union of two embryologically developing teeth a) Klinefelter's syndrome b) Sturge weber syndrome
is referred as c) Down syndrome d) Turner syndrome
a) Gemination b) Twinning (AIPG-10)
c) Concrescence d) Fusion 97. Static bone cyst is a cyst developing from
(COMEDK-05} a) Infection of Salivary gland in the mandible
b) Tissue of the odontogenic apparatus

75) C 76) B 77) B 78) A 79) D 80} C 81} C 82} C 83} A 84} A 85} D 86} A 87) D
88} A 89} D 90} C 91} B 92} D 93} B 94} A 95} D 96} A 97) D
r ORAL PATHOLOGY & MEDICINE
c) Tissue of the oral mucosa d) None of the above c) Alveolar ridge d) Palate
(AP-10) (KAR- 2013)
98. Most common site for melanotic neuroectodermal tumour 108. Brittle bone syndrome is caused by
of infancy is (M NTI): a) Improper synthesis of procollagen
a) Maxilla b) Mandible b) Polymerisation of collagen
c) Ethmoid bone d) Cervical spine c) Increase osteoclastic activity
(AIIMS-2011) d) Increase fibroblastic activity
99. A girl suffering from browning of teeth, wearing of (AIIMS-2012)
enamel but not cavitory. 0PG shows obliteration of 109. A 7-year-old child having yel lowish discoloured spot of
pulp with narrowing of canal & deposition of secondary maxillary central incisor. His mother presents a history
dentin, she gave the history that out of her 4 brothers, of injury to deciduous tooth 3 years back with recurrent
2 are suffering from the same disease. She is suffering infections & swelling. The diagnosis is
from a) Turners hypoplasia b) Generalised dental fluorosis
a) Amelogenesis im[Perfecta c) Syphillitic hypoplasia d) Rickets hypomineralisation
b) Dentinogenesis imperfecta (AIIMS MAY 2012)
c) Flourosis d) Odontodysplasia 110. Marked reduction in amount of dentin, widening of
(AIIMS-2011) predentin layer, presence of large area of interglobular
100. Cysts associated with vital teeth are: dentin and irregular pattern of dentin is seen in
a) Dentigerous cyst, Globulomaxillary cyst, lateral a) Hypocalcified dentin b) Odonto dysplasia
periodontal cyst, OKC c) Dentin dysplasia d) Dentinogenesis imperfecta
b) Dentigerous cyst, Globulomaxillary cyst, radicular cyst (AIIMS MAY 2012)
c) Dentigerous cyst, OKC, radicular cyst 111. Amelogenesis imperfecta is a disorder of:
d) 'B' & 'C' a) Ectoderm b) Mesoderm
(AIIMS-2011) c) Endoderm d) Ecto and Mesoderm
101. Failure of descent of thyroid analage can be seen in the (NEET-2013)
tongue 112. A child has marked difference of crown, root size,
a) In anterior 2/3 of dorsal aspect eruption pattern of left & right side of jaw. The condition
b) In posterior 1/3 of dorsal aspect is:
c) Near the base of tongue close to foramen caecum a) Crouzan syndrome b) Hemifacial hypertrophy
d) In anterior 2/3 of inferior surface c) Cherubism d) Achondroplasia
(COMED-2012) (NEET-2013)
102. Premaxilla-premaxillary cyst 113. Hypodontia is mainly found in all, except:
a) Nasoalveolar cyst b) Nasopalatine cyst a) Papillon levefre syndrome
c) Incisive canal d) Globulomaxillary cyst b) Osteopetrosis
(AIPG-2012) c) Cleidocranialdysostosis d) Ectodermal dysplasia
103. Geographic tongue is of red colour because of (NEET-2013)
a) Infiltration of eosinophils 114. Most characteristic feature of mandibulofacial dysostosis
b) Infiltration of neutrophils is
c) Both of the above d) None of the above a) Normal hearing
(PGI-2011) b) Normal vision with coloboma
104. According to Veau's classification of cleft lip & palate, c) Mandibular prognathism
isolated cleft palate falls under: d) Vertical maxillary excess
a) Group I b) Group II (AIIMS MAY-13)
c) Group III d) Group IV 115. How will you differentiate between fibrous dysplasia and
(PGI-2011) facial hemiatrophy?
105. Developmental enamel defects are most commonly seen a) Size of the face
in b) Distance between the anatomical soft tissue landmarks
a) Primary incisors b) Primary 2nd molar from midline
c) Permanent inciso,rs d) Permanent 1st molar c) Shape, size and difference in eruption pattern of teeth
(PGI-2011) d) Shape of tooth and supporting structures
106. Based on the degree to which apical displacement of the (AIPG-14)
pulpal floor is seen, "Cynodont" is the name given to 116. A true cyst which is derived from stellate reticulum and
a) Hypotaurodont b) Mesotaurodont grows in place of tooth is known as?
c) Hypertaurodont d) Normal tooth a) Primordial cyst b) Eruption cyst
(KAR- 2013) c) Dentigerous cyst d) Radicular cyst
107. Fordyce granules mostly occurs in all of the following (AIPG-14)
areas EXCEPT
a) Vermillion of the lip b) Posterior tonsillar pillar

98) A 99) B 100) A 101) C 102) D 103) D 104) B 105) C 106) D 107) C 108) A 109) A 110) B
111) A 112) B 113) B 114) B 115) C 116) A
Dental [fu£6e
117. Supernumerary tooth, tooth with in tooth is most 123. A 40 year old male patient presents with a painless
commonly seen in? swelling causing fullness of upper lip and elevation
a) Maxillary lateral incisors of nostril on left side. The only positive radiographic
b) Mandibular 2nd premolar finding is unilateral elevation of floor of nasal cavity.
c) Mandibular incisor s The probable diagnosis is:
d) Maxillary central and lateral incisors a) Globulomaxillary cyst b) Polyp in the nasal cavity
(AIPG-14) c) Radicular cyst d) Nasolabial cyst
118. The gene that defines the localization of the tooth germs (MHCET-15)
is 124. Following are true about thyroglossal cyst EXCEPT
a) Fgf-8 b) Bmp-2 a) Often occur in children; they may also present in adults
c) Dlx-5 d) Pax-9 b) Rarely may contain only functioning thyroid tissue
{APPG-15) c) Treated surgically by Sistrunks operation
119. Which condition is depicted in the CT picture given d) Moves downwards on tongue protrusion
below? (KERALA-2015)
125. Laugier-Hunziker syndrome is associated with
a) Idiopathic macular oral hyperpigmentation
b) Multiple impacted supernumerary teeth
c) Ankyloglossia
d) Hemifacial atrophy
(KERALA-2015)
126. Goldenhar syndrome is?
a) Facial deformity, auricular abnormalities and vertebral
deformity
a) Mandibular tori b) Torus palatinus b) Facial deformity, auricular abnormalities, vertebral
c) Complex odontoma d) Sialolithiasis deformity and genital lesion
(APPG-15) c) Facial and auricular abnormalities only
120. The condition highlighted by the arrow in the occlusal d) Auricular abnormalities only
radiograph in a 8 year old child could be due to (PGI JUNE-2011)
127. Enamel Hypo-calcification and hypo-maturation of tooth
is seen in?
a) Osteogenesisimperfecta
b) Dentinogenesisimperfecta
c) Trichodento-osseus syndrome
d) Turner tooth
(PGI DEC-2013)

a) Non-vital tooth b) Impacted tooth


c) Ghost tooth d) Mesiodens
(APPG-15)
121. Maximum incidence of developmental defects is seen in?
a) Primary incisor
b) Permanent incisors
c) Permanent mandibular first molar
d) Primary mandibular second molar
(PGI DEC-2011)
122. True about enameloma
a) A globule like structure usually found on root due to
misplaced ameloblast
b) True neoplasm of ameloblast
c) Tumor resembling enamel present on gingiva
d) Tumor of ameloblast which fall to differentiate terminally
(AIIMS MAY-14)

117) A 118 D 119 A 120 D 121 B 122 A 123 D 124 D 125 A 126 A 127 C
_____ r ORAL PATHOLOGY & MEDICINE

1. DEVELOPMENTAL DISTURBANCES - ANSWERS


1. 'A' [Shafer 7th ed 40, 41/ 6th ed 40] 6. 'B' [Shafer 7th ed 67/ 6th ed 66]

• An attempt at division of a single Radiologic feature Cyst


tooth before calcification, resulting Ovoid or heart shaped Median anterior
in incomplete formation of two teeth. radiolucency between or above maxillary cyst
(AIIMS-2012). the roots of maxillary central (Nasopalatine /
• The resultant structure will have two incisors Incisive canal cyst)
completely or incompletely separated Inverted pear shaped Globulomaxillary cyst
Gemination
crowns with a single root and root radiolucent area between lateral (COMED-14)
canal. incisor and cuspid
• It is difficult to differentiate between Multilocular or unilocular well Median mandibular cyst
gemination and a case in which there circumscribed lesion in midline
has been fusion between a normal of mandible
tooth and a supernumerary tooth.
Nasolabial or klestadt's
• The division of a single tooth resulting No radiologic feature
cyst (a soft tissue cyst)
in one normal and one supernumerary
Twinning
tooth. The number of teeth will be
7. 'B' [Shafer 7th ed 22/ 6th ed 28]
more than normal.
Fissured tongue is seen in Melkersson-Rosenthal
• A form of fusion of cementum after syndrome, Down syndrome and in some cases of benign
completion of root formation migratory glossitis.
• The teeth are united by cementum
Concrescence
only. Melkerson-Rosenthal syndrome is characterized by triad of
• Maxillary molars are frequently bell's Palsy, persistent or recurring lip or facial swelling
involved (cheilitis granulomatosa), and fissured tongue or lingua
plicata. (COM EDK-10)
• Union of two normally separated
tooth germs. 8. 'A' [Shafer 7th ed 67/ 6th ed 66]
• If the union occurs before calcification, Nasolabial or klestadt's cyst is a soft tissue cyst and involves
the two teeth are completely united the bone rarely.
to form a single large tooth.
Fusion • If the union occurs after calcification, 9. 'D' [Shafer 7th ed 721/ 6th ed 717]
these is union of roots only. The Micrognathia of maxilla is due to deficiency in premaxillary
tooth may have separate or fused root area.
canals.
• Fusion of teeth is more common in 10. 'C' [Shafer 7th ed 53/ 6th ed 52]
deciduous teeth.
11. 'B' [Shafer 7th ed 59/ 6th ed 48]
Natal teeth are the teeth that are present in the oral cavity
2. 'A' [Shafer 7th ed 41/ 6th ed 40] at the time of birth.
3. 'B' [Shafer 7th ed 52 for option 'A' 12. 'K [Shafer 7th ed 53/ 6th ed 52]
637 Fig 19.1 for 'B'
806 for option 'C' 13. 'A' [Shafer 7th ed 28/ 6th ed 27]
47 for option 'D'] Tongue tie or an kyloglossia may cause speech defects,
especially articulation of the sounds 1, r, t, d, n, th, sh,
4. 'C' [Shafer 4th ed 28] and z.
Erythema migrans / wandering rash / geographic tongue
(KCET-10) is other name for benign migratory glossitis. 14. 'C' [Shafer 7th ed 39/ 6th ed 38]
Example of true microdontia is "pituitary dwarfism". Example
5. 'B' [Shafer 7th ed 64/ 6th ed 64] for true macrodontia is "pituitary gigantism".
It is also called as "pre maxilla-maxillary cyst". It
characteristically appears as inverted, pear shaped 15. 'B' [Shafer 7th ed 40/ 6th ed 39]
radiolucent area between roots of lateral incisor and cuspid.
Histologically this cyst resembles odontogenic keratocyst. 16. 'A' [Shafer 7th ed 42/ 6th ed 101]
Dental 9'ul-6e
17. 'B' [Shafer 7th ed 58/ 6th ed 57] Clinically both types of dentinogenesis imperfecta exhibit
Ghost teeth or regional odontodysplasia is due to defective a characteristic unusual translucent or opalescent hue. The
mineralization. Both the enamel and dentin appear very usual scalloping of DEJ, which tends to form an interlocking
thin and the pulp chamber is exceedingly large. The union between the enamel and dentin is absent and
roentgenograms characteristically show a marked reduction this results in early loss of enamel. This results in rapid
in radiodensity so that the teeth assume a "ghost" attrition of enamel and the occlu sal surfaces of primary and
appearance. Maxillary anterior teeth are more frequently permanent molars are severely flattened.
involved than the mandibular anterior teeth
21. 'B' [Shafer 7th ed 48/ 6th ed 46]
18. 'B' [Shafer 7th ed 39/ 6th ed 37] 'Mesiodens' is the most common supernumerary teeth. It is a
Static or stafne cyst is due to developmental inclusion of cone-shaped crown with a short mot and is usually present
salivary glandular tissue on the lingual surface of mandible between the two maxillary central incisors. The various
BELOW the mandibular canal. morphological types of supernumerary teeth are
• Conical
19. 'B' [Shafer 7th ed 55/ 6th ed 54-55]
• Supplemental
In TYPE I dentinogenesis imperfecta the pulp chambers are
smaller than normal or completely obliterated but the teeth • Odontone (KCET-1O)
in TYPE II shows enormous pulp chambers and so known as
shell teeth. 22. 'A' [Shafer 7th ed 30/ 6th ed 29]
Option 'B', "Geographic tongue or benign migratory
20. 'D' (Shafer 7th ed 55/ 6th ed 54-55] glossitis" is characterized by multiple areas of desquamation
of the filiform papillae of the tongue. The desquamated
Revised shields classification of dentinogenesis areas appears in one location, then heal and reappear in
imperfecta another location giving rise to idea of migration.

• Dentinogenesis imperfecta is seen with out 23. 'C' (Shafer 7th ed 24/ 6th ed 23-24]
osteogenesis imperfecta. Fordyce's spots are heterotopic collections of sebaceous
• Caused by mutation in Dentin glands at various sites in the oral cavity (COMED-14).
sialophosphoprotein (DSSP) gene that maps They appear as yellow spots and are found on cheek mucosa
to chrosome#4. lateral to angle of mouth, mostly on the mucosa of the
• Affected teeth are amber brown and opalescent cheeks opposite the molar teeth in a bilaterally symmetrical
and have broad crowns with constriction pattern.
of cervical area resulting in 'Tulip' shape.
24. 'C' (Shafer 7th ed 46, 47/ 6th ed 45- 46]
• The teeth have bulbous crowns, with narrow
roots and root canals. The pulp chambers are
smaller than normal. Most common super-numerary
Type I Mesiodens
tooth
• Some researchers found an increase in
glycosaminoglycans. Maxillary 4th molars
2nd most common
followed by mandibular
• The dentin is composed of irregular tubules supernumerary teeth
4th molars
that tend to be larger in diameter and less
numerous than normal. This is due to limited Most commonly ankylosed tooth
Deciduous mandibular
ability of odontoblasts to form well organized or most commonly ankylosed
second molar (AP-14)
dentinal matrix. primary tooth/submerged tooth
• The water content in teeth is greatly increased Most commonly missing Maxillary and mandibular
as much as 60% above normal and the micro deciduous teeth lateral incisors
hardness of dentin closely approximates 3rd molars followed by
cementum. Most commonly missing
maxillary laterals and
permanent teeth
• Also known as Brandywine type or shell teeth mandibular or maxillary
(shields type III). second premolars.
• Apart from 0SSP, three other genes, DMP-1, Supernumerary roots are Mandibular canine &
lBSP and SPPl are also involved. common in premolars
• The dentin is extremely thin and the pulp Permanent premolars
Teeth commonly involved in
Type II chambers are enormous due to insufficient and followed by permanent
turner's hypoplasia
defective dentin formation & multiple pulp maxillary incisors
exposures are more common than in type I. Teeth that are commonly Maxillary lateral incisors
• In roentgenograms, the teeth appear as shells affected with microdontia followed by third molars.
of enamel and dentin surrounding extremely
large pulp chambers and root canals.
25. 'A [Shafer 7th ed 52/ 6th ed 51]
r ORAL PATHOLOGY & MEDICINE
39. 'C' [Shafer 7th ed 32/ 6th ed 31]
Below lyr Centrals, laterals, first molar and tip of cuspids Papilla involved Condition
are involved Multiple areas of desquamation Benign migratory glossitis
After 3yrs Premolars, 2 & 3rd molars are involved of filiform papillae or geographic tongue
Hypertrophy of filiform papillae Black hairy tongue
26. 'D [Shafer 7th ed 24/ 6th ed 23-24] from 1mm to 15 mm (lingua villosa)
Option 'B' heck's disease or focal epithelial dysplasia occurs
predominantly in children. It presents as multiple nodular Atrophy of filiform papilla with
lesion occurring most commonly on the lower lip. The lesions normal or engorged fungiform Riboflavin or B2 deficiency.
of focal epithelial hyperplasia contains virus like particles, papillae
which are similar to human oral papillomas. Swollen, hyperemic fungiform Strawberry tongue
papillae (scarlet fever)
27. 'C' [Shafer 7th ed 48/ 6th ed 47] Absence of fungiform and
Gardner's syndrome consists of: Familial dysautonomia
circumvallate papillae
• Multiple polyposis of large intestine
Absence of filiform papillae Median rhomboid glossitis
• Osteomas of bones
• Multiple epidermoid/sebaceous cysts 40. 'C' [Shafer 7th ed 41/ 6th ed 39]
• Impacted supernumerary and permanent teeth
• Occasional desmoid tumors 41. 'C' [Shafer 7th ed 67/ 6th ed 66]

28. 'fl{ [Shafer 7th ed 34/ 6th ed 33] 42. 'A' [Shafer 6th ed 38]
Lingual tonsil, one of the largest oral lymphoid aggregates, Microdontia most commonly affects maxillary lateral incisor
is located on the posterior portion of tongue on dorsolateral followed by 3rd molars.
aspect. It is surrounded by a crypt lined by stratified
squamous epithelium. It may become inflammed and 43. 'B' [Shafer 7th ed 45/ 6th ed 44]
enlarged bilaterally. Taurodontism (Bull Teeth) is due to failure of hertwig's
sheath to invaginate at proper horizontal level (AIPG-
29. 'A' [Shafer 7th ed 55/ 6th ed 54] 14). Mostly molars are involved and the body of tooth is
enlarged at expense of roots resulting in rectangular teeth.
30. 'B' [Shafer 7th ed 55, 56/ 6th ed 54-55] The roots are short and the pulp chambers are extremely
Refer Q. No. 20 large lacking the usual constriction at cervix of tooth.

31. 'A' [Shafer 7th ed 6 7/ 6th ed 93] It has been reported that many patients with Klinefelters
"Bohn's nodules" are seen at the junction of hard and soft syndrome exhibit taurodontism but is not constant feauture
palates whereas "Epstein pearls" are seen along the median of this syndrome (AIPG-10).
raphae of hard palate. A third type known as "Dental lamina
cysts of new born" are seen on the alveolar ridges. 44. 'B' [Shafer 7th ed 53/ 6th ed 52]

32. 'B' [Shafer 7th ed 46/ 6th ed 45] 45. 'C' [Shafer 7th ed 64 Tab 4-1 260/ 6th ed 63]
False anodontia / Induced anodontia is due to multiple
extracted teeth while pseudo anodontia is due to multiple 46. 'C' [Shafer 7th ed 28/ 6th ed 27]
unerupted teeth Bifid tongue / cleft tongue is one of feature of "oro - facial
-digital syndrome".
33. 'D' [Shafer 7th ed 53/ 6th ed 52]
47. 'C' [Shafer 7th ed 806/ 6th ed 45]
34. 'D' [Shafer 7th ed 64/ 6th ed 46-47]
Supernumerary teeth are most common in anterior maxilla. 48. 'B' [Shafer 7th ed 60/ 6th ed 59]
DELAYED ERUPTION is seen in conditions like:
35. 'B' [Shafer 7th ed 55/ 6th ed 695]
a) Systemic conditons
36. 'A' [Shafer 7th ed 41/ 6th ed 39] • Rickets
Refer Q. No. 1 • Down syndrome
• Hypopituitarism
37. 'C' [Shafer 7th ed 48/ 6th ed 47]
• Cleidocranial dysplasia
38. 'A' [Shafer 7th ed 32/ 6th ed 31] • Achondroplasia
• Cretinism (Congenital Hypothyroidism)
• Fibromatosis gingiva
• Cherubism
Dental 9'ul-6e
b) Local conditions 57. 'C' [Shafer 7th ed 47/ 6th ed 46]
• Fibromatosis gingivae Anamolies affectin ermanent maxilla incisor
• Premature exfoliation of primary teeth • Dens-in-dente
• Thickened gingiva • Localized microdontia (peg lateral)
• Talon cusp
In hyperthyroidism, premature shedding of deciduous teeth • Most commonly missing permanent tooth after third
and accelerated eruption of permanent teeth is seen. molars (maxillary lateral incisor is the most commonly
missing deciduous tooth)
49. 'C' [Shafer 7th ed 30/ 6th ed 31]
• May be affected by Turner's Hypoplasia & Regional
Odonto dysplasia.
50. 'B' [Shafer 7th ed 69]
58. 'A' [Shafer 7th ed 52/ 6th ed 51]
51. 'D' [Shafer 7th ed 55/ 6th ed 54]
The incisors are screw driver shaped. The mesial and distal
surfaces of crown are tapered and the incisal edge is usually
52. 'B' [Shafer 7th ed 46/ 6th ed 45]
notched. In congenital syphilis the molars are irregular and
the occlusal one third of tooth is arranged in an agglomerate
53. 'B' [Shafer 7th ed 30/ 6th ed 29]
mass of globules. The molars are called Mulbery molars or
Median rhomboid glossitis or posterior midline atrophic
Moon's molars or Fournier's molars.
candidiasis shows male predilection in 3:1 ratio. In some
cases, a midline soft palate erythema may be foend in the area
59. 'A' [Shafer 7th ed 64/ 6th ed 65]
of routine contact with the underlying tongue involvement
and this is commonly referred as kissing lesion.
60. 'D' [Shafer 7th ed 58/ 6th ed 5 7]
(Note:- Kissing disease is other name for infectious
61. 'A' [Shafer 7th ed 67/ 6th ed 66]
mononucleosis)
"Kissing lesion" is representative of [KCET-2012] 62. 'C' [Shafer 7th ed 58/ 6th ed 57]
a) Candidiasis b) Cytomegalovirus infection The etiology of odontodysplasia may be due to latent virus
c) Histoplasmosis d) Infectious mononucleosis residing in odontogenic epithelium or due to local vascular
defect. Recently it has been suggested that odontodysplasia
54. 'B' [Shafer 7th ed 63/ 6th ed 62] may represent a somatic mutation.
Submerged teeth are deciduous teeth that are ankylosed
and prevent subsequent replacement by permanent teeth. 63. 'B' [Shafer 6th ed 44]
The affected teeth lack mobility and upon percussion show
a characteristic solid sound in contrast to dull cushioned 64. 'C' [Shafer 7th ed 52/ 6th ed 52]
sound of a normal tooth. X - Ray shows partial or complete Turner's hypoplasia involves most commonly one of the
absence of periodontal ligament space. permanent maxillary incisor or a maxillary or mandibular
premolar.
55. 'D' [Shafer 7th ed 67 / 6th ed 66]
• Derived from epithelial remnants of 65. 'C' [Shafer 7th ed 56/ 6th ed 55]
developing palatal salivary glands. According to Shafer 4th edition (shields classification)
Bohn's correct answer is type III (option'B'). But according to
nodules • Usually scattered over the hard palate
Shafer 5 th edition (revised shields classification) shell teeth
and tend to be most numerous along the
are seen in type II (option 'C').
junction of hard and soft palate.
Usually present along the median raphae For further information please refer both editions.
Epstein's of hard palate and appeared to be derived
pearls from entrapped epithelial remnants 66. 'B' [Shafer 7th ed 56/ 6th ed 55]
along the line of fusion (AIPG-14). Refer Q. No 65
Dental Found on alveolar ridges and cysts of new
lamina cysts are derived from remnants of born dental 67. 'D' [Shafer 7th ed 727/ 6th ed 722]
of new born lamina. Important Manifestations of cleidocranial dysostosis or
Marie and Sainton's disease:
56. 'C' [Shafer 7th ed 42/ 6th ed 41] • Partial or complete absence of clavicles
Permanent maxillary lateral incisors are most frequently • Delayed closure of fontanels
involved.
• Wormian bones are common
• The sagittal suture is characteristically sunken, giving
the skull a flat appearance
• Hypoplasi a of maxilla

r ORAL PATHOLOGY & MEDICINE
Prolonged retention of deciduous teeth (AP-14) and • Numerous unerupted supernumerary teeth, which are
subsequent delay in eruption of succedaneous teeth most prevalent in premolar and incisor areas.
• Absence or paucity of cellular cementum on the roots of
the permanent teeth with no increased thickening of the 68. 'C' [Shafer 7th ed 52/ 6th ed 51)
primary acellular cementum.
69. 'C' (Shafer 7th ed 42/ 6th ed 41)

70. 'A' (Shafer 7th ed 57/ 6th ed 57]


Dentin dysplasia (rootless teeth) is characterized by normal enamel but atypical dentin formation with abnormal pulpal
morphology.
• Both the dentitions are clinically normal
• The teeth exhibit extreme mobility and are exfoliated prematurely as a result of their abnormally short roots.
• In deciduous teeth, the pulp chambers and root canals are completely obliterated, while in permanent dentition,
Type I or a crescent shaped pulpal remnant may be seen in the pulp chamber.
radicular • Some times periapical radiolucencies, representing granulomas, cysts, or abscesses involving apparently
dentin otherwise intact teeth.
dysplasia • The pulp is obliterated with calcified tubular dentin, osteodentin and fused denticles.
• Normal dentinal tubule formation appears to be blocked, so that new dentin forms around obstacles and takes
the characteristic appearance described as "Lava flowing around boulders" and this pattern of "cascades of
dentin" results from repetitive attempts to form root structure.
• The clinical appearance of permanent dentition is normal, but the deciduous teeth
• shows brown, opalescent hue as seen in dentinogenesis imperfecta.
Type II or
• The pulp chambers of deciduous teeth are obliterated as in type I and in dentinogenesis imperfecta.
Coronal
dentin • The permanent teeth exhibit abnormally large pulp chambers in the coronal portion, often described as "thistle
dysplasia tube" in shape.
• The deciduous teeth exhibits amorphous and atubular dentin in radicular portion, while the coronal dentin is
normal. The permanent teeth contains multiple pulp stones.

71, 'D' [Shafer 7th ed 158/ 6th ed 154] 76. 'B' (Shafer 6th ed 4 7]
"Mesiodens", a small peg-shaped conical tooth is the most
72. 'B' (Shafer 7th ed 58/ 6th ed 57] common supernumerary tooth found between two maxillary
Abnormally large pulp chambers are seen in regional permanent central incisors.
odontodysplasia (Ghost teeth) and TYPE II dentinogenesis
imperfecta (shell teeth) 77. 'B' (Shafer 7th ed 55/ 6th ed 54]

73. 'B' (Shafer 7th ed 720/ 6th ed 716] 7 8. 'A' (Shafer 7th ed 25/ 6th ed 24]
Clinical features of mandibulofacial dysostosis or Focal epithelial hyperplasia (Heck's disease) is highly
Treacher Collins syndrome contagious HPV-induced epithelial proliferation. The
• Antimangoloid palpebral fissures hyperplastic lesions appear as dusters and give the entire
mucosa[ area a Cobblestone or fissured appearance.
• Hypoplasia of the facial bones, especially of the malar
bones and mandible.
79. 'D' [Shafer 7th ed 56 for Option A, 45 for Option B
• Malformation of external ear, middle ear and internal ears. 57 for Option CJ
• Macrostomia with abnormally high palate.
• Facial clefts, skeletal deformities and malocclusion 80. 'C' (Shafer 7th ed 42/ 6th ed 41)
• Patients have Bird like or fish like face.
81. 'C' [Shafer 7th ed 45/ 6th ed 44)
Permanent molars are most commonly affected by
74. 'A' (Shafer 7th ed 721/ 6th ed 717]
Taurodontism.
Characteristics triad of Pierre Robin syndrome
• Micrognathia or Retrognathia 82. 'C' (Shafer 7th ed 23/ 6th ed 22)
• Cleft palate Important features of Peutz - Jeghers syndrome
• Glossoptosis, accompanied by airway obstruction • Also known as hereditary intestinal polyposis syndrome.
• Circum oral and perinasal pigmentation (1-5mm macules)
75. 'C' [Shafer 7th ed 47 Table 1-12/ 6th ed 46 Table 1-11] crossing the vermilion border.
Mandibular 2 nd premolar followed by maxillary 2 nd premolar
are most commonly congenitally missing teeth.
Dental 9'ul-6e
• Pigmentation may also be present on the fingers and • The number of teeth in case of successful gemination -
toes, on the hanc:ls and feet and around the anus and 29 teeth.
genitalia. • The number of teeth in case of incomplete gemination -
• Prolapse of tissue from the rectum. 28 teeth (27 normal+ 1 macrodontic).
• Precocious puberty (also seen in Albright syndrome) • The number of teeth in case of fusion between normal and
• Gynecomastia and growth acceleration ( due to Sertoli supernumerary teeth-28 (27 normal+ 1 macrodontic)
cell tumor)
94. 'A' (Shafer 7th ed 64/ 6th ed 63]
83. 'A' (Shafer 7th ed 41/ 6th ed 40] Nasopalatine duct cyst/Nasopalatine canal cyst/ Incisive
canal cyst is most common non-odontogenic cyst. Its
84. 'A' (Shafer 7th ed 36/ 6th ed 34] location is peculiar and specific in that it affects the midline
"Parulis" or pus pocket is an inflammatory enlargement seen anterior maxilla.
at the terminus of fistula or sinus tract.
Option 'B' nasoalveolar cyst is a rare fissural cyst, that
85. 'D' [Shafer 7th ed 40/ 6th ed 39] arise at the junction of globular process, the lateral nasal
process and the maxillary process.
86. 'A' (Shafer 7th ed 47/ 6th ed 46 Tab.1-11&1-12]
Option 'C' globulomaxillary cyst is found within the bone at the
87. 'D' (Shafer 7th ed 159/ 6th ed 155] junction of the globular portion of the medial nasal process,
Torus mandibularis is commonly seen on the lingual surface maxillary process, the globule maxillary fissure, usually
of mandible opposite to the premolar teeth. It is usually between the maxillary lateral incisor and cuspid teeth.
present above the myalohyoid line.
95. 'D' (Oral & Maxillofacial Pathology by Neville 2nd ed 78]
88. 'A' (Shafer 7th ed 63/ 6th ed 62] Talon cusp is an additional cusp that is usually located on
Submerged teeth are deciduous teeth that have undergone the lingual surface of anterior teeth. It extends atleast half
a variable degree of root resorption and then become the distance for the cemento-enamel junction to the incisal
ankylosed to bone. Mandibular second molars are most edge. It is composed of normal enamel and dentin and
commonly involved teeth. contains a horn of pulp tissue.

89. 'D' [Shafer 7th ed 64, 65/ 6th ed 64] It is seen in:
Nasoalveolar cyst [also known as Nasolabial or Klestadt's • Mohr syndrome
cyst (AIPG-09)] arises at the junction of globuar portion • Rubinstein Taybe syndrome
of medial nasal process, the lateral nasal process and the • Sturge-Weber syndrome
maxillary process. It is a soft tissue cyst, involving bone
rarely and shows no radiographic features.
96. 'A' (Shafer 7th ed 45/ 6th ed 44]
Taurodontism / bull teeth are seen in:
90. 'C' (Shafer 7th ed 3/ 6th ed 4,5]
• Klinefelter's syndrome (XXY Chromosomes)
91. 'B' (Shafer 7th ed 66/ 6th ed 64] • Tricho-dento-osseous syndrome
As the question is about the cyst found wihthin the bone,
option 'B' globulomaxillary cyst is correct answer. 97. 'D' (Shafer 7th ed 39/ 6th ed 37]
Static bone cyst / latent bone cyst / Stafne cyst is
92. 'D' (Shafer 7th ed 58/ 6th ed 57] developmental inclusion of salivary gland tissue adjacent to
lingual surface of body of the mandible.
93. 'B' [Shafer 7th ed 41/ 6th ed 39]
In case of fusion, the patient will be having one tooth less 98. 'A' (Shafer's 7th ed 205/ 6th ed 201-2)
than normal dentition. In case of successful gemination, MNTI is an uncommon osteolytic, pigmented neoplasm,
the patient will be having one tooth in excess of normal primarily affecting the jaws of new born infant. Its origin is
dentition. But it is always difficult to diagnose between linked to neural crest cells. More than 90% occur in the head
fusion between a normal tooth and a supernumerary tooth and neck region especially the anterior part of the maxillary
and incomplete gemination. This is because of that the ridge. They appear as rapidly growing, non-ulcerated,
patient in both the conditions will have normal dentition, darkly pigmented lesions, with a radiographic appearance
but with a macrodontic tooth. of an invasive malignant neoplasm.

In case of a patient of age 16 years:


• The number of teeth to be present - 28 (third molars are
not erupted)
• The number of teeth in case of fusion - 27 (26 normal+
1 macrodontic tooth)
99. 'B' [Shafer's 7th ed 55/ 6th ed 54]
r ORAL PATHOLOGY & MEDICINE
• Amelogenesis imperfecta: All teeth of both dentitions
Dentinogenesis imperfecta is an autosomal dominant are affected to some degree. In some cases, the teeth
condition that manifests as gray - yellowish brown crowns may appear essentially normal.
in both deciduous and permanent dentitions, lacking pulp
• Hypoplasia due to nutritional deficiency and
chambers and root canals on radiograph. Amelogenesis
exanthematous fevers: The teeth most frequently
imperfecta, fluorosis does not cause obliteration of pulp
involved are central & lateral incisors, cuspids and first
chambers. Regional Odontodysplasia causes shell like teeth,
molars
with reduced radiodensity of enamel and dentin.
• Enamel hypoplasia due to congenital syphilis:
100. 'A' [Shafer's 7th ed 273/ 6th ed 268, 64] Maxillary & mandibular permanent incisors & the first
Radicular cyst or the periapical cyst is the most common molars are involved.
inflammatory cyst, associated with tooth necrosis, hence the
• Enamel Hypoplasia due to local infection or trauma:
involved teeth are non-vital. The rest are developmental cysts
Most commonly one of the permanent maxillary incisors
that arise from cell rests of Serrae, reduced enamel epithelium
or a maxillary or mandibular premolar are involved.
or epithelial entrapment. Recently, an odontogenic origin
had been suggested for globulomaxillary cysts, the clinical
and radiologic features being entirely compatible with a 106. 'D' [Shafer's 7th ed 45/ J Indian Soc Pedod Prev Dent -
lateral periodontal, lateral dentigerous or primordial cyst. March 2006: Taurodontism of deciduous and permanent
molars: Report of two cases]
101. 'C' [Shafer's 7th ed 33/ 6th ed 32] Teeth with pulp chambers that are relatively small and
The thyroid gland and the tongue form the pharyngeal floor have constriction of the pulp chamber approximately at the
at the same time and both are anatomically associated with CEJ and are called as cynodonts. The term taurodontism
the thyroglossal tract and its lingual remnant, the foramen was introduced by Sir Arthur Keith to describe a dental
caecum. Lingual thyroid is an anomalous condition, arising anomaly where the body of the tooth is enlarged at the
from the thyroid analogue that failed to migrate to its expense of the roots (bull -Ii ke teeth). The taurodontic
predestined position or from the analogue remnants left teeth are identified by elongated pulp chambers and apical
behind. It is seen usually in the vicinity of the foramen displacement of bifurcation or trifurcation of the roots. Due
caecum. to this, the chamber has a greater apicoocclusal height than
in cynodont teeth and lacks the constriction at the level of
102. 'D' [Shafer's 7th ed 65/ 6th ed 64] the CEJ giving it a rectangular shape.
Globulomaxillary cyst had been traditionally described
as a fissural cyst, that forms in the incisive suture (bone Shaw further classified taurodont teeth into hypo, meso and
suture between the premaxilla and maxilla). Hence the term hypertaurodont.
premaxilla- maxillary cyst was used by Ferency to describe • Hypotaurodont is the mildest form where the
the apt origin of globulomaxillary cyst. bifurcation/ trifurcation is slightly apical to the normal
level.
103. 'D' [Shafer's 6th edl 31, Differential diagnosis of Oral &
• Hypertaurodont is the extreme form in which the
Mxillofacial lesions-Wood and Goaz 5th ed 90]
bifurcation/ trifurcation occurs near the apices of the
The smooth depapillated areas of the geographic tongue
roots.
show a thin layer of epithelium with thin elongated rete
ridges, thus contributing to the red colour. • Mesotaurodont is in between the hypo and
hypertaurodont forms.
104. 'B' [Shafer's 7th ed 19/ 6th ed 18]
107. 'C' [Shafer 7th ed 24/ 6th ed 23]
Veau classification system of clefts Fordyce's granules are ectopic sebaceous glands that appear
Group I (A) Defects of the soft palate only as small yellow spots, found frequently in a bilaterally
Group II (B) Defects involving the hard palate and soft symmetrical pattern on the buccal mucosa, labial mucosa,
palate vermilion border of the lips, retromolar region and
occasionally on the tongue, gingival, frenum and palate.
Group III (C) Defects involving the soft palate to the Ectopic sebaceous glands are also found in the esophagus,
alveolus, usually involving the lip. female and male genitilia, nipples, palms and soles, parotid
Group IV ( D) Complete bilateral clefts. Hence, the best gland, larynx and the orbit.
option is group II
108. 'K [Shafer's 7th ed 699-700/ 6th ed 694,695]
105. 'C' [Shafer's 7th ed 52/ 6th ed 51] Osteogenesis imperfecta, also known as Brittle Bone
Clinical studies indicate that most cases of enamel hypoplasia Disease, or 'Lobstein syndrome' is disorder of congenital
involve those teeth that form within the first year of birth, bone fragility caused by mutations in the genes that
mainly the central and lateral incisors, cuspids and the first codify for type I procollagen. This deficiency arises from an
molars. amino acid substitution of glycine to bulkier amino acids
in the collagen triple helix structure. If the body does not
destroy the improper collagen, the relationship between the
Dental 9'ul-6e
collagen fibrils and hydroxyapatite crystals to form bone is 113. 'B' [Shafer 7th ed 46, 704/ 6th ed 700)
altered causing brittleness. Osteopetrosis is a rare bone disorder caused by failure of
the osteoclastic bone resorption leading to increased bone
The following 4 types of osteogenesis imperfecta have been mass. The dental manifestations include retardation of
reported. Type I -mild form, type II -extremely severe, type tooth eruption, along with increased risk of osteomyelitis
III -severe, type IV -undefined. These partly depend on the (following dental infection/ extraction). The teeth are prone
genetic subtype. Type I and IV are the most common forms to caries.
of osteogenesis impenfecta.
Congenital lack of teeth comprises the most common dental
109. 'A' [Shafer 7th ed 52/ 6th ed 52) abnormalities in humans. Oligodontia or agenesis of more
Turner's hypoplasia is an unusual hypoplasia seen in the than six teeth (excluding the third molars), and its less
permanent maxillary incisors or maxillary or mandibular severe form, hypodontia, can be inherited in autosomal
premolars due to trauma or periapical infection of the dominant, autosomal recessive, or X-linked inheritance
deciduous predecessors. The bacterial infection of the mode.
periapical tissues may disturb the ameloblastic layer of the • Papillon levefre syndrome
permanent tooth and result in a hypoplastic crown. The • Ectodermal dysplasia
hypoplasia may range from a mild brownish discolouration
to a severe pitting and irregularity of the tooth crown. The • Cleidocranial dysostosis
affected teeth are called Turner's teeth. • Down Syndrome
• Van der woude syndrome, etc.
110. 'B' [Shafer 7th ed 58/ 6th ed 57) • Also there is the possibility that this defect depends on
Regional Odontodysplasia is an unusual anomaly affecting
a moniliasis (Candidial endocrinopathy syndrome)
one or several teeth in a localised area, where the shape
of teeth may be markedly altered with delay in eruption.
114. 'B' [Shafer 7th ed 720)
The radiographic appearance is 'ghost appearance' due to
Teacher Collins syndrome or mandibulofacial dysostosis is
reduction in the radiodensity of the teeth. The enamel
a group of closely related defects of the head and face,
and dentin appear very thin and the pulp chamber is very
characterised by
large. The characteristic histological appearance is marked
reduction of dentin, widening of predentin, presence of • Anti mongoloid palpebral fissures with a coloboma of the
large interglobular derntin and an irregular pattern of dentin. outer portion of the lower eyelid
The other names for this disease are Odontodysplasia, • Hypoplasia of the malar bones and mandible
Odontogenesis imperfecta, Odontogenic dysplasia, ghost • Malformation of the external ear
teeth. • Macrostomia, high palate and malocclusion of teeth
111. 'A' [J Oral Maxillofacial Pathol 2009; Jul 13(2): 70-7. • Blind fistulas between angles of ears and angles of the
Amelogenesis imperfecta: report of a case and review of mouth
literature]
Amelogenesis Imperfecta is a diverse collection of inherited 115. 'C' [Shafer 7th ed 15,711)
diseases that exhibit quantitative or qualitative tooth Facial Hemiatrophy is characterised by progressive wasting
enamel defects in the absence of systemic manifestations. of the subcutaneous fat, skin, cartilage, muscle, bone
The defect is entirely ectodermal, since the mesodermal accompanied by contralateral Jacksonian epilepsy, trigeminal
components of the teeth are basically normal. neuralgia. The most common sign is a painless cleft, coup
de sabre near the midline of the face or forehead that marks
112. 'B' [Shafer 7th ed 13/ 6th ed 13) the boundary between normal and atrophic tissue. Dental
Hemifacial hypertrophy or hyperplasia is a rare developmental abnormalities include incomplete root formation, delayed
anomaly characterised by asymmetric growth overgrowth eruption; growth of the teeth may also be affected.
of one or more body parts. An anatomic classification of
hemihyperplasia given by Hoyme et al classifies it into Fibrous dysplasia results in expansion and deformity of the
complex hemihyperplasia (involvement of half the body), jaws and eruption pattern of the teeth may be disturbed
simple hemihyperplasia (involvement of a single limb) or due to the loss of normal support of developing teeth or the
hemifacial hyperplasia (involvement of one side of face). endocrine disturbances. However the formation of teeth is
Facial hemihypertrophy is characterised by enlargement on not affected.
one side of the face, unilateral macroglossia, premature
development, eruption and increased size of dentition. 116. 'A' [Karjodkar 2nd ed 497)
Dentition on the hypertrophic side is abnormal in 3 aspects Primordial cyst develops due to cystic changes in the stellate
- crown size, root size and rate of development. Permanent reticulum of a tooth germ before it mineralises. Therefore
teeth may be larger, rate of development and eruption is the tooth does not develop. However, now this is considered
more rapid along with premature shedding of deciduous not to exist and the similar cyst with aggressive behaviour
predecessors. had been termed as keratocystic ,odontogenic tumor (KOT).
117. 'A' [Shafer 7th ed 43, 48]
r ORAL PATHOLOGY & MEDICINE
the underlying bone or distortion of inferior border of nasal
Permanent maxillary lateral incisor is the most common fossa.
supplemental kind of supernumerary tooth. Dens in dente
is a developmental variation caused by invagination in the 124. 'D' [Shafer 7th ed 68)
surface of tooth crown before calcification has occurred Thyroglossal duct cyst results from the dilatation of remnants
and is most commonly seen in permanent maxillary lateral of thyroglossal duct. They often present as asymptomatic
incisors. midline neck mass at or below the level of hyoid bone and
move upwards during swallowing and tongue protrusion.
118. 'D' [Shafer 7thed 10] They may contain functional thyroid tissue and before
During tooth development, several mesenchymal molecules excision, presence of normally functioning thyroid glands
and their receptors act as mediators of epithelial - in usual locations should be assessed by thyroid scans and
mesenchymal interactions. Bone morphogenetic proteins function tests. Simultaneous removal of the central portion
(BMPs) 2,4,7 mRNAs shift between epithelium and of hyoid bone to ensure complete removal of the tract is
mesenchyme during regulation of tooth morphogenesis. known as Sistrunk procedure.
Fibroblast growth factor (FGF8, FGF9) have been localised
in the dental mesenchyme and act via downstream factors 125. 'A' [Burket 11th ed 121]
MSXl and PAX9. Muscle specific homeobox genes MSX-1 Laugier Hunziker pigmentation is an acquired, idiopathic,
and Msx-2, appear to be involved in epithelial mesencymal macular hyperpigmentation of the oral mucosal tisues,
interactions and craniofacial development, particularly in especially lips, buccal mucosa, along with esophageal,
the initiation of developmental position (Msx-1) and further genital, and conjunctiva! surfaces. They may also have nail
development (Msx-2) of tooth buds.Pax-9 is transcription involvement in the form of longitudinal melanotic streaks
factor necessary for tooth morphogenesis. without any evidence of dystrophic change. No systemic
abnormalities had been identified. The patients present with
119. 'A' [Shafer 7th ed 159) multiple, discrete, brown or dark brown macules, no more
Torus mandibularis is an exostosis or outgrowth of bone than 5mm in diameter and this condition is a diagnosed by
found on the lingual surface of bone, usually opposite exclusion.
bicuspid teeth and may vary in size, shape and number.
126. 'A' [Shafer 7th ed 721/ White & Pharoah 6th ed 564)
120. 'D' [Shafer 7th ed 48) Goldenhar syndrome or oculoauriculovertebral dysplasia is
Mesiodens is usually a peg shaped conical supernumerary caused by abnormality in the vascular supply of the head.
tooth found in between the permanent incisors and may Hemifacial microsomia is second most common craniofacial
cause the rotation or displacement of the permanent anomaly, causing reduced growth and development of one
incisors. The long axis may be normally inclined or may half of the face. The mandible, maxilla, zygoma, external
inverted or may be seen in a horizontal position. & middle ear, hyoid bone, parotid gland, 5th & 7th nerves,
musculature and other soft tissues. When hemifacial
121. 'B' [Shafer 7th ed 40,42,43,47) microsomia is associated with vertebral abnormalities and
Permanent maxillary lateral incisors are the teeth most epibulbar dermoids, it is known as Goldenhar syndrome.
commonly associated with microdontia involving a single
tooth (peg lateral), dens in dente (accentuation in the 127. 'C' [Check Explanation Below]
development of lingual pits), true partial anodontia after Tricho-dento-osseous (TOO) syndrome is a rare, autosomal
the third molars. Talons cusp, an anomalous structure dominant disorder principally characterised by curly hair at
projecting lingually from the cingulum area is commonly infancy, severe enamel hypomineralization and hypoplasia
seen in maxillary or mandibular permanent incisors. and taurodontism of teeth, sclerotic bone, and other defects.
Diagnostic criteria are based on the generalized enamel
122. 'A' [White & Pharoah 6th ed 311] defects, severe taurodontism especially of the mandibular
Enameloma is a globule of enamel, of 1-3 mm in diameter first permanent molars, an autosomal dominant mode of
that occurs on roots of molars. It is probably formed by inheritance, and at least one of the other features (i.e., nail
hetwig's epithelial root sheath before the epithelium loses defects, bone sclerosis, and curly, kinky or wavy hair present
its enamel forming ability. at a young age that may straight,en out later). Confusion with
amelogenesis imperfecta is common; however, taurodontism
123. 'D' [Shafer 7th ed 66/White & Pharoah 6th ed 360) is not a constant feature of any of the types of amelogenesis
Nasoalveolar cyst or nasolabial cyst was once thought to imperfecta.
arise at the junction of globular process, lateral nasal process
and the maxillary process due to proliferation of entrapped
epithelium along the fusion line. It is now thought to
originate from the lower anterior part of nasolacrimal duct
rather than from entrapped epithelium. It may cause a
swelling in the mucolabial fold as well as floor of the
nose, being located near the attachment of the ala over
the maxilla. It is a soft tissue cyst, thus not being visible
on a radiograph. Occasionally, it may produce erosion of
Dental 9'ul-6e � - - - - - -

2. BENIGN AND MALIGNANT TUMOURS OF ORAL CAVITY


1. Which of the following lesions are seen in van b) Gingival cyst of the newborn
recklinghausen's disease of skin c) Haemorrhagic bone cyst
a) Hemangioma b) Ameloblastoma d) All of the above
c) Neurofibroma d) Giant cell fibroma (MAN -98, AP -01)
(KAR -98) 13. Frequent bouts of epistaxis are a conspicuous feature of
2. In which of the following conditions pulsations or a) Encephalotrigeminal angiomatosis
murmur may be detected b) Hereditary haemorrhagic telangiectasia
a) Capillary hemangioma b) Osteogenic sarcoma c) Nasopharyngeal angiofibroma
c) Epidermoid carcinoma d) Osteoid osteoma d) Vascular nevus
(KAR -98) (MAN-95)
3. Hereditary hemorrhaglic telangiectasia is seen commonly on 14. Bence - Jones are laboratory finding of
a) Lips b) Buccal mucosa a) Paget's disease b) Multiple myeloma
c) Tongue d) Palate c) Malignant melanoma d) �ibrous dysplasia
(AP -04) (MAN-98, AIPG-06)
4. Numbness of lip seen with no previous dental treatment 15. Which of the following conditions is not considered
a) Meta static carcinoma b) Central nervous system lesion as premalignant
c) Osteomyelitis d) Infection a) Erosive lichen planus b) Syphilitic glossitis
(AP -04) c) Leukoedema d) Leukoplakia
5. Which of the following produces osteoblastic secondaries (KAR-04, COMEDK-05)
a) Carcinoma lung b) Carcinoma breast 16. "Warty" or "Cauliflower" like grrowth is
c) Carcinoma urinary bladder a) Papilloma b) fibroma
d) Carcinoma prostate c) Lipoma d) Torus
(AIIMS -04) (MAN-2K)
6. An Inflammed capillary hemangioma of the oral cavity 17. Starry sky appearance is seen in
looks similar to a a) Pagets disease b) Cherubism
a) Nevus b) Neurofibroma c) Garrey's osteomyelitis d) Burkitts lymphoma
c) Pyogenic granuloma d) Angiosarcoma (COMEDK-14)
(AP -04, APPSC-99) 18. Bowen's disease is
7. Reed-sternberg cells are characteristically seen in a) Benign neoplasm of the G.I.T
a) Alpha-thalassemia b) Glandular fever b) Intra epithelial carcinoma
c) Hansan's disease d) Hodgkin's disease c) Vesiculobullous lesion of skins
(COMEDK -04,MAN -94,AIPG -98) d) Ulcerative lesion of G.I.T
8. Which of the following is the most common site for the (MAN-02)
occurrence of a basal cell carcinoma? 19. Granular cell myoblastoma of tongue is
a) Buccal mucosa b) Hard palate a) Benign tumor
c) Skin of the lower Lip d) Dorsum of the tongue b) Malignant tumor of the tongue
(AIIMS -93) c) Lymphatic enlargement of the tongue
9. Which of the following is the most likely (among them) d) Developmental anomaly of the tongue
to turn malignant? (MAN -02)
a) Intradermal nevus b) Junctional nevus 20. Abtropfing affect is seen in
c) Lichen planus d) Papilloma a) Junctional nevus b) Pemphigus
(AIIMS -93, MAN-94,98) c) Apthous ulcer d) Erythema multiformae
10. A rhabdomyoma is a tumour originating from (MAN -02, 2K)
a) Nerve tissue b) Smooth muscle 21. Carcinoma of lip
c) Striated muscle d) Vascular endothelium a) occurs mostly in femalesb) metastases easily
(MAN -94, AIPG-97,AIIMS-93) c) occurs mostly in the lower lip
11. The term poikilokaryosis refers to d) is mostly basal cell type
a) Alteration of nuclear cytoplasmic ratio (MAN -99)
b) Division of nuclei without division of cytoplasm 22. On stretching the cheek the lesion disappears in:
c) Large, prominent nuclei a) Leukoplakia b) Focal hyperkeratosis
d) Loss of polarity and disorientation of cells c) Leukoedema d) Typhoid
(COMEDK -04, MAN -98, AIIMS -93) (C0MEDK -04, AIIMS -2K)
12. Which of the following is a true cyst?
a) Aneurysmal bone cyst

1) C 2) A 3) A 4) A 5) 0 6) C 7) 0 8) C 9) B 10) C 11) B 12) B 13) B


14) B 15) C 16) A 17) 0 18) B 19) A 20) A 21) C 22) C
___ r
23. The most common precancerous lesion for oral
ORAL PATHOLOGY & MEDICINE
c) Smoker's keratitis d) Candidiasis
malignancy is (AIPG-94)
a) chronic hypertrophic candidiasis 34. The most common malignancy of the oral cavity is:
b) leukoplakia a) Basal cell carcinoma b) Transitional cell carcinoma
c) dental ulcers d) atrophic glossitis c) Melanoma d) Squamous cell carcinoma
(KAR -99) (AIPG-91)
24. Satellite lesion with locally invasive property is seen in 35. The oral mucosa becomes rigid, blanched and opaque in
a) chronic hypertrophic candidiasis which of the following conditions?
b) leukoplakia a) Pemphigus vulgaris b) Lupus erythematoses
c) dental ulcers d) Hemangioma c) Ehlers-Danlos syndrome d) Submucous fibrosis
(AP -99) (AIPG -95)
25. The most common site for metastasis from carcinoma of 36. Which of the following is most likely to be fatal?
cheek: a) 0steochondroma b) Giant cell tumour
a) Brain b) Regional lymph nodes c) Paget's disease d) Multiple myeloma
c) Lung d) Liver (AIPG -90, 11)
(KAR -01) 37. The sign or symptom most suggestive of metastatic
26. During oral examination of a 57-year-old-man, a large disease is:
keratotic patch that covers the entire palate is noted. a) Paraesthesia b) Sudden swelling
Some 'Red Spots' are also seen in the patch. The patient c) Root resorption d) Diffuse radiolucency
most likely is a: (KAR -91)
a) Pipe smoker b) Cigar smoker 38. Kaposi's sarcoma is a tumour of:
c) Snuff chewer d) Tobacco chewer a) Blood vessels b) Reticuloendothelial system
(AIPG -90, 91,01) c) Striated muscles d) Smooth muscles
27. The most reliable single histologic criterion for diagnosis (AIPG -95)
of oral squamous cell carcinoma is: 39. Acanthosis with intraepithelial vacuolation and
a) Invasion b) Degeneration hyperparakeratosis is seen in:
c) Pleomorphism d) Encapsulation a) Hairy tongue (lingua villosa)
(AIPG -92) b) Hyperplastic candidiasis
28. Which of the following has been implicated due to c) Speckled leukoplakia d) Oesquamative gingivitis
sunlight (AIPG -95)
a) Basal cell carcinoma b) Lymphoepithelioma 40. Increased incidence of squamous cell carcinoma of the
c) Junctional nevus d) Verruca vulgaris skin is due to all except:
(AIPG -01) a) Ultraviolet radiation b) Actinic keratitis
29. Commonest site for carcinoma of tongue is: c) Alcohol d) None of the above
a) Lateral margin b) Tip of tongue (AIPG -99)
c) Posterior 1/3 d) Ventral surface 41. Squamous cell carcinoma with best prognosis is:
(KAR -01) a) Lip b) Tongue
30. Which of the following viruses are not implicated in c) Palate d) Floor of the mouth
human malignancies: (AIPG -96)
a) Epstein-Barr virus b) Herpes simplex virus type I 42. Which of the following does not have a viral etiology?
c) HTLV 1 d) Papilloma virus a) Burkitt's lymphoma b) Nasopharyngeal carcinoma
(AIPG -99) c) Hodgkin's lymphoma d) Hepatocellular carcinoma
31. A non-painful, slowly enlarging benign neoplasm appears (AIPG-96)
as a submucosal lump and exhibits pseudoepitheliomatous 43. Sarcoma of the soft tissues spread by:
hyperplasia in the overlying epithelium, is most probably: a) Blood vessels b) Lymphatics
a) Fibroma b) Rhabdomyoma c) Direct invasion d) Local infiltration
c) Granular cell tumour d) Papilloma (AIPG -96)
(AIIMS -2K, AIPG -91) 44. Which of the following is not a feature of torus
32. Which of the following conditions is characterized by mandibularis?
cafe-au-lait spots, non-encapsulation and potential for a) Common in Mongoloids.
malignant transformation: b) Present on the lingual surface of mandible below the
a) Neurilemmoma b) Neurofibroma mylohyoid line.
c) Traumatic Neuroma d) Solitary plasmocytoma c) Usually Bilateral.
(AIPG-97, 98) d) May or may not associated with torus palatinus.
33. Oral hairy leukoplalda is seen in which of the following (AIPG -04)
conditions? 45. Brachytherapy is:
a) AIDS b) Hepatitis B a) Irradiation of tissues from a distance
b) Irradiation of tissues from a distance of 3 cm

23) B 24) D 25) B 26) A 27) A 28) A 29) A 30) B 31) C 32) B 33) A 34) D 35) D
36) D 37) A 38) A 39) C 40) D 41) A 42) C>N0NE 43) A 44) B 45) C
Dental 9'ul-6e
c) Irradiation of tissues by implants within the tissues d) Ground-glass appearance
d) Irradiation of tissues by radiopharmaceuticals (AIPG -90, AP-05)
(AIIMS-99,01) 52. Features characteristic of leukoplakia include all except:
46. In which of the following type of tumour the HPV-6 can a) Hyperkeratosis.
be detected? b) Plasma cell infiltration within the dermal papillae.
a) Papilloma b) CEOT c) Clinically, a paint like patch.
c) Sarcoma d) Pyogenic Granuloma d) A moist shiny lesion.
(KAR -03) (KAR -97)
47. Hemangiopericytoma, resembles: 53. A patient is diagnosed of oral cancer of stage T3 N 2M0
a) Hemangioma b) Glomous tumour a) Surgery b) Surgery +radiotherapy
c) Ewings tumour d) Plasmacytoma c) Chemotherapy alone d) Surgery+ chemotherapy
(KAR -97) (AIIMS -99)
48. Which of the following epithelial changes commonly 54. Metastases from carcinoma of tongue by blood stream is
signify precancerous condition: more likely when the carcinoma involves the
a) Dyskeratosis b) Hyperkeratosis a) anterior third b) middle third
c) Parakeratosis d) Acanthosis c) posterior third d) lateral margin
(AIPG -01) (KAR -99)
49. A 20yr old patient reports with multiple swelling of the 55. Most common site of oral leukoplakia is
jaws. Clinical examination reveals multiple hard swellings a) angle of mouth b) cheek mucosa
involving the jaws and intra orally several missing teeth c) Soft palate. d) Gingiva
are noticed. Panoramic radiograph reveals multiple (KAR -99)
radio opaque lesions in the maxilla and the mandible 56. Papillomatous tongue is observed in
with multiple impacted teeth and supernumerary teeth a) Lymphangioma
indicative of Gardeners' syndrome. b) Hyalinia cutus et mucosa syndrome
c) Fetal face syndrome d) Tuberous sclerosis
A) The above mentioned condition is (AP-ZK)
a) Autosomal dominant disease 57. Commonly involved lymph nodes during metastasis from
b) Autosomal recessive disorder carcinomas of oral cavity is
c) X-linked disease a) jugulo omohyoid and jugulo digastric
d) A syndrome of unknown cause. b) Sub mandibular and cervical Lymph nodes
c) Submental and submandibular lym phnodes
B) One of the following is also a manifestation in the d) none of the above
above mentioned condition. (AP -2001)
a) Cafe-au-lait pigmentation of the skin 58. A 3 cm squamous cell carcinoma of the retromolar trigone
b) Multiple epidermoid or sebaceous cyst of the skin and invading the mandible and the medial pterygoid
c) Basal cell carcinonna of the skin muscle is at what TNM stage:
d) Peri oral dermatitis. a) Stage I b) Stage II
c) Stage III d) Stage IV
C) One of the following manifestations in Gardener's {PGI -01)
syndrome has a risk of malignant transformation. 59. Asymmetric widening of the periodontal ligament around
a) Multiple polyposis of large intestine two or more teeth is seen in
b) Osteomas of the jaws a) metastatic breast carcinoma
c) Desmoid tumors of the skin b) osteosarcoma
d) Epidermal and trichilemmal cysts c) Paget's disease d) Fibrous dysplasia
(AIIMS -ZK)
D) Multiple impacted teeth as seen in Gardener's 60. Storiform pattern of fibrous tissue is seen in
syndrome may also be seen in one of the following. a) Fibrosarcoma
a) Tricho-dento-osseous syndrome b) Malignant fibrous histiocytoma
b) Hereditary intestinal polyposis c) Neruofibroma d) Ameloblastic fibroma
c) Cowden's syndrome d) Rubinstein- Taybi syndrome (KAR -03)
(COMEDK-2011, 2013) 61. Increased incidence of carcinoma is observed with
50. Leukoplakia with the worst prognosis is seen on the: a) Homogenous leukoplakia
a) Dorsum of tongue b) Floor of mouth b) Verrucous leukoplakia
c) Buccal mucosa d) Palate c) Nodular leukoplakia d) None of the above
(AIPG -01) (AP -2K)
51. Osteosarcoma presents a radiographic picture resembling: 62. Persons with the greatest risk of oral cancer:
a) Sun-burst appearance b) Cotton -wool appearance a) Have a poor oro-dental hygiene, nutritional deficiencies
c) Soap-bubble appearance and are chronic alcoholics

46) A 47) B 48) A 49A) A 49B) B 49C) A 49D) A 50) B 51) A 52) B 53) B 54) C 55) B
56) A 57) B 58) D 59) B 60) B 61) C 62) C
_r ORAL PATHOLOGY & MEDICINE
b) Are middle aged, have a poor oro-dental hygiene and 73. Which one of the following is a connective tissue tumour:
regularly use tobacco a) Lipoma b) Melanoma
c) Are old, Chronic alcoholics and regularly use tobacco c) Carcinoma d) Papilloma
d) Are young, have poor oro-dental hygiene and are heavy (AIIMS -98)
smokers 74. Etiology of multiple neurofibromatosis is:
(AIIMS -90) a) Genetic b) Viral
63. Which among the following shows pseudo- c) Injury d) Endocrine
epitheliomatous hyperplasia? (AIIMS -96)
a) Sq. cell carcinoma b) Basal cell carcinoma 75. Which of the following is most malignant?
c) Verrucous carcinoma d) Granular cell myoblastoma a) Neurolemmoma b) Neurofibroma
(AIIMS- 2K) c) Neurogenic fibroma d) Traumatic neuroma
64. The most common bone cancer is: (AIPG -92)
a) Osteosarcoma b) Metastatic bone cancer 76. Most common route for metastasis of oral cancer is by:
c) Multiple myeloma d) Squamous cell carcinoma a) Direct extension b) Lymphatics
(AIIMS -94, AIPG -99) c) Blood vessels d) Aspiration of tumor cells
65. Which of the following is untrue about malignant (AIIMS -03)
melanoma? 77. Presence of Verocay bodies and having predeliction for
a) Occurs with equal frequency in both sexes occurence in the tongue are seen in:
b) Rare in children a) Granular cell myoblastoma
c) Palate is the most common intraoral site b) Neurilemmoma
d) Is very painful c) Neurofibroma d) Metaplasia
(AIPG -90) (KAR -01)
66. Which of the following is a reactive lesion of the gingiva 78. Presence of epithelial pearls in spinous layer of
that may demonstrate bone radiographically and often epithelium is characteristic of
even microscopically? a) Carcinoma b) Pre-cancer
a) Osteoma b) Peripheral ossifying fibroma c) Dysplasia d) Metaplasia
c) Traumatic neuroma d) Irritation fibroma (AP -03)
(AIIMS -94) 79. Oral submucous fibrosis is diagnosed by
67. Sunlight is one of the etiological causes of: a) Juxta, epithelial fibrosis (Changes)
a) Squamous cell carcinoma b) Changes in epithelium
b) Malignant melanoma c) Changes in submucosa
c) Basal cell carcinoma d) Port-wine stain d) All of the above
(AIIMS -94) (KAR -97)
68. White rough pedunculated lesion on palate is most likely: 80. Best results are obtained in oral submucous fibrosis is by:
a) Pleomorphic adenoma b) Papilloma a) Intralesional cortisone therapy, with hyaluronic acid
c) Nevus d) Fibroma b) Oral cortisone with intralesional therapy (cortisone)
(AIIMS -95) c) Vitamin E and oral cortisone
69. Russell's bodies are found in: d) Intralesional placental extract
a) Activated macrophages b) Histiocytes (KAR -97)
c) Plasma cells d) Erythrocytes 81. All of the following are precancerous conditions except:
(AIIMS- 99) a) SLE b) Peutz-Jeghers syndrome
70. Hodgkin's disease is considered to be: c) Plummer-Vinson syndrome
a) Follicular reticulosis b) Inflammatory disease d) Xeroderma pigmentosum
c) Chronic granulomatous disease (PGI -98)
d) A malignant neoplasm 82. Carcinoma of the cheek:
(AIIMS- 99) a) Characteristically is columnar-celled.
71. Which of the following is carcinoma of the skin, spreads b) Has a recognized association with chewing gum.
by local invasion and has no tendency to metastasise? c) Has a recognized association with chewing betel nut.
a) Malignant melanoma b) Basal cell carcinoma d) Has a recognized association with geographic tongue.
c) Fibrosarcoma d) Leukoplakia (KAR -03)
(AIIMS - 92) 83. The most common site of metastasis from the mandibular
72. A man who had suffered from displaced fracture of sarcoma is
mandible complains of pain in-old fracture site near a) Lung b) Liver
mental foramen. There is a movable tender mass on c) Spleen d) Heart
palpation in the area. The best preoperative diagnosis is: (COMEDK - 03)
a) Neuroleiomyoma b) Neurofibroma 84. A patient complains of numb1ness of lower lip. There is
c) Trigeminal neurallgia d) Traumatic neuroma no history of tooth extraction. It could be
(AIIMS - 92) a) Infection b) metastatic neoplasia

63) D 64) C 65) A 66) B 67) C 68) B 69) C 70) D 71) B 72) D 73) A 74) A 75) B
76) B 77) B 78) A 79) D 80) A 81) A 82) C 83) A 84) B
Dental 9'ul-6e
c) dental Manipulation d) CNS disease 96. On clinical examination a 60 years old female had a
(AIIMS -2K) tumor in the right buccal mucosa. The size of the tumor
85. The most common malignant tumor of the gingiva is: was about 2 cm in diameter. There was no involvement of
a) Malignant melanoma b) Sarcoma regional lymph nodes and also had no distant metastasis.
c) Fibre-sarcoma d) Squamous cell carcinoma The TNM stage of the tumor is:
(KAR -02) a) Tl No Mo. b) Tl Nl Mo.
86. Swelling and redness of the orifices of minor salivary c) Tl N2 Mo. d) T2 No Mo.
glands of the palate occurs in: (AIPG -04)
a) Nicotinic stomatitis b) Leukoplakia 97. Which of the following is a pseudo cyst?
c) Fovea palatine d) Erythroplakia a) Mucous retention cyst b) Radicular cyst
(AIIMS - 99) c) Aneurysmal bone cyst d) Dentigerous cyst
87. Osteosarcoma of the jaw: (COMEDK -04)
a) Occurs mostly in the maxilla 98. Which of the following is benign in nature
b) Seen in old age a) Lymphoma b) lynnphangioma
c) Highly malignant tumour which shows early metastasis c) Melanoma d) leukemia
d) Shows a soap bubble type of radiolucency in radiographs (KAR - 97)
(AIPG - 01) 99. Leiomyoma is a tumor of
88. Tumour that doesn't show spontaneous regression: a) cerebral tissue b) smooth muscle
a) Neuroblastoma b) Retinoblastoma c) striated muscle d) cardiac muscle
c) Malignant melanoma d) Osteosarcoma (KAR -99)
(AIPG -01) 100. 'Onion Skin' appearance of radiographs is seen in
89. Cancer which most coimmonly metastasizes to jaw bone is: a) fibrous dysplasia b) osteosarcoma
a) Breast b) Prostatic c) Ewing's sarcoma d) Chondrosarcoma
c) Lung d) Kidney (KAR -98)
(PGI -95) 101. What could be the most appropriate provisional diagnosis
90. Melanoma is for multiple nodular exophytic reddish lesions of oral
a) Benign tumour of skin and mucous membrane mucosa in an AIDS patient?
b) Malignant tumour of skin and mucous membrane a) Hemangioma b) Focal epithelial hyperplasia
c) Malignant tumour of melanophores c) Acute pseudo-membranous ca.ndidiasis
d) Benign tumor of melanophores d) Kaposi's sarcoma
(PGI - 2003) (UPSC-2001)
91. An epithelial lining is typically found in all of the 102. Teratoma is best describad as:
following except: a) Hamartous developmental manifestation consisting of
a) Sinuses b) Fistulae three different layers
c) Aneurismal bone cysts d) Keratocytes b) Occurs most commonly in stomach and lungs
(AIIMS - 93) c) Originates from mesodermal tissue
92. Acanthosis is: d) Associated with very painful condition
a) Increase in mitotic division (PGI-05)
b) Increase in thickness of superficial layer 103. In TNM classification T3 stands for tumour size:
c) Increase in thickness of spinous layer a) >2 cm b) >4 cm
d) Disruption of basal lamina c) <4 cm
(AIIMS - 95, 96) d) >4 cm with invasion of adjacent structure
93. Which of the following statements is true? (AIPG-06)
a) Kaposi sarcoma is a benign tumour 104. Multiple punched out lesions are seen in:
b) Kaposi sarcoma is found in HIV positive subjects a) Paget's dissease b) Osteosarcoma
c) Kaposi sarcoma is malignant fast growing tumour c) Ewing sarcoma d) Multiple myeloma
d) Kaposi sarcoma is an ectodermal derivative. (AIIMS-06)
(TNPSC -99) 105. Which of the following is the diagnostic characteristic of
94. Tobacco usage has been associated with peripheral giant cell granuloma?
a) Hyperkeratosis b) Erythema a) Mass of granulation tissue
c) Ulceration d) None of the above b) Multinuclear giant cells
(APPSC -99) c) Keliod like enlargement
95. Metastatic disease to the oral region is most likely to d) Epithelium is atrophic in some areas
occur in which of these locations? (AIPG-06)
a) Tongue b) Posterior maxilla 106. Small palpable mass elevated above the epithelial
c) Posterior mandible d) Floor of the mouth surface is
(APPSC -99) a) Papule b) Macule

85) D 86) A 87) C 88) D 89) A 90) B 91) C 92) C 93) B 94) A 95) C 96) A 97) C
98) B 99) B 100) C 101) D 102) A 103) B 104) D 105) B 106) A
c) Plaque
r
d) Vesicle
ORAL PATHOLOGY & MEDICINE
c) Afflicted by trauma d) Hormonal disturbance
(KCET-2011) (AIPG-05)
107. A lesion composed of microscopic vessels is called as 119. Benign tumour of voluntary muscle
a) Haemangioma b) Angioma a) Leiomyoma b) Rhabdomyoma
c) None of Haemangioma and Angioma c) Rhabdomyosarcoma d) Leiomyosarcoma
d) Haemangioma and Angioma (AIPG-05)
(KAR-06) 120. The most common bone tumor that occurs in children is:
108. All of the following malignancies metastasize except a) Osteosarcoma b) Ewing's sarcoma
a) Basal cell carcinoma b) Adenocarcinoma c) Metastatic carcinoma d) Multiple myeloma
c) Squamous cell carcinoma (AIIMS-06)
d) Melanoma 121. Which of the following statements is true for MEN type 1
(KAR-06) a) There is hyperplasia of parathyroid gland
109. Among pre-malignant oral lesions b) Chromophobe adenoma of pituitary gland may result in
a) Leukoplakia should be proved by biopsy acromegaly
b) Leukoplakia does not disappear even after cessation of c) Pancreatic tumours may produce gastrin, insulin
smoking glucagon, somatostain
c) Erythroplakia has a higher risk for malignancy than d) All the above
leukoplakia (COMEDK-07)
d) Oral submucous fibrosis is seen in all parts of the world 122. Peripheral giant cell granuloma occurs most commonly
(KAR-04,COMED-05) on the
110. In which one of the following perineural invasion in a) Gingiva b) Alveolar ridge
head and neck cancer is most commonly seen? c) Palate d) Floor of the mouth
a) Adenocarcinoma b) Adenoid cystic carcinoma (COMEDK-07)
c) Basal cell adenoma d) Squamous cell carcinoma 123. Squamous papilloma is induced by -
(AIPG-05) a) EBV b) HSV
111. Antoni type A and type B are seen in c) HPV d) CMV
a) Neurofibroma b) Neurilemmoma (KCET-09)
c) Neurofibrosarcoma d) Traumatic neuroma 124. Which of the following conditions is least likely to
(AP-2011) present as an eccentric osteolytic lesion:
112. Mobility of teeth in carcinoma of Maxillary sinus is due a) Aneurysmal bone cyst b) Giant cell tumour
to involvement by tumor of c) Fibrous cortical defect d) Simple bone cyst
a) Anterior wall of the sinus (AIPG-06)
b) Posterior wall of the sinus 125. White lesion with definite precancerous potential
c) Roof of the sinus d) Floor of the sinus a) Leukoplakia b) Discoid lupus Erythematosus
(COMEDK-2011) c) Lichen Planus d) All of the above
113. Kaposi's sarcoma is more commonly seen in patient with (KCET-08)
a) AIDS b) Amyloidosis 126. Ossifying fibroma manifests as:
c) Leukemia d) HSV infection a) Unilocular radiolucency b) Unilocular radio opacity
(KAR-04) c) Multilocular radiolucency
114. Arecanut chewing is aetiological factor in d) Multilocular radio opacity
a) Leukoedema b) Oral submucous fibrosis (AIIMS-07)
c) Erythema multiforme d) Oral lichen planus 127. Phleboliths are seen in
(KAR-04) a) Dental Pulp b) Salivary Gland duct
115. The most common benign tumor occurring in oral cavity is: c) Cavernous Hemangiomas
a) Papilloma b) Fibroma d) Subepithelial Bulla
c) Adenoma d) Epulis (KCET-08)
(AIPG-06) 128. The condition involved with an unerupted tooth or
116. The tissue of which lesion has been described as impacted tooth is
resembling a 'blood soaked sponge with large pores'? a) Dentigerous cyst b) Mural ameloblastoma
a) Cavernous hemangioma b) Capillary hemangioma c) Adenomatoid odontogenic tumor
c) Aneurysmal bone cyst d) Eruption hematoma d) All of the above
(COMEDK-05) (AP-08)
117. Port wine stain is characteristic feature of 129. Hemosiderin particles are seen histologically in case of
a) Peutz Jegers Syndrome b) Sturge Weber Syndrome a) Fibroma b) Pregnancy tumor
c) Albright's Syndrome d) Lymphangioma c) Peripheral giant cell granuloma
(COMEDK-05) d) Papilloma
118. Hamartoma is (KCET-08)
a) Neoplastic b) Non-neoplastic

107) D 108) A 109) C 110) B 111) B 112) D 113) A 114) B 115) B 116) C 117) B 118) B 119) B
120) B 121) D 122) A 123) C 124) D 125) A 126) B>A 127) C 128) D 129) C
Dental 9'ul-6e
130. White, spongy, folded thick mucosa is seen in: c) Burkitt's lymphoma d) None of the above
a) Oral hairy leukoplakia (AIPG-09)
b) Aspirin burn 142. Oral ulceration resembling apthae are encountered in -
c) White spongy nevus a) Gluten enteropathy b) Chronic smokers
d) Pseudomembranous Candidiasis c) Excess of B-complex d) Anti-malarial medication
(AIIMS-07) (COMEDK-09, 11)
131. Which of following is an oral precancer? 143. The following lesion is a doubtful premalignant lesion
a) Oral hairy leukoplakia b) White spongy naevus for oral malignancy
c) Leukemia d) Speckled Leukoplakia a) Leukoplakia
(AIIMS-07) b) Chronic hyperplastic candidiasis
132. Tumor represented T 3N 2M 0 is classified as : c) Erythroplakia
a) Stage I b) Stage II d) Oral lichen planus
c) Stage III d) Stage IV (KCET-09)
(AIIMS-07) 144. Hairy leukoplakia is associated with all of the following
133. Tumors metastatic to the jaw bones are most likely to be EXCEPT
found in the a) Filiform to flat patch on lateral tongue
a) Mandibular condyle b) Posterior moral region b) Bilateral appearance
c) Anterior maxilla d) Maxillary tuberosity c) Epstein Barr virus d) AIDS
(BHU-07) (KAR- 2013)
134. Sturge Weber syndrome is characterized by all EXCEPT: 145. Definitive diagnosis of oral cancer is made by-
a) Exophytic oral hemangioma a) Complete radiographic survey
b) Facial hematoma b) Biopsy
c) Tramline calcification of dura on lateral cephalogram c) Exfoliative cytology d) Pantograph
d) Facial hemangioma (KCET-09)
(MCET-07) 146. Presence of Epstein Barr virus in hairy leukoplakia can
135. A patient with Carcinoma cheek has tumor of 2.5 ems be demonstrated using following methods EXCEPT -
located close to and involving the lower alveolus. A single a) Tzanck smear b) Polymerase chain reaction
mobile homolateral node measuring 6 cm is palpable. c) In situ hybridization d) Electron microscopy
Based on these clinical findings TNM stage of the tumor is (KCET-09)
a) Tl Nl Mo b) T2 N2 Mo 147. Diffuse erythema (pin-point petechiae) and white
c) T3 Nl Mo d) T4 N2 Mo patches in smoker's palate occur due to
(COMEDK-10) a) Complete obstruction to minor salivary gland orifices
136. Clear cells are seen in b) Keratinization with partially occluded minor salivary
a) Chondrosarcoma b) Osteosarcoma glands
c) Fibrosarcoma d) Ewing's sarcoma c) Diffuse parakeratinization of the palate
(COMEDK-10) d) Excessive orthokeratinization of the palate
137. Commonest site of carcinoma of tongue - (AP-10)
a) Posterior one third b) Ventral surface 148. Lateral skull view of palatal tori is used to know
c) Tip of tongue d) Lateral margin a) Extent b) Pneumatization
(KCET-09) c) Bone pattern d) Location
138. Neoplastic transformation in leucoplakia is seen most (AP-2011)
commonly in - 149. Histochemical demonstration of glycogen in the cells can
a) Buccal mucosa b) Floor of mouth help in the diagnosis of
c) Lateral border of tongue a) Malignant melanoma b) Squamous cell carcinoma
d) palate c) Kaposi's sarcoma d) Ewing's sarcoma
(MCET-10) (AP-2012)
139. Brachytherapy means 150. Benign tumor which shows metastasis
a) Chemotherapy a) Warthin's tumor b) Ameloblastoma
b) Radiation administered interstially through catheters c) Keratocanthoma d) Neurofibroma
c) Lasers (COMED-2012)
d) Radiation involving normal tissues 151. On biopsy report of CGCG on the basis of histologically &
(COMEDK-10) morphological similarities differential diagnosis is made
140. Keratoacanthoma is found most commonly on? between:
a) Lip b) Gingiva a) Fibrous dysplasia b) Hyperparathyroidism
c) Tongue d) Hard palate c) Osteitis deformans d) Hyperthyroidism
(AIPG-09) (AIIMS-2011)
141. Plasma cell tumour of bones with B-lymphocytic origin is? 152. Which of the following is not a type of leukoplakia:
a) Plasmacytoma b) Multiple myeloma a) Ulcerative b) Bullous

130) C 131) D 132) D 133) B 134) B 135) B 136) A 137) D 138) B 139) B 140) A 141) A&B 142) A
143) D 144) A 145) B 146) A 147) B 148) C 149) D 150) B 151) B 152) B
c) Speckled
r
d) Homogenous
ORAL PATHOLOGY & MEDICINE
d) Infectious mononucleosis
(PGI-2011) (GCET-14)
153. Fibroma combined with glandular tissue is 160. Elephantiasis neuromatosa is a feature of
a) Neurofibroma b) Fibroadenoma a) Von Recklinghausen's disease
c) Fibrolipoma d) Fibromyoma b) Neurilemmoma
(COMEDK- 2013) c) Paraganglioma
154. Treatment for verrucous carcinoma is: d) Multiple endocrine neoplasia syndrome
a) Surgery+ radiotherapy b) Excision (AP-14)
c) Laser technique d) Alpha (a) interferon 161. In a child on lower lip exophytic growth, finger like
(NEET-2013) projections and hyperkeratosis, same type of growth on
155. Which of the following is false about ossifying fibroma? dorsal surface of left thumb which is shorter than right
a) Affects people in 3-4 decade of life thumb is?
b) Maxilla affected more than the mandible a) Squamous cell carcinoma
c) Circumscribed radio opacity b) Acantholysis
d) Continuous growth c) Papilloma d) Verrucous carcinoma
(KAR- 2013) (AIPG-14)
156. Which of the following is NOT true of Sturge Weber 162. In screening of oral cancer not true about exfoliative
syndrome? cytology is?
a) Hemangiomatous involvement of skin a) Keratin interrupts cell collection
b) Mostly bilateral b) It is not very good diagnostic tool
c) Port wine nevus c) Greater numbers of cells are not superficial
d) Vascular gingival hyperplasia d) Cells include trans-epithelial cells
(KAR- 2013) (AIPG-14)
157. To study the malignant tumour of the jaw bone, the 163. Characteristic histopathological feature of basal cell car
radio-nuclide used is cinoma is
a) Gallium 67 citrate a) Nuclear palisading b) Keratin pearls
b) Technitium 99m labelled iodine c) Psammoma bodies d) Foam cells
c) Technitium 99m ·per technitate (AIIMS NOV-14)
d) Technitium 99m methylene diphosphate (MOP) 164. Which of the following occurs in OSMF?
(MCET-14) a) Stomatopyrosis
158. A 70 Year old male complains of ulceration on lower lip b) Vesicles, melanosis, and xerostomia
since 1 moth with lymph node involvement. Patient has c) Blanched pallor hard subepithelial stiffness
history of tobacco chewing since 30 years 6-7 times per d) All
day, patient also complains of loss of weight since 2 (PGI JUNE-2012)
months and change in voice since one month 165. A 40 year old tobacco chewer was given a routine oral
(COMEDK-14) examination; on examination non-scrapable white patch
158A.A reliable clinical condition to consider for the diagnosis was noticed in his left buccal mucosa.
as malignancy is (COMEDK-15}
a) An ulcerative lesion 165A. Provisional diagnosis for the white lesion will be
b) An ulcerative and painful lesion a) Lichen planus b) Erythroplakia
c) Ulceration and fixation of the lymph node c) Leukoplakia d) Submucous fibrosis
d) Bilateral involvement of lymph node
(COMEDK-14) 165B. Diagnosed disease
158B.In this condition the lymph node enlargement may be a) Normal variation of mouth
due to b) Developmental disorder
a) Infection of the malignant lesion c) Premalignant lesion d) Premalignant condition
b) Reactive hyperplasia secondary to infection
c) Infiltration of lymph node by bacteria or virus 165(. Disease can be further evaluated by
d) Infiltration of lymph node by tumor cell a) Toluidine blue solution b) Rose Bengal stains
c) Caleoflaw white stains d) Noneof the above
158C.The patient is com plaining of changes in voice, which
nerve could be affected? 165D. Histological examination will reveal
a) Recurrent laryngeal b) Internal laryngeal a) Normal cells b) Dysplastic cells
c) External laryngeal d) Superficial laryngeal c) Aplastic cells d) Hyperplastic cells

159. 'Pel-Ebstein fever' is seen in


a) Hodgkins lymphoma b) Non-hodgkins lymphoma
c) Lymphogranuloma venereum

153) B 154) B 155) B 156) B 157) D 158A) C 158B) D 158C) A 159) A 160) A 161) C 162) D 163) A
164) D 165A) C 165B) C 165C) A 1650) D
Dental 9'ul-6e � - - - - - -

166. Autoinnoculation is seen with?


a) Ameloblastoma b) Papilloma
c) Benign pemphigoid d) Leukoplakia
(AIIMS MAY-14)
167. Giant cell lesions of periodontium are best considered
as?
a) Benign neoplasms
b) Non-neoplasm reactive lesions
c) Malignant neoplasm d) None
(PGI DEC-2O11)
168. Which of the following is true?
a) Lesser is the numerical value, lesser is the differentiation
b) Grading of tumor is based on metastasis
c) Grade I means poorly differentiated tumor
d) More number of cells, nuclear pleomorphism, decreases
production of keratin shows highly differentiated tumor
(PGI DEC-2O13)
169. Granularity of oncocyte is due to?
a) Overabundance of golgi bodies
b) Overabundance of mitochondria
c) Deficiency of mitochondria
d) Deficiency of golgi bodies
(PGI JUNE-2O11)
170. A surgeon removed section of tissues how would he
confirm that cut margin doesn't contain any suspicious
tissue intraoperatively?
a) Punch biopsy b) Brush biopsy
c) Frozen section d) Exfoliative cytology
(PGI DEC-2O13)

166) B 167) B 168 B 169) B 170) C


_____ r ORAL PATHOLOGY & MEDICINE

2. BENIGN AND MALIGNANT TUMOURS OF ORAL CAVITY - ANSWERS


1. 'C' [Shafer 7th ed 202/ 6th ed 198] • Malignant
schwannoma Malignant tumor of nerve
2. 'A' [Burket's 11th ed 139]
• Olfactory tissue
3. 'A' [Shafer 7th ed 148/ 6th ed 144] neuroblastoma
Hereditary hemorrhagic telangiectasia or Osler-Weber-Rendu
syndrome is an autosomal dominant disorder charecterized 11. 'B' [Shafer 6th ed 91]
by triad of telangiectasia, recurrent epistaxis and a positive Criteria for diagnosis of epithelial dvsplasia in carcinoma
family history for the disorder. The most commonly affected include.
areas of the oral mucosa are lips, gingiva, buccal mucosa • Increased abnormal mitoses
and palate as weel a:s floor of the mouth and the tongue. • Individual cell keratinisation
• Epithelial pearls in the spinous layer
4. 'JJ. [Shafer 7th ed 209/ 6th ed 205]
• Alterations in nuclear Cytoplasmic ratio
5. 'D' [Shafer 7th ed 209-10/ 6th ed 205] • Loss of polarity and disorientation of cells
• Hyper chromatism
6. 'C' [Shafer 7th ed 147/Burket's 10th ed 142] • Large nucleoli

7. 'D' [Shafer 7th ed 187/ 6th ed 182] • Dyskaryosis or nuclear atypism including giant nuclei.
Reed - Sternberg cellls are multinucleated and are considered • Poikilokarynosis or division of nuclei without division of
to be malignant cells in Hodgkin's disease. They are derived cytoplasm
either from B-lymphocyte or monocyte - macrophage. • Basilar hyperplasia

8. 'C' [Shafer 7th ed 102/ 6th ed 100] Individual cell keratinisation arnd epithelial pearl formation
Basal cell carcinoma is 2nd most common cancer of skin, is absent in carcinoma in situ or intra epithelial carcinoma.
mostly involving the middle 3 rd of face (AIIMS MAY 2012).
It is rarely seen in oral cavity. 12. 'B' [Shafer 7th ed 267/ 6th ed 262)
Pseudo cysts are
9. 'B' [Shafer 7th ed 86/ 6th ed 85]
A nevus is congenital, development tumor like malformation • Stafne's cyst
of skin or mucous membrane, composed of nevus cells. • Aneurysmal bone cyst
Common mole is intra dermal nevus. The nevus cells are • Hemorrhagic bone cyst/ traumatic bone cyst
situated within the connective tissue and are not in contact • Mucocele-Extravasation type
with surface epithelium.
13. 'B' [Shafer 7th ed 148/ 6th ed 144]
In junctional nevus, the epithelium is thin and irregular and Hemorrhage telengiectasia is a congenital hereditary
shows cells crossing the junction and growing down into the disease characterized by numerous telengiectatic or
connective tissue. This is called abtropfung or drooping angiomatous areas and common Ly involves the lips, gingiva,
off effect. Because of this junctional activity it undergoes buccal mucosa and palate. Repeated hemorrhages in form of
transformation into malignant melanomas. epistaxis and bleeding from ora.l cavity are earliest signs of
this disease.
10. 'C' [Shafer 7th ed 193/ 6th ed 188]
• Leiomyoma Benign tumor of smooth 14. 'B' [Shafer 7th ed 189/ 6th ed 183]
• Angiomyoma muscle Bence Jones protein is an unusual protein, which coagulates
when urine is heated to 40-60 ° ( and disappears when urine
Benign Tumor of striated
Rhabdomyoma is boiled. It reappears when urine is cooled.
(voluntary) muscle
Malignant tumor of smooth Bence Jones protein is also seen in patients with diseases
Leiomyosarcoma
muscle such as leukemia and polycythemia.
Malignant tumor of striated
Rhabdomyosarcoma 15. 'C' [Shafer 6th ed 92-93)
muscle (AIIMS MAY-2012)
Precancerous lesions (clinical classification)
• Traumatic neuroma
• Leukoplakia
• Neuro fibroma Benign tumor of nerve tissue
• Erythroplakia
• Neurilemmoma
• Palatal keratosis associated with reverse smoking
Dental 9'ul-6e
Precancerous lesions (histological classification) 26. 'A' [Shafer 7th ed 96/ 6th ed 95]
• Squamous epithelial dysplasia The condition is called as Leukokeratosis Nicotina
Palati, (stomatitis nicotiana or pipe smoker's palate)
• Squamous cell carcinoma - in - situ
seen frequently in heavy smokers. The palate develops
• Solar keratosis multinodular appearance with a small red spot in centre
of each nodule representing dilated or partially occluded
Precancerous conditions orifice of accessory palatal salivary gland duct around which
• Sideropenic dysphagia inflammatory cell infiltration is present.
• Lichen planus
27. 'A' [Shafer 7th ed 109/6th ed 105 Fig 2- 19]
• Oral Submucous fibrosis
• Syphilis 28. 'A' [Shafer 7th ed 102/ 6th ed 100]
• Discoid Lupus erythematosus Basal cell carcinoma involves the exposed surfaces of skin
• Xeroderma pigmentosum mostly the middle 3 rd of face (AIIMS MAY 2012) with no
tendency for metastasis because of which it is known as
• Epidermolysis bullosa
"Benign carcinoma". Sunlight(UV Light) is considered as
important etiologic agent. Men are more commonly involved
16. 'A' (Shafer 7th ed 81/ 6th ed 80)
because they are exposed to environment more than women.
17. 'D' [Shafer 7th ed 185/ 6th ed 181]
29. 'A' [Shafer 7th ed 116/ 6th ed 111]
Burkitt's lymphoma/ African Jaw lymphoma is a B - cell
Lateral margins are commonly involved followed by ventral
neoplasm confined exclusively to children between 2 and
surface. Dorsum of tongue is usually involved in patients
14 yrs of children. "Macrophages" are found uniformly
with a history of syphilitic glossitis
throughout the tumor producing the very characteristic
"Starry sky" effect.
30. 'B' [Shafer 7th ed 106, 125, 184, 360/6th ed 80, 81,
180, 334-35]
18. 'B' [Shafer 7th ed 94/ 6th ed 93]
Bowen's disease is a form of intraepithelial carcinoma Virus Malignancy
occurring in patient's who have had arsenical therapy and is HSV - II or
often associated with internal or external cutaneous cancer. Carcinoma of uterine cervix
Genital herpes

19. 'A' [Shafer 7th ed 193/ 6th ed 189] African jaw lymphomas or Burkitt's
HSV - IV or
lymphoma and Nasopharyngeal
Epstein-Barr virus
20. 'A' [Shafer 7th ed 86/ 6th ed 85) carcinoma.
HSV - VIII Kaposi's sarcoma
21. 'C' (Shafer 7th ed US/ 6th ed 109] Papovaviridae
About 88% of lip cancers occurs on lower lip. Cervical, vulvar, penile cancers,
(Human papilloma
squamous cell carcinoma
virus)
22. 'C' [Shafer 7th ed 93/ 6th ed 92-93]
Hepadna viridae
Leukoedema mostly occurs bilaterally and involves the Hepatocellular carcinoma
(Hepatitis B virus)
buccal mucosa along the occlusal line in the bicuspid and
molar region. Leukoedema resembles early leukoplakia but Retroviridae
Adult T-cell leukemia
leukoedema disappears on stretching while leukoplakia (HTL virus)
persists on stretching.
31. 'C' [Shafer 7th ed 194/ 6th ed 184]
The hallmarks of histological features of leukoplakia are
epithelial dysplasia and surface hyperkeratosis. Dysplasia is 32. 'B' [Shafer 7th ed 202)
present more frequently in leukoplakia involving floor of Neurofibroma arises from schwann cells and perineurial
mouth. cells. "cafe - au - lait" spots are seen in neurofibroma and
polyostotic type of fibrous dysplasia.
23. 'B' [Shafer 7th ed 89, 111/ 6th ed 88,93]
Leukoplakia is the most common precancerous lesion 33. 'A' [Shafer 7th ed 360/ 6th ed 354]
whereas Erythroplakia is considered as the most malignant The term 'greenspan lesion' is suggested for 'hairy
precancerous lesion leukoplakia' and it is not premalignant in nature.

24. 'D'

25. 'B' [Shafer 7th ed 118/ 6th ed 113]


The most common sites of metastases are submaxillary
lymphnodes.
34. 'D' [Shafer 7th ed 103/ 6th ed 101]
r ORAL PATHOLOGY & MEDICINE
37. 'K [Shafer 7th ed 209/ 6th ed 206]
Most common malignancy Breast cancer followed
in females by lung cancer 38. 'A' [Shafer 7th ed 167/ 6th ed 163]

Breast cancer followed 39. 'C' [Shafer 6th ed 90 Fig 2- 7]


Most common malignancy
by Carcinoma of uterine
in females in India
cervix {Chennai) 40. 'D' [Shafer 7th ed 102, 107]
Most common malignancy
Lung cancer
in males 41. 'A' [Shafer 7th ed 115/6th ed 109-110]
Most common malignancy Most lip carcinoma's are well-differentiated lesions and are
Oral cancer classified as grade I carcinomatous lesions. They metastasize
in males in India
late in course of disease. So the prognosis is considered as
Basal cell carcinoma good.
followed by squamous
Most common skin cancer
cell and malignant 42. 'C>None' [Shafer 7th ed 184-86/ 6th ed 182]
melanoma. Even EBV could either play a clirect or indirect role in the
Most common oral cancer Squamous cell carcinoma. pathogenesis of Hodgkin's dise.ase, but, the association of
Most deadly of all human EBV with Hodgkin's disease seems to depend on factors such
Malignant melanoma as country of residence, histological subtype, sex, ethnicity,
neoplasms
and age.
Most common benign bone
Osteochondroma
tumor Hodgkin's disease is characterized by bimodal age incidence
Most common bone tumour of peak, one in young adults and second in 5th decade of life.
Osteosarcoma
long bones The etiology is unknown. However, it had been found that
Most common primary 50% of cases are EBV-positive. Patients with HIV also have a
Multiple Myeloma higher incidence of Hodgkin's disease. There is characteristic
malignant bone tumour.
enlargement of one or more cervical Lymph nodes which are
Third most common cancer in
Osteosarcoma "firm and rubbery" in consistency.
adolescence
Most common malignant bone
Osteosarcoma Types of Hodgkin's lymphoma
tumour in India
• Comprises 60-80% of all cases
Most common malignant bone
Ewing's sarcoma • The broad bands of fibrosis divide the
tumour in children
node into nodules
Most common leukemia in Nodular • The characteristic cell is - lacunar type
ALL
children sclerosis reed-sternberg (RS) cell, which has a
Most common leukemia in monolobated or multilobated nucleus
CLL • Usually involves the mediastinum and
elders
other supradiaphragmatic sites.
• Most common malignant
salivary gland tumour • Comprises 15-30 % of all cases
• Most common malignant • RS cells are of classic type (large, with
Mucoepidermoid bilobate, double or multiple nuclei,
salivary gland tumour in
carcinoma and a large eosinophilic inclusion like
children Mixed
• Most common salivary gland cellularity nucleosus).
tumor in bone • Usually involves the abdominal lymph
nodes and spleen.
Most common salivary gland
Pleomorphic adenoma • EBV positivity is higher in this group.
tumor or most common tumor
(Benign tumor)
of Parotid • Comprises <1 % of all cases
nd
2 most common salivary • Associated with older age and HIV
Warthins tumor Lymphocyte­
gland tumor positivity.
depleted
• Large numbeTs of RS cells and bizarre
Most common benign soft sarcomatous variants are present.
Fibroma
tissue neoplasm in oral cavity
• Comprises 5% of all cases
Lymphocyte-
35. 'D' [Shafer 7th ed 98/ 6th ed 97] • RS cells are of classic or lacunar type with
rich
a background infiltrate of lymphocytes
36. 'D' [Shafer 7th ed 188-90/ 6th ed 183] • Comprises 5% of all cases.
Nodular
Multiple myeloma is a malignant neoplasm of plasma cells. • RS cells appear infrequently, instead
lymphocyte­
The prognosis is poor with median survival time of 2-3 yrs. popcorn cells, a variety of lymphocytic
predominant
Infection, anemia and kidney failure are common causes of and histiocytic cells are seen.
death.
Dental 9'ul-6e
43. 'A '[Shafer 7th ed 160/ 6th ed 155] epithelia of the dental organ. It is possible, then, that
The sarcomas differ from carcinomas by their typical the three dental manifestations of the TOO syndrome are
occurrence in young person and their tendency to metastasize caused by the product of mutant gene acting on the dental
through the blood stream rather than lymphatics. epithelia.

All carcinomas metastasize by lymphnodes except renal cell One of the following is seen in tricho-dento-osseous
carcinoma (occurs by hematogenous spread). All sarcomas synodrome (KCET-2012]
metastasize by blood stream except Rhabdomyosarcoma a) Dentinogenesis imperfecta
(occurs by lymphatics). b) Amelogenesis imperfecta
c) Odontogenesis imperfecta
44. 'B' [Shafer 7th ed 159/ 6th ed 155) d) Osteogenesis im perfecta
Torus mandibularis is usually present on lingual surface of
mandible above the myalohyoid line, usually opposite to 50. 'B' [Burket's 11th ed 86]
bicuspid (premolar) teeth. Floor of mouth is the area with worst prognosis followed
by tongue and lips. About 50% of lesions in floor of mouth
45. 'C' [Burket's 11th ed 165-166) shows features of epithelial dysplasia, carcinoma in situ or
invasive carcinoma.
46. 'A' [Shafer 7th ed 81/ 6th ed 80)
H PV types 6 and 11 are commonly associated with squamous
papillomas. Multiple papillomatous lesions are seen in Modified classification for oral leukoplakia
Cowden's syndrome. It is considered as cutaneous marker Ll Size of leukoplakia <2 cm
of breast cancer. L2 Size of leukoplakia 2-4 cm
47. 'B' (Shafer 4th ed 174) L3 Size of leukoplakia >4cm
Staghorn pattern is an important histologic feature of Lx Size not specified
hemangiopericytoma. (AIPG-09) PO No epithelial dysplasia
48. 'A' [Shafer 7th ed 113/ 6th ed 92-93] Pl Distinct epithelial dysplasia
The classic microscopic histopathologic alterations observed Px Dysplasia not specified in the pathology report
in squamous cell carcinoma are:
• Enlarged nuclei as well as cell size OLEP (oral leukoplakia) staging system
• Large and prominent nuclei LlPO
Stage 1
• Increased nuclear/ cytoplasmic ratio
Stage 2 L2PO
• Hyperchromatic (dark staining) nuclei
Stage 3 L3PO (OR) L1L2P1
• Dyskeratosis (premature keratinization of cells)
Stage 4 L3P1
• Increased and / or abberant mitotic activity
51. 'A' (Shafer 7th ed 176/ 6th ed 172]
49A. 'A' (Shafer 7th ed 48/6th ed 47]
Gardeners syndrome is a autosomal dominant disorder, due 52. 'B' (Shafer 7thed 91/6thed 90-91]
to a single pleiotropic gene with complete penetrance and
variable expression. It is characterised by multiple polyposis 53. 'B' [Burket's 11th ed 163]
of large intestine, osteomas of long bones, multiple Usually T 1 and most T2 lesions without metastasis are treated
epidermoid cysts of the skin, occasional desmoids tumors, by either surgery or radiotherapy. Larger T2, T3 and T4 lesions
impacted supernumerary and permanent teeth. are generally treated by combined approach.
49B. 'B' (Shafer 7th ed 48/ 6th ed 47]
Treatment
I nd'icati.on
49C. 'A' [Shafer 7th ed 48/ 6th ed 48) Modality
The intestinal polyps are premalignant and may develop into • For T 1 and T2 lesions.
carcinoma of the large intestine. • For tumors involving bone.
• For tumors that lack sensitivity to
49D. 'A' [Shafer 7th ed 728/ Oral Surgery, Oral Medicine, Oral
Pathology Volume 3 6, Issue 5, November 1973, 693- radiation.
Surgery
700] • For tumors showing recurrence after
The tricho-dento-osseous (TOO) syndrome is an autosomal maximum dose of radiotherapy.
dominant disorder characterized primarily by abnormalities • In cases of clinically positive cervical
of the hair and teeth. The dental anomalies consist of lymph node involvement.
amelogenesis imperfecta, taurodontia, and impacted
teeth. Each of these traits has been attributed to defective
_____ r ORAL PATHOLOGY & MEDICINE

• Used as adjunct to other treatment Definition of the TNM categories of malignancy


modalities to promote initial tumor T-primary tumor
Chemotherapy
reduction and to provide early • TX - Primary tumour cannot be assessed
treatment of micrometastasis • TO - No evidence of primary tumour
• For T 1 and T 2 lesion. • Tis - carcinoma in situ.
• Exophytic, radiosensitive and well • Tl - tumor 2 cm or less in diameter
Radiation • T2 - tumor greater than 2 cm, but not greater than 4cm
oxygenated tumors
in diameter.
• Tumors with small cervical metastasis.
• T3 - tumor greater than 4 cm in diameter.
• Primary treatment modality for • T4 - Tumor invades adjacent structure
i) Brachy therapy
localized tumors in anterior 2/3rds Eg:- through cortical bone, tongue, Max. sinus
(interstitial and
of oral cavity, for boosted doses of
intra cavitary N-Regional lymph nodes
radiation to a specific site and for
implants) • NX - Regional lymph nodes cannot be assessed
treatment following recurrence.
• No - clinically palpable lymph nodes, but metastasis not
• Used in the region of head and neck suspected
by using external beam radiation. • N1 - Metastasis in a single ipsilateral lymph node, 3 cm
• Indicated for primary tumors or less in dimension
of posterior third of tongue • N2a - Metastasis in a single ipsilateral lymph node, >3
oropharynx and tonsillar pillar 0 cm and < 6 cm in dimension
are best treated by external beam • N2b - Metastasis in a multiple ipsilateral lymph node, <
ii) External therapy. 6cm in dimension
beam therapy • External beam therapy with wedged • N2c - Metastasis in a bilateral or contralateral lymph
(Tele therapy) - pair field allows a therapeutic dose node, < 6cm in dimension (KAR- 2013)
to unilateral dose while sparing a • N3 - metastasis in a lymph node , more than 6 cm in
high dose to opposite dose. greatest dimension.
• External beam therapy with a parallel (Note:- midline nodes are considered as ipsilateral nodes)
opposed - field set or three - field - M-Distant metastasis
set-up is used for the tumors present • MX - Distant metastasis cannot be assessed
in midline. • MO - No distant metastasis
Combined • Larger T2, T 3 and T4 lesions. • Ml - clinical and/ or radiographic evidence of metastasis
radiation and
surgery Clinical stage grouping of carcinoma of the oral cavity
54. 'C' [Shafer 7th ed 116/ 6th ed 111] Stage O Tls NO MO
The lesions on posterior past of tongue metastasize earlier Stage I Tl No Mo
with a poorer prognosis because of their inaccessibility Stage II T2 No Mo (AIIMS May 2012)
for treatment and cross lymphatic drainage, which is
contralateral in respect to tongue lesion. Stage III Tl Nl Mo, T2 Nl Mo, T3 Nl Mo, T3 No Mo
T4 NO, Nl MO
55. 'B' [Shafer 4th ed 95) Stage IV ANYT N2, N3 MO
56. 'A' [Shafer 7th ed 151/ 6th ed 146) ANYT ANY N Ml

57. 'B' [Burket's 11th ed 158/ Shafer 6th ed 107] 59. 'B' [Shafer 7th ed 176/ 6th ed 171]
Metastases chiefly involve the sub maxillary and cervical Widening of periodontal ligament is seen in
lymph nodes followed by pre auricular and post auricular • Osteosarcoma
lymph nodes. • Chondrosarcoma
• Scleroderma
58. 'D' [Shafer 7th ed 112/ 6th ed 108)
• Trauma from occlusion

Obliteration of eriodontal Li ament s ace is seen in


• Hypercementosis
• Ankylosis
• Hypo function of teeth
• Paget's disease

60. 'B' [Shafer 7th ed 163/ 6th ed 158]


Dental 9'ul-6e
61. 'C' [Burket's 11th e d 86/Shafer 7th ed 91] NOTE: Pseudoepitheliomatous hyperplasia is also seen in
Clinically Leukoplakia is of three types. • Granular cell myoblastoma
• Blastomycosis
• Appears as translucent white, raised • Papillary hyperplasia
area. • DLE
Homogenous • It is non-palpable i.e., same as 64. 'C' [Shafer 7th ed 188/ 6th ed 183]
surrounding mucous membrane.
• Differential diagnosis is hyperplastic
65. 'A' [Shafer 7th ed 126/ 6th ed 122-23]
lichen planus.
Malignant melanoma is twice as common in men as in
• Raised from surface with red and white women occurring between 40 - 70 yrs of age with a definite
areas. predilection for palate and maxillary gingiva or alveolar ridge.
• Mostly seen at the angle of mouth and
Speckled or commisures of lips in chronic smokers. 66. 'B' [Shafer 7th ed 133/ 6th ed 128]
nodular • Indurations, fissuring and ulcer
furmdliun b ���n. 67. 'C' (Shafer 7th ed 102/ 6th ed 100]
• The epithelial dysplasia is more common
68. 'B' [Shafer 7th ed 81/ 6th ed 80]
and has more tendency for malignancy.
Fibroma is smooth, mostly sessile ,or occasional pedunculated
• First described by Hansen and is lesion.
associated with a high risk of progression
to squamous cell carcinoma. 69. 'C' [Shafer 7th ed 189/ 6th ed 185]
• May be associated with human papilloma
virus (HPV) 70. 'D' [Shafer 7th ed 186/ 6th ed 181]
Proliferative
verrucous • Seen as white papilliferous or cauliflower
like growth 71. 'B' [Shafer 7th ed 102/ 6th ed 100]
leukoplakia
Because of these reasons, basal cell carcinoma is also known
(PVL) • Commonly seen in the region, where the
as benign carcinoma.
quid is kept for long time
• No fixity is seen 72. 'D' [Shafer 7th ed 200/ 6th ed 196]
• Differential diagnosis is verrucous
carcinoma. 73. 'A' [Shafer 7th ed 141/ 6th ed 137]

Note: 74. 'A' [Shafer 7th ed 202/ 6th ed 198]


The forms of leukoplakia according to CURRENT Neurofibromatosis is inherited as simple autosomal dominant
classifications are: trait.
1) Homogeneous: Lesions that are uniformly white.
75. 'B' [Shafer 7th ed 202/ 6th ed 198]
a) Smooth
b) Furrowed (Fissmed) 76. 'B' [Shafer 7th ed 110/ 6th ed 107]
c) Ulcerated Blood stream metastasis from oral cancer is uncommon
2) Nonhomogeneous nodulospeckled: Lesion with
well demarcated raised white areas, interspersed with 77. 'B' [Shafer 7th ed 204/ 6th ed 200]
reddened areas. It is applicable to both color (mixed Refer synopsis
red and white lesion - erythroleukoplakia) and texture
(exophytic, papillary or verrucous) 78. 'A' [Shafer 7th ed 109/ 6th ed 105]

Proliferative verrucous leukoplakia is a term used to describe 79. 'D' [Shafer 7th ed 99/ 6th ed 98]
a clinically aggressive form or oral leukoplakia with a strong
potential for malignant transformation. 80. 'A' [Shafer 7th ed 101/ 6th ed 100]
Treatment of oral submucous fibrosis:
62. 'C' [Burket's 11th ed 154/ Shafer 6th ed 102] • Intralesional injection of dexamethasone sodium phosphate
(2ml) + 1500 LU of hyaluronidase + xylocaine (0.2cc)+
63. 'D' [Shafer 7th ed 194/ 6th ed 190] peripheral vasodilator like Nylindrin hydrochloride).
Granular cell myoblastoma is a benign tumor of muscle tissue • Vit.A supplements of 50,000-1,00,000 LU.
origin, most commonly involving tongue. The epithelium • Placentrex is given in HTN and diabetic patients.
exhibits pseudoepitheliomatous hyperplasia because of
which it is confused with epidermoid carcinoma. 81. 'A' [Shafer 7th ed 111/ 6th ed 106]
Discoid Lupus erythematosus (DLE) is precancerous
condition, not SLE (option 'A'}
Precancerous conditions
_r ORAL PATHOLOGY & MEDICINE
periosteum is created. This is known as codman's triangle
• Siderophinic dysphagia and is highly suspicious for osteosarcoma. (GCET-14)
• Lichen planus
• OSF Osteosarcoma occurs in young person (10 -25 yrs age) chiefly
involves the femur and tibia with sunburst appearance
• Syphilis
radiologically.
• DLE
• Xeroderma pigmentosum 88. 'D' [Shafer 6th ed 172]
• Epidermolysis bullosa
89. 'A' [Shafer 7th ed 209/ 6th ed 206]
82. 'C' [Shafer 7th ed 118/ 6th ed Pg113] The mandible molar area is more affected than maxilla.
Breast cancer commonly metastasizes to jaw bones followed
83. 'A'[Shafer 7th ed 177] by lung, bone or colorectum and kidney.
Staging of osteosarcoma:
1. Stages 90. 'B' [Shafer 7th ed 126/ 6th ed 120]
Stage I- low grade lesions 91. 'C' [Burket's 11th ed 146]
Stage II - high grade lesions Aneurysmal bone cyst is a lesion of young persons mostly
Stage III - metastatic disease occurring in long bones and vertebral column with history
of trauma. Upon entering the lesion there is excessive
bleeding, the blood "welling up" from the tissue.
2. Substages The tissue resembles a blood - soaked sponge with large
A- intramedullary lesion pores representing the cavernous spaces of lesion.
B- local extramedullary
92. 'C' [Burket's 11th ed 77]
3. Site
93. 'B' [Shafer 7th ed 167 / 6th ed 163-64]
Distal proximity - best prognosis According to cofactor model "the combined effects of
Distal femur - intermediate prognosis numerous agents (human herpes virus 8, HIV virus) host
Axial skeleton - worst prognosis factors, and environmental factors encourage Kaposi's
sarcoma proliferation".
In osteosarcoma cumulus cloud densities form within
Early lesion characterized by a proliferation
the intramedullary and soft tissue components caused by
Patch stage of small lesions and capillaries around one
mineralizing tumor osteoid
or more preexisting dilated vessels.
84. 'B' [Shafer 7th ed 209/ 6th ed 206] Increased numbers of small capillaries or
Plaque stage
dilated vascular channels.
Carcinoma with no tendency for
Basal cell arcinoma Nodular stage All the histologic features are prominent.
metastasis
Most common route for metastasis
Lymphatic spread 94. 'A' [Burket's 11th ed 101/ Shafer 6th ed 95]
of oral cancer is by
Commonly involved lymphnodes 95. 'C' [Shafer 7th ed 209/ 6th ed 206]
Submaxillary and
during metastasis from carcinomas
cervical Lymphnodes
of oral cavity is 96. 'A' [Shafer 7th ed 112/ 6th ed 108]
The most common site of metastasis
Lung 97. 'C' [Burket's 11th ed 146]
from mandibular sarcoma is
The cancer which commonly Lung cancer in men
98. 'B' [Shafer 7th ed 151/ 6th ed 146]
metastasize to oral mucosa & jaw and breast cancer in
bones is woman
99. 'B' [Shafer 7th ed 192/ 6th ed 188]
Common site of metastatic Mandibular molar
carcinoma in oral cavity area 100. 'C' [Shafer 7th ed 170/ 6th ed 165]
Periosteal new bone formation or neoperiostosis is seen in:
85. 'D' [Shafer 7th ed 119/ 6th ed 113] • Infantile cortical hyperostosis (Caffey's disease)
• Hypeavitaminosis A
86. 'A' [Shafer 7th ed 96/ 6th ed 95] • Syphilis
• Leukemia
87. 'C' [Shafer 7th ed 17 6/ 6th ed 171]
• Ewing's sarcoma
In osteosarcoma, at the point where the periosteum begins
to merge, an acute angle between the bone surface and the • Matastatic neuroblastoma
• Garre's osteomyelitis
Dental 9'ul-6e
101. 'D' [Shafer 7th ed 168/ 6th ed 163] 113. 'A' [Shafer 7th ed 168/6th ed 352, Table 6-5]

102. 'A' [Shafer 4th ed 312] 114. 'B' [Shafer 7th ed 99/ 6th ed 98]
Teratoma is made up of a number of different types of tissue
which are not native to that area. Teratoma may occur in 115. 'B' [Shafer 7th ed 81/ 6th ed 126)
various parts of body and is made of various epithelial
appendages such as hair, sweat glands, sebaceous glands 116. 'C' [Shafer 7th ed 140/ 6th ed 136]
and salivary glands, etc. The teeth are usually normal and
resemble premolars. Inflammatory gingivitis may be seen. 117. 'B' [Shafer 7th ed 149/ 6th ed 140,145)

103. 'B' [Shafer 7th ed 112/ 6th ed 108) 118. 'B' [Shafer 6th ed 33)

104. 'D' [Shafer 7th ed 189/6th ed 185 Fig 2-9) 119. 'B' [Shafer 7th ed 193/ 6th ed 188)

105. 'B' [Shafer 7th ed 136/ 6th ed 130-31] 120. 'B' [Shafer 7th ed 169/ 6th ed 164)

106. 'A' [Burket's 10th ed St/11th ed 42) 121. 'D' [Shafer 4th ed 201/ 6th ed 196]
Solid lesions raised above the skin surface that are smaller Multiple Endocrine Neoplasia (MEN) Syndrome
than 1cm in diameter are called papules. Solid raised sesions • Hyperplasia of pituitary gland with acromegaly
that are over 1 cm in diameter are called plaque. Macules are
• Hyperplasia of parathyroid and adrenal cortex
well circumscribed, flat lesions that are noticeable because
of change of colour. Vesicles are elevated blisters containing MEN-I • Hyperplasia of pancreatic islets with
clear fluid that are under 1 cm in diameter. So, the best increased production of gastrin, insulin
option given is Option 'ft: papule. and glucagon with peptic ulcers and gastric
hypersecretion
107. 'D' [Shafer 6th ed 143] • Hyperplasia of parathyroid gland
Hemanqiomas are associated with the following syndromes: MEN-II • No tumors of pancreas. No peptic ulcer.
• Rendu-osler-Weber syndrome (Sipple's • Patients may have pheochromo-cytomas of
• Sturge-Weber syndrome Syndrome) the adrenal medulla and medullary carcinoma
• Kasabach-Merritt syndrome of thyroid gland
• Pheochromocytomas and medullary
108. 'A' [Shafer 7th ed 102/ 6th ed 101) carcinoma of thyroid gland.
MEN-III • Oral neuromas that are common on lips,
109. 'C' [Shafer 7th ed 94/ 6th ed 94] tongue and buccal mucosa.
Different studies have demonstrated that 80-90% of
• The lips are described as 'Bumpy lips'
erythroplakias are histopathologically either severe epithelial
dysplasia, carcinoma-i nsitu, or invasive carcinoma.
122. 'A' [Shafer 7th ed 136/ 6th ed 130)
110. 'B' [Shafer 7th ed 238/6th ed 236)
123. 'C' [Shafer 7th ed 81/ Burket's 11th ed 137]
Note: Keratocanthoma also may invade perineural spaces.
HPV associated lesions or conditions are:

111. 'B' [Shafer 7th ed 204/6th ed 200) • Heck's disease


Histopathological picture of neurilemmoma shows Antoni • Proliferative verrucous leukoplakia
type A and Antoni type B. Antoni type A tissue is made • Squamous papilloma
of cells with elongated or spindle shaped nuclei which are
• Carcinoma of cervix
aligned to form a palisading pattern. Intercellular fibers
are arranged in parallel fashion between rows of nuclei. • Condyloma acuminatum
Antoni type B tissue shows disorderly arrangement of cells • Verrucous vulgaris (common wart of skin)
and fibers. Verocay bodies are small hyaline structures • Molluscum contagiosum (caused by POX group virus with
characteristically seen in this tumor. characteristic Henderson-Paterson or molluscum bodies)
• Oral squamous cell carcinoma
112. "D' [Shafer 7th ed 121/6th ed 116)
The clinical features of the neoplasm depend on the
The high-risk. oncogenic subtypes that are commonly
direction of spread of the neoplasm or the involvement
detected in oral squamous cell carcinomas are primarily HPV
of the surrounding structures. The floor of the sinus is
16 followed by HPV 18, 31, 33, 35.
associated with oral structures like mobility of teeth. Medial
wall involvement causes nasal obstruction. Superior wall
124. 'D' [Shafer 7th ed 530]
involvement produces displacement of the eyes. Lateral wall
involvement produces bulging of the cheek.
125. 'A' [Shafer 7th ed 111/ 6th ed 106]
r ORAL PATHOLOGY & MEDICINE
characteristically manifests encephalotrigeminal
Though all the above mentioned conditions are either angiomatosis. The patient may give history of seizures and
classified as premalignant lesion (Leukoplakia) or the skull radiograph may disclose vessel wall calcifications
premalignant conditions (LP and OLE), the malignant that yield bilamellar radiopaque tracks referred as
transformation with LP & OLE occur commonly with the "tram line" calcifications.
erosive form and the atrophic scars respectively, both of
which appear red irn colour. Hence the white lesion with Thrombosis is commonly encountered in hemangioma. These
definite precancerous potential is leukoplakia. thrombi may eventually calcify, turns radiopaque and known
as phleboliths.
126. 'B>A' [Shafer 7th ed 135/6th ed 128-30]
• Mandible is commonly involved 128. 'D' [Shafer 7th ed 259, 282, 286/6th ed 254-257, 282]
Refer synopsis.
• Displacement of teeth is an early clinical
Central feature
ossifying 129. 'C' [Shafer 7th ed 137/6th ed 131]
fibroma of • In early stages appear as radiolucent
bone area, which becomes calcified later 130. 'C' (Shafer 7th ed 821/6th ed 813 Fig. 19-17)
• Ulti mately, the lesion appears as a White sponge Nevus
relatively uniform radiopaque mass • Described by Cannon in 1935.
• Common in young below 30 years of age • Follows hereditary pattern as autosomal dominant trait.
Central
• Usually shows no signs and symptoms but • The oral lesions are widespread and appears as thickened
giant cell
sometimes may cause expansion of cortex and folded or corrugated with a soft or spongy texture
granuloma
and perforation, mobility, displacement and a peculiar white opalescent hue.
(and) Giant
and root resorption of associated tooth. • It can be removed sometimes by gentle rubbing without
cell tumor
of bone • MNG cells and hemosiderin pigment are any ensuing bleeding.
also seen.
• Involves gingiva and shows no radiological This condition is due to a defect in the normal
Peripheral features keratinization of the oral mucosa (COMEDK- 2013)
ossifying
• Shows different forms of calcifications a) Erythema Multiformae b) White Sponge Nevus
fibroma
histologically c) Incontinentia Pigmenti d) Systemic Sclerosis
• Involves gingival or alveolar process most
131. 'D' [Shafer 7th ed 91, 95, 111/6th ed 106]
frequently anterior to molars
• Seems to originate from either periodontal • Oral hairy leukoplakia - viral in origin
ligament or Mucoperiosteum • White sponge naevus - genetic in origin
• Histologically shows MNG cells, which • Leukemia - A malignant condition
resemble osteoclasts • Speckled leukoplakia - Oral pre-cancerous lesion high
• Numerous capillaries, Foci of hemorrhage carcinomatous change
with liberation of hemosiderin
pigment, and its subsequent ingestion 132. 'D' [Shafer 7th ed 112/6th ed 108]
Peripheral by mononuclear phagocytes, as well
giant cell as inflammatory cell infiltration are 133. 'B' [Shafer 7th ed 209/6th ed 205]
grauloma characteristic features. Metastatic tumors of the jaw:
• The giant cells might be derived from • In the skeleton, bones with red bone marrow are the
proliferating giant cells associated with preferred sites for metastatic deposits. Remnants of
resorption of deciduous tooth roots and red bone marrow can be detected in posterior areas
this would account for the predominance of mandible. That is why the metastasis is common to
of the lesions anterior to the permanent posterior areas of jaws.
molars. • Metastatic tumors to the oral region are uncommon and
• Radiologically it exhibits superficial account for approximately 1 % of malignant oral tumors.
erosion of bone with pathognomonic On average patients with metastases to the jaw bones are
"peripheral cuffing" of bone younger than those with metastases to oral soft tissues.
• In dentulous patients, 80% of the metastatic tumors to
127. 'C' [Burket's 10th ed 127/White & Pharoah 6th ed 532] oral soft tissue mucosa are found in attached gingiva,
Most oral hemangio,mas are located on the tongue, but whereas in edentulous patients, the metastatic lesions
lip mucosa is another common site for hemangiomas in are equally distributed between the tongue and the
children. Most hemangiomas are raised and nodular, but alveolar mucosa.
some may be flat, macular and diffuse, particularly on • The breast is most common primary site for tumors that
the facial skin, where they are referred to as port-wine metastasize to jaw bone, whereas the lung is the most
stains. Sturge-Weber syndrome, a variant of hemangioma,
Dental 9'ul-6e
common source foir cancers that metastasize to the oral 139. 'B' [Burket's 11th ed 165, 166]
soft tissues. In Brachy therapy, interstitial and intracavitary implants
• Also, the breast is the most common primary cancers are used to treat primary cancers in head and neck. It is
that metastasize to the oral region in females while it is primary treatment modalities for localized tumors in the
lung carcinomas in males. anterior 2/3rds of oral cavity, for boosted doses of radiation
• to a specific site, or for treatment following recurrence. The
Breast carcinoma
isotopes used include cesium, iridium and gold.
- Commonly metastasize to jaw bones.
- Commonly primary cancers that metastasize to the Directly implanted sources may be used to deliver radiation,
oral region in females. or an after loading technique may be used in which the
• Lung cancers radiation source is placed by using previously inserted
catheters or guide tubes.
Common metastasize to oral soft tissues.
- Common primary cancers that metastasize to oral 140. 'A' (Shafer 7th ed 83/6th ed 82]
region in males. Keratoacanthoma (self healing carcinoma, molluscum
pseudo-carcinomatosum, molluscum sebaceum) is a
134. 'B' (Shafer 7th ed 149, 150/Burket's 10th ed 127, 147] relatively common low-grade malignancy that originates in
Though the skin and oral lesions of hemangioma are most the pilosebaceous glands. Trauma, HPV virus, genetic factors
deforming and disfiguring, the CNS involvement often and immuno compromised status have been implicated as
results in serious problems of epilepsy, hemiplegia, mental etiologic factors. It occurs twice commonly in men than
retardation and retinal changes. Sturge-weber syndrome is women, usually on sun exposed areas. Lips and the vermillion
probably the most common of these malformations. bolder of both the upper and lower lip are affected with
equal frequency.
It is characterized by angiomatosis of face, laptomeningeal
angiomas, contralateral hemiplegia, massive gingival The clinical course of the lesion is one of its unusual aspects.
growths and asymmetric jaw growth. The patients are It begins as a small firm nodule that develops to full size
treated for many years with phenytoin. over period 4 - 8 wks and then undergoes spontaneous
regression over the next 6-8 wks (self healing carcinoma)
135. 'B' (Shafer 7th ed 112/6th ed 108]
• Tumor size> 2 cms = T2 If spontaneous regression does not occur, the lesion is usually
• Metastasis in a single epsilateral lymphnode more than treated by surgical excision parakeratin or orthokeratin surface
layer with central plugging is important histologic feature.
3cm but not more than 6cm in greatest dimension = N 2
• No distant metastasis = M0 141. 'A&B' [Shafer 7th ed 187,190/6th ed 183]
• So the TNM stage of tumour is T2 N 2 M0 Multiple myeloma is the most common primary neoplasm of
the skeletal system. The disease is a malignancy of plasma
136. 'A' (Shafer 7th ed 172/6th ed 167] cells, which are subset of B-cells.
Clear cells are seen in:
• Chondrosarcoma Plasmacytoma is a discrete, solitary mass of neoplastic
• monoclonal plasma cells in either bone marrow or a soft
Clear cell carcinoma
tissue site. Plasma cells arise from B-cells. Histologically,
• Clear cell acanthoma solitary plasmacytoma and multiple myeloma are similar.
137. 'D' (Shafer 7th ed 116/6th ed 111] 142. 'A' (Shafer 7th ed 765/6th ed 758]
Lateral borders followed by ventral surface of tongue are Celiac Sprue/ Gee-Herter disease/ gluten-sensitive enteropathy
commonest sites of carcinoma of tongue. The prognosis is characterized by malabsorption of ingested fat (steatorrhea)
depends on the site of lesion. The lesions on the posterior resulting in excessive amount of fat in the stools.
portion of tongue are usually of a higher grade of malignancy,
metastasize earlier and offer a poorer prognosis, especially The oral changes are similar to those of pernicious anemia.
because of their inaccessibility for treatment. In rare cases, There is severe glossitis with atrophy of filiform papillae,
carcinoma occurs on the dorsum of tongue (AIIMS MAY although the fungiform papillae often persist for some time
2012 ), usually in a patient with a past or present history of on the atrophic surface. Painful, burning sensations of the
syphilitic glossitis. tongue and oral mucosa are common and small, painful
erosions may occur.
138. 'B' [Burket's 11th ed 86]
Oral leukoplakia may be found at all sites of oral cavity. Tyldesley has reviewed this problem recently and concluded
Non-smokers have a high percentage of leukoplakias at the that there is an association between recurrent aphthous
border of tongue compared with smokers. The floor of mouth ulcers and celiac disease and proper dietary treatment leads
and the lateral boldness of tongue are high risk sites for to remission of the oral lesions.
malignant transformation.
r
143. 'D' [Shafer 7th ed 111, 811 Table 2-9]
ORAL PATHOLOGY & MEDICINE
148. C [Shafer 7th ed 158/6th ed 154]
Clinical and histological classification of precancerous The torus may be made up of dense compact bone or of a
lesions and conditions had not included chronic hyperplastic shell of compact bone with a center of cancellous bone. It
candidiasis, but have included lichen planus. Moreover, the appears as a dense radiopaque shadow below and attached
malignant transformation of candidal leukoplakia has been to the hard palate.
linked to characteristics of speckled leukoplakia rather than
that of candidal superinfection. 149. 'D' [Shafer's 7th ed 170/6th ed 165]
Ewing's sarcoma also called round cell sarcoma, seen in
144. 'A' [Shafer 7th ed 360/6th ed 354] children and young adults, is characterized by painful
Oral hairy leukoplakfa is a painless, faint, white streaks or swelling of the involved bone, facial neuralgia and lip
thickened, furrowed areas with a corrugated appearance paresthesia. It produces an irregular diffuse radiolucency,
seen bilaterally on the lateral borders of tongue, caused onion-skin and sun ray appearance of bone. Histologically,
by Epstein - Barr virus. It is most commonly seen in the neoplasm is composed of solid sheets of round cells,
immunocompromisecl patients like HIV, kidney transplant arranged in Filigree pattern. Intracytoplasmic glycogen of
patients etc. these cells helps in the differentiation from reticulum cell
sarcoma.
145. 'B' [Shafer 6th ed 593/ Burket's 11th ed 162]
150. 'B' [Shafer's 7th ed 276/6th ed 297]
146. 'A' [Burket's 11th ed 84, 85] Traditionally, ameloblastoma has been regarded as
Hairy leukoplakia is the second most common HIV­ a benign tumor that can be locally aggressive, but
associated oral mucosal lesions after candidiasis. It is occasionally can metastasize. Metastatic nodules develop
non-malignant and is not pathognomonic for HIV since in the lung, cervical lymph nodes or extragnathic bones.
other immunodeficiences such as cancer chemotherapy are Ameloblastoma with metastasizing potential (malignant
also associated with Hairy leukoplakia. The condition is ameloblastoma) is histologically indistinguishable from
commonly observed on the lateral borders of tongue in conventional ameloblastomas. Ameloblastic Carcinoma
form of vertical whit,e folds. is a malignant epithelial proliferation that is associated
with an ameloblastoma or histologically resembles an
Diagnosis Of Hairy Leukoplakia ameloblastoma.
Characteristic gross appearance with or 151. 'B' [Shafer's 7th ed 653/6th ed 652]
Provisional
without non-responsiveness to antifungal The histologic appearance of CGCG and hyperparathyroidism
diagnosis
therapy are very similar, with numerous capillaries, RBCs, yellow or
Light microscopy of histologic sections brown hemosiderin, multinucleated giant cells. Therefore,
revealing hyper keratosis, koilocytosis, any patient who has a lesion diagnosed as a Central giant
acanthosis, and absence of inflammatory cell lesion, must be evaluated to rule out the possibility of
Presumptive cell infiltrate hyperparathyroidism.
diagnosis (or)
light microscopy of cytologic operations 152. 'B' [Shafer's 7th ed 91/6th ed 90]
demonstrating nuclear beading and Leukoplakia may be classified into homogeneous (further
chromatin margination divided into smooth, furrowed/ fissured, ulcerated) and
Insitu hybridization of histologic or nonhomogeneous or nodulospeckled. There is no bullous
cytologic specimen revealing positive variant of leukoplakia.
staining for EBV DNA
(or) 153. 'B' [Atlas of Pathology 2nd edition)
Definitive Electron microscopy of histologic or Fibroadenoma is most commonly seen in breast of women
diagnosis cytologic specimens showing herpes-like of child bearing age. It is a lump composed of fibrous and
particles glandular tissue and is a firm, painless mobile mass.
(or)
Epstein-Barr virus was demonstrated with 154. 'B' [Ref: European Journal of Dentistry. 2010 April;
poly1merase chain reaction technique 4(2): 202-207. Oral Verrucous Carcinoma: A Study of 12
Cases]
147. 'B' [Shafer 7th ed 9 6] Surgery is considered the primary mode of treatment for
Stomatitis nicotina or pipe smoker's palate is first manifested verrucous carcinoma. Irradiation alone or in combination
by redness and inflammation of the palate. The palate with surgery is rarely performed. Combined therapy can
develops multinodular appearance with a small red spot be useful when the tumor extends to the retromolar area.
in the center of each tiny nodule, representing the dilated McClure et al reported that extensive lesions in the oral
and sometimes partially occluded orifice of an accessory cavity may benefit from combined therapy. When surgery is
palatal salivary gland duct around which inflammatory cell not indicated, other treatment modalities such as cytostatic
infiltration is prominent. The epithelium around the ducts drugs may be preferred. Various dosages of cytostatic drugs
shows excessive thickening and keratinization. have been proven to show beneficial effects in reducing
tumor size; Alpha-interferon (IFN) seems to support the
Dental 9'ul-6e
therapy by delaying the growth of the tumor but does not and is less frequently associated with obvious peripheral
take the place of surgery alone. neurofibromatosis or oral lesions.

155. 'B' [Shafer 7th ed 13 5/6th ed 130] 161, 'C' [Shafer 7th ed 81]
Ossifying fibroma is more common in young adults, with a Papillomas caused by HPV virus consist of exophytic growth
predilection for occurrence in the mandible and females. made up of numerous small finger like projections resulting
The radiographic features may vary depending on the stage in a cauliflower like roughened surface. The analogous skin
of development. However the lesion is always demarcated. wart is called verruca vulgaris, common on the hands and
Early lesions may be totally radiolucent, with increase in fingers.
calcification in the form of radiographic flecks and ultimately
may appear as a uniform, radiopaque mass. 162. 'D' (Shafer 7th ed 596]
Exfoliative cytology is the study of cells that exfoliate or
156. 'B' [Shafer 7th ed 149/6th ed 144] abrade from the body surfaces. The use of cytological smear
Sturge -Weber syndrome is associated with congenital depends not only on the proper preparation of the smear but
hamartomatous malformations that affect the eye, skin also the experience of the pathologist. Cytological smears
and CNS. It is characterised by venous angioma of the can be inconclusive due to the paucity of the cells collected.
leptomeninges with ipsilateral angiomatous lesions of the Routine exfoliative cytology involves the use of a tongue
face, skull, jaw and oral soft tissues. The port wine nevi blade that collects only superficial epithelial cells and does
(cuatneous capillary-venous angiomas) are usually the first not allow transepithelial collection.
component observed at birth, exclusive to the skin area
supplied by trigeminal nerve. The oral features include 163. 'A' [Shafer 7th ed 102, 109, 144]
angiomatous lesions o,f the gingiva and the buccal mucosa. • In nodular basal cell carcinoma, the tumor cells typically
have large, hyperchromatic,. oval nuclei with little
According to Can J Neurol Sci. 2008 Jul;35(3):301-7; cytoplasm and nests of tumor cells show peripheral cell
Sturge-Weber syndrome: study of 55 patients, the facial palisading.
nevus flammeus was unilateral in 35 (63.5%) patients, • Keratin pearls are found in squamous cell carcinoma,
bilateral in 17 (31%) and absent in 3 (5.5%) of the patients
while Psammoma bodies are concentric lamellated
with leptomeningeal angiomas.
structures found in papillary thyroid carcinoma,
meningioma, papillary serous cystadenocarcinoma of the
157. 'D' [White & Pharoah 6th ed 218]
ovary.
The various radioisotopes used for scintigraphy include
Iodine 131, Gallium 67, Selenium 74, Technetium 99m. The • Foam cells are found in verruciform xanthoma.
most commonly used is Technetium 99m. To image bone, Tc
99m is typically bound to methylene diphosphonate (MDP) 164. 'D' (Shafer 7th ed 98]
and a dose of 20-30 mCi is injected intravenously. The Oral sub mucous fibrosis is a chronic progressive, scarring
phases that follow are flow phase (known as radionuclide disease characterised by burning sensation while consuming
angiography), second blood pool phase and the third bone spicy food (stomatopyrosis), blisters, ulcerations, excessive
scintigraphy phase (2-3 hrs after injection). Most metastatic salivation/ xerostomia, blanching of the oral mucosa, and
tumors in bone induce new bone formation and thus may be fibrous band formation in oral mucosa.
detected.
165A. 'C' (Shafer 7th ed 89]
158A. 'C' [Shafer 7th ed 103] The common tobacco induced white lesions include
leukoplakia and tobacco pouch keratosis. Leukoplakia is
158B. 'D' [Burket's 10th ed 203] the most common potentially malignant lesion defined as a
Lymph nodes associated with cancer become enlarged predominantly white lesion of the oral mucosa that cannot
and firm to hard in consistency. The fixation of nodes to be characterised as any other definable lesion.
adjacent tissue due to invasion of cells through the capsule
is a late occurrence. 165B. 'C' (Shafer 7th ed 90]
It is considered a premalignant or potentially malignant
158(. 'A' [Burket's 10th ed 347] lesion as it has an annual malignant transformation rates of
Carcinoma of larynx presents with hoarseness. 0.3% -0.06%

159. 'A' [Burket's 10th ed 449] 165C. 'A' (Shafer 7th ed 92)
Pel Ebstein fever is a cyclic spiking of high fever seen in Diagnostic procedures for leukoplakia include the use of
Hodgkin's lymphoma. toluidene blue staining, Lugol's iodine.

160. 'A' [Burket's 10th ed 168] 165D, 'D' [Shafer 7th ed 92)
Two distinct variants of Elephant man syndrome or von The hall marks of histopathological aspects of leukoplakia
Recklinghausen's neurofibromatosis are known. Type 1 which are epithelial hyperplasia and surface keratosis. Epithelial
is often associated with oral lesions and neurofibromatosis dysplasia may or may not be present.
2 (bilateral acoustic neuroficromatosis) is less common
___ r ORAL PATHOLOGY & MEDICINE
Advantages of frozen section biopsy
True about staining with Toluidine blue is?
a) 2% solution of toluidine blue is applied and area • If more tissue is needed to make an accurate diagnosis,
brushed to collect cells the surgeon is able to obtain an additional sample,
b) 2% solution of toluidine blue is washed with saline avoiding a second operation.
c) 1 % solution of toluidine blue is washed with tap water • If the tissue is determined to be cancerous and is amenable
d) 1 % solution of toluidine blue is washed with acetic to surgery, the mass can be removed at that time.
acid • If the tissue is determined to be benign (not cancerous),
then the mass may not always need to be removed and
166. 'B' [Shafer 7th ed 82) the surgery can end.
Squamous papilloma, associated with H PV 6 & 11 is an
• The frozen section biopsy can help ensure that the mass
exophytic growth made up of numerous small finger like
projections resulting in a cauliflower like surface. It is being removed is the intended tissue for removal.
analogous to common wart or verruca vulgaris of the skin. • It can help ensure that the entire mass and its
Verruca vulgaris is contagious and spreads by autoinoculation. surrounding borders are removed.
Other diseases spreading by autoinoculation include • It allows for the collection of proper tissue samples for
molluscum contagiosum and condyloma acuminatum. further scientific research.
• The surgeon and pathologist are able to collaborate to
167. 'B' [Shafer 7th ed 136) care for the patient.
Peripheral giant cell granuloma has an unknown etiology
and is believed to be a reactive lesion originating from
periodontal ligament or mucoperiosteum. It is found in very
young children as well as in dentulous or edentulous elderly
persons, in 4th-6th decade, on the gingival or alveolar
process.

168. 'B' [Shafer 7th ed 112, 113)


Carcinomas are classified based on TNM staging which include
tumor size, lymph 111ode and distant metastasis. Tumors
that more closely resemble their native tissues are considered
to be well differentiated and have a better prognosis. Tumors
with abundant cellular and nuclear alterations with little or
no resemblance to squamous epithelium or those that lack
keratin production may be classified as poorly differentiated
or high grade or aggressive tumors.

169. 'B' [Shafer 7th ed 231)


Oncocytoma is a rare benign salivary gland tumor composed
of oncocytes with granular eosinophilic cytoplasm and
a large number of atypical mitochondria. The name
oncocytoma is derived from the resemblance of these tumor
cells to apparently normal cells termed oncocytes found in
salivary glands, respiratory tract, breast, thyroid, pancreas,
parathyroid, pituitary, testicle, fallopian tube, liver and
stomach. Electron microscopic studies have shown that
cytoplasm of oncocyte is choked with mitochondria.

170. 'C' [Check Explanation Below]


The principal use of the frozen section procedure is the
examination of tissue to make rapid microscopic diagnosis
of a specimen while surgery is taking place. It is used most
often in oncological surgery. The technical name for this
procedure is cryosection. The report given by the pathologist
is usually limited to, a "benign" or "malignant" diagnosis,
and communicated to the surgeon operating via intercom.
When operating on a previously confirmed malignancy, the
main purpose of the pathologist is to inform the surgeon if
the surgical margin is clear of residual cancer, or if residual
cancer is present at the surgical margin.
Dental 9'ul-6e � - - - - - -

3. ODONTOGENIC CYSTS AND TUMOURS


1. All of the following lesions may be classified as 10. The most common odontogenic tumour which occurs in
odontogenic tumours EXCEPT relation to an unerupted tooth in the anterior maxilla:
a) Acanthomatous ameloblastoma a) Odontogenic adenomatoid tumour
b) Branchial cleft cyst b) Odontoma
c) Myxoma d) Simple ameloblastoma c) Myxoma d) Cementifying fibroma
(MAN -94) (KAR -97, AIIMS - 98)
2. Bifid ribs, multiple radiolucent lesions of the jaws, 11. Which of the following shows the presence of cholesterol
multiple basal cell nevi and falx cerebri calcification are crystals:
found in a) Keratocyst b) Periodontal cyst
a) Basal cell nevus syndrome c) Aneurysmal cyst d) Hemorrhagic cyst
b) Sturge weber syndrome (AIPG - 01)
c) Horner syndrome d) Hereditary internal polyposis 12. Nodular growth of alveolus is seen in
(MAN -98,97) a) Paget's disease b) Osteomas
3. Ameloblastoma most frequently occurs in c) Cementifying fibroma d) All of the above
a) Mandibular molar region (AP - 99, 03)
b) Maxillary molar region 13. Basal layer in primordial cyst is arranged in the form of:
c) Mandibular premolar region a) Tennis racket b) Picket fence
d) Maxillary premolar region c) Linear d) Irregular
(MAN -98) (KAR -97)
4. Compound odontoma shows on a radiograph as 14. Which of the following is the most common lesion of the
a) Supernumerary teeth mandible?
b) Radiolucent and radiopaque areas a) Adamantinoma b) Osteogenic sarcoma
c) Masses of calcified areas c) Squamous cell carcinoma
d) Distinguishable tooth -like structures d) Osteoclastoma
(MAN -99) (AIIMS-01)
s. Which of the following is a true neoplasm of functional 15. One of them is not a true cyst:
cementoblasts a) Hemorrhagic cyst b) Median palatal
a) Periapical cemental dysplasia c) Globulomaxillary d) Nasolabial
b) Familial cemental dysplasia (AIPG -97)
c) Benign cementoblastoma 16. Dentigerous cyst is likely to cause which neoplasia
d) Hypercementosis a) Ameloblastoma b) Adeno carcinoma
(MAN -2K) c) Fibrosarcoma d) All of the above
6. Botryoid odontogenic cyst is a variant of (AIIMS -2004)
a) Lateral periodontal cyst 17. Odontogenic keratocyst has the following feature
b) Apical periodontal cyst a) Occurs due to infection periapically
c) Gingival cysts of new born b) Is developmental in origin
d) Gingival cysts of adult c) Can be treated by aspiration
(MAN -02) d) Has low recurrence rate
7. Eruption cyst (AIIMS -96)
a) Transforms into dentigerous cyst 18. Primordial cyst develops:
b) Regresses after eruption of the tooth a) In place of missing teeth
c) Is found in the place of the missing tooth b) In teeth in which crown development is completed
d) Is a type of dentigerous cyst c) In periapical region d) In mandibular body
(AP -01) (AIPG - 97,95,AIIMS -2K)
8. Leisegang rings are found in 19. Robinson's classification of ameloblastoma does not
a) Calcifying epithelial odontogenic cyst include:
b) Primordial cyst a) Multicentric b) Non-functional
c) Calcifying epithelial odontogenic tumor c) Anatomically benign d) Clinically persistent
d) Odontoma (AIIMS- 99)
(AP -01) 20. Radiographic finding in Pindborg tumour is:
9. Which of the following is an odontogenic tumor? a) Sun-burst appearance b) Onion-peel appearance
a) Arrhenoblastoma b) Astrocytoma c) Driven-snow appearance
c) Ameloblastoma d) Granular cell tumor d) Cherry-blossom appearance
(KAR -01) (AIIMS -01, AIPG-06)

1) B 2) A 3) A 4) D 5) C 6) A 7) B 8) C 9) C 10) A 11) A 12) C 13) B


14) A 15) A 16) A 17) B 18) A 19) A 20) C
21.
r
Unicentric, non-functional, anatomically
ORAL PATHOLOGY & MEDICINE
benign, 31. Adenomatoid odontogenic tumour is most commonly
clinically persistent tumor is found in
a) CEOT b) Enameloma a) Anterior mandible b) Posterior maxilla
c) Odontoma d) Ameloblastoma c) Anterior maxilla d) Ramus of mandible
(AP -01) (AIPG -04)
22. The most ideal explanation for recurrence of odontogenic 32. Adamantinoma is
keratocyst is a) A tumour from embryonal cells of developing teeth
a) Increased mitotic activity of the epithelial lining b) also known as Ameloblastoma
b) Friability of the epithelial lining c) is a complication of dentigerous cyst
c) Presence of satellite cysts or daughter cysts d) all of these
d) Continued proliferation of rests of dental lamina (TNPSC -99)
(KAR -99) 33. Ghost (shadow) cells are seen in
23. The cyst with highest recurrence rate is: a) Ameloblastic fibroodontoma
a) Keratocyst b) Periapical cyst b) calcifying odontogenic cyst
c) Nasoalveolar cyst d) Globulomaxillary cyst c) compound odontoma d) all of the above
(KAR-04, AP-98,04) (AIPG -97, KAR -04)
24. A multilocular cyst of the jaw is more likely: 34. A 25 year old male patient reports with a bony expansile
a) Dental cyst. b) Dentigerous cyst. swelling of the right body of the mandible & mild
c) Keratocyst. d) Simple bone cyst. paresthesia of the right ION. 0PG shows a multi locular
(KAR -99) radiolucency without root resorption. (COM EDK-2011}
25. Keratocyst has all of the following features except: A) What would be your choice of next investigation
a) It is more common in mandible a) Excision biopsy b) Aspiration Cytology
b) May be filled with thin straw coloured fluid c) CT Scan d) PET Bone scan
c) Low recurrence rate
d) Expansion of bone clinically seen B) A dirty white aspirate with a protein estimation of
(AIPG -96) <4gm% is suggestive of
26. Each of the following cyst is associated with an impacted a) Ossifying fibroma b) Dentigerous cyst
tooth except: c) Muco-epidermoid carcinoma
a) Dentigerous cyst d) Odontogenic Keratocyst
b) Calcifying epithelial odontogenic cyst
c) Keratocyst d) Primordial cyst C) 0dontogenic Keratocyst is noted for its
(AIPG -01) a) Malignant transformation
27. Which of the following is wrong about keratocyst: b) Daughter cysts and high rate of recurrence
a) Haw low recurrence rate c) Impacted teeth d) Nodal metastasis
b) Has low protein content
c) High recurrence rate d) B and C D) Management of odontogenic keratocyst involves
(AIPG -2K) a) Marsupialization b) Enucleation
28. A 40 year old woman has ameloblastoma, the c) Enucleation with peripheral ostectomy
histomorphologic features will be d) Resection and Radiation
a) Peripheral palisading cellular strand with central loose
stellate reticulum 35. Treatment for cementoma
b) Peripheral palisading with central stromal retraction a) No treatment b) Pulpectomy
artefact c) Resection of jaw d) None of the above
c) Peripheral palisading cellular strand with peripheral (AP -04)
loose stellate reticulum 36. Lesions associated with vital tooth
d) Central loose stellate reticulum shows marked nuclear a) condensing osteitis b) cementoma
atypia and numerous mitotic c) periapical abscess d) none of the above
(AIIMS-2012) (KAR -98, AP-05)
29. Which histopathological type of odontogenic keratocyst 37. Destructively invasive, locally malignant with rare
is commoner, more invasive & has a greater tendency for metastasis, the lesion is
recurrence? a) Fibroma b) Ameloblastoma
a) Orthokeratinised b) Parakeratinised c) Papilloma d) None of the above
c) Non-keratinised d) Diskeratinised (AP -02)
(COMEDK-04, 05) 38. Compound odontoma shows
30. Multiple odontogenic keratocyst are associated with: a) Mixed tissue of dental origin with no resemblance to
a) Gardner's syndrome b) Gorlin-Goltz syndrome. tooth structure
c) Goldenhar's syndrome. d) Grinspan syndrome. b) Numerous tooth Like structure with denticles commonly
(AIPG -04) found in maxillary lateral incisors.

21) D 22) C 23) A 24) C 25) C 26) D 27) A 28) A 29) B 30) B 31) C 32) D 33) D
34A) B 34B) D 34C) B 34D) C 35) A/D 36) B 37) B 38) B
Dental 9'ul-6e
c) haphazardly arranged calcified mass 49. The pathogenesis of periapical cyst is?
d) all of the above a) Increased pressure within the cyst
(PGI -2003) b) Immune mediated bone destruction
39. Dentigerous cyst is suspected if the follicular space is c) Proliferation of epithelium
more than d) None
a) 2-3 mm b) 3-4mm (PGI-08)
c) 1-2 mm d) > 5 mm 50. Adenomatoid odontogenic tumour is characterized
histologically by
40. After entering a radiolucent lesion in a 30 year old man, a) Polyhedral epithelial cells
hollow cavity without epithelial lining is seen, the most b) Tubular / duct like cells
probable diagnosis is c) Stellate shaped cells
a) Aneurysmal bone cyst b) Static bone cavity d) Stratified squamous epithelial cells
c) Hemorrhagic bone cyst d) Ameloblastoma (COMEDK-10)
(AP-10) 51. Cyst arising from dental lamina
41. A patient with ameloblastoma of the jaw can best be a) Radicular cyst b) Pa.radental cyst
treated by c) Eruption cyst d) Glandular odontogenic cyst
a) Irradiation b) Excision (COM ED-2012)
c) Enucleation 52. The most common odontogenic cyst is
d) Surgical removal followed by cauterization a) Primordial cyst b) Dentigerous cyst
(KCET-10) c) Radicular cyst d) Mucocele
42. Dentigerous cyst is associated with the following (KCET-2012)
a) Impacted 3rd molar 53. Standard treatment of ameloblastoma
b) Impacted supernumerary tooth a) Segmental resection with 1 cm of normal bone
c) Odontome d) All the above b) Enbloc resection
(COMEDK-05) c) Enucleation
43. The epithelium of a dentigerous cyst is: d) Enucleation with cauterization
a) 15-20 cell thick b) 6-10 cell thick (AIPG-2012)
c) 2-4cell thick d) 1-2 cell thick 54. The most aggressive and destructive cyst is
a) Periapical cyst b) Dentigerous cyst
44. Clear cells are commonly seen in which of the following c) Globulomaxillary cyst d) Nasopalatine cyst
lesions? (AIPG-2012)
a) Pleomorphic b) Warthins tumor 55. Facial nerve paralysis is common with
c) Mucoepidermoid a) Pleomorphic adenoma b) Epidermoid carcinoma
d) Adenomatoid odontogenic tumor c) Warthin'stumour d) Lymphoepithelial carcinoma
(COMEDK-07) (AIPG-2012)
45. Multipie periapical radiolucencies are seen in 56. COC is now called as
a) Jawcyst basal cell Nevus Syndrome a) Odontogenic ghost cell tumor
b) Odontogenic keratocyst b) Dentinogenic ghost cell tumor
c) Cherubism d) Thyroid disorders c) Keratcysticodontogenic tumor
(AP-08) d) A & C
46. A 36 year old male with an asymptomatic swelling in (PGI-2011)
the body of the mandible with radiographic features of 57. Multiple bilatertal dentigerous cysts are seen in
radiolucency with radiopaque flecks is suffering from a) Down's syndrtome b) Maroteaux Lamy syndrome
a) Odontogenic keratocyst c) Treacher Collin syndrome
b) Calcifying eipthelial odontogenic tumor (CEOT) d) Gorlin Goltz syndrome
c) Ameloblastoma d) None of the above (AP- 2013)
(AP-08) 58. The cyst that remains behind in the jaws after removal of
47. Pindborg tumor arises from the tooth is
a) Basal layer of cells b) Stratum intermedium a) Lateral Periodontal Cyst
c) Stratum corneum d) dental lamina b) Radicular Cyst
(AIIMS-09) c) Residual Cyst d) None of the above
48. A six year old child patient has blue-dome shaped (MCET-14)
swelling in posterior mandibular region, what will be the 59. Corrugated collagenous rings surrounding lymphocytes
treatment plan? and plasma cells in the walls of inflammatory cysts are
a) Reassure the patient without any treatment called
b) Excise the lesion a) Ruston bodies b) Hyaline bodies
c) Marsupialization d) Surgical Excision c) Howell-Jolly bodies d) Pa penheimer bodies
(AIPG-09) (MCET-14)

39) D 40) C 41) D 42) D 43) C 44) C 45) A 46) B 47) B/D 48) A 49) C 50) B 51) D
52) C 53) A 54) B 55) D>B 56) B 57) B 58) C 59) B
r ORAL PATHOLOGY & MEDICINE
60. Potential complications stemming from dentigerous cyst
are
a) Ameloblastoma b) Epidermoid carcinoma
c) Mucoepidermoid carcinoma
d) All of the above
(GCET-14)
61. Permanent mandibular first molar is a common site for
all the lesions EXCEPT
a) Pulp polyp b) Condensing osteitis
c) Lateral periodontal cyst
d) Benign cementoblastoma
(MHCET-15)
62. Most common cyst associated with adjoining vital teeth?
a) Dentigerous cyst b) Globulomaxillary cyst
c) Periapical cyst d) Lateral periodontal cyst
(AIIMS MAY-14)
63. Which of the following statement is TRUE about a
malignant ameloblastoma
a) Cytopathologic features of malignancy seen in primary,
recurrent or any metastatic tumor foci
b) Cytopathologic features of malignancy seen only in
primary tumor
c) Cytopathologic features of malignancy seen only in
metastatic tumor
d) Metastasizing tumor that is cytopathologically similar to
its benign counterparts in both primary and metastatic
deposits.
(COMEDK-15)

60) D 61) C 62) A 63) D


Dental 9'ul-6e � - - - - - -

3. ODONTOGENIC CYSTS AND TUMOURS - ANSWERS


1. 'B' [Shafer 7th ed 35,260/ 6th ed 34] 6. 'A' [Shafer 7th ed 271/ 6th ed 264]
Brachia[ cleft cyst is a fissural cyst that occurs on lateral Botryoid odontogenic cyst is a m ulticystic variant of lateral
aspect of neck. It originates from remnants of pharyngeal periodontal cyst.
pouches or pharyngeal arches.
7. 'B' [Shafer 7th ed 263/ 6th ed 258]
2. 'A' [Shafer 7th ed 267/6th ed 261-262] Eruption cyst is a form of dentigerous cyst associated with
Basal cell nerus syndrome is also called as Gorlin Goltz erupting deciduous or permanent tooth.
syndrome. Option 'C', Homer's syndrome is characterized by
miosis, ptosis and anhidrosis over face due to interruption 8. 'C' (Shafer 7th ed 285/ 6th ed 281]
of sudomotor and vasomotor control. Option 'D', hereditary Liesegang rings are formed by amyloid like material
intestinal polyposis is seen in Peutz-Jeghers syndrome. composed of conventional amyloid or immunamyloid or
APUDamyloid.
3. 'A' [Shafer 7th ed 277/ 6th ed 272]
Tumour common site 9. 'C' (Shafer 7th ed 276/ 6th ed 271]
Ameloblastoma is an ectodermal tumour of odontogenic origin.
• Composite compound odontoma
• Adenomatoid odontogenic tumor Anterior maxilla Option 'A, arrhenoblastoma is a rare ovarian tumor that
• Squamous odontogenic tumor produces masculinization and often contains tubules and
Bifurcations and luteinized cells.
trifurcations of
Enameloma 10. 'A' [Shafer 7th ed 286/ 6th ed 282]
maxillary posterior
teeth. Adenamatoid Odontogenic tumor occurs commonly in the
anterior maxilla (76%) and is associated with an unerupted
Composite complex odontoma Posterior maxilla
tooth (74%). Odontoma is the most common odontogenic
• Ameloblastoma tumor, with 67% occurring in the maxilla. However,
Mandibular molar
• Pindborg tumor compound odontoma occurs in the anterior maxilla, and
area
• Metastatic carcinoma complex odontoma commonly occurs in posterior jaws.
Since the type of odontoma had not been mentioned in the
4. 'D' (Shafer 7th ed 29 2-93/ 6th ed 289] question, Option A is more appropriate. The most distinctive
Odontoma is of 2 types. If the involved tissue resembles microscopic feature of AOT is varying numbers of duct like
normal tooth it is called as compound odontoma. If the (COMEDK-10) structures lined by eosinophilic rim of varying
tooth has no morphologic similarity with a normal tooth it thickness, which are called as hyaline ring.
is called complex odontoma.
Option 'C', odontogenic myxoma is more common in mandible
5. 'C' (Shafer 7th ed 73 6/ 6th ed 732] and it may or may not be associated with a embedded or
missing tooth. Option 'D' cementifying fibroma is common
• The lesion occurs in and near the
in mandible.
periodontal ligament around the apex
Periapical of a tooth, usually a mandibular incisor.
11. 'A' [Shafer 7th ed 265/ 6th ed 260]
cementa[ • The initial stage is radiolucent, the It got the name keratocyst because of production of so
dysplasia second stage is mixed radiopaque and much keratin by cyst epithelium that fills the cyst lumen.
(cementoma) radiolucent, and the third stage is Sometimes cholesterol and hyaline bodies are present
completely radiopaque at sites of inflammation. Also called as 'ortokeratinized
• Tooth is vital odontogenic cyst'.
• Also known as true cementoma,
because it is a true neoplasm functional Note: WHO has reclassified OKC as Keratocystic Odontogenic
cementoblasts. Tumor ( KOT) based on:
• Mandibular first permanent molar is the a) Behaviour: KOT is locally destructive with high recurrence
Benign most frequently affected tooth. rates.
cemento-of b) Histopathology: Basal epithelial layer shows
blastoma • Associated tooth is vital
proliferatoin & budding into the underlying connective
• Also known as familial multiple
tissue as daughter cysts; Mitotic figures are found in
cementoma, which usually presents as
suprabasal layer.
diffuse radiopaque masses scattered
throughout the jaws. c) Genetics: PTCH, a tumor suppressor gene, on 9q22.3-q31
chromosome is associated with syndrome - associated
Gigantiform • Involves all the four quadrants of the
KOT and sporadic KOT.
cementoma jaw
ORAL PATHOLOGY & MEDICINE
12. 'C' [Shafer 7th ed 135] • Gorlin cyst
Central cementifying, central ossifying, central cementa­ • Unicystic or mural ameloblastoma
ossifying fibroma have a centrifugal growth pattern. They grow
• Adena ameloblastoma
by expansion equally in all directions and present as a round
tumour mass. Fibrous dysplasia expands linearly and the expanded • Primordial cyst
mandible is not in continuity with the outline of lesion.
19. 'A' [Shafer 7th ed 276/ 6th ed 271]
13. 'B' [Shafer 4th ed 259] Robinson classified Ameloblastoma as "usually unicentric,
Picket fence or tomb stone pattern of Basal layer with no rete non-functional, intermittent in growth, anatomically benign
peg formation is seen in both primordial and odontogenic and clinically persistent".
keratocysts.
20. 'C' [Shafer 7th ed 284/ 6th ed 279]
14. 'A' [Shafer 7th ed 277/ 6th ed 272]
Odontoma is the most common neoplasam. Second is 21. 'D' [Shafer 7th ed 276/ 6th ed 271]
ameloblastoma. Ameloblastoma incidence is equal to the
incidence of all other odontozenic tumors combined. 22. 'C' [Shafer 7th ed 267 / 6th ed 260-61]
The high recurrence rate of OKC (now called Keratocystic
Ameloblastoma replaced the term adamantinoma and the Odontogenic tumour (KOT)) may be due to the nature of
term ameloblastoma was suggested by Churchill. the lesion itself. It had also been noted that recurrence
rates were more frequent in O KCs of nevoid basal cell
Types of Ameloblastoma:- carcinoma syndrome. Keratocysts enucleated in on piece had
less recurrence. Multilocular lesions had higher recurrence
• Follicular type, which is simple and most common than unilocular lesions. Parakeratinized O KCs have greater
• Plexiform type, which contains ameloblast like tumor recurrence than orthokeratinized O KCs.
like cells arranged in irregular masses.
23. 'A' [Shafer 7th ed 266/ 6th ed 261]
• Acanthomatous type, in which the stellate reticulum The highest recurrence rate is due to very thin epithelial
undergo squamous metaplasia, with keratin formation in lining and presence of daughter cysts.
central portion of tumor islands. This usually occurs in
follicular amelobllastoma. Sometimes keratin pearls may 24. 'C' [Shafer 7th ed 265/ 6th ed 260]
be seen
• Granular cell ameloblastoma, in which the cytoplasm 25. 'C' [Shafer 7th ed 266/ 6th ed 261]
of Stellate reticulum cells shows a very coarse, granular,
eosinophilic appearance. Most aggressive with metastasis. 26. 'D' [Shafer 4th ed 259]
Primordial cyst is usually found in place of a tooth rather
• Basal cell type, which bears resemblance to basal cell than directly associated with the tooth.
carcinoma. Rarest histologic subtype.
• Desmoplastic type, which has a greater tendency to 27. 'f( [Shafer 7th ed 266/ 6th ed 261]
grow in thin strands and cords of epithelium rather than
islands. Characteristically found in a dense collagen 28. 'A' [Shafer's 7th ed 280/ 6th ed 274]
stroma that may appear hyalinized and hypocellular. Histologically, most common types of ameloblastomas are
follicular and plexiform varieties and the uncommon variants
include acanthomatous, granular cell types, desmoplastic,
In some instances, the ameloblastic epithelium may basal cell and clear cell. In general, moderately to densely
be proliferative, with extension of the ameloblastic collagenised tissue constitutes stroma, while the epithelial
epithelium into the lumen of the cystic cavity. This component proliferates in disconnected islands, strands,
feature has been termed intraluminal proliferation. cords. The peripheral cells are tall columnar cells with
hyperchromatic nuclei that are arranged a palisading
15. 'A' [Shafer 7th ed 529/6th ed 528] pattern. The most common follicular ameloblastoma shows
peripheral palisading and central reticulum stellate pattern.
16. 'A' [Shafer 7th ed 262/ 6th ed 257]
This type of amelobla:stoma is known as mural ameloblastoma. 29. 'B' [Shafer 7th ed 266/ 6th ed 259-60]

17. 'B' [Shafer 7th ed 260 Table 4-1/ 6th ed 254 Table 4-1] 30. 'B' [Shafer 7th ed 267/ 6th ed 259]

18. 'A' [Shafer 4th ed 259] 31. 'C' [Shafer 7th ed 286/ 6th ed 282]
Lesions associated with clinically or radiologically missing
teeth. 32. 'D' [Shafer 7th ed 276-77/ 6th ed 271]
• Eruption cyst
33. 'D' [Shafer 7th ed 272, 293-95/ 6th ed 292]
• Dentigerous cyst
Dental 9'ul-6e
34A. 'B' [Check Explanation Below] Histologically the rete peg formation is absent except in
Expansile multilocular lesions of the bone with paresthesia cases of secondary infection.
of the inferior alveolar nerve suggest odontogenic or central
tumors and cysts. Cysts usually cause displacement of roots, 40. 'C' [Shafer 7th ed 529/6th ed 528]
though occasionally root resorption may also be seen. The Hollow cavity without epithelial lining means, the lesion is a
common chair side investigation for cysts is aspiration pseudocyst. Options'/( 'B' &'C' are examples of pseudocysts.
cytology. So, immediate choice of investigation would be In Option 'fl Aneurysmal bone cyst, upon entering the
aspiration cytology. lesion, excessive bleeding is encountered like the blood
welling up from the tissue.
34B. 'D' [Shafer 7th ed 265/6th ed 260]
The electrophoretic measurement of fluid from odontogenic Option 'B' static bone cavity or developmental lingual
keratocyst shows a very low content of soluble protein when mandibular salivary grand depression or stafne cyst is
compared with the patient's own serum. Hence a low protein an unusual form of slightly aberrant salivary gland tissue
content of 4 gm¾ is suggestive of odontogenic keratocyst. wherein a developmental inclusion of glandular tissue is
Ossifying fibroma and mucoepidermoid carcinoma are solid found on the lingual surface of body of the mandible. It can
tumors with no aspirate. be differentiated from the traumatic or hemorrhagic bone
cyst which by location lies superior to inferior alveolar canal.
34C. 'B' [Shafer 7th ed 265-66/6th ed 261, 257]
The connective tissue wall of odontogenic keratocyst shows Option 'C' traumatic cyst / hemorrhagic cyst or idiopathic
small islands of epithelium known as satellite or daughter bone cavity is a pseudocyst without epithelial lining. The
cysts. These actually represent ends of folds of lining widely accepted theory of origin of hemorrhagic cyst is from
epithelium of the main cystic cavity, that has been cut in intramedullary hemorrhage following traumatic injury. When
cross section. The cyst has an extraordinary recurrence rate the cavity is opened surgically it is found to contain either
between 13 and 30%. Recurrence is due to the nature of a small amount of sero-sanguinous fluid, shreds of necrotic
lesion itself i.e the presence of additional remnants of dental blood clot, fragments of fibrous connective tissue or
lamina from which a cyst may develop. Hence odontogenic nothing. The dentist is frequently astonished to open into
keratocysts are known for their daughter cysts and high rate an empty space in bone and find that it has no clinically
of recurrence. Malignant transformation is more common in demonstrable membrane.
dentigerous cysts, where the mucosa lining may give rise to
epidermoid or mucoepidermoid carcinoma. 41. 'D' (Shafer 7th ed 282-3/6th ed 278]

34D. 'C' [Shafer 6th ed 261] 42. 'D' (Shafer 7th ed 259/ 6th ed 255]
The most important feature of an odontogenic keratocyst is
its extraordinary recurrence rate. Hence inadequate removal 43. 'C' [Shafer 6th ed 256]
by enucleation, especially when broken into pieces may be As the lining is derived from re·duced enamel epithelium,
associated with recurrence. To ensure complete removal dentigerous cyst is 2-4 cell layer in thickness.
and no possible recurrence, removal of peripheral bone is
necessary. However, Browne found no significant differences 44. 'C' (Shafer 7th ed 236/ 6th ed 232]
in recurrence rate following treatment with enucleation Important features about mucoepidermoid carcinoma:
with primary closure, enucleation and packing open or • Most common malignant neoplasm observed in the major
marsupialisation and considered recurrence to be due to the and minor salivary glands.
nature of lesion itself. • Most common malignant salivary gland tumor of children.
35. 'A/D' [Shafer 4th ed 298] • Histologically shows clusters of clear cells, which are
Note: True cementoma is benign cementoblastoma which generally mucin and glycogen free.
is treated by excision & extraction of involved tooth. • Intraosseous or central mucoepidermoid carcinoma,
Cementoma is another name for PCOD, where treatment may a variant of mucoepidermoid carcinoma, is the most
not be required normally. common malignant salivary gland tumor in bone.

36. 'B' [Shafer 6th ed 732/White & Pharoah 6th ed 433] 45. 'A' [Shafer 6th ed 259)
The multiple periapical radiolucencies are due to multiple
37. 'B' [Shafer 7th ed 276/ 6th ed 271, 272] odontogenic keratocysts.

38. 'B' [Shafer 7th ed 292/6th ed 289-90] 46. 'B' [Shafer 7th ed 283-4/6th ed 279]
Compound odontoma shows resembles to normal teeth and • Option 'K is incorrect because odontogenic
common in anterior maxilla. Complex odontoma shows no keratocyst is always radiolucent
similarity to normal teeth and are common in posterior jaws. • Option 'C' can be ruled out because of multilocular
radiographic appearance.
39. 'D' (Shafer 7th ed 260/ 6th ed 255]
The normal follicular space is 3-4 mm and a dentigerous
cyst can be suspected if the space is more than 5 mm.

r ORAL PATHOLOGY & MEDICINE
Option 'B' is correct because the condition occurs released due to necrotic pulps in infected teeth. Hence it
frequently in middle age group, with definite had been classified as an inflammatory cyst.
predilection in mandible molar area and sometimes
appear as radiopaque - radiolucent lesion (Driven 53. 'A' [Differential Diagnosis Oral and Maxilofacial lesions
snow appearance). Wood and Goaz 5th ed 340)
Multilocular or solid ameloblastomas are treated radically by
CEOT characteristically show homogenous, eosinophilic resection including 1-2 cm of normal bone, since it had been
substance interpreted as amyloid. The amyloid like material reported that ameloblastoma extends 2.3-8mm beyond its
is composed of atleast three different types of fibrils, which radiographic margins. This is done to avoid higher chances
are smaller in size compared to fibrils of conventional of recurrence.
amyloid. Another characteristic feature of CEOT is presence
of calcifications in form of Liesegang rings. 54. 'B' [Shafer's 7th ed 260/6th ed 255]
Dentigerous cyst is potentially capable of becoming an
47. 'B/D' [Shafer 7th ed 283/6th ed 279] aggressive lesion, with considerable bone expansion,
Regarding the pathogenesis of Pindborg tumor, some facial asymmetry, hollowing out of the entire ramus upto
suggested that epithelial cells of Pindborg tumor are the coronoid process or condyle. Moreover the epithelium
reminiscent cells of stratum intermedium layer of enamel of the dentigerous cysts is pluripotential, and can give
organ. Some hypothesize that the pindbold tumor arises rise to ameloblastoma, epidermoid carcinoma or the
from remnants of primitive dental lamina found in initial mucoepidermoid carcinoma.
stages of odontogenesis.
Most aggressive type of cyst which causes hollowing of
48. 'A' [Shafer 7th ed 263/6th ed 258] mandible is ____
The blue-dome shaped swelling is nothing but eruption a) Dentigerous cyst. (NEET-2013)
cyst that usually surrounds a tooth crown that has erupted
through bone but not soft tissue. No treatment is necessary 55. 'D>B' [Shafer's 7th ed 246/6th ed 222]
as the cyst often ruptures spontaneously. Surgical exposure Pleomorphic adenoma (PA) & Warthin's tumours are benign
of crown may require in a few cases. salivary gland neoplasms and are firm, slowly enlarging
nodular lesions. Facial paralysis if at all, occurs only in
49. 'C' [Shafer 7th ed 2 73/6th ed 268] case of pleomorphic adenoma, but is rare. Malignant
The development of periapical cyst is inflammatory in nature. transformation of PA of parotid may cause facial paralysis in
The epithelial cell rests of malassez present in the midst of 1/3rd of patients.
periapical granuloma. proliferate and eventually form a large
mass of cells. With continuous growth, the inner cells of the Epidermoid carcinoma or squamous cell carcinoma is a rare
mass are deprived of nourishment and undergo liquefaction malignant epithelial neoplasm of major salivary glands, and
necrosis. This leads to the formation of a cavity which is may be associated with pain or facial weakness.
located in the center of granuloma, giving rise to radicularcyst.
Lymphoepithelial carcinoma is an undifferentiated tumour
Islands of squamous epithelium which have developed from of salivary glands associated with a dense lymphoid stroma.
cell rests of malass,ez can also be found in a periapical Pain & facial palsy (20% of patients) and a parotid/
granuloma without cystic formation. Endodontists refer to submandibular mass are the common manifestations. Hence
these granulomas as "Bay cyst". option (D) is more appropriate.
50. 'B' [Shafer 7th ed 287 /6th ed 282] If option "D" is not given, then Option 'B would be the right
Adenomatoid odontogenic tumor occurs mostly in association answer.
with unerupted maxillary cuspid. The most distinctive
microscopic feature of AOT is varying number of duct like 56. 'B' [Shafer's 7th ed 295/6th ed 267, 291]
structures with lumina of varying size. These duct like or COC is a distinct clinicopathological entity characterized
microcyst lumina frequently are lined by an eosinophilic rim by ghost cell keratinization of the odontogenic epithelium
of varying thickness (the so-called hyaline ring). and it exists in a cystic form, solid neoplastic form or a
third malignant counterpart. The neoplastic variant termed
51. 'D' [Shafer's 7th ed 272/6th ed 254] as dentinogenic ghost cell tumor, is a rare lesion that may
Option 't,; Radicular cyst and residual cysts arise from the occur in the bone or gingival soft tissue.
cell rests of Malassez whereas dentigerous and Option 'C'
eruption cysts arise from the reduced enamel epithelium. 57. 'B' [Shafer 7th ed 260/6th ed 255]
COC and option 'B' paradental cyst remain unclassified as to Multiple dentigerous cysts are usually found in association
the tissue of origin and the rest are derived from the rests with syndromes like cleidocranial dysplasia and Maroteaux
of Serrae (dental lamina rests) -Lamy syndrome.
52. 'C' [Shafer's 7th ed 273/6th ed 268]
Periapical or root end or apical periodontal or radicular cyst
is the most common odontogenic cyst, caused by the toxins
Dental 9'ul-6e
58. 'C' [Shafer 7th ed 27 4] lymph nodes (15%) or extragnathic bones. The median
Residual cysts are actually retained periapical cysts from survival of patients after the discovery of metastases is
teeth that have been removed. It may be found in any of the about 2 years. Ameloblastoma with metastatic potential
tooth bearing areas of the jaws. They may be accidentally is histologically indistinguishable from conventional
discovered on routine radiologic examination as a well­ ameloblastoma i.e, does not show any greater cytologic
defined radiolucency varying in size from few mm to few cm. atypia or mitotic activity than seen in primary and
However they may become symptomatic when secondarily demonstrates the biologic behaviour of a well differentiated
infected. low grade carcinoma.

59. 'B' [Check Explanation Below]


The occurrence of hyaline body or Rushton body is seen
in great numbers in the epithelium of apical periodontal
or residual cyst. These hyaline bodies are tiny linear or
arc- shaped bodies, generally associated with the lining
epithelium, that appear amorphous in structure, eosinophilic
in reaction and brittle in nature. They appear to have no
diagnostic significance and their origin is unknown (Thought
to represent some form of epithelial product.)

60. 'D' [Shafer 7th ed 26 2]


The epithelium lining the dentigerous cyst is pluripotential
and is capable of developing into an ameloblastoma,
epidermoid carcinoma or mucoepidermoid carcinoma. The
source for the ameloblastoma and SCC is lining epithelium
or the islands of odontogenic epithelium. Mucoepidermoid
carcinoma develops from mucus secreting cells of the lining
epithelium.

61. 'C' [Shafer 7th ed 270, 480, 495]


The lateral periodontal cyst is a slow growing, non expansile,
developmental cyst derived from one or more rests of dental
lamina, containing ain embryonic lining of 1-3 cuboidal
cells and distinctive focal thickenings (plaques). Most of
the cysts are seen in bicuspid/cuspid/incisor areas, however
lacks any rational explanation for this Localization.

Pulp polyps are characterised by exuberant proliferation and


is most commonly seen in deciduous molars and permanent
first molars. Condensing osteitis also commonly affects first
mandibular molar.

62. 'A' [Shafer 7th ed 259,273,269)


Dentigerous cyst is one of the most common odontogenic
cyst, estimated to be 20% of all jaw cysts. It is estimated
that about 10% of impacted teeth have a dentigerous cyst
and the associated tooth is always vital.

Radicular cyst, though is the most common odontogenic


cyst, is always associated with a non-vital tooth. Lateral
periodontal cyst is an uncommon developmental cyst
arising from cystic degeneration of clear cells of dental
lamina. Lateral radicular cysts must be differentiated from
development lateral periodontal cysts.

63. 'D' [Shafer 7th ed 301]


Metastasizing ameloblastoma, ambiguously called malignant
ameloblastoma, is a term used to describe a tumor that
shows histological features of classic ameloblastoma in
the gingiva or jaw and has metastatic deposits elsewhere.
After a mean interval of 11 years of primary ameloblastoma,
metastatic nodules develop in the lung (80%), cervical
_____ r ORAL PATHOLOGY & MEDICINE

4. DISEASES OF SALIVARY GLANDS


1. Salivary gland stone most commonly involves 10. Non-inflammatory, non-neoplastic enlargement of the
a) Submandibular gland b) Parotid gland salivary gland is called as:
c) Sub lingual glands d) Lingual glands a) Sialadentis b) Sialosis
(MAN -98, 01, COMEDK-06) c) Ptyalism d) Sialorrhea
2. In the clinical evaluation, the most significant, finding (KAR -03)
of the parotid mass may be accompanying 11. Most common tumour of parotid gland is:
a) Rapid progressive painless enlargement a) Pleomorphic adenoma
b) Nodular consistency b) Adenoid cystic carcinoma
c) Supramental and preauricular lymphadenopathy c) Cylindroma d) Epidermoid carcinoma
d) Facial paralysis (AIIMS -94, AP-2K, KAR -02)
(MAN -98, KAR -97) 12. A painful crater like 1.5 cm ulcer develops within one
3. Which of the following statements is FALSE week on the hard palate mucosa of a 40 year old female.
a) A salivary duct obstruction can cause a unilateral The most likely diagnosis is:
swelling in the floor of the mouth that is largest before a) Actinomycosis b) Squamous cell carcinoma
a meal and smallest after a meal. c) Pleomorphic adenoma d) Necrotizing sialometaplasia
b) The lesion termed a ranula is associated with the (AIPG -92, AIIMS -90)
sublingual salivary gland. 13. A condition of the mouth which increases the caries
c) The sublingual sa.livary gland is the most common site of activity in the oral cavity is:
salivary gland neoplasia a) Xerostomia b) Malignancy
d) A pleomorphic adenoma is the most common salivary c) Hairy tongue d) Watery saliva
gland neoplasm (AIPG -89,90)
(MAN-95) 14. Which of the following parotid malignancy shows
4. Warthins tumor is perineural spread:
a) An adenolymphoma of the parotid gland a) Pleomorphic adenoma b) Adenoid cystic carcinoma
b) A pleomorphic adenoma of parotid gland c) Warthin's tumor d) Ouctal papilloma
c) Carcinoma of the parotid gland (AIPG -99)
d) None of the above 15. Which of the following is of salivary gland origin?
(MAN -98, KAR -97) a) Acinic cell carcinoma b) Granular cell myoblastoma
5. The common site for necrotizing sialometaplasia c) Chondrosarcoma d) All of the above
a) cheeks b) dorsum of tongue (AIPG -96)
c) palate d) gingival 16. Reduction in flow of saliva is not generally seen in:
(KAR -2K, AIIMS -97) a) Elderly diabetics
6. Pleomorphic adenoma arises from b) Patient undergoing radiation therapy
a) Myoepithelial cells b) Acinar cells c) Patients suffering from parkinsonism
c) Connective tissue d) Stem cells d) Patients on phenothiazine drugs
(KAR -01) (AIPG -91)
7. The most common salivary gland malignant neoplasm in 17. Sialography is used to detect anomaly of:
bones a) Salivary duct only b) Salivary gland
a) Plemorphic adenoma b) Adenoid cystic carcinoma c) Salivary gland & duct d) Salivary gland tumours
c) Muceopidermoid carcinoma (AIPG -97)
d) Adenolymphoma 18. Acute non-suppurative sialdenitis is seen in:
(KAR -01, COMEDK -05) a) Acute bacterial sialadenitis
8. Mikulicz's disease is: b) Mumps
a) An inflammatory disease c) Chronic bacterial sialadenitis
b) Neoplastic disease d) Necrotizing sialometaplasia
c) An autoimmune disease (KAR -02)
d) Viral infection 19. A cyst occurs under the tongue, caused by obstruction of
(KAR -02) a salivary gland. Such a cyst is called:
9. Bimanual palpation technique is carried out for a) Mucocele b) Ranula
a) Submandibular gland b) Sublingual gland c) Dermoid cyst d) Oentigerous cyst
c) Ranula (AIPG -91,99, AIIMS-91,MAN -01)
d) Cervical lymph nodes when they are enlarged due to 20. Mucocele most commonly arise as a result of:
inflammation a) Rupture of a salivary duct
(AIIMS -94) b) Partial or complete compression of the salivary acini

1) A 2) D 3) C 4) A 5) C 6) A 7) C 8) C 9) A 10) B 11) A 12) D 13) A


14) B 15) A 16) C 17) C 18) B 19) B 20) A
Dental 9'ul-6e
c) Inflammatory changes in the glandular interstitial tissue 33. Salivary gland aplasia is seen in
d) Partial or complete obstruction of the salivary duct by a) Hemifacial microstomia b) LADD syndrome
calculus c) Mandibulo-facial dysostosis (Treacher Collins)
(AIPG -92, 03) d) All of the above
21. The most common complication of mumps is:
a) Myocarditis b) Orchitis 34. Acinic cell carcinomas of the salivary gland arise most
c) Uveitis d) Conjunctivitis often in the:
(AIPG -03) a) Parotid salivary gland b) Minor salivary glands
22. Adenoid cystic ca rein oma is also known as c) Submandibular salivary gland
a) Cylindroma b) Pindborg tumor d) Sublingual salivary gland
c) Warthins tumor d) Pleomorphic adenoma (COMEDK-05)
(AP -ZK) 35. All of the following is the extraglandular manifestation
23. Sialolith in the excretory duct, will result in: of primary sjogren's syndrome except:
a) chronic sialadenitis b) mucous retention cyst a) Raynaud's phenomena b) Arthritis
c) pleomorphic adenoma d) rupture of the duct c) Lymphoma d) Thrombocytopenia
(KAR -98) (PGI-06)
24. Parotid fatty change is sign of: 36. Mixed tumours of the salivary glands are:
a) Aging b) Alcoholism a) Most common in submandibular gland
c) Malnutrition d) None of the above b) Usually malignant
(AIPG -93) c) Most common in parotid gland
25. Which tumour does not occur in minor salivary gland? d) Associated with calculi
a) Pleomorphic adenoma b) Adenocarcinoma (AIPG-06)
c) Mucoepidermoid carcinoma 37. "Xerostomia" is seen in all of the following EXCEPT in
d) Warthin's tumour a) Anticholinergic drugs b) Dehydration
(AIPG -93) c) Sjogren's syndrome d) Oral sepsis
26. Leafless fruit laden tree or Cherry-blossom appearance (COMEDK-05)
on a sialogram indicates: 38. Cribriform, Honey comb (or) swiss cheese histology
a) Mucoepidermoid cell carcinoma pattern is seen in
b) Acinar cell carcinoma a) Adenoid cystic carcinoma
c) Sjogren's syndrome d) Pleomorphic adenoma b) Pleomorphic adenoma
(PG! -99) c) Acinic cell carcinoma d) Clear cell carcinoma
27. Sjogren's syndrome includes all except: (COMEDK-06)
a) Xerostomia b) Keratoconjunctivitis 39. Commonest salivary gland tumour in children
c) Arthritis d) Lymphoma a) Lymphoma b) Pleomorphic adenoma
(AIIMS -92) c) Adenoid cystic carcinoma
28. Tender submandibular swelling is mostly due to: d) Mucoepidermoid carcinoma
a) Ludwig's angina b) Stone or Sialolithiasis (COMEDK-06)
c) Enlarged lymph nodes d) All of the above 40. Adeno lymphoma refers to -
(AIPG -2K) a) Adeno carcinoma b) Adeno cystic lymphoma
29. Dilation of the salivary duct secondary to epithelial atrophy c) Warthin's tumour d) Pleomorphic adenoma
as a result of repeated inflammatory or infectious process is (COMEDK-09)
a) Sialodochitis b) sialadenitis 41. Spindle Cell Carinoma is a variant of
c) Sialolithiasis d) Mucocele a) Pleomorphic Adenoma b) Adenoid cystic carcinoma
(KCET-2011) c) Basal cell carcinoma d) Squamous cell carcinoma
30. Saliva is increased by: (COMEDK-07)
a) Cholinergic drugs b) Anticholinergic drugs 42. Commonest site for ectopic salivary gland tumour is:
c) Adrenergic drugs d) None of the above a) Tongue b) Cheek
(AIIMS -89) c) Palate d) Neck
31. The recurrence of pleomorphic adenoma is attributed to (COMEDK-07)
a) Presence of an incomplete capsule 43. Chemical mumps' is synonymous with
b) Mixed origin a) Epidemic parotitis b) Iodine mumps
c) Absence of capsule d) Perineural spread c) Nutritional mumps d) Nonspecific mumps
(UPSC -2001) (KCET-07)
32. Xerostomia, enlargement of salivary and lacrimal glands 44. Break up time (BUT) test is done in
is seen in a) Sjogren's syndrome b) Multiple sclerosis
a) Sicca syndrome b) Sjogren's syndrome c) SLE d) Myasthenia gravis
c) Mickulicz's disease d) None of the above (COMED-2K5)
(KAR -98)

21) B 22) A 23) B 24) B 25) D 26) C 27) D 28) C 29) A 30) A 31) A 32) C 33) D
34) A 35) D 36) C 37) D 38) A 39) D 40) C 41) D 42) C 43) B 44) A
45. Salivary duct calculi
_r ORAL PATHOLOGY & MEDICINE
a) Parotid b) Submanidbular
a) Produce pain on eating c) Sublingual d) Minor
b) Are commonest in the parotid ducts (KAR- 2013)
c) Are common cause of acute parotitis 56. The following are the high grade salivary gland
d) Are associated with hypercaleaemic states malignancies, EXCEPT
(KCET-08) a) Basal cell adenocarcinoma
46. Sialoliths are stones found in the salivary duct and gland b) Mucoepidermoid carcinoma
and are primarily composed of c) Adenoid Cystic carcinoma
a) Hydroxyapatitie b) Potassium chloride d) Salivary duct carcinoma
c) Unknown compounds of phosphates (KAR- 2013)
d) Calcium chloride 57. Treatment for recurrent ranula is
(COMEDK-08) a) Incision b) Excision
47. In Xerostomia the salivary pH is: c) Excision with adjacent glands
a) Unaffected b) Low d) Marsupialization
c) High (COMEDK-14)
d) increased in morning & decreases in day 58. Sialograhy of normal salivary gland reveals the ductal
(AIIMS-07) architecture as having an appearance of
48. Pleomorphic adenoma is a) Sausage string b) Fruit-laden tree
a) a teratomatous tumour of the salivary gland c) Leafless tree d) Ball in hand
b) a neuroendocrine cell tumour (COMEDK-14)
c) multiple ectopic tissue proliferating in the salivary gland 59. Microscopically epimyoepithelial islands are typically
d) myoepithelial or ductal reserve cell origin seen in
(AP-09) a) Sjogren's syndrome b) Mucoepidermoid carcinoma
49. Which of the following statements about Sjogren's c) Epithelial-myoepithelial carcinoma
syndrome is incorrect - d) Myoepithelioma
a) MRI shows salt aind pepper appearance (AP-14)
b) Lacrimal gland enlargement is common 60. Which of the following features distinguish Necrotizing
c) Minor salivary gland biopsy is a diagnostic test Sialometaplasia from squamous Cell carcinoma?
d) Pilocarpine is the most useful and convenient drug in its a) Majority of intraoral cases present with ulceration and
treatment pain in the later stages
(KCET-09) b) Varying degrees of lobar necrosis is seen
so. Chocolate-coloured fluid is seen in the cystic space of c) Squamous metaplasia of excretory ducts and acini are
a) Dentigerous cyst b) Odontogenic keratocyst noted
c) Unicystic amelob lastoma d) Nests of squamous epithelium with several residual
d) Papillary cystadenoma lymphomatosum ductal lamina within them are noticed.
(COMEDK-06) (COMEDK-15)
51. Intra oral pleomorphic adenoma usually occurs in 61. The Salivary gland tumor which is associated with
a) Buccal mucosa b) Upper lip smoking
c) Floor of the mouth d) Palate a) Cannicular adenoma b) Pleomorphic adenoma
(C0MEDK- 2013) c) Warthin tumour d) Ductal papilloma
52. A Salivary gland tumour, which histologically shows a (APPG-15)
double layer of epithelial cells based on a reactive 62. The soft, fluctuating, translucent bluish lesion depicted
lymphoid stroma, is in the above picture could be due to
a) Pleomorphic adenoma b) Mucoepidermoid carcinoma
c) Acinic cell tumour d) Warthin tumour
(C0MEDK- 2013)
53. Sjogren's syndrome affects
a) Exocrine glands b) Paracrine glands
c) Endocrine glands d) Autocrine glands
(C0MEDK- 2013)
54. In which one of the following salivary gland tumors, the a) Hemangioma b) Gingival cyst
tumor is composed of "intermediate cells" histologically? c) Ranula d) Mucocoele
a) Adenoid cystic carcinoma
b) Mucoepidermoid carcinoma 63. What percentage (%) of sub-lingual salivary gland
c) Pleomorphic adenoma d) Warthin's tumour tumours is malignant?
(AP- 2013) a) 80 b) 85
55. Which of the following salivary gland tumors has highest c) 90 d) 95
rate of malignant transformation? (COMEDK-15)
45) A 46) A 47) B 48) D 49) B 50) D 51) D 52) D 53) A 54) B 55) D 56) A 57) C
58) C 59) A 60) D 61) C 62) C 63) B
Dental 9'ul-6e � - - - - - -

4. DISEASES OF SALIVARY GLANDS - ANSWERS


1. 'A' [Shafer 7th ed 546/ 6th ed 544) Sialosis / Non-inflammatory, non-neoplastic
The higher rate of sialolith formation in the submandibular Sialadenosis enlargement of the salivary gland.
gland is due to (80-90% cases occur in submandibular gland)
Sialdenitis Inflammatory disease of salivary glands.
• Torturous course of Wharton's duct.
Sialadenoma Papillomas of salivary gland excretory
• Higher calcium and phosphate levels.
papilliferum ducts.
• Dependent position of submandibular glands that make
Congenital occlusion or absence of one or
them more prone to stasis. Atresia
major salivary gland ducts.
Sialoliths are most common in the
Sialorrhea Increased salivation (AP-2012)
a) Wharton's duct (AP- 2013)
Occurrence of salivary duct stones in
Sialolithiasis
2. 'D' (Shafer 7th ed 227 Tab 3-6/ 6th ed 222 Tab 3-6) salivary glands and its ducts.
Parotid tumor may cause facial palsy, while adenoid cystic Sialodochitis Inflammation of salivary gland ducts.
carcinoma can cause multiple nerve lesions especially of
Drugs that increase the salivation
lingual, facial or hypoglossal nerves. Sialogogue
Eg:- Para sympathomimetics, Cholinergics
3. 'C' (Shafer 7th ed 224 Table 3-1/ 6th ed 544) Drugs that decrease the salivation
About 60-70% of salivary gland tumors arise in parotid glands. Eg:- Anticholinergic (Atropine)
Anti- Sympathomi metics,
4. 'A' [Shafer 7th ed 229/ 6th ed 226) sialogogues Antihistamines,
Papillary cystadenoma lymphomatosum or Warthins tumor Antidepressents
is a benign salivary tumor occurring almost exclusively in Antihypertensives.
parotid gland.
11. 'A' (Shafer 7th ed 224 Table 3-1/ 6th ed 220)
5. 'C' (Shafer 7th ed 248/ 6th ed 256) Pleomorphic adenoma is most common salivary gland tumor
Necrotising sialometaplasia is a benign, inflammatory {60% cases) and most common parotid gland tumor {85%
reaction of salivary gland that mimics a salivary gland cases).
malignancy. Etiology is mostly due to local ischemia. Most
common intraoral site· is palate followed by buccal mucosa 12. 'D' (Shafer 7th ed 248/ 6th ed 244-45]
and the type of necrosis is coagulative necrosis. Similar type
of lesions in skin are called as syringometaplasia. 13. 'A' [Shafer 7th ed 37/ 6th ed 36)
Caries associated with xerosto mia typically affect the
6. 'A' [Shafer 7th ed 224/ 6th ed 220) gingival 3 rd and the incisal cusp tips.

7. 'C' (Shafer 7th ed 235/ 6th ed 233] 14. 'B' [Shafer 7th ed 238/ 6th ed 234)
Central mucoepidermoid carcinoma of jaw is more common
in mandible, especially in premolar-molar area. These lesions 15. 'A' (Shafer 7th ed 234/ 6th ed 230)
originate due to
• Entrapment of retromolar glands within the mandible.
16. 'C' [Burket's 11th ed 216 Table 5/ 10th ed 222)
• Due to inclusion of remnants of submaxillary glands
17. 'C' [Burket's 11th ed 195/ 10th ed 238)
within the mandible.
18. 'B' [Shafer 7th ed 351/ 6th ed 346)
8. 'C' (Shafer 7th ed 249/ 6th ed 245) Bilateral, rubbery or elastic salivary gland swelling frequently
Mikulicz's disease or benign lymphoepithelial lesion is a
elevating the ear is characteristic of mumps.
autoimmune disease closely related to sjogren's syndrome and
it exhibits both inflammatory and neoplastic characteristics.
19. 'B' [Shafer 7th ed 544/ 6th ed 543)
Ranula is a form mucocele frequently associated with
9. 'A' [Burket's 11th ed 194/ 10th ed 242) sublingual gland. Thus it causes a swelling in the floor of
During bimanual palpation of submandibular gland, one mouth below the tongue.
hand is placed intra orally on the floor of mouth and other
extra orally below mandible. 20. 'A' [Shafer 7th ed 543/ 6th ed 543)
Mucocele mostly originates due to rupture of salivary duct
10, 'B' [Shafer 7th ed 24 7/ 6th ed 244] following trauma that results in pooling of saliva and
Sialosis / sialadenosis is asymptomatic salivary gland
consequent swelling. It is of 2 types.
enlargement. The preauricular or retromandibular portion of
a) Extravasation mucocele - more common type
parotid gland is commonly affected. Etiology may include
hormonal problems, alcoholic cirrhosis or due to malnutrition. b) Retention mucocele - true retention cyst
21. 'B' [Shafer 7th ed 351/ 6th ed 346]
r ORAL PATHOLOGY & MEDICINE
32. 'C' [Shafer 7th ed 249/ 6th ed 245]
Mumps is characterized by triad of pathological involvement­
epidemic parotitis, orchitis-oophon"tis , and pancreatitis. 33. 'D' [Shafer 7th ed 36/ 6th ed 35]
Meningoencephalitis, epididymitis, and encephalitis are Aplasia occurs for unknown reasons as an isolated finding
other complications of mumps. or in conjunction with other developmental defects such as
hemifacial microsomia, the LADD syndrome and mandibulo­
When mumps involves adult male, orchitis is a great danger. facial dysostosis (Treacher Collins). In the more severe cases,
Involvement of pancreas causes an increase in serum lipase enzyme. the ensuing xerostomia causes clinical problems. Salivary loss
leads to increased caries, burning sensations, oral infections,
22. 'A' [Shafer 7th ed 238/ 6th ed 234] taste aberrations and difficulty with denture retention.
In cylindroma, the basal cells are characteristically
arranged in "Honey-Comb" or "Swiss cheese" pattern. 34. 'A' [Shafer 7th ed 234/ 6th ed 230]
The connective tissue surrounds the tumor cells in form of The acinic cell carcinoma closely resumbles the pleomorphic
cylinders, from which it derives the name "Cylindroma". adenoma in gross appearance, tending to be encapsulated and
lobulated. Although this tumor has been reported occurring
23. 'B' [Shafer 7th ed 543/ 6th ed 544] chiefly in the parotid, with more than 80% of the cases
occurring in the parotid glands, it does occur occasionally in
24. 'B' [Burket's 11th ed 210/ 10th ed 256] the other major glands and in the accessory intraoral glands.

25. 'D' [Shafer 7th ed 230/ 6th ed 226] 35. 'D' [Shafer 7th ed 250/ 6th ed 247]
Warthin's tumor (second most common tumor in salivary • Dry mouth
glands) occurs almost exclusively in parotid glands. Very
rarely it involves submaxillary gland. • Dry eyes
• Associated with HLA system.
26. 'C' [Shafer 7th ed 251/ Burket's 10th ed 256] • HLA-DR3 and H LA-B8 are associated
Sialographs of Sjogren's syndrome demonstrate, punctuate, with primary form and HLA-DRW52
cavitary defects which produce a "cherry blossom" or is associated with both primary and
"branchless fruit laden tree" effect. secondary form
Primary • Extraglandular Manifestations
27. 'D' [Shafer 7th ed 250/ 6th ed 247]
Sjogren's - Arthritis
Keratoconjunctivitis sicca, xerostomia and rheumatoid
syndrome or - Raynauds Phenomenon
arthritis are characteristic triad of Sjogren's syndrome.
Sicca complex
- Lymphoma
28. 'C' [Shafer 7th ed 509] - Pulmonary Involvement
The important contents of submandibular space are
submandibular salivary gland and submandibular - GIT and Hepatobiliary Features
lymph nodes. Infections of the drainage area including - Renal Involvement
dental infections commonly causes submandibular - Vasculiti s
lymphadenopathy. Hence option 'C' is the best answer. - Neuromuscular Involvement
29. 'A' [Text book of dental and maxillofacial radiology Freny - Autoimmune Thyroid Disease
R Karjodkar 2nd ed 869] • Dry mouth
Sialodochitis or ductal sialadenitis is an inflammation of the Secondary • Dry eyes
ductal system of the salivary glands, commonly parotid and Sjogren's • Connective tissue disorders like SLE,
submandibular glands. Dilatation of the ducts is a common syndrome polyarteritis nodosa, polymyositis or
feature. When interstitial fibrosis develops, characteristic scleroderma.
sialographic picture of sausage - string appearance is seen,
where ductal dilatation is seen alternatively with strictures. • Chronic bilateral lacrimal gland
Benign lympho­ enlargement associated with
Management includes antibiotics ( when secondary infection epithelial
sets in), increased fluid intake and sialogogues to promote enlargement of salivary glands
lession or
drainage. mikulicz • Lacrimal gland enlargement, parotid
syndrome gland enlargement, dry mouth and
30. 'A' [Burket's 11th ed 214/ 10th ed 260] dry eyes are classic signs.

31. 'A' [Shafer 7th ed 226/ Burket's 10th ed 261] 36. 'C' [Shafer 7th ed 224/ 6th ed 219-20]
One of characteristic feature of pleomorphic adenoma is the
microscopic projections of tumor outside the incomplete 3 7. 'D' [Shafer 7th ed 3 7/ 6th ed 36 J
capsule. Because of its microscopic projections this tumor
requires a wide resection to avoid recurrence. During surgery, 38. 'A' [Shafer 7th ed 239/ 6th ed 234]
care should be taken not to damage the facial nerve.
39. 'D' [Shafer 7th ed 236/ 6th ed 232]
Dental 9'ul-6e
40. 'C' [Shafer 7th ed 229/ 6th ed 2] 46. 'A' [Burket's 11th ed 201/ 10th ed 245]
Adenolymphoma or papillary cyst adenoma Lym phomatosum The sialoliths are usually crystalline in nature and are
are other names of Warthin's tumor. composed primarily of hydxyapatite. The chemical
composition is calcium phosphate and carbon, with trace
41. 'D' [Shafer 7th ed 123/ 6th ed 118] amounts of magnesium, potassium chloride, and ammonium.
Spindle cell carcinoma resembles a squamous cell carcinoma 50% of parotid gland sialoliths and 20% of submandibular
but is associated with a spindle cell component. Proliferation sialoliths are poorly calcified. This is clinically significant
and "dropping off" of basal cells to spindle cell elements is because such sialoliths are radiographically detectable.
an important histologic phenomenon.
4 7. 'B'
42. 'C' [Shafer 7th ed 224 Table 3-2]
48. 'D' [Shafer 7th ed 224/ 6th ed 220]
43. 'B' [Shafer 7th ed 354/ 6th ed 347) Numerous theories have been advanced in explaining the
Types of mumps histogenesis of pleomorphic adenoma. Currently, these
A) True mumps or viral mumps: thoeries center around the myoepithelial cell and a reserve
Caused (RNA) by polymyxovirus cell in the fotercalated duct. Studies have confirmed
the presence of both ductal and myoepithelial cells in
B) Non specific mumps: pleomorphic adenomas. It follows that possibly either or
i) Chronic non-sepcific sialdenitis due to salivary both may play active roles in the histogenesis of the tumor.
duct calculi.
ii) Acute postoperative parotitis due to retrograde 49. 'B' [Shafer 7th ed 250-1/ 6th ed 247)
infection of parotid gland by microorganisms ascending
parotid duct. The microorganisms involved are usually 50. 'D' [Shafer 7th ed 231/ 6th ed 226]
Staph. aureus, Staph. pyogenes, and Strep. Viridans. Warthin's Tumor is made up of two histologic components:
epithelial and lymphoid tissue. As the name would indicate,
iii) Nutritional "mumps" occurs as chronic,
the lesion is essentially an adenoma exhibiting cyst
asymptomatic, bilateral enlargement of the salivary
formation, with papillary projections into the cystic spaces
glands in populations suffering from malnutrition.
and a lymphoid matrix showing germinal centers. There is
iv) Chemical mumps or iodine mumps due to frequently an eosinophilic coagulum present within the
administration of either inorganic or organic iodine. cystic spaces, which appears as a chocolate - colored fluid
in the gross specimen.
44. 'A' [Burket's 9th ed 426)
• Break-up Time (BUT) (KCET-11), Schirmer's Test, and the 51. 'D' [Shafer 7th ed 225/ 6th ed 221]
quantitative rose bengal dye test are the three tests used Pleomorphic adenomas or mixed tumor occur most commonly
by ophthalmologist to evaluate lacrimal gland function in the parotid gland. About 8% occur in minor salivary
in Sjogren syndrome patients. glands, intraorally; the most common site being the palate.
• The salivary gland function in Sjogren syndrome patients
is measured by: 52. 'D' [Shafer 7th ed 230/ 6th ed 226]
- Salivary flow rate Warthin's tumor is histologically made of 2 components­
epithelial and lymphoid tissue. The other names for this
- Minor salivary gland biopsy
neoplasm are papillary cystadenoma lymphomatosum and
- Salivary scintigraphy (99m Tc - Pertechnetate) adenolymphoma that suggests an adenoma exhibiting cystic
• The normal salivary flow rate of each parotid gland is 5ml/ spaces and a Lymphoid matrix showing germinal centers.
min but in SS patients, this flow rate will be 0.5ml/min. The epithelial component consists of a bilayered oncocytic
• The total uptake of radioisotope ggm Tc - Pertechnetate epithelium thrown into papillary projections into the cystic
is less in SS patients compared to normal salivary glands. space. There is an eosinophilic coagulum present within the
cystic spaces, which appears as a chocolate-coloured fluid
• The sialography reveals formation of punctate, cavitary
in the gross specimen.
defects filled with radiopaque contrast media producing
a Cherry blossom or branchless fruit-laden effect. 53. 'A' [Shafer 7th ed 250/ 6th ed 245)
• MRI of SS patients shows a "Salt and Pepper" appearance Cystic fibrosis is also a disease of exocrine glands.
of the glands.
54. 'B' [Shafer 7th ed 236/ 6th ed 232)
45. 'A' [Shafer 7th ed 546/ Burket's 10th ed 245) Mucoepidermoid carcinoma is composed of mucous secreting
In sialolithiasis, the degree of symptoms are dependent cells, epidermoid (squamous) cells and intermediate cells.
on the extent of salivary gland duct obstruction. Typically Intermediate cells are a group of highly prolific, basaloid
eating will iniate the salivary gland swelling. The stone cells, larger than basal cells, but smaller than squamous
totally or partially blocks the flow of saliva, causing salivary cells and are believed to be the progenitor of epidermoid
pooling within the ducts and the gland body. Since the and mucous cells.
glands are encapsulated, there is little space for expansion
and any attempt to enlarge causes pain.
55. 'D' [Check Explanation Below]
r ORAL PATHOLOGY & MEDICINE
60. 'D' [Shafer 7th ed 248]
Tumours of salivary gland tissue constitute about 5% of Necrotising sialometaplasia is a non-neoplastic
head and neck tumors and affect major salivary glands five inflammatory condition of the salivary glands, with clinical
times more than the minor salivary glands. The incidence of and histopathologic features similar to squamous cell
malignancy among salivary gland tumours varies inversely or mucoepidermoid carcinoma. They usually present as
with the size of the gland. About 15% of parotid tumors, crateriform painless ulcers in the palate, sub mucosa[ nodules
50% of submandibular gland tumours, and 90% of minor and may cause erosion of the underlying cortical bone
salivary gland tumours are malignant. Approximately 70% of frequently. The microscopic features include coagulative
all minor salivary gland cancers of oral cavity occur on the necrosis of glandular acini and squamous metaplasia of its
hard palate. ducts. Pseudoepitheliomatous hyperplasia may be seen but
the cytological features of atypia are absent.
Since 70% of salivary gland tumours occur in the parotid
and 85% of these are benign, the majority of salivary gland 61. 'C' [Shafer 7th ed 230]
tumours are benign. The most common benign salivary Warthins tumor or papillary cystadenoma lymphomatosum
gland tumour is the benign mixed tumour or pleomorphic or adenolymphoma is the second most common tumor of
adenoma, which accounts for 70% of parotid tumours and salivary glands, found almost exclusively in parotid gland. A
50% of all salivary gland tumours. Mixed tumours are more strong association between development of Warthins tumor
common in women than in men, with peak incidence in and smoking had been documented, though the precise
5th decade. Warthin's tumour is the next most common mechanism is unclear.
benign, accounts for about 5% of parotid tumours. Warthin's
tumours are usually cystic, typically occur in men in the 62. 'C' [Shafer 7th ed 544]
sixth decade and occur almost exclusively in the parotid Ranula is a form of a large mucocele that occurs in the floor
gland. Mucoepidermoid cancer is the most common parotid of the mouth in association with ducts of submaxillary or
cancer (AIIMS -2012). sublingual gland. It is a slowly enlarging, painless mass on
one side of floor of the mouth, causing elevation of tongue.
56. 'ti: [Shafer 7th ed 229/ 6th ed 237] When superficial, it gives a bluish colour, similar to the belly
Treatment for stage I salivary gland cancer depends on of a frog, hence the name ranula.
whether the cancer is low-grade (slow growing) or high­
grade (fast growing). Basal cell adenocarcinoma of salivary
glands is an uncommon lesion that occurs almost exclusively
at major salivary glands. It is a low grade malignant
neoplasm, cytologically similar to basal cell adenoma, but is
infiltrative and has a small potential for metastases.

57. 'C' [Shafer 7th ed 544]


If the mucous retention phenomenon is simply excised,
there is a possibility of recurrence. However, if the
associated adjacent salivary gland is also removed, chances
of recurrence are minimised.

58. 'C' [White & Pharoah 5th ed 663]


On a sialogram of a rnormal salivary gland, the ductal system
appears as tree limbs, followed by a bloomed appearance
after acinar filling. Sausage string appearance is seen in
sialodochitis (Inflammation of the ductal system), filling
defects are seen in radiolucent salivary stones, branch
less fruit laden appearance in Sjogrens syndrome due to
sialectases and ball in hand appearance in benign salivary
gland tumors.

59. 'A' [Shafer 7th ed 251]


The three types of histologic alterations seen in Sjogren's
syndrome are as fol.lows:
• Intense lymphocytic infiltration of the gland replacing
the acinar structures, but preserving the lobular
architecture.
• Proliferation of the ductal epithelium and myoepithelium
to form epimyoepithelial islands
• Atrophy of the glands sequential to the lymphocytic
infiltration.
63. 'B' (Check Explanation Below]
SALIVARY GLAND TUMORS

Adults (85%) Children '(15%)


• Parotid is most commonly involved
• About 65% of all salivary tumors are benign.
• Mucoepidermoid carcinoma & Acinic cell

......
carcinomas are most common malignant tumors

Parotid (80%) Submandibular (10-15%) Sublingual & Minor salivary (5-10%)


t t C
CD
=
Benign (50%) Malignant (50%) Benign (50%) Malignant (50%) Sublingual Minor salivary -+
0
• Pleomorphic adenoma is most common Mucoepidermoid Pleomorphic Adenoid cystic • About 7% of all pleomorphic • Pleomorphic adenoma is
tumor (90%) carcinoma is the adenoma is most carcinoma adenomas are found in most common benign tumor �
common tumor (cylindroma) is sublingual and minor salivary
• Warthin's tumor (Papillary Cystadenoma most common • Approximately 60% �
malignant tumor of the most common glands (according to Burket 12th �
Lymphomatosum) is 2nd most common.
the parotid gland. malignant tumor • Approximately 60% ed.) or 90% of all minor
Commonly occurs in inferior pole of the
gland. (according to Burket 12th salivary gland tumors
ed.) or 85% of all sublingual (according to Bailey & love
• Oncocytoma & Sebaceous adenomas
salivary gland tumors 25th ed.) are malignant in
almost exclusively occurs in parotid
(according to Bailey & love nature
gland.
25th ed.) are malignant in • Adenoid cystic carcinoma
• Most myoepitheliomas occur in the nature and mucoepidermoid
parotid gland; the palate is the most
carcinoma are the most
common intraoral site.
common malignant tumors

Benign (75-80%) Malignant (20-25%) Others


• The pleomorphic adenoma is the most common tumor of the salivary • Mucoepidermoid • Most common ectopic salivary tissue is stafne cyst.
glands; overall, it accounts for about 60% of all salivary gland carcinoma is the most
• Superficial parotidectomy is commonest procedure for parotid
tumors. It is often called a mixed tumor because it consists of both common malignant
gland pathology.
epithelial and mesenchymal elements. About 85% of these tumors tumor of the salivary
are found in the parotid glands, 8% are found in the submandibular glands.
glands, and the remaining tumors are found in the sublingual and • Mucoepidermoid
minor salivary glands. This tumor represents the most common carcinoma is the most
neoplasm in each of the salivary glands and accounts for about 50% common malignant
of salivary tumors in the minor salivary glands. tumor of the salivary
• 0uctal papilloma is salivary gland tumor that arises from the qlands in bone.
excretory ducts.
_____ r ORAL PATHOLOGY & MEDICINE

5. DENTAL CARIES
1. Lactobacilli are numerous in carious lesions because they d) Decalcification zone, zone of sclerosis, bacterial invasion
a) Are the main causative agent (UPSC -2001)
b) Can produce insoluble extracellular polysaccharides 11. S. mutans is involved in dental caries initiation. Other
c) Can attach to smooth enamel surfaces bacteria also involved is:
d) Are secondary invaders a) S. sarcinus b) 5. macae
(MAN -94) c) S. sanguis d) 5. salivarius
2. Starch is considered to be less cariogenic than (PGI -2K)
monosaccharides and disaccharides because it 12. The dietary carbohydrate most likely involved in etiology
a) Does not diffuse through plaque of dental caries in man is:
b) Is rapidly hydrolysed in the mouth a) Glucose b) Sucrose
c) Enhances remineralisation c) Dextran d) Polysaccharide
d) Raises the pH in the oral cavity (AIPG -91)
(MAN -95) 13. Bacteria responsible for initiation of caries is:
3. The extra - cellular polysaccharide synthesized by a) Actinomyces b) Strep. mutans
cariogenic streptococci in the presence of excess sucrose c) Lactobacillus d) Strep. viridans
are best described as (PGI -95, AP -98)
a) Mucopolysaccharide b) Dextran-like glucan 14. What is the pH at which initiation of caries begins?
c) Glycogen-like glucan d) Amylopectin a) 4.3-4.5 b) 5.2-5.5
(MAN -98) c) 4.9-5.1 d) 3.5-4.5
4. Organism involved in smooth surface caries is (AIPG -95)
1. Streptococcus mutans 2. Actinomyces viscosus 15. Oral foci of miller's are seen in:
3. Lactobacillus 4. Campylobacter a) Dental caries b) Lichen planus
a) only 1 b) only 2 c) Herpes simplex d) Syphilis
c) 1 and 3 d) all of the above (AIPG -97)
(MAN-99) 16. In the earliest stages of carious lesion. There is loss of:
5. In a patient with reduced salivary flow the carious a) Enamel cuticle b) Interprismatic substance
incidence is c) Organic matrix d) Enamel lamellae
a) more than the patient with normal salivary flow (COMEDK-04, AIPG-92)
b) less than the patient with normal salivary flow 17. Smooth surface caries is characterized by spread of
c) unaffected d) none of the above caries in enamel and dentin as cones. These alignment
(MAN -99) in enamel and dentin is
6. Which of the following is cariogenic? a) Base to base b) Apex to base
a) Selenium b) Vanadium c) Apex to apex d) None of the above
c) Strontium d) Molybdenum (KAR -98)
(MAN -2K) 18. Least cariogenic among these
7. The number of dentinal zones in dentinal caries is a) Raw starch b) Cooked starch
a) One b) Two c) Sucrose d) Fructose
c) Four d) Five (PGI -2003)
(MAN -02) 19. Which tooth in the permanent dentition is the most
8. Dental caries is associated with susceptible to dental caries?
a) Streptococcus agalactiae a) Maxillary 1'1 premolar b) Maxillary 2 nd molar
b) Streptococcus mutans c) Mandibular 1st molar d) Mandibular 2 nd molar
c) Streptococcus bovis d) Streptococcus anginosus (KAR -01)
(MAN -01) 20. Turbid dentin in carious tooth is all EXCEPT -
9. Widely accepted theory of dental caries a) Zone of bacterial invasion
a) Proteolytic theory b) Proteolytic chelation theory b) Zone which can not be remineralized
c) Acidogenic theory d) Autoimmune theory c) Zone in which collagen is irreversibly denatured
(MAN -01) d) Zone that need not be removed before restoration
10. The correct order of microscopic zones of dentinal caries (KCET-09)
starting from the D.E. junction is 21. Streptococcus mutans is considered to be a principal
a) Zone of sclerosis, decalcification zone, bacterial invasion etiologic agent of caries because it produces organic
b) Bacterial invasion, decalcification zone, zone of sclerosis acids and it:
c) Zone of sclerosis, bacterial invasion, a) Forms a gelatinous matrix
decalcification zone b) Metabolizes substrate from saliva

1) D 2) A 3) B 4) A 5) A 6) A 7) D 8) B 9) C 10) B 11) C 12) B 13) B


14) B 15) A 16) B 17) B 18) A 19) C 20) D 21) A
Dental 9'ul-6e
c) Derives energy from enamel constituents 32. Which of the following organisms is found in deep
d) Lives symbiotically with lactobacillus carious lesions rather than in incipient lesions?
(AIIMS -94) a) Streptococci b) Lactobacilli
22. Miller put forth the acidogenic theory of dental caries in c) Veillonella d) Bateroides
the year­ (KAR -2K)
a) 1890 b) 1920 33. Pioneer bacteria in dental caries are in
c) 1924 d) 1980 a) Enamel b) Dentin
(KAR -99) c) Pulp d) Cementum
23. For a bacterium to be seriously considered in the etiology (AP -03)
of dental caries, it must: 34. The most pronounced effect on the oral microflora of a
a) Exist regularly in the dental plaque reduction in rate of salivary flow is a
b) Produce extracellular amylopectins a) Significant increase in number of oral bacteria
c) Be lethal for gnotobiotic animals b) Shift towards more acidogenic microflora
d) Produce intracellular dextrans c) Significant decrease in number of oral bacteria
(AIPG -03) d) Shift towards more aerobic microflora
24. Initiation of dental caries depends upon (KAR-04)
a) formation of large amount of acid. 35. Ammonia causes
b) Availability of carbohydrate food. a) Increase in plaque formation
c) Viscosity of saliva b) Increase in calculus formation
d) Localization of acid over tooth surface. c) Decrease in plaque formation
(AIPG -99) d) Causes precipitation of salivary proteins
25. Caries, all are true except: (AIPG-06)
a) Lactobacillus is main causative organism in plaque 36. Cavity formation in a tooth, due to dental caries is due to:
b) smooth surface caries occur due to streptococcus mutans a) Destructive potential of streptococcus mutans
c) Pit and Fissure caries can be prevented by using pit and b) Destructive potential of lactohacillus acidophilus
tissue sealants. c) Lateral spread of caries along DE junction and weakening
d) Fluorides help in reducing caries incidence of the outer covering enamel
(AIPG -02) d) Mastectomy force and unrelated to the extent of carious
26. The enzyme glucosyl transferase secreted by process
Streptococcus mutans synthesizes glucans from (AIPG-05)
a) Glucose b) Fructose 37. Animals maintained in a germ free environment did not
c) Sucrose d) Lactose develop caries even when fed on a high carbohydrate diet
(COMEDK-09) is given by:
27. The lateral spread of dental caries is facilitated mostly a) Gottlieb b) Miller
by the: c) Snyder d) Orland and Fizgerald
a) Enamel spindles b) Dentinoenamel junction. (COMEDK-06)
c) Enamel lamellae d) Striae of Retzius 38. Which of the following factors in the stephan's curve is
(AIIMS -03) related to the caries incidence and sugar intake:
28. The probable reasons for a high incidence of dental a) Physical form of sugar b) Frequency of sugar intake
caries in the teenage population relates most directly to: c) Ph of plaque d) Quantity of sugar intake
a) Rapid growth b) Frequency of sucrose intake (AIPG-06)
c) Negligence in visiting the dentist 39. Initiation of caries by streptococcus mutans is by the
d) Carelessness in oral hygiene habits. production of
(AIIMS -03) a) Dextranase and soluble dextran
29. In a caries-free individual the saliva has: b) Insoluble dextan and glucosyl transferase
a) Low buffering capacity for acids c) Soluble dextran and glycosyl itransferase
b) Medium buffering capacity for acids d) None of the above
c) High buffering capacity for acids (AIPG-04)
d) Independent of buffering capacity for acids 40. The attachment of the Actinomyces species to the tooth
(AIIMS -98) surface is facilitated by
30. Early invading bacteria in carious lesions are called a) Fimbriae b) Cilia
a) Microcosm b) Pioneer bacteria c) Flagella d) Pseudopodia
c) Advancing bacteria d) Anaerobic bacteria (COMED-2012)
(COMEDK-05) 41. Caries associated with pre-eruptive enamel hypoplasia?
31. Chemico-parasitic theory of dental caries is proposed by: a) Secondary caries b) Chronic caries
a) Miller b) G.V. Black c) Occult caries d) Incipient caries
c) Gottlieb d) Schwartz (AIIMS MAY-13)
(KAR -90, 99)

22) A 23) A 24) D 25) A 26) C 27) B 28) B 29) C 30) B 31) A 32) B 33) B 34) B
35) C 36) C 37) D 38) C 39) B 40) A 41) C
42.
r
Bacteria free zone of dentinal caries is
ORAL PATHOLOGY & MEDICINE

a) Zone of decomposed dentin


b) Zone of bacterial invasion
c) Zone of decalcification d) Zone of dentinal sclerosis
(AIIMS MAY-13)
43. The legend of worms, endogenous theories, chemical
theories & parasitic theory are early theories explaining
a) Dental Caries b) Gingivitis
c) Periodontitis d) Osteomyelitis

42) D 43) A
Dental 9'ul-6e � - - - - - -

5. DENTAL CARIES - ANSWERS


1. 'D' [Shafer 7th ed 424/ 6th ed 415] 10. 'B' [Shafer 7th ed 455/ 6th ed 451]

2. 'A' [Shafer 7th ed 438/ 6th ed 431] 11. 'C' [Shafer 7th ed 425/ 6th ed 416]

3. 'B' [Shafer 7th ed 425/ 6th ed 415-16] 12. 'B' [Shafer 7th ed 438/ 6th ed 430-31]
Streptococcus mutans by its action on sucrose synthesize
glucan and glucosyltransferase. Glucan or dextran causes 13. 'B' [Shafer 7th ed 423-24 Table 9-1/ 6th ed 415-16]
plaque to adhere to tooth surface and also acts as barrier Streptococcus Initiation of smooth surface mutans caries
against diffusion of saliva.
Streptococcus Another group of streptococci to cause
4. 'A' [Shafer 7th ed 423 Table 9-1/ 6th ed 415-16] sanguis caries after streptococcus mutans (PGI-05)
Pit & fissure caries in initiated by a wide variety of organisms Secondary invaders and helps in progression
Lactobacillus
like S. mutans, Lactobacillus and Actinomyces species as of carious lesion.
they colonize in these retentive areas. However, only Actinomyces Root surface caries.
S.mutans (very significantly) & S. salivarius (by chance)
have been proved to colonise smooth surfaces. 14, 'B' [Shafer 7th ed 428-29/ 6th ed 426]
5. 'A' [Shafer 7th ed 433/ 6th ed 427] 15. 'A' [Shafer 7th ed 454/ 6th ed 450]
Saliva contains anticariogenic substances like amylase or "Liquefaction foci" are seen in Dentin in advanced cases
ptyalin, lysozyme and buffering agents. So patients with of carious destruction. Liquefaction foci are ovoid areas of
reduced salivary secretion are more prone to caries. destruction parallel to dentinal tubules and are formed by
focal coalescence and breakdown of a few dentinal tubules.
6. 'A' [Shafer 7th ed 439/ 6th ed 433]
Dental caries is higher in persons residing in seleniferous 16. 'B' [Shafer 7th ed 447/ 6th ed 447]
areas and a decrease in caries rate is associated with The first change is loss of inter prismatic substance or inter­
increasing vanadium concentration. rod substance of enamel with increased prominence of rods.
Also, there is accentuation of incremental lines of retzius
7. 'D' [Shafer 7th ed 455/ 6th ed 451] (AIPG-09). Clinically, the earliest manifestation is an area of
Beginning pulpally alt the advancing edge of the lesion decalcification that resembles a smooth chalky white area.
adjacent to normal dentin, the dentinal zones are as follows.
17. 'B' [Shafer 7th ed 447-48/ 6th ed 444]
Zone 1 Zone of fatty degeneration of tome's fibres
Zone of dimtinal sclerosis characterized by 18. 'A' [Shafer 7th ed 422/ 6th ed 431]
Zone 2
deposition of calcium salts in dentinal tubules
Zone of decalcification of dentin, a narrow zone,
19. 'C' [Shafer 7th ed 432/ 6th ed 441]
Zone 3 Caries susceptibility of individual teeth in permanent
preceding bacterial invasion.
dentition:
Zone of bacterial invasion of decalcified but
Zone 4 • l't molars - Most commonly involved. (Mandibular 1 st
intact dentin.
molars followed maxillary 1 st molar)
Zone 5 Zone of decomposed dentin. • 2 nd molars
• Max. l't premolar and Mand. 2 nd premolar
Option 'B' four zones are seen in enamel caries. They are
• Max. incisors
• Max. canines and lower l't premolar
Translucent zone and lies at advancing front of
Zone 1 • Mandibular incisors and canines are least likely to
enamel lesion
develop lesions
Zone 2 Dark zone
Zone 3 Body of lesion Maxillary arch is more frequently involved by caries than
Zone 4 Surface zone mandibular arch.

8. 'B' [Shafer 7th ed 424/ 6th ed 415] In deciduous teeth, mandibular second molar is commonly
involved. (KAR- 96)
9. 'C' [Shafer 7th ed 421-2/ 6th ed 412]
Acidogenic theory of chemico-parasitic theory was proposed
by Miller. According to this theory, caries occurs in 2 stages
- Initial decalcification of enamel and dentin followed by
dissolution of softened residue.
20. 'D' [Check Explanation Below]
r ORAL PATHOLOGY & MEDICINE
31. 'K [Shafer 7th ed 421/ 6th ed 412]
According to sturdevant, clinically 5 different zones of
caries progression ar,e seen in dentin. They are 32. 'B' [Shafer 7th ed 424/ 6th ed 415]
Zone 1 Normal Dentin
33. 'B' [Shafer 7th ed 453/ 6th ed 449]
Zone 2: Zone of demineralization created Pioneer bacteria are seen in earliest stage of caries, when
Subtransparent by the acid from caries. No bacteria only few tubules are involved without any clinical evidence.
Dentin found. Capable of remineralization. The dentinal tubules may be filled with pure forms of bacteria
Zone 3: i.e., one tubule may be filled with caecal forms while the
Softer than normal dentin. No bacteria
Transparent adjacent may contain either bacilli or thread forms, but not
present. Capable of remineralization.
Dentin both.
Zone 4: Zone of bacterial invasion. Tubules
Turbid zone must with bacteria. Not capable of 34. 'B' [Shafer 6th ed 427-28]
be removed. remineralization.
35. 'C' [Shafer 7th ed 462/ 6th ed 459]
Outermost zone. Decomposed dentin
Zone 5:
filled with bacteria. Must be totally 36. 'C' [Shafer 7th ed 441-42/ 6th ed 436]
Infected dentin
removed.
37. 'D' [Shafer 6th ed 414]
21. 'A' [Shafer 7th ed 425/ 6th ed 430-31]
38. 'C' [Shafer 7th ed 428/ 6th ed 420 Fig. 9-3]
22. 'K [Shafer 6th ed 412] Stephen curves:
23. 'A' [Shafer 7th ed 426)
• Previously it was thought that dental plaque is permeable
to carbohydrates at a restricted site for a relatively long
'D' [Shafer 7th ed 426/ 6th ed 416] time. But Stephen showed that carbohydrates entering
24.
the plaque were degraded rapidly.
25. 'A' [Shafer 7th ed 424/ 6th ed 415] • With the help of antimony microelectrode, he found
that the pH of plaques in different persons varied with
26. 'C' [Shafer 7th ed 424-25/ 6th ed 416) an average of about 7 .1 in caries-free persons and 5.5
The most important substrate for the involvement of S. mutans in persons with extreme caries activity. The maxillary
in the caries is the sucrose. The transport of sucrose into the anterior teeth exhibited a greater pH drop in the plaque
cell interior is accompanied by direct phosphorylation for than mandibular teeth.
energy utilization through the glycolytic pathway leading • Stralfors carried out studies similar to Stephen, and in
to lactic acid production and degradation of sucrose to free addition, correlated the lowest level of pH fall after
glucose and fructose by invertase. S.mutans polymerize the the carbohydrate rinse with the lactobacillus count.
glucose and the fructose moieties of sucrose to synthesize He found that persons with a higher pH have a lower
glucans and fructans, which are two types of extracellular lactobacillus count. He also reported that the plaque
polymers. The enzymes responsible for the synthesis of has a much greater buffering capacity than saliva due to
the extracellular glU1cans and fructans are called glucosyl presence of carbohydrates a11d protein.
transferase and fructosyl transferases respectively.
39. 'B' [Shafer 7th ed 425/ 6th ed 430-31]
27. 'B' [Shafer 7th ed 452/ 6th ed 436]
40. 'A' [J Bacteriol. 2011 July; 193(13): 3197-3206]
28. 'B' [Shafer 6th ed 413] The fimbriated cell surface of Actinomyces naeslundii give
the microbe a " fuzzy" appearance. It is the type 1 and type
29. 'C' [Shafer 7th ed 434/ 6th ed 426) 2 fimbriae that allow the bacteria to adhere to different
surfaces. The type 1 fimbriae, for example, help them attach
30. 'B' [Shafer 7th ed 453/ 6th ed 449) to tooth surfaces and collagen through adhesion-receptor
In early stages of caries, when only a few tubules are binding, while type 2 fimbriae bind to �-linked galactose
involved, microorganisms may be found penetrating these and galactosamine appendages on epithelial and bacterial
tubules before clinical evidence of the carious process. surfaces (Tang et al., 2004).
These organisms are called as "Pioneer bacteria". Later the
study of individual tubules show pure forms of bacteria 41. 'C' [White & Pharoah 6th ed 246]
in each tubuie i.e., one tubule may be filled with caecal Occult disease refers to disease that presents no clinical
forms while the adjacent tubules may contain only bacilli or signs or symptoms. Occult diseases in the jaws include a
thread forms. combination of dental and intraosseous findings. Dental
findings may include incipient carious lesions, resorbed or
Cocci are commonly responsible for smooth surface caries dilacerated roots or hypercementosis. Intraosseous findings
while filamentous organisms are responsible for root surface include osteosclerosis, unerupted teeth, periapical disease
caries. and a wide variety of cysts and benign and malignant tumors.
Dental 9'ul-6e � - - - - - -

Small carious lesions, resorption of root structure and bony


lesions may go unnoticed until signs and symptoms develop.

42. 'D' [Shafer 7th ed 454]


Sclerotic dentin seals off the dentinal tubules and prevents
further penetration of microorganisms. The various zones in
dentinal caries include fatty degeneration of the odontoblast
process, dentinal sclerosis, and decalcification of dentinal
tubule and zone of microbial invasion.

The zone of microbial invasion contains proteolytic


organisms, predominantly in deeper layers and acidogenic
microbes more in early caries. Decalcification zone may have
pioneer bacteria, those bacteria that may have penetrated
the dentinal tubules.

43. 'A' [Shafer 7th ed 421]


The etiology of dental caries has been explained earlier by
the legend of worms, endogenous theories, chemical theory
and the parasitic theory. Legend of worms states that dental
decay or toothache was caused by worms. Endogenous
theories postulated that tooth decay originated within a
tooth itself. Chemical theory opined that an unidentified
'chymal agent' was responsible for dental caries. Parasitic
theory has attributed dental caries to microorganisms.
_____ r ORAL PATHOLOGY & MEDICINE

6. PULP AND PERIAPICAL INFECTIONS


1. All of the following statements about the typical features c) Staphylococci d) Legionella infection
of a periapical gran uloma are true EXCEPT? (KAR -98)
a) It consists of proliferating granulation tissue 12. The most common organism involved in a periapical
b) It can form only if the periapical bone is resorbed abscess is:
c) It shows evidence of local antibody production a) Strep. pyogens b) Strep. viridans
d) It results from immunologically mediated tissue damage c) p hemolytic streptococci
(MAN -04) d) Non hemolytic streptococci
2. Phlegmon is a (AIPG -96)
a) Sexually transmitted disease 13. The earliest radiographic sign of osteomyelitis is
b) Type of cellulites a) Solitary or multiple small radiolucent areas
c) Type of osteomyelitis d) Venereal disease b) Increased granular radioopacity
(MAN -94) c) Blurring of trabecular outlines
3. Most common cyst in oral region is d) rormation of sequestrum appearing as radiopaque patches
a) Medial cyst b) Radicular cyst ( KAR -98, AP-04)
c) Follicular cyst d) Naso labial cyst 14. Osteomyelitis begins as an inflammation of
(MAN -98, AP-06) a) Cortical bone b) Periosteum
4. Dental cyst c) Medullary bone d) Periosteum and inner cortex
a) Occurs from the reduced enamel epithelium (KAR -98)
b) Replaces the tooth to which it it attached 15. Focal sclerosing osteomyelitis is:
c) Is frequently seen with a missing tooth on the X-ray. a) Due to excessive periosteal !bone formation
d) Cystic lining of stratified squamous epithelium b) An extremely painful condition
(MAN -2K) c) Due to low grade chronic infection
5. The earliest response of pulpitis is: d) A common sequel following sequestrectomy
a) Cyst formation b) Calcification (AIIMS -03)
c) Hyalinization 16. A tooth with a 3 month history of pain, which was worse
d) Formation of dental granuloma when hot liquid were in mouth. After extraction, the
(AIPG -96) tooth was split open. The pulp chamber was completely
6. Garre's chronic non suppurative sclerosing osteomyelitis filled with pus. A few remnants of pulp tissue were found
is characterized clinically by in apical end. The condition is:
a) Endosteal bone formation a) Acute partial pulpits b) Acute total pulpits
b) Periosteal bone formation c) Suppurative pulpitis d) Strangulation of pulp
c) Resorption of medullary bone (AIPG -02)
d) Resorption of cortical bone 17. Chronic hyperplastic pulpitis i s
(AP - 98) a) necrotizing b) suppurative lesion
7. Abscess formation is particularly characteristic of c) proliferation of a chronically inflamed pulp
infections with which of the following microorganisms: d) also called as phoenix abscess
a) Viruses b) Rickettsiae (AIIMS -2K)
c) Streptococci d) Staphylococci 18. Periapical cyst is usually preceded by:
(AIPG -97) a) Periapical granuloma b) Periodontal abscess
8. Organism involved in cellulitis is: c) Periapical abscess d) All of the above
a) Strept. mutans b) Strept. pyogenes (AIPG -90)
c) Pneumococci d) Klebsiella 19. Acute osteomyelitis is most frequently caused by which
(AIPG -97) of the following microorganisms?
9. An acute apical abscess is usually a result of: a) Gonococcus b) Enterococcus
a) Periodontal pocket b) Occlusal interference c) Streptococcus d) Staphylococcus
c) Necrotic pulp d) Chronic gingivitis (AIPG -02, 03)
(KAR -98) 20. An asymptomatic tooth has deep caries on occlusal
10. Which of the following periapical conditions is often surface. Radiograph shows radiopaque mass at apex of
associated with a vital pulp? the tooth; this mass is most likely to be:
a) Apical cyst b) Apical scar a) Cementoma b) Condensing osteitis
c) Condensing osteitis d) Chronic apical periodontitis c) Chronic apical periodontitis
(KAR -01, AP -05) d) Acute apical periodontitis
11. The main causative agent of Ludwig's angina is: (AIIMS -95, 98)
a) Anaerobic streptococci b) Aerobic streptococci

1) C 2) B 3) B 4) D 5) C 6) B 7) D 8) B 9) C 10) C 11) B 12) A 13) C


14) C 15) C 16) C 17) C 18) A 19) D 20) B
Dental 9'ui-6e
21. A person experiences throbbing pain at night. It is due to: 31. Chronic periostitis in children is known as -
a) Acute pulpal degeneration a) Cherubism b) Garre's osteomyelitis
b) Acute periodontal abscess c) Histiocytosis X d) Tuberculous osteomyelitis
c) Chronic pulpitis d) Cellulitis (KCET-09)
(PGI -01) 32. Which of the following is more prone to osteomyelitis
22. Pain due to acute irreversible pulpitis is: a) Maxilla b) zygoma
a) Spontaneous b) Sharp-shock like c) palatine bone d) mandible
c) Lasting for short time d) Continuous (PG! -97)
(PGI -95) 33. The chronic osteomyelitis of the jaw consists of
23. Reversible pulpitis change to irreversible pulpitis a) Condensing osteitis b) Sclerotic cementa[ mass
primarily because of: c) Chronic diffuse sclerosing osteomyelitis
a) Vascular strangulation b) Reduced host resistance d) All of the above
c) Invasion of microorganisms
d) An increase in microbial virulence 34. The fascial spaces involved in Ludwig's angina are
(AIPG -03) a) Unilateral -submandibular & sublingual spaces
24. Which is not true of Ludwig's angina? b) Bilateral - submandibular & sublingual spaces
a) Usually arises from an infected molar c) Unilateral - submandibular, sublingual & submental spaces
b) Involves submandibular space. d) Bilateral -submandibular, sublingual & submental spaces
c) May need emergency tracheostomy (AIPG -04)
d) None of the above 35. Three stages in progression of acute odontogenic
(KAR -99) infection are
25. 0dontogenic epithelium responsible for the formation of a) Periapical osteitis, cellulitis, abscess
dental cyst is: b) Abscess, cellulitis, periapical osteitis
a) Cell rests of seirre b) Enamel organ c) Cellulitis, Abscess, periapical osteitis
c) Reduced enamel epithelium d) Periapical osteitis, abscess, cellulitis
d) Cell rests of malassez (KAR-04)
(KAR -90, 99) 36. The tooth most commonly involved in chronic focal
26. The caries of enamel surface Leads to accentuation of? sclerosing osteomyelitis is
a) Incremental lines of retzius a) Maxillary second molar b) Maxillary third molar
b) Perikymata c) Maxillary first molar d) Mandibular first molar
c) Imbrication lines of pickerill (COMEDK-06)
d) Wickham's striae 37. Cyst arising from rests of malassez is
(AIPG-09) a) Dental cyst b) Dentigerous cyst
27. Constant feature associated with a radicular cyst c) Radicular cyst d) Kerato cyst
a) An impacted tooth b) A missing tooth (COMEDK-06)
c) A non-vital tooth d) An anomalous tooth 38. Low grade infection which leads to Localized periosteal
(AP -98) reaction is:
28. Which of the following differentiates between condensing a) Garre's osteomyelitis b) Acute osteomyelitis
osteitis and benign cementoblastoma c) Condensing osteitis d) Local alveolar osteitis
a) Condensing osteitis is associated with vital teeth where (AIIMS-06)
as cementoblastoma is associated with non-vital teeth 39. Tiny linear or arc-shaped bodies, amorphous, brittle and
b) In condensing osteitis radiopacity is attached to tooth eosinophilic in reaction, found in association with some
where as in cementoblastoma it is not odontogenic cysts, are called
c) Cementoblastoma is associated with vital tooth where as a) Civattle bodies b) Russell bodies
condensing osteitis is associated with non-vital tooth c) Guarneri bodies d) rushton bodies
d) In cementoblastoma radiopacity is attached to tooth (COMEDK-07)
where as in condensing osteitis it is not 40. The cells most frequently found in a granuloma are
(AIPG -02) a) Mast cells b) Giant cells
29. Best way to differentiate a periapical cyst and a perapical c) Lymphocytes d) Neutrophilis
granuloma is (BHU-07)
a) Radiographically b) Histologically 41. Most used selective medium for streptococcus mutans is
c) Clinically d) None of the above a) Mac conkey medium b) Mitus salivarius bacitracin agar
(AP-10) c) Nutrient agar d) Tellurite medium
30. A diffuse spreading inflammatory lesion is due to (AIIMS -07)
bacterial enzyme 42. Most demineralised zone in enamel caries
a) Coagulase b) Hyaluronidase a) Translucent zone b) Body of lession
c) Peroxidase d) Bradykinin c) Dark zone d) Surface zone
(AIIMS -2K) (BHU-07)

21) A 22) A 23) C 24) D 25) D 26) A 27) C 28) D 29) B 30) B 31) B 32) D 33) D
34) D 35) A 36) D 37) C 38) A 39) D 40) C 41) B 42) B
r ORAL PATHOLOGY & MEDICINE
43. The gelatinous deposit adherent on the tooth surface is
called as:
a) Materia alba b) Plaque
c) Calculus d) All of the above
(MCET-07)
44. Streptococcus mutans produces an adhesive polymer
from sucrose, known as
a) Levans b) Lectins
c) Glucans d) Polyfructans
(COMEDK-08)
45. Liquefaction foci of Miller is a histopathological
observation in
a) Cemental caries b) Early enamel caries
c) Advanced enamel caries
d) Advanced dentinal caries
(UPSC-09)
46. Which of the following is cariostatic?
a) Selenium b) Magnesium
c) Cadmium d) Molybdenum
(KCET-10)
47. Progression of dental caries on pit and fissure occurs
from
a) Apex of the pit and fissure
b) Wide end of the pit and fissure
c) Lateral surface of the pit and fissure
d) Bottom of the pit and fissure
(COMEDK-10)
48. Which of the following represents a soluble polysaccharide
found in dental plaque and is formed from the fructose
moiety of sucrose ?
a) Levan b) Dextran
c) Amlyopectin d) Hyaluronic acid
(AP-09)
49. Plaque microflora can split carbohydrates. What does it
mean?
a) Sacchrolytic b) Saprophytic
c) Virulant d) Avirulant

50. True about caries, all except


a) Infectious and transmissible
b) Not due to microorganisms
c) Can develop in the absence of sucrose
d) Microorganisms play the most essential role

51. A 30 year old patient with radiographic appearance of


endosteal bone formation, sclerosed bone is likely to be
suffered from?
a) Garre's osteomyelitis
b) Chronic focal sclerosing osteomyelitis
c) Acute osteomyelitis d) Chronic osteomyelitis
(AIPG-14)

43) B 44) C 45) D 46) D 47) C 48) A 49) A 50) B 51) B


Dental 9'ul-6e � - - - - - -

6. PULP AND PERIAPICAL INFECTIONS - ANSWERS


1. 'C' [Shafer 7th ed 484-85/ 6th ed 481] 12. 'A' [Shafer 6th ed 490]

2. 'B' [Shafer 7th ed 504/ 6th ed 501) 13. 'C' [Shafer 7th ed 493)
In acute osteomyelitis little or no radiographic features
3. 'B' [Shafer 7th ed 488/ 6th ed 487) are seen for at least one or two weeks. Later, diffuse lytic
changes in the bone begin to ap'pear. Individual trabeculae
4. 'D' [Shafer 7th ed 489/ 6th ed 488] become fuzzy and indistinct and radiolucent areas begin to
appear.
5. 'C'
In earlier stage of chronic osteomyelitis, "mottled
6. 'B' [Shafer 7th ed 498/ 6th ed 497) appearance" is seen in radiographs. In established cases
Periosteal new bone formation is seen in: "moth-eaten appearance" is seen because of enlargement
• Caffey's disease or infantile cortical hyperostosis. ot medullary spaces and widening ot Volkmann's canals.
• Hypervitaminosis A.
14. 'C' [Shafer 7th ed 493/ 6th ed 492]
• Syphilis, leukemia, Ewing's sarcoma, etc.
• Garre's osteomyelitis. 15. 'C' [Shafer 7th ed 495/ 6th ed 494]

7. 'D' [Shafer 4th ed 504/ 6th ed 501] 16. 'C' [Shafer 7th ed 478/ 6th ed 479]
Most of streptococci produce hyauronidase, an enzyme that
dissolves hyaluronic acid (universal intercellular substance) 17. 'C' [Shafer 7th ed 479/ 6th ed 479]
and helps in spread of infection. Usually staphylococci Chronic hyperplastic pulpitis is due to excessive, exuberant
are not potent producers of hyaluronidase. So, there is no proliferation of chronically inflamed pulp tissue. It occurs
spread of infection and the infection becomes localized in exclusively in children and young adults and commonly
form of abscess. involves with large, open carious lesions. The teeth most
commonly involved are deciduous molars and first permanent
8. 'B' [Shafer 7th ed 504/ 6th ed 501) molars as they have an excellent blood supply because
Streptococcus are potent producers of Hyaluronidase and of their large root opening. The high tissue resistance
fibrinolysin that dissolve the hyaluronic acid and fibrin and reactivity in young persons accounts for the unusual
respectively and causes spread of infection into tissue proliferative property of the pulp tissue.
spaces resulting in cellulitis.
18. 'A' [Shafer 7th ed 488/ 6th ed 268)
9. 'C' [Shafer 7th ed 491/ 6th ed 490] The cells of periapical granuloma (predominantly
Abscess may develop directly from a necrotic pulp or more lymphocytes) increase in size by division at the periphery
commonly it originates in an area of chronic infection like and the cells in the center are separated from their source
periapical granuloma. of nutrition, becomes degenerated and liquefy. This results
in epithelium lined cavity filled with fluid, the Apical
10. 'C' [Shafer 7th ed 495/ 6th ed 494] periodontal cyst.

11. 'B' [Shafer 7th ed 510/ 6th ed 508) 19. 'D' [Shafer 7th ed 493/ 6th ed 492]
Ludwig's angina is severe form of cellulitis, usually begins in The most common organism cultured from the lesions
submaxillary space and secondarily involves the sublingual include staphylococcus aureus ( KCET-09} and Staph. albus.
and submental spaces. The 2 nd and 3 rd mandibular molars are
chief source of infection. Most cases of Ludwig's angina are 20. 'B' [Shafer 7th ed 495/ 6th ed 494]
mixed infections, streptococci are invariably present. In condensing osteitis, the periodontal ligament space is
widened and this is an important feature in distinguishing
If 2nd & 3 rd molars Submaxillary space is first involved it from benign cementoblastoma.
are involved as their apices lie below mylohyoid
ridge. 21. 'A' [Shafer 7th ed 477/ 6th ed 476-477)

If 2 nd premolar Sublingual space is first involved 22. 'A' [Shafer 7th ed 477/ 6th ed 479)
& l't molars are as their apices lie above mylohyoid
involved ridge. 23. 'C' [Shafer 7th ed 476-77/ 6th ed 478)

Ludwig's angina is caused due to pus collection in 24. 'D' [Shafer 7th ed 510/ 6th ed 508]
a) Submandibular region b) Parotid region The patients with Ludwig's angina manifests rapidly
c) Cardiac muscle d) Nasal cavity (COMEDK-14) developing board like swelling offlloor of mouth and elevation
r ORAL PATHOLOGY & MEDICINE
of tongue with difficulty in swallowing and breathing. If the 40. 'C' [Shafer 7th ed 485/ 6th ed 482)
infection involves neck, edema of glottis occurs and there Periapical granuloma mainly composed of macrophages,
is risk of death by suffocation. In such cases emergency lymphocytes and plasma cells, and less frequently with mast
tracheostomy is done to prevent suffocation. cells and eosinophils.

25. 'D' [Shafer 6th ed 488/ 5th ed 677) 41. 'B' [Shafer 7th ed 424/ 6th ed 415)

26. 'A' [Shafer 7th ed 447/ 6th ed 442) 42. 'B' [Shafer 7th ed 451/ 6th ed 445)
Refer synopsis.
27. 'C' [Shafer 7th ed 488/ 6th ed 487]
43. 'B' [Shafer 7th ed 426/ 6th ed 417)
28. 'D' [Shafer 7th ed 495/6th ed 494) Dental Plaque is the self, non-mineralized gelatinous
In condensing osteitis, radiographs demonstrate radiopacity bacterial deposit found on teeth and dental prosthesis.
which is not attached to tooth, entire root outline is visible, Materia alba is thick mass found on the teeth and calculus
laminadura intact and periodontal ligament is widened. is mineralized plaque.
These features differentiate it from cementoblastoma, in
which the radiopacity is attached to tooth, laminadura 44. 'C' [Shafer 7th ed 425/ 6th ed 416)
absent and root outline is not visable. S. mutans polymerize the glucose and fructose moieties of
sucrose to synthesize glucans (contains two homo-polymers
Note: In condensing osteitis the tooth may be vital namely Dextran and mutan) and fructans (also known as
or non-vital. Levans). Glucans are more significant as they promote the
accumulation of S. mutans on teeth. Fructans are highly
29. 'B' [Shafer 7th ed 483-84) soluble and can be degraded by plaque bacteria, thus
Many cases of periapical granuloma are entirely serving as a reservoir of fermentable sugars for oral bacteria.
asymptomatic. Some patients may complain of mild pain On the other hand, mutan is an important constituent of
on biting / chewing' solid food. In some cases, the tooth fibrillar plaque matrix and is less soluble and more resistant
may be slightly elongated in the socket. Radiographs to enzymatic attack than dextran.
cannot be used to accurately distinguish between forms of
periapical disease, clue to overlapping features. Hence, an 45. 'D' [Shafer 7th ed 454/ 6th ed 450)
accurate distinction is possible only by histopathological Advanced dentinal changes:
examination. Tiny 'liquefaction foci', described by Miller, are formed by
focal coalescence and breakdown of a few dentinal tubules.
30. 'B' [Shafer 7th ed 504/ 6th ed 501) In areas of globular dentine, de calcification and confluence
of tubules occur rapidly. The presence of considerable
31. 'B' [Shafer 7th ed 498/ 6th ed 496] amounts of globular dentin accounts for the rapid spread of
caries in so-called malacotic or soft teeth.
32. 'D' [Shafer 6th ed 492/ White & Pharoah 6th ed 332)
46. 'D' [Check Explanation Below]
33. 'D' [Shafer 7th ed 494-99/ 6th ed 493-97) A review by "Navia et al" revealed the effects of various
Types of chronic osteomyelitis are: trace elements on dental caries as follows:
• Chronic suppurative osteomyelitis • Flourine
Definitely cariostatic
• Chronic focal sclerosing osteomyelitis (condensing osteitis). • Phosphorous
• Chronic diffuse sclerosing osteomyelitis. • Vanadium (KCET-11)
• Sclerotic cementa[ masses. • Molybdenum
• Chronic osteomyelitis with proliferative periostitis • Strontium
Mildly cariostatic
(Garre's osteomyelitis). • Boron
• Calcium
34. 'D' [Shafer 7th ed 510/ 6th ed 508)
• Gold
35. 'A' [Shafer 7th ed 482 Fig. 10-8/ 6th ed 481) • Selenium
• Magnesium
36. 'D' [Shafer 7th ed 495/ 6th ed 494) • Cadmium
Caries promoting
• Platinum
37. 'C' [Shafer 7th ed 488/ 6th ed 487]
• lead
38. 'A' [Shafer 7th ed 498/ 6th ed 497) • Silicon
• Aluminium
39. 'D' [Shafer 7th ed 490/ 6th ed 489) No effect • Barium
• Iron
Dental 9'ul-6e � - - - - - -

47. 'C' [Shafer 7th ed 449/ 6th ed 435, 436]

48. 'A' [Shafer 7th ed 425/ 6th ed 416]


Refer Q. No 45 in dental caries

49. 'A' [International Congress Series 1284 (2005} 103 -


112]
With the respect to microbial flora, the oral cavity is one
of the most densely populated sites of the human body.
The environmental diversity of the oral cavity promotes
the establishment of distinct microbial communities, such
as supragingival plaque, subgingival plaque and tongue
coating. The properties of the environment determine which
microorganisms can occupy a site, while the metabolic
activities of those microbial communities subsequently
modify the properties of the environment.

Saccharolytic microorganisms in supragingival sites split


carbohydrates into, principally, lactic acid and create a
temporarily acidic environment. Conversely, in subgingival
sites, asaccharolytic microorganisms metabolize nitrogenous
compounds derived from gingival crevicular fluid (GCF) and
create a neutral pH and anaerobic environment abundant in
short-chain fatty acicls and ammonia. In tongue coating,
asaccharolytic activity toward cysteine and methionine
produces sulfur compounds, the major components of oral
malodor.

50. 'B' [Not Required]

51. 'B' [Shafer 7th ed 495, 498]


Garre's osteomyelitis is chronic osteomyelitis with
proliferative periostitis that produces a focal periosteal
thickening to a mild stimulus. It is essentially a periosteal
osteoslerosis unlike the endosteal osteosclerosis seen in
chronic focal & diffuse sclerosing osteomyelitis.

Chronic focal sclerosing osteomyelitis, also called condensing


osteitis, is a reaction to a mild bacterial infection in patients
with high degree of tissue resistance (the infection acts as a
stimulant rather than an irritant).
, ORAL PATHOLOGY & MEDICINE

7. PHYSICAL AND CHEMICAL INJURIES


1. A patient comes with emotional problems, 11. Eighty percent of all salivary stones occur in
increased salivation; pallor of oral mucosa and a a) Parotid b) Submandibular
grayish blue discoloration of the gingiva. These findings c) Sublingual d) Minor salivary glands
are most consistent with a clinical impression of: (KAR-04)
a) Cherubism b) Cretinism 12. Internal resorption is characterised by
c) Pierre Robin syndrome d) Lead poisoning a) Pain on percussion
(AIPG -02) b) Slow dull continuous pain
2. Silver tatoo is due to c) No characteristic feature, symptom free
a) deposition of Ag Amalgam in Mucosa d) Increased pulpal pain when lying down
b) deposition of Ag Amalgam in Bone (KA T-04)
c) deposition of Ag Amalgam in Dentin 13. Mucooceles are rarely seen in the
d) deposition of Ag Amalgam in Enamel a) Lower lip b) Upper lip
(PGI -97) c) Buccal mucosa d) Floor of the mouth
3. The etiology of angioedema is: (COMEDK-06)
a) Sensitivity to acrylic resin 14. In a 2 year old child, a bluish dome shaped swelling on
b) Using ACE inhibitors the inner side of the lip is mostly
c) Hereditary d) All of the above a) Hematoma b) Mucocoele
(AIPG -95) c) Hemangioma d) None of the above
4. Bismuth intoxication is manifested as (AP-10)
a) pigmentation in areas of inflammation 15. Punctate basophilia is characteristically seen in
b) Burtonian line a) Lead poisoning b) Acute myeloid leukaemia
c) Gingiva d) Blackish line in the mucosa c) Chronic myeloid leukaemia
(AP -01) d) Megaloblastic anaemia
5. Cyst without lining is (COMEDK-15)
a) Radicular cyst b) Dentigerous cyst 16. The above picture depicts which one of the following
c) Naso palatine duct cyst conditions
d) Hemorrhagic/ Traumatic bone cyst
(AP -04, AP-06)
6. Perleche is ca used by
a) decreased interdental space
b) increased interdental space
c) Lack of Vit-C
d) Trauma to the corners of the mouth
(KAR -98)
7. Angular stomatitis
a) can be a concomitant of any candidal infection
b) is seen mainly in adults
c) may be associated with streptococcus
d) may be a sign of anaemia
(APPSC -99) a) Amyloidosis b) Nicotinic stomatitis
8. Sloughing of necrotic epithelium is characteristic of:
c) Leukoplakia d) Denture stomatitis
a) Aspirin burn b) Denture sore mouth (APPG-15)
c) Traumatic ulcer d) Contact dermatitis 17. Difference in nicotinic stomatitis from papillary
(AIPG -90) hyperplasia
9. Enlargement of lip occured with in seconds to 24 hours is
a) Multiple dots at periphery
a) Angioedema b) Mucocoele b) Rings at centre
c) Herpes d) Fibroma c) Small centrally placed dot
(AIPG-06) d) Rings at periphery
10. Loss of tooth surface because of chemomechanical action (PG! JUNE-2014)
is known as
a) Abrasion b) Erosion
c) Abfraction d) Attrition
(COMEDK-05)

1) D 2) A 3) D 4) A 5) D 6) B 7) A 8) A 9) A 10) B 11) B 12) C 13) B


14) B 15) A 16) B 17) C
Dental 9'ul-6e �- - - - - -

7. PHYSICAL AND CHEMICAL INJURIES - ANSWERS


1. 'D' [Shafer 7th ed 558/ 6th ed 657] 10. 'B' [Shafer 7th ed 573-74/ 6th ed 571]
Loss of both enamel and dentin at the cervical
2. 'A' [Shafer 7th ed 560/ 6th ed 558] areas of teeth caused by mechanical loading
forces such as swallowing and chewing, etc.
3. 'D' [Shafer 7th ed 675/ 6th ed 671] Abfraction
The abfractive lesions were caused by flexure
Common causes of Anqio or Quincke's edema and ultimate fatigue of susceptible teeth at
• allergic angioedema doe to mast cell degranulation locations away from point of loading.
that leads to histamine release. Seen in IgE mediated It is the physiologic wearing away of a tooth
hypersensitivity reactions. as a result of tooth-to-tooth contact as in
• Use of ACE inhibitors, which increase the levels of Attrition mastication. This phenomenon is physiologic
bradykinin and this form of angioedema usually arises rather than pathologic.
within hours of initial use of the drug.
It is the pathologic wearing away of
• Hereditary in which there is reduction of an inhibitor,
tooth substance through some abnormal
that prevents the transformation of Cl to Clesterase
mechanical process. The most common cause
that cleaves C4 and CZ and results in angioedema.
of abrasion of root surfaces is the use of an
• Due to presence of high levels of antigen-antibody Abrasion
abrasive dentifrice. It is usually manifested
complexes (Eq: LE patients) as V-shaped or wedge shaped ditch on the
root side of the CEJ in teeth with gingival
4. 'A' [Shafer 7th ed 555/ 6th ed 554] recession.
Heavy metals cause gingival pigmentation in areas of
inflammation only, where the increased permeability permits It is irreversible loss of dental hard tissue
the seepage of metal into the surrounding tissue. by a chemical process that does not involve
Dental bacteria. It is dissolution of mineralized
Lead line Bluish red or deep blue linear erosion tooth surface by acids that are introduced
(Burtonian line) pigmentation in marginal gingiva into oral cavity through vomiting, acidic
Silver (argyria) Violet marginal line beverages and citrus fruits, etc.
• More or less black marginal line 11. 'B' [Shafer 7th ed 546/ 6th ed 544]
Bismuth, arsenic
and mercury • Blue- b lack line in seen in bismuth
pigmentation 12. 'C' [Shafer 7th ed 585/ 6th ed 584]
Delicate, brown or black pigmented
non cariogenic plaque found on the 13. 'B' [Shafer 7th ed 543/ 6th ed 541)
Mesentric Line
enamel at the cervical margin of the
tooth 14. 'B' [Shafer 7th ed 543/ 6th ed 541]

5. 'D' [Shafer 7th ed 529/ 6th ed 528] 15. 'A' [Shafer 7th ed 558)
Basophilic stippling or punctate basophilia refers to an
6. 'B' [Shafer 7th ed 925/ 6th ed 916] observation found when observing a blood smear in which
Causes of angular cheilitis or Perleche: erythrocytes display small dots at the periphery. These dots
are the visualization of ribosomes and can often be found in
• Decreased vertical dimension or increased freeway space
the peripheral blood smear, even in some normal individuals.
in old age
• Nutritional deficiencies such as iron deficiency, Vit. B or 16. 'B' [Shafer 7th ed 96)
folic acid deficiency. Leukokeratosis nicotina palati or smokers palate consists
of grayish white palate with small nodular excrescences
7. 'A' [Shafer 7th ed 925/ 6th ed 916] having small central red spots corresponding to the inflamed
orifices of minor salivary glands, commonly associated with
8. 'A' [Shafer 7th ed 553/ 6th ed 552] smoking.
Aspirin tablet is mistaken as local obtundant and are placed
against the offending tooth for relief of tooth ache, allowing 17. 'C' [Shafer 7th ed 96]
the lip or cheek to hold it in position. Leukokeratosis nicotina palati consists of grayish white
palate with small nodular excrescences having small
Option 'C" traumatic ulcer is the most common cause of the central red spots corresponding to the inflamed orifices
single ulcer. of minor salivary glands, commonly associated with
smoking. Papillary hyperplasia is considered to be a form
9. 'A' [Shafer 7th ed 675/ 6th ed 541] of inflammatory hyperplasia associated with ill - fitting
dentures due to frictional irritation (& poor oral hygiene)
r ORAL PATHOLOGY & MEDICINE
and is characterised by numerous, closely arranged red
edematous papillary projections and lacks the central red
dots.
Dental 9'ul-6e � - - - - - -

8. DISEASES OF BONES & JOINTS


1. 0steogenesis imperfecta 10. Clavicle is absent in
a) Is a sex-linked disorder of bones that develop in cartilage a) Osteogenesis imperfecta b) Cleidocranial dysostosis
b) manifests with blue sclera which are pathognomonic of c) Fibrous dysplasia d) Osteopetrosis
this disease (MAN -01)
c) May be associated with deafness 11. "Cotton-wool" appearance is seen in
d) Has associations with amelogenesis imperfecta a) Paget's disease b) Osteosclerosis
(MAN -94) c) Periapical cemental dysplasia
2. Which of the following are characteristic feature of d) Ossifying fibroma
cherubism (MAN -01)
a) Premature exfoliation of primary teeth 12. A 3-year-old patient reports of painless progressive
b) Hypo plastic defects bilateral facial swellings. The tentative diagnosis is
c) Progressive painless symmetric swelling of cheek bones a) Cherubism b) Monostotic fibrous dysplasia
d) A+ C c) Polyostotic tibrous dysplasia
(MAN -97) d) Central giant cell granuloma
3. Precocious puberty is most characteristic of which of the (AIIMS -04,AP-02)
following? 13. Generalised hyper cementosis is seen in
a) Jaffe's Syndrome b) Monostotic fibrous dysplasia a) Hypophosphatasia b) Pa.gets disease
c) Albright's syndrome d) Osteogenesi imperfecta c) Fibrous dysplasia d) Cherubism
(MAN -97} (AIPG -03, 05}
4. Which of the following diseases of the bone 14. The primary causative factor for myofacial pain
characteristically exhibits (in contrast to the other three dysfunction syndrome of the TMJ is
conditions) a single lesion in a single bone? a) Infratemporal space infection
a) Central giant cell granuloma b) Auriculotemporal neuritis
b) Osteopetrosis c) Muscular overextension and over contraction
c) Paget's disease of the bone d) Otitis media
d) Polyostotic fibrous dysplasia (MAN -01)
(MAN -95, AP-98,AIPG-99) 15. Class III malocclusion is seen in all of the following except
5. Serum alkaline phosphatase levels are increased in a) Pierre robinson syndrome
a) Osteoarthritis b) Dentinogenesis imperfecta b) Cleft palate
c) Paget's disease d) Rheumatoid arthritis c) Cleidocranial dysplasia d) Craniofacial dysostosis
(MAN -98,AIPG-97,98) (MAN -02)
6. Which of the following has the potential of undergoing 16. Down's syndrome is associated with all except
'Spontaneous' malignant transformation a) Retrognathia b) Periodontal disease
a) Osteomalacia b) Albright's syndrome c) Premature loss of deciduous teeth
c) Paget's disease of bone d) Osteogenesis imperfecta d) Delayed eruption of deciduous teeth
(MAN-98) (AP -03)
7. Which of the following are a triad of the sign and 17. Delayed eruption of teeth occurs in:
symptoms of osteogenesis imperfecta a) Craniofacial dysostosis b) Hyperthyroidism
a) blue sclera, sparse hair, anhydrosis c) Cleidocranial dysostosis d) Osteitis deformans
b) enlarged hand, feet, maxilla, mandible (AP -95}
c) blue sclera, brittle bones, opalescent dentin 18. The most common cause of TMJ ankylosis is:
d) blue sclera, arachnodactyly, brittle bones. a) Trauma b) Osteoarthritis
(MAN -99) c) Childhood illness d) Rheumatoid arthritis
8. In a middle-aged man the radiograph shows 'cotton- wool' (AIPG -91 ,92)
appearance and blood investigation reveal an elevated 19. False about Cherubism:
alkaline phosphatase levels. The tentative diagnosis is a) Unilocular lesion b) Bilateral
a) Paget's disease b) Cherubism c) Presence of Giant cell
c) Fibrous dysplasia d) Osteogenesis imperfecta d) Delayed eruption of Permanent teeth
(MAN -99) (AIPG -02)
9. Generalised thickening of cortical and cancellous bones 20. Alkaline phosphatase increases in:
is seen in a) Pagets disease b) Osteopetrosis
a) Osteopetrosis b) Pagets disease c) Cherubism d) Fibrous dysplasia
c) Osteogenesis imperfecta d) Infantile hyperostosis e) A and D
(MAN -02) (KAR -98)

1) B 2) D 3} C 4) A 5) C 6) C 7) C 8) A 9) D 10) B 11) A 12) A 13} B


14) C 15) A 16) A 17) C 18) A 19) A 20) E
r
21. The histopathology of osteopetrosis shows:
ORAL PATHOLOGY & MEDICINE
32. Amber coloured tooth translucency, blue sclerae and
a) Endosteal bone formation and lack of normal bone bone fragility and a history of previous bone fractures
resorption are characteristic findings in:
b) Periosteal bone formation and lack of normal bone resorption a) Osteoporosis b) Osteogenesis imperfecta
c) Presence of extra collagen fibres and less calcification c) Osteitis deformans d) Osteitis fibrosa cystica
resulting in resistance of bones to fracture (KAR -2K)
d) Presence of numerous osteoclasts and a few osteoblasts 33. Mosaic pattern of bone is seen in radiographic features of:
(AIPG -99) a) Fibrous dysplasia b) Paget's disease
22. Complication of Rheumatoid arthritis of condyle is c) Osteopetrosis d) Osteogenesis imperfecta
a) Fibrous ankylosis b) Subluxation (AIIMS -96, AIPG-11)
c) Dislocation d) None of above 34. Normal serum, Ca and alkaline PO 4 are in:
(AP -04) a) Cherubism b) Hypothyroidism
23. Multiple fractures are seen in: c) Hyperparathyroidism d) Paget's disease
a) Rickets b) Osteogenesis imperfecta (AIPG -2K)
c) Osteomyelitis d) Osteoma 35. The most common complication following rheumatoid
(KAR -98) arthritis of the TMJ is:
24. Of the following which is most common disorder causing a) Ankylosis b) Synovial chondromatosis
pain about the masticatory apparatus including the TMJ: c) Subluxation d) Osteoarthritis
a) Traumatic arthritis b) Trigeminal neuralgia (AIIMS -98)
c) Myofacial pain dysfunction syndrome 36. A patient with multiple impacted supernumerary teeth,
d) Degenerative arthritis and can bring his shoulders together is suffering from:
(AIPG -98) a) Klinefelter's syndrome b) Trisomy 21
25. In MPDS which muscle is most apt to exhibit tenderness: c) Down's syndrome d) Cleidocranial dysostosis
a) Temporalis b) Buccinator (KAR -03,AIPG-02)
c) Masseter d) Lateral pterygoid 37. Which of the following statements is false in relation to
(KAR -97) myofacial pain dysfunction sy1ndrome?
26. Biochemical abnormality associated with osteogenesis a) Mainly affects young females
imperfecta is increase in: b) Is caused by muscle fatigue due to chronic oral habits as
a) Alkaline phosphatase b) Acid phosphatase grinding and clenching
c) Bicarbonate ion d) Phosphorylase enzyme c) Treatment involves construction of occlusal guard and
(AIPG -90,92) stress free emotional condition
27. Orange peel and Ground glass radiographic appearance is d) The perioral musculature becomes hypotonic
observed in case of (AIIMS -94)
a) Pagets disease b) Ewing's sarcoma 38. Corticosteroids are useful in treatment of TMJ arthritis
c) Osteosarcoma d) Fibrous dysplasia because they have:
(AP -2K) a) An analgesic effect
28. The treatment for a child with cherubism is b) An anti-inflammatory effect
a) surgical excision c) Inhibitory effect on synovial membrane
b) cosmetic surgery after puberty d) Analgesic and anti-inflammatory effect
c) radiation d) Enbloc dissection (AIPG -89)
(KAR -98) 39. A child with Down's syndrome has Moon facies, retarded
29. Delayed dentition with multiple supernumerary teeth is mentally and which of the facial characteristics:
seen in a) Maxillary prognathisrn b) Mandibular retrognathia
a) Hypoparathyroidi sm b) Cleidocranial dysplasia c) Mandibular prognathism
c) Pierre Robin Syndrome d) Mongolism(Down's Syndrome) d) Maxillary hypoplasia
(AIPG -01) (AIIMS- 98)
30. A patient complains of loss of visual acuity, deafness 40. A 10-year-old child presents with anemia and recurrent
and enlargement of maxilla: fractures. The X-ray shows diffuse hyper density of bone.
a) Paget's disease b) Osteornalacia The diagnosis is most likely to be:
c) Fibrous dysplasia d) Osteogenesis imperfecta a) Osteogenesis imperfecta
(AIIMS -99) b) Osteopetrosis
31. Large anterior fontanelles, open sutures, slanting eyes, c) Osteochondroma
decreased sexual development, macroglossia and enamel d) Hyperparathyroidism
hypoplasia are seen in: (AIPG -02)
a) Craniofacial dysostosis b) Down's syndrome 41. Ground glass appearance in bone is seen in:
c) Treacher Collins syndrome a) Hyper parathyroidism b) Fibrous dysplasia
d) Marfan's syndrome c) Condensing osteitis d) Osteopetrosis
(PGI-97,98,AIIMS- 99) (AIPG-06)

21) A 22) A 23) B 24) C 25) D 26) D 27) D 28) B 29) B 30) A 31) B 32) B 33) B
34) A 35) A 36) D 37) D 38) B 39) D 40) B 41) A
Dental 9'ul-6e
42. Pain, muscle tenderness, clicking or popping noise in T.M 51. Preauricular pain, grating sensation and partial trismus
joint and limitation of jaw motion are the four cardinal are the symptoms of
signs and symptoms of a) TMJ fibrous ankylosis b) TMJ bony ankylosis
a) Costen's syndrome c) TMJ pain dysfurction symdnome
b) Traumatic arthritis of T.M J d) Ear infection
c) Ostearthritis (KAR-04)
d) Myofacial pain dysfuction syndrome 52. In Cleidocranial dysostosis, sometimes the roots of the
(COMEDK-06} permanent teeth are
43. A non-neoplastic hereditary bone lesion, histologically a) Thin and long b) Thin and short
similar to central giant cell granuloma, affects children c) Thick and short d) Fused
and shows a bilateral involvement of the jaws with 'eye (KCET-08)
to heaven' appearance clinically is 53. Polydactyly, craniosynostosis, late closure of fontanelles
a) Fibrous dysplasia b) Cherubism is a feature of:
c) Craniofacial dysostosis d) Chondro-ectodermal dysplasia a) Apert's syndrome b) Crouzon's syndrome
(KCET-09) c) Pierre robin syndrome d) Down' syndrome
44. Albers-Schonberg's disease is (AIIMS-07)
a) Osteomyelitis b) Osteopetrosis 54. The most likely diagnosis in a 23 year old, mentally
c) condensing osteitis d) Osteomalacia alert, male dwarf with disproportionate arm and leg to
(COMEDK-05) body growth, prominent forehead and retruded maxilla is
45. Treacher Collins Syndrome is a) Cretinism b) Pituitary dwarfism
a) Maxillofacial Dysostosis b) Mandibulofacial Dysostosis c) Acromegaly d) Achondroplasia
c) Maxillomandibulofacial Dysostosis (BHU-07)
d) Condylar Dysostosis 55. Osteosarcoma characteristically may develop in some
(COMEDK-05) cases of
46. Immature bony trabeculae are found in a) Osteopetrosis b) Osteogenesis imperfecta
a) Fibrous dysplasia b) Paget's disease c) Acromegaly d) Osteitis deformans
c) Rickets d) Cleidocranial Dysplasia (BHU-07)
(AP-10) 56. A patient with fibrous dysplasia can be treated by
47. Osteosclerosis of bone occurs due to a) Surgical excision
a) Decreased host resistance b) Removal of adjacent teeth
b) Increase in the virulence of organisms causes infection c) Irradiation of the lesion
c) Increased host res·ponse d) Conservative surgery
d) Occurs in immunocompromised patients (KCET-10)
(AIPG-05) 57. Histopathologically reversal lines are seen in
48. During a routine checkup a 70 year old male is found to a) Cherubism b) Fibrous dysplasia
have Serum Alkaline Phosphatase three times the upper c) Paget's disease of the bone
limit of normal. Serum Calcium, Serum Phosphorous and d) Craniofacial dysplasia
liver function tests are normal. The most Likely diagonsis is (AP-09)
a) Primary hyperparathyroidism 58. In a 60 year old adult, which of the following diseases
b) Paget's disease of the bone cause expansile maxillary lesions?
c) Osteomalacia a) Paget's disease b) Acromegaly
d) Metastatic bone disease c) Fibrous Dysplasia d) Rickets
(COMEDK-07) (AP-10)
49. Venous malformations involving the leptomeninges of 59. Most common primary malignant bone tumor is:
the cerebral cortex is a salient feature of a) Osteosarcoma b) Ewing sarcoma
a) Rendu-Osler-Weber disease c) Metastatic carcinoma d) Multiple Myeloma
b) Maffuci's syndrome (AIPG-10)
c) Angioosteohypertrophy syndrome 60. Transformation into osteosarcoma is seen with
d) Sturge weber syndrome a) Pagets disease and polyostotic fibrous dysplasia
(KCET-07} b) Pagets disease and osteopetrosis
50. A patient 18 years of age with sebaceous cysts on the c) Cherubism and polyostotic fibrous dysplasia
scalp and back of the neck, an osteroma on the right d) Cherubism and Pagets disease
mandible; Radiographs reveal multiple impacted (AIIMS-09)
supernumerary teeth in both jaws. These findings suggest 61. Which of the following is NOT a clinical feature of Pierre­
a) Cleidocranial dysostosis b) Ectodermal dysplasia Robin syndrome?
c) Gardner's syndrome d) Osteogenesis imperfecta a) Micrognathia b) Retrognathia
(KCET-07) c) Glossoptosis cl) Coloboma of lower eyelid
(KCET-09)

42) D 43) B 44) B 45) B 46) A 47) C 48) B 49) D 50) C 51) C 52) B 53) A 54) D
55) D 56) D 57) C 58) A 59) D 60) A 61) D
62.
r ORAL PATHOLOGY & MEDICINE
A 15-year-old boy reports with a rapidly growing swelling 68. The granulomatous tissue that is responsible for
of the mandible with intermittent pain. Patient history destruction of articular surfaces of TMJ in rheumatoid
is that the swelling occurred after an episode of trauma. arthritis is known as
Radiographs reveal formation of new subperiosteal bone a) Pannus b) Pulse granuloma
producing Onion Skin appearance. The patient also has c) Baker's cyst d) Immune granuloma
lip paresthesia elevated white blood cell count (AP- 2013)
69. Hypercementosis of whole dentition is seen in
A) Based on the cli111ical and radiographic picture, one of a) Hypophosphatasia b) Achondroplasia
the following conditions could be considered in the c) Paget's disease
provisional diagnosis d) Fibrous dysplasia polyostotic type
a) Fracture of the jaw with cancellous bone formation (AP-14)
b) Chronic suppurative osteomyelitis 70. Which among the following are associated with increased
c) Burkitt's Lymphoma incidence of osteosarcoma?
d) Ewing's sarcoma a) Cherubism and osteopetrosis
b) Paget and polystotic fibrous dysplasia
B) There is considell'able clinical and histological overlap c) Paget and cherubism
of Ewing's sarcoma with one of the following: d) Polystoticfibrous dysplasia and cherubism
a) Primitive neuro ectodermal tumor (PNET)
b) Osteosarcoma 71. Osteoporosis is seen in all except?
c) Fibrosarcoma a) Prolonged corticosteroid therapy
d) Chondrosarcoma b) Malnutrition
c) Old age/audit d) Hypervitaminosis
C} One of the following statements is true for Ewing's (PGI JUNE-13)
sarcoma
a) An episode of trauma often precedes development of
Ewing's sarcoma
b) Trauma is the main etiology in the development of
Ewing's sarcoma
c) Relationship between trauma and Ewing's sarcoma has
never been reported
d) Trauma can be considered as etiology for Ewing's sarcoma
provided there is secondary infection
(COMED-2012)
63. Eruption fails in this bone disease, as there is no bone
resorption:
a) Primary hyperparathyroidism
b) Phantom bone disease
c) Paget's disease d) Osteopetrosis
(COMED-2012)
64. Paget's disease of bone is a chronic disease of the
a) Prepubertal skeleton b) Pubertal skeleton
c) Infantile skeleton d) Adult skeleton
(BHU-2012)
65. Mutation in GNAS 1 gene is associated with
a) Fibrous dysplasia b) Ossifying fibrorna
c) Focal cementoosseous dysplasia
d) Periapical cementoosseous dysplasia
(AP-2012)
66. A patient aged 50 years presented with a history of jaw
expansion and enlargement of maxilla
a) Paget's disease b) Acromegaly
c) Fibrous dysplasia d) Hyperparathyroidism
(PGI-2011)
67. Multiple myeloma has been seen commonly after
exposure to
a) Radioactive isotopes b) Gold
c) Asbestos d) Organic dyes
(COMEDK- 2013)

62A) D 62B) A 62C) A 63} D 64} D 65) A 66} A 67) A 68} A 69} C 70) B 71) D
Dental 9'ul-6e � - - - - - -

8. DISEASES OF BONES & JOINTS - ANSWERS


1. 'B' [Shafer 7th ed 699/ 6th ed 695) The other conditions associated with increased incidence of
Osteogenesis imperfecta is a hereditary autosomal dominant osteosarcoma are
disorder. Pale blue sclera is one of characteristic features • Polyostotic fibrous dysplasia
of osteogenesis imperfecta. Blue sclera are also seen in
• Enchondromatosis
osteopetrosis, fetal rickets, marfan's syndrome, and Ehlers
- Danlos syndrome. • hereditary multiple exostosis

Class III occlusion due to maxillary hypoplasia, posterior and 7. 'C' [Shafer 7th ed 699-700/ 6th ed 695)
anterior cross bites, open bite, large number of impacted
first and second molars are important features. 8. 'A' [Shafer 7th ed 733/ 6th ed 729)

2. 'D' [Shafer 7th ed 71 7/ 6th ed 711] 9. 'D' (Shafer 6th ed 700]


Periosteal new bone formation (Neoperiostosis) occurs in
Grading system for Cherubism infantile cortical hyperostosis leading to formation of bony
Grade I Involvement of both mandibular ascending rami hard swelling and unusual cortical thickening.
Involvement of both maxillary tuberosities
Grade II Osteopetrosis is a hereditary bone disease with a failure
and mandibular ascending rami
of osteoclastic resorption, leading to increase in defective
Characterized by McCune-Albright syndrome osseous tissue that tends to replace bone marrow and hence
Grade III with involvement of whole maxilla and the bone deposition is endosteal only. Endosteal bone
mandible. deposition tends to compromise on the marrow elements
before demonstrating an increase in the size of the bone.
3. 'C [Shafer 7th ed 714/ 6th ed 705) Radiologically , the vertebrae are extremely radiodense. They
Polyostotic Fibrous dysplasia is of 2 types. In Jaffe's type may show alternating bands and this is known as 'rugger­
there is multiple bone involvement associated with cafe-au­ jersy' sign.
lait spots. Extreme or diffuse multiple bone involvement,
cafe-au-lait pigmentation and endocrine disturbances 10. 'B' [Shafer 7th ed 726/ 6th ed 722)
are important features of McCune-Albright's syndrome. In Cleidocranial dysplasia is characterized by abnormalities of
polyostotic form, the curvature of femoral neck and proximal skull, shoulder girdle, jaws and teeth. In the skull, delayed
shaft of the femur increase markedly causing a shepherd's closing of sutures and wormian bones are common. The
crook deformity. characteristic skull abnormalities are sometimes referred to
as "Arnold head".
The association of fibrous dysplasia and intramuscular
myxoma is known as mazabraud's syndrome. In the shoulder girdle, there is partial or complete absence
of clavicle and the patients can bring their shoulder forward
Radiologically the radiolucent lesions in fibrous dysplasia until they meet in the midline.
are surrounded by a thick sclerotic border and this is known
as rind sign. There is prolonged retention of deciduous teeth and
subsequent delay in eruption of succedaneous teeth.
4. 'A' [Shafer 7th ed 137 / 6th ed 132) Numerous unerupted supernumerary teeth are found in
Central giant cell granuloma commonly involves mandible. mandibular premolar and incisor areas.
5. 'C' [Shafer 7th ed 733/ 6th ed 729]
Serum alkaline phosphatase level may exceed 50 bodansky Narrow, high arched palate, prolonged retention of
units in monostotic type while in polyostotic type the levels deciduous teeth and failure in the eruption of permanent
are increased to 250 bodansky units. teeth are characteristic _____ of (COM EDK-14)
a) Pagets disease b) Cherubism
Also urinary hydroxy proline (a product of collagen c) Cleidocranial Dysplasia d) Osteogenesis imperfect
breakdown) levels are elevated as they reflect increased
osteoclastic activity and bone resorption.. More recently 11. 'A' (Shafer 7th ed 733/ 6th ed 729)
urinary excretion of pyridinium collagen cross links, urinary In osteoblastic phase of pagets disease, the osteoblastic
N-telopeptide (NTX), and alpha -C telopeptide (CTX) have areas appear as opacities in x-ray and are patchy in
been found more sensitive index of bone resorption. distribution giving the characteristic "cotton wool"
appearance.
6. 'C' [Shafer 7th ed 735/ 6th ed 169]
Development of osteosarcoma is the most serious 12. 'A' [Shafer 7th ed 715/ 6th ed 713)
complication of paget's disease or osteitis deformans. Cherubism manifests by the age of 3-4 yeas in the form
(BHU-07). of painless symmetric swelling of mandible or maxilla,
r ORAL PATHOLOGY & MEDICINE
producing the typical chubby, CHERUB face. The deciduous • Anterior and posterior crossbite
dentition sheds prematurely and numerous teeth are absent • ANUG
in permanent dentition. X-ray reveals numerous unerupted
• Candidiasis
teeth floating in cyst like spaces.
• Juvenile periodontitis
"Floating Tooth Syndrome" is seen in • Partial anodontia
a) Cherubism (AP-2013) • Enamel hypoplasia

The presence of numerous unerupted teeth and the 17. 'C' [Shafer 7th ed 727/ 6th ed 722)
destruction of alveolar bone may displace the teeth,
producing an appearance referred to as floating tooth 18. 'A' [Shafer 7th ed 741/ 6th ed 735)
syndrome. Floating teeth radiological feature is also
seen in two variants of histiocytosisi.e, eosinophilic 19. 'K [Shafer 7th ed 717/ 6th ed 713)
granuloma and Hand-Schuller-Christian disease.
20. 'E' [Shafer 7th ed 711, 733/ 6th ed 711, 729)
13. 'B' (Shafer 7th ed 733/ 6th ed 729] • Pagets disease
Increased serum alkaline • Caffey's disease
14. 'C' [Shafer 7th ed 7 49/ 6th ed 742)
phosphatase levels • Fibrous dysplasia
15. 'A' [Shafer 7th ed 721/ 6th ed 717) • Hyperparathyroidism
Jaw abnormality Disease Reduced serum alkaline • Scurvy
• Cleidocranial dysplasia phosphatase levels • Hypothyroidism
• Craniofacial dysostosis Increased serum alkaline
Underdeveloped maxilla phosphatase and lactate
• Achondroplasia
(Class III malocclusion) dehydrogenase with reduced Odontogenic myxoma
• Down syndrome acid phosphatase and G-6-P-D
• Osteogenesis im perfecta activity
Underdeveloped mandible • Treacher Collins syndrome Increased serum acid
Osteopetrosis
(Class II malocclusion) • Pierre Robin syndrome phosphatase
Hypopituitarism Increased phosphorylase enzyme Osteogenesis
Underdeveloped maxilla levels imperfecta
(development of mandible is
& mandible
more retarded than maxilla) • Normal serum calcium &
"Leontiasis ossea" or a phosphorous Cherubism
Enlargement of maxilla type of monostotic fibrous • Normal alkaline phosphatase
dysplasia.
Enlargement of mandible Acromegaly Serum alkaline phosphatase is raised in all of the
Enlargement of both following except: (AIIMS-2012)
Pagets disease a) Paget's disease b) Fibrous dysplasia
maxilla and mandible
c) 0steomyelitis d) Hyperparathyroidism
Residual asymmetric
deformity of mandible Striking feature of Caffey's
21. 'A' [Shafer 7th ed 705/ 6th ed 701)
(due to cortical disease
Serum acid phosphatase levels are increased in osteopetrosis
thickening)
or marble bone disease.
An unusual prominence
Thalassemia
of premaxilla 22. 'K [Shafer 7th ed 746/ 6th ed 740)
Underdeveloped
Congenital syphilis 23. 'B' [Shafer 7th ed 699/ 6th ed 695)
premaxilla
Osteogenesis imperfecta is characterized by extreme
16. 'A' [Shafer 7th ed 728/ 6th ed 725) fragility and porosity of bones leading to multiple fractures.
Down's syndrome or mongolism is due to trisomy of 21st The basic defect lies in the organic matrix because of failure
chromosome in transformation of fetal collagen into mature collagen.

Dental problems in down syndrome 24. 'C' [Shafer 7th ed 748/ 6th ed 742)
• Hypoplastic maxilla
25. 'D' [Burkitts 9th ed. Page No 312)
• Macroglossia Lateral pterygoid followed by masseter are the two muscles
• Class III tendency that are commonly involved in MPDS.
• Scrotal/fissured tongue
Dental 9'ul-6e
36. 'D' [Shafer 7th ed 726/ 6th ed 722)

27. 'D' [Shafer 7th ed 713/ 6th ed 708) 37. 'D' [Shafer 7th ed 749/ 6th ed 742)
"Ground-glass" appearance is also seen in hyper
parathyroidism. 38. 'B' [Shafer 4th ed 711)

28. 'B' [Shafer 7th ed 718/ 6th ed 714] 39. 'D' [Shafer 7th ed 729/ 6th ed 725)

29. 'B' [Shafer 7th ed 727/ 6th ed 723) 40. 'B' [Shafer 7th ed 705/ 6th ed 699)
In osteopetrosis there is endosteal production of bone
30. 'A' [Shafer 7th ed 732/ 6th ed 728) and lack of physiologic bone resorption. Osteoblasts
In Paget's disease, the patients suffer from deafness are prominent but osteoclasts are seldom found. The
(due to compression of cochlear nerve) blindness (due to hemopoietic bone marrow is replaced by bone causing
involvement of optic nerve) and facial paralysis (due to myelopthisic anemia.
pressure on facial nerve). There is progressive enlargement
of skull, and maxilla because of which the patient has to 41. 'A' [Shafer 7th ed 653/ 6th ed 652)
change the hats and dentures frequently. Ground glass appearance is seen in
• Hyperparathyroidism
Platybasia, characterized by descent of cranium onto
• Cherubism
cervical spine due to softened bone at the base of the skull,
is also an important feature of paget's disease. • Monostotic fibrous dysplasia

31. 'B' [Shafer 7th ed 728-29/ 6th ed 725) 42. 'D' [Shafer 7th ed 749/ 6th ed 742)
The four cardinal signs and symptoms of MPDS are:
32. 'B' [Shafer 7th ed 699-700/ 6th ed 695) • Pain
• Muscle tenderness
33. 'B' [Shafer 7th ed 729/ 6th ed 729)
• Clicking or popping noise in TMJ
The term "Mosaic" bone is due to partially resorbed and
repaired bone, leaving deeply staining hemotoxyphilic • Limitation of jaw motion unilaterally or bilaterally
reversal lines (AP-O9). These lines give the "Jig saw
puzzle" histological a.ppearance to bone. 43. 'B' [Shafer 7th ed 717-18/ 6th ed 711-13)
Cherubism is non-neoplastic autosomal dominant lesion
of jaws that is histologically similar to central giant cell
• Mosaic bone Pagets disease/ granuloma. It affects the jaws of children bilaterally and
• Jigsaw-Puzzle bone osteitis symmetrically, usually producing the so-called cherubic
• Cotton wool bone deformans look, with a 'eye to heaven' appearance clinically.
"Orange Peaud" or ground glass
44. 'B' [Shafer 7th ed 704/ 6th ed 699)
or orange peel appearance of Monostotic fibrous
bone with (-shaped or Chinese dysplasia
45. 'B' [Shafer 7th ed 720/ 6th ed 716)
character shaped trabeculae
Salt and pepper bone Thalassemia 46. 'A' [Shafer 7th ed 714/ 6th ed 706)
Total disappearance of bone Massive osteolysis /
Phantom vanishing 47. 'C' [Shafer 7th ed 495/ 6th ed 494)
bone
48. 'B' [Shafer 7th ed 733/ 6th ed 729)
Drill bone / Riders bone / Traumatic myositis
Exercise bone ossificans
49. 'D' [Shafer 7th ed 149/ 6th ed 140)
Brittle bone Osteogenesis
imperfecta 50. 'C' [Shafer 7th ed 48/ 6th ed 47)
Wormian bones Cleidocranial dysplasia
Coarse woven bone Fibrous dysplasia 51. 'C' [Shafer 7th ed 749/ 6th ed 742)

34. 'A' [Shafer 6th ed 713) 52. 'B' [Shafer 7th ed 727/ 6th ed 722)
The outstanding oral findings of cleidocranial dysostosis
35. 'A' [Shafer 7th ed 746/ 6th ed 740) include prolonged retention of the deciduous teeth and
TMJ involvement is common in "Marie-strumpell type of subsequent delay in eruption of the succedaneous teeth.
rheumatoid arthritis". Rheumatoid arthritis in children is The roots of teeth are often SHORT and THINNER than
called stills disease (COMEDK-O9) and it results in class II normal and may be deformed. Also there is an absence or
division I malocclusion with anterior open bite. paucity of cellular cementum which may be related to failure
of eruption.
ORAL PATHOLOGY & MEDICINE
53. 'A' [Shafer 7th ed 723/ 6th ed 718] In less widespread involvement, osteosarcoma occurs in
Important features of A pert syndrome: fewer patients with Paget's disease. Men are affected with
• More common in Asians. sarcotamous degeneration slightly more frequently than
women.
• Craniostenosis resulting in acrocephaly, brachycephaly,
large late-closing fontanels, a gaping midline defect, 61. 'D' [Shafer 7th ed 721/ 6th ed 717)
hypertelorism, shallow orbits, depressed nasal bridge The characteristic triad of Pierre-Robin syndrome
with parrot-beaked appearance are other features. includes: (AIIMS-2012)
• Poly dactyly, syndactyly with the fusion of 2nd, 3rd and 4th • Micrognathia or retrognathia
digits (mitten hands and sock feet) are other features.
• Cleft palate
• Prominent mandible, high arched palate, bifid uvula,
cleft palate, anterior crowding, malocclusion, delayed • Glossoptosis
and ectopic eruption, shovel-shaped incisors,
supernumerary teeth are oral features. Option 'D' is seen in treacher-collin syndrome

54. 'D' [Shafer 7th ed 724/ 6th ed 720] 62A. 'D' [Shafer's 7th ed 169/ 6th ed 165]
Option 'D' Ewing's sarcoma is also called round cell sarcoma
55. 'D' [Shafer 7th ed 735/ 6th ed 169] and is seen in children and young adults. It is characterized
by painful (intermittent pain) swelling of the involved bone,
56. 'D' [Shafer 6th ed 7 06] facial neuralgia and lip paresthesia. It is often preceded
Treatment of fibrous dysplasia is usually conservative and by history of trauma. It produces an irregular diffuse
primarily to prevent deformity. Any underlying endocrine radiolucency, onion skin and sun ray appearance of bone.
disturbances should be treated. Histologically, the neoplasm is composed of solid sheets of
round cells, arranged in Filigree pattern. Intracytoplasmic
Surgical therapy with curettage and replacement of the glycogen of these cells helps in the differentiation from
bone defect with autograft or allograft usually results in reticulum cell sarcoma.
resorption of the graft at the surgical site. Use of allograft or
cortical autograft usually delays this conversion, as it is more 62B. 'A' [Shafer's 7th ed 169/ 6th ed 164]
resistant to resorption and replacement by dysplastic bone. Option 'N most investigators believe that Ewing's sarcoma
and Primitive neuroectodermal tumor (PNET) are different
57. 'C' [Shafer 7th ed 733/ 6th ed 730] morphological expressions of the same tumor type. There
is considerable clinical and histologic overlap, but Ewing's
58. 'A' [Shafer 7th ed 7 31/ 6th ed 728] sarcoma commonly arises within bone, whereas PNET arises
Paget's disease is recognized most commonly after age of 50 within the soft tissues.
years and the ratio of involvement of maxilla to mandible
is approximately 2.3 to 1. The maxilla exhibits progressive 62C. 'A' [Shafer's 7th ed 169/ 6th ed 165]
enlargement, the alveolar ridge becomes widened and the An episode of trauma often precedes the development of the
palate is flattened. If teeth are present, they may become tumor.
loose and migrate, producing some spacing.
63. 'D' [Shafer's 7th ed 705/ 6th ed 700]
59. 'D' [Shafer 7th ed 187 / 6th ed 183] Osteopetrosis is a hereditary bone disease, in which failure of
Multiple myeloma is the most common primary neoplasm of osteoclastic resorption leads to increased bone mass. Failure
the skeletal system. The disease is a malignancy of plasma of osteoclasts to resorb bone, results in thickened sclerotic
cells, which are subset of B-cells. The diagnostic laboratory bones. This may cause retardation of tooth eruption.
findings are monocl.onal hypergamma globulinemia. IgG
myeloma is the most common followed by IgA myeloma. 64. 'D' [Shafer's 7th ed 731/ 6th ed 727]
Paget's disease, characterized by excessive and abnormal
Option 'N osteosarcoma is the most common primary remodeling of bone, is a common disorder in middle aged
malignant tumor of Long bones. and elderly patients. Hence it affects the adult skeleton. The
excessive remodeling gives rise to bones that are extensively
60. 'If [Shafer 7th ed 711, 733/ 6th ed 706, 730] vascularised, weak, enlarged and deformed.
The association of polyostotic fibrous dysplasia (McCune­
Albright syndrome) and intramuscular myxoma is a 65. 'If [Shafer's 7th ed 710/ 6th ed 705]
rare disease known as Mazabraud's syndrome. The Fibrous dysplasia is caused by the mutation in the GNASl
osteosarcomatous transformation in fibrous dysplasia is gene. It encodes a G protein that stimulates the production
most common in people with Mazabraud's syndrome. of cAMP. The mutation causes a continuous activation of
G protein, leading to overproduction of cAMP, that results
Sarcomatous degeneration of pagetic bone is a deadly in hyperfunction of endocrine glands, proliferation of
complication. Sarcomatous degeneration may occur in 5-10% melanocytes and effects osteoblastic differentiation.
of patients with extensive pagetic skeletal involvement.
Dental 9'ul-6e
66. 'A' [Shafer's 7th ed 7 33/ 6th ed 728] 71. 'D' [Shafer 7th ed 617 Burkefs 12th ed 605]
• In Paget's disease, the patients suffer from deafness Osteoporosis is caused due to a long term negative calcium
(due to compressi,on of cochlear nerve) blindness (due balance and is defined as a disease characterised by low
to involvement of optic nerve) and facial paralysis bone mass and micro - architectural deterioration of bone
(due to pressure on facial nerve). There is progressive tissue, leading to bone fragility and an increase in fracture
enlargement of skull, and maxilla because of which the risk. The main risk factors for osteoporosis are old age,
patient has to change the hats and dentures frequently. malnutrition, estrogen deficiency, vitamin D insufficiency,
hyperparathyroidism, hyperthyroidism, malabsorption,
• Platybasia characterized by descent of cranium onto
glucocorticoid therapy and smoking.
cervical since due to softened bone at the base of the
skull, is also an important feature of Paget's disease.

67. 'A' [Shafer 7th ed 187-88/ 6th ed 183]


Predisposing factors do not appear to be important in
the etiology of multiple myeloma. However some events
suggested are radiation exposure (radiologists, radium-dial
workers), occupational exposure (agricultural, chemical,
metallurgical, rubber plant, leather tanners) and chemical
exposure to benzene, formaldehyde, hair dyes etc.

68. 'A [Karjodkar 2nd ed 723]


Rheumatoid arthritis (RA) is an autoimmune disorder
affecting the synovial membrane of several joints. This
disease starts as synovitis (inflammation of the synovial
membrane), which leads to the formation of granulomatous
disease (also known as pannus), that destroys articular
surface of underlying' bone. RA typically starts in small
joints and affects the TMJ in 30% of the patients.

69. 'C' [Shafer 7th ed 753]


Paget's disease or Osteitis deformans is characterised by
excessive and abnormal remodelling of bone in elderly
patients. The oral manifestations include progressive
enlargement of the jaws, most commonly the maxilla,
resulting in exposure of teeth in dentulous patients (lips
are too small to cover the enlarged jaw) or inability to
wear dentures in edentulous patients. Radiologically, it is
characterised by initial phase of deossification & softening,
followed by bizarre, dysplatic reossification. Initially there
may be multiple destructive lesions of the skull resulting
in osteoporosis circu1mscripta. In the osteoblastic phase,
patchy radiopacities may result in cotton-wool appearance.
The teeth characteristically produce pronounced
hypercementosis and often a loss of well-defined lamina
dura.

70. 'B' [Shafer 7th ed 711,735]


The malignant transformation rate in fibrous dysplasia
is 0.4-1% and the interval between diagnosis of fibrous
dysplasia and development of malignancy is usually years or
decades. Skull and facial bones undergo malignant change
in monostotic disease and femoral & facial bones undergo
malignant change in polyostotic disease. The most common
tumors are osteosarcoma & fibrosarcoma.

The complications of Paget's disease include pathological


fractures and sarcomatous degeneration (5-10% of cases
with extensive pagetic skeletal involvement). Pagetic
sarcoma follows a rapid and fatal course.
_____ r ORAL PATHOLOGY & MEDICINE

9. DISEASES OF NERVES & MUSCLES


1. Trigeminal neuralgia (tic doulourex) is characterized by c) Eagle's syndrome d) Reiter's syndrome.
a) Paralysis of one side of the face (KAR-03,AIPG-95)
b) Uncontrollable twitching of muscles 12. Facial paralysis is tested by:
c) Sharp, excruciating pain of short duration a) Whistling b) Chewing
d) Prolonged episodes of pain on one side of the face c) Protruding the tongue d) Swallowing
(MAN -97) (AIPG -94)
2. Trotter's syndrome involves: 13. Which syndrome consists of flushing, warmness and
a) Pharynx b) Oropharynx perspiration over the cheek and pinna of the ear on the
c) Larynx d) Nasopharynx side following the ingestion of highly seasoned food ?
(KAR -01) a) Fanconi's b) Auriculotemporal
3. Homer's syndrome DOES NOT include c) Homer's d) Cushing's
a) Ptosis b) Anhydrosis (AIIMS -02)
c) Flushing d) Mydriasis 14. The latest drug of the choice in the management in
(COMEDK-04) trigeminal neuralgia is:
4. All of the following are true about trigeminal neuralgia a) Valproic acid b) Carbamazepine
EXCEPT c) Diphen hydantoin d) None of the above
a) it is unilateral b) it is of throbbing nature (KAR-01)
c) it is triggered by touching cheeks, mucosa etc 15. Carbamazepine has been utilized to successfully
d) occurs in bouts diminish attacks in trigeminal neuralgia. During this
(MAN -2K) therapy which of the following is indicated?
5. Frey's syndrome results from surgery of the a) Clinical observation only
a) Submandibular salivary gland b) Clinical observation and complete blood and platelet
b) Parotid gland counts prior to and at frequent intervals during therapy
c) Sublingual salivary gland c) No monitoring
d) TMJ d) Complete blood investigation only if adverse symptoms
(AIPG-99; MAN, AIIMS-2K; KAR-03) arise
6. Geniculate neuralgia is caused in the nerve (AIIMS -02)
a) VII b) IX 16. Patient suffering form Eagle's syndrome complains of
c) X d) II a) burning sensations in mouth
(AIIMS -02) b) excessive salivation.
7. Easy fatigability of muscles seen in: c) Glossodynia d) Dysphagia
a) Epilepsy b) MPDS (KAR -99, PGI-99)
c) Myasthenia gravis d) Cerebral palsy 17. Trigeminal neuralgia:
(PGI -95) a) Does not disturb the patient during sleep
8. If a patient with Raynaud's disease puts his hand in cold b) Can be treated with NSAID's.
water, the hand appears: c) Always bilateral in distribution
a) Red b) Yellow d) Is a hereditary condition
c) White d) Blue (KAR -02)
(AIIMS- 02) 18. Severe pain which arise after injury to or sectioning
9. A neuralgia with trigger zones in the oropharynx and pain of a peripheral sensory nerve is called as:
in the ear pharynx, nasopharynx, tonsils and posterior a) Temporal arteritis b) Neuralgia
tongue is most likely: c) Neuritis d) Causalgia
a) Trigeminal neuralgia b) Bell's palsy (KAR -02)
c) Glossopharyngeal neuralgia 19. The following site is the common involvement in cases of
d) Sphenopalatine neuralgia Myositis ossificans
(AIPG-91,KAR-97,AIIMS-95) a) Massetor b) Hyoglossus
10. Which of the following structures are associated with c) Stylohyoid d) Lateral pterygoid
bells palsy (KAR -03,PGI JUNE- 13)
a) Sub mandibular gland b) Seventh cranial nerve 20. "Fothergill's disease" is one of the synonyms of
c) Temporomandibular joint a) Sarcoidosis b) Multiple sclerosis
d) Glosso pharyngeal nerve c) Trigeminal neuralgia d) Lupus erythematosis.
(AP -98) (KAR -03)
11. Facial pain due to elongated styloid process is called 21. Bell's palsy is triggered by
a) Cowden syndrome b) Tic doulourex a) Exposure to Cold b) Tooth extraction

1) C 2) D 3) D 4) B 5) B 6) A 7) C 8) C 9) C 10) B 11) C 12) A 13) B


14) B 15) B 16) D 17) A 18) D 19) A 20) C 21) D
Dental 9'ul-6e
c) Local and systemic infection 32. TENS therapy is useful in -
d) Any of the above a) MPDS b) Trigeminal neuralgia
(KAR -03) c) Facial palsy d) Neurosis
22. Lesion of facial nerve at level of stylomastoid foramen (KCET-09)
leads to: 33. Bell's Palsy is characterized by
a) Loss of taste sensation from Ant. 2/3 of tongue a) Bilateral involvement of the side of the face
b) Paralysis of orbicularis oculi muscle b) Inability to whistle
c) Loss of innervation to stapedius c) No loss of muscular control
d) Loss of lacrimal secretion d) Closing of the eyes
(AIPG -02) (COMED-2012)
23. A patient shows inability to close the right corner of the 34. Which one of the following is seen in facial palsy?
mouth is most probably suffering from: a) Drooping of eyelid
a) Myasthenia gravis b) Bell's palsy b) Loss of taste sensation in anterior 2/3 of tongue
c) TMJ dysfunction syndrome c) Excess of saliva secretion
d) Multiple sclerosis d) Loss of general sensation in cheek
(AIIMS -99) (COMEDK-14)
24. What is non characteristic of Eagle's syndrome: 35. The characteristic alarm clock headache is a feature of
a) Excessive lacrimation a) Auriculotemporal Neuralgia
b) Pain during mandibular movement b) Trigeminal Neuralgia
c) Stabbing type pain orginate in the tonsillar regions c) Sphenopalatine Neuralgia
d) When the jaws are closed the pain subsided d) Glossopharyngeal Neuralgia
(AIPG-05) (KCET-10)
25. Mask-like appearance of face with narrowing of aperture 36. Which of the following Orofacial pain is not associated
and rigidity of the mucosa is characteristic of: with vascular origin?
a) Progressive systemic sclerosis a) Cluster headache b) Giant cell arteritis
b) Tetanus c) Anaesthesia dolorosa
c) Multiple sclerosis d) Osteomalacia d) Chronic paroxysmal hemicrania
(AIPG-09, 11)
26. Which of the followi1ng drugs is not effective in case of 37. For myositis ossificans, main cause is?
Trigeminal Neuralgia? a) Infection b) Hereditary
a) Carbamazipine b) acetaminophen c) Benign pathology d) Trauma
c) Phenytoin sodium d) Baclofen (PGI JUNE-2012)
(COMEDK-05) 38. Paroxysmal lacrimation during mastication occurs due to
27. In an acute attack of migraine, the during of choice damage to
a) Ergontamine tortrate b) Methysergide a) Facial nerve b) Auriculotemporal nerve
c) Propranolol d) Caffeine c) Occulomotor nerve d) Mandibular nerve
(MHCET-15)
28. Patient comes with pain in pharyngeal region and is 39. Features of trigeminal neuralgia do not include
having carcinoma of nasopharynx. The diagnosis is ? a) Paresthesia b) Paroxysmal pain
a) Homer's syndrome c) Refractory period d) Presence of trigger zone
b) Glossopharyngeal neuralgia (APPG-15)
c) Trotter's syndrome d) Eagles syndrome
(AIPG-09)
29. Burning Mouth Syndrome describes pain associated with
a) Oral lichen planus b) Oral submucous fibrosis
c) Aphthous stomatitis d) No detectable oral disease
(COMEDK-10)
30. Anti-convulsants frequently used in management of
trigeminal neuralgia are
a) Phenytoin b) Gabapentin
c) Baclofen d) All of the above
(KCET-10)
31. An attack of cluster headache can be aborted by -
a) Morphine administration b) Breathing oxygen
c) Aspirin administration
d) Sublingual nitroglycerine administration
(KCET-09)

22) B 23) B 24) A 25) A 26) B 27) A 28) C 29) D 30) D 31) B 32) A 33) B 34) B
35) C 36) C 37) D 38) A 39) A
_____ r ORAL PATHOLOGY & MEDICINE

9. DISEASES OF NERVES & MUSCLES - ANSWERS


1. 'C' [Shafer 7th ed 853/ 6th ed 844] 11. 'C' [Shafer 7th ed 862/ 6th ed 853]
Trigeminal neuralgia ,or "Fothergill's disease" is characterized Eagles syndrome is due to either elongation of styloid process
by searing, stabbing or lancinating type of pain that is or ossification of stylohyoid ligament causing dysphagia,
initiated when the patient touches the "Trigger Zone". The sore throat, glossodynia, headache, and vague pain along
term "tic doulourex" is applied only when the patient suffers distribution of internal and external carotid arteries. Pharyngeal
from spasmodic contractions of facial muscles. pain and dysphagia are chief complaints in eagle's syndrome.

2. 'D' [Shafer 7th ed 854/ 6th ed 845] 12. 'A' [Burket's 10th ed 21/ 6th ed 848-49]
Trotter's syndrome is caused by "nasopharyngeal tumour"
and the patient will have symptoms like pain in lower jaw, 13. 'B' [Shafer 7th ed 857/ 6th ed 848]
tongue, and side of head with an associated middle ear
deafness; The patients also exhibit asymmetry and defective 14. 'B' [Shafer 7th ed 855/ 6th ed 846]
mobility ot sott palate. As the tumor progresses, trismus ot
internal pterygoid muscle may develop. The cause of pain is 15. 'B' [Burket's 11th ed 280/ 10th ed 329]
due to involvement of mandibular nerve in foramen ovale Aplastic anemia and leucopoenia are rare complications
through which tumour invades the calvarium. of carbamazepine therapy. So, the patients receiving
carbamazepine must have periodic hematological laboratory
3. 'D' [Shafer 7th ed 863/ 6th ed 854] investigations.
Miosis, ptosis, anhydrosis, and flushing are a features of
homer's syndrome. 16. 'D' [Shafer 7th ed 862/ 6th ed 853]

4. 'B' [Shafer 7th ed 854/ 6th ed 844] 17. 'A' [Burket's 10th ed 328]
Carbamazepine is the drug of choice in trigeminal neuralgia.
5. 'B' [Shafer 7th ed 857/ 6th ed 848]
Auriculo temporal syndrome or Frey syndrome or gustatory 18. 'D' [Shafer 7th ed 862/ 6th ed 853]
sweating is due to damage of auriculotemporal nerve Causalgia manifests as severe pain after injury or sectioning
following surgery in parotid and mandibular ramus areas, and of a peripheral sensory nerve following a difficult extraction.
subsequent innervation of sweat glands by parasympathetic The other names are complex regional pain syndrome and
salivary fibres. The patient typically exhibits flushing and reflex sympathetic dystrophy syndrome.
sweating of the involved side of the face, chiefly in the
temporal area, during eating. 19. 'A' [Shafer 7th ed 869/ 6th ed 860]
Myositis ossificans manifests by masses of fibrous tissue and
Neurotransmitter of Frey's syndrome is [AIIMS-2011] bone in the interstitial tissues of muscle, leading to atrophy
a) Acetylcholine b) Epinephrine and destruction of associated muscles. Masseter is commonly
c) Norepinephrine d) Serotonin involved in face and fixation of jaw occurs. The patient
becomes transformed into rigid like a "petrified man'.
6. 'A' [Burket's 11th ed 281/ 10th ed 329]
Geniculate neuralgia (Nervus intermedius neuralgia) results 20. 'C' [Shafer 7th ed 853/ 6th ed 844]
from herpes zoster infection of geniculate ganglion and
nervus intermedius branch of seventh cranial nerve. 21. 'D' [Shafer 6th ed 848]

7. 'C' [Shafer 7th ed 867/ 6th ed 858] 22. 'B' [Burket's 9th ed 335]
Involvement of "chordatympanic nerve" near its point of
8. 'C' [Burket's 11th ed 449/ 10th ed 492] origin in the facial canal is accompanied by paralysis of
Raynaud's phenomenon is due to intense vasospasm motor, gustatory and autonomic functions of nerve.
of peripheral arteries that results in change in colour of
fingertips as a response to cold or emotion. It is seen in Involvement of nerve after the level of stylomastoid foramen
scleroderma and L. E. (Eg: sarcoidosis of parotid, melkerson rosenthal syndrome)
results in paralysis of facial muscles without dysguesia.
9. 'C' [Burket's 11th ed 281/ Shafer 6th ed 849]
23. 'B' [Shafer 7th ed 858/ 6th ed 848]
10. 'B' [Shafer 7th ed 857/ 6th ed 848] Bell's palsy is manifested by drooping of corner of mouth,
Recurrent attacks of facial paralysis identical with bells drooling of saliva, watering of eye, inability to blink the
palsy, non-pitting, non-inflammatory painless edema of eye. The patient has a typical mask like or expressionless
face, cheilitis granulomatosa, and fissured tongue or lingua appearance. The patient will have speech difficulty and
plicata are features of 'Melkerrson-Rosenthal syndrome'. occasionally the taste sensation to anterior position of
tongue is lost or altered.
Dental 9'ul-6e
In supranuclear lesions of facial nerve, only the lower part • Topiramate
of the face is paralysed. The upper part (frontalis and part • Pimozide
of orbicularis oculi) escapes due to bilateral representation
in the cerebral cortex. Recently, first line management of trigeminal
neuralgia: (AIIMS Nov-13)
In infranuclear lesions of facial nerve (Bells palsy) half of the a) Peripheral neurectomy b) Anticonvulsant drugs
whole of face is paralysed. The face becomes asymmetrical, and c) Radiofrequency ablation
any attempt to smile draws the mouth to the NORMAL SIDE. d) Diagnostic regional nerve block

24. 'A' [Shafer 7th ed 863/ 6th ed 853] 31. 'B' [Burket's 11th ed 294]
Cluster headache is unilateral and among the most painful
25. 'A' [Shafer 7th ed 839/6th ed 831) of all headache disorders. Periorbital pain is commonly
associated with autonomic symptoms including homolateral
26. 'B' [Burket's 11th ed 280/ Shafer 6th ed 845-846) lacrimation. Abortive medicines including the use of 100%
oxygen at the outset of an episode, are often useful in
27. 'A' [Burket 11th ed 292/ 10th ed 335] confirming diagnosis. Oxygen inhalation results in rapid
Headaches are believed to be either vascular or muscular in resolution of symptoms over 70% cases.
origin. Migraine is most common of the vascular headaches,
and it occasionally causes pain of the face and jaws. Triptan therapy, short courses of glucocorticoids are also
Migraine is due to vasoconstriction of intracranial vessels effective.
(that causes neurologic symptoms) followed by vasodilation
that results in pounding headache. Basilar migraine is most 32. 'A' [Burket's 11th ed 243)
common in young women. Passive modalities such as ultrasound, laser and
transcutaneous electrical nerve stimulation (TENS) are often
Drugs that are useful in aborting migraine include used to reduce myofascial pain. TENS uses a low-voltage
ergotamine and sumatriptan. These drugs must be used biphasic current of varied frequency and is designed for
cautiously since they may cause hypertension and other CNS sensory counter stimulation for the control of pain. It is
problems. Methysergide or monoamine oxidase inhibitors thought to increase the action of modulation that occurs
such as phenelzine can be used to manage difficult cases. in pain processing at the dorsa.l horn of the spinal cord
and the trigeminal nucleus of the brain stem. Ultrasound
28. 'C' [Shafer 7th ed 854/ 6th ed 845) relies on high-frequency oscillations that are produced and
converted to heat as they are transmitted through tissue; it
29. 'D' [Shafer 7th ed 856/ 6th ed 847) is a method of producing deep heat more effectively than
Burning Mouth Syndrome (BMS) is burning or stinging of the patient could achieve by using surface warming.
the mucosa, lips, and/or tongue in the absence of visible
mucosal lesions. Van der Waal defined the term Burning 33. 'B' [Shafer's 6th ed 848)
mouth syndrome to refer only to idiopathic cases. There Bell's palsy is an abrupt, unilateral, peripheral facial nerve
is strong predilection with most female patients being paralysis. It is characterized by drooping of the corners of
menopausal and the age of onset being approximately 50 the mouth, watering of the eye, inability to close or wink
years. the eye, absence of forehead wrinkles on the affected side,
mask like expression less face, inability to whistle.
30. 'D' [Burket's 11th ed 280)
Anticonvulsant drugs are most frequently used and are 34. 'B' [Shafer 7th ed 858)
most effective in treatment of trigeminal neuralgia. Facial palsy is the paralysis of facial musculature (motor
Carbamazepine is the most commonly used drug and is an activity) and not the general sensation of the face (Face is
effective therapy for greater than 85% of newly diagnosed supplied by trigeminal nerve). Facial palsy is characterised
cases. by inability to close the eye, drooping of the corner of the
mouth resulting in drooling of saliva, absence of wrinkles
Patients who do not respond to Carbamazepine alone may on the forehead and a mask like face. Drooling of saliva
obtain relief from Baclofen or by combining Carbamazepine is due to the drooping of corner of the mouth and not
with Baclofen. overproduction or ptyalism. Speech and eating becomes
difficult and occasionally taste sensation on the anterior
Other drugs of choice for trigeminal neuralgia are: 2/3rd of tongue may be lost or altered.
• Gabapentin (less reliable than carbamazepine)
• Pregablin (gabapentine analogue)
• Oxycarbamazepine (less risk of blood dyscrasias than
Carbamazepine)
• Phenytoin
• Lamotrigine
3 5. 'C' [Shafer 7th ed 855/ 6th ed 846]
r ORAL PATHOLOGY & MEDICINE

• Also known as Tic douloureux or Fothergills' disease


• Trigeminal nerve on the right side of face is commonly involved.
Trigeminal neuralgia • The pain is searing, stabbing or lancinating type with a trigger zone on face
• Conditions with similar pain to trigeminal neuralgia are Trotter's syndrome and post-herpetic
neuralgia

• Trotter's syndrome is characterized by pain in pharyngeal region and carcinoma or tumor of


Trotter's syndrome
nasopharynx.

Post herpetic • Post herpetic neuralgia occurs after attacks of herpes zoster of the trigeminal nerve. The condition
neuralgia commonly involves ophthalmic division of trigeminal nerve

• Special form of zoster infection of the enuculate ganglion, with the involvement of external ear
Hunt's and oral mucosa
(or)
James Ramsay-hunt's • Clinical features include facial paralysis, pain of the external auditory meatus and pinna
syndrome of the ear and vesicular eruptions in oral cavity and oropharynx with hoarseness, tinnitus, vertigo
and occasional other disturbances.

• Characterized by severe headache or pain in the area of trigeminal distribution with signs of ocular
Para-trigeminal sympathetic paralysis.
(or)
Reader's syndrome • The sympathetic symptoms and homolateral pain the head or eye occur without vasomotor or
tropic disturbances.

• Described by Sudian, who concluded the involvement of Vidian nerve.


Sphenopalatine
neuralgia • Pain is thought to be generated at the level of pericarotid or cavernous sinus complex.
(or) • Characterized by unilateral paroxysms of pain in the region of eyes; the maxilla, the ear
lower half head and mastoid, and beneath the zyqoma.
ache • There is no trigger zone.
(or) • Unfortunately, the attacks develop regularly, usually atleast once a day, over a prolonged period
Sluder's head ache of time.
(or)
Horton's syndrome • Interestingly in some patients, the onset occurs at exactly the same time of day, because of which
it is referred to as alarm clock headache

• Characterized by triad of miosis, ptosis and anhidrosis & vasodilation over face
Homer's syndrome
(Note:- Hurler syndrome is a type of mucopolysaccharidoses)

Auriculo temporal (or) • Frey syndrome arises as a result of damage to auriculo temporal nerve and subsequent innervation
Frey syndrome of sweat glands by parasympathetic salivary fibres.
(or) Gustatory • The patient typically exhibits flushing and sweating of the involved side of the face, chiefly in
sweating temporal area, during eating.

• Unilateral in nature and manifests itself as a sharp, shooting pain in the ear, the pharynx, the
nasopharynx, the tonsil or the posterior portion of tongue.
Glossopharyngeal • Like trigeminal neuralgia, the patient usually has a 'trigger zone' in the posterior oropharynx or
neuralgia tonsillar fossa.
• The paroxysmal, rapidly subsiding type of pain which is characteristic of trigeminal neuralgia is
also a feature here.

• Rare neuralgia in the distribution of the sensory branches of the cervical plexus, most commonly
Occipital neuralgia in the neck and occipital region and is usually unilateral in distribution.
• Palpation below the superior nucal line may reveal a tender spot.

• Characterized by deep, dull pain in the orpharynx, and posterior auricular region, dysphagia, and
Stylohyoid (or) limited range of neck motion.
Eagle's syndrome • The disorder is due to an elongated styloid process or calcification of stylohyoid ligament.
• Pain relief following styloidectomy confirms the diagnosis.
Dental 9'ul-6e � - - - - - -

36. 'C' [Burket's 9th ed 338]

TAXONOMY OF PAIN

Anesthesia • It is a paradoxic pain in the region of sensory loss following an injury to a cranial nerve or a nerve root.
dolorosa • It is seen most often after surgical treatment of the trigeminal ganglion or root for neuralgia
Allodynia • Pain due to stimulus non-painful stimulus that does not normally produce pain (NEET-2013)
Hyperalgesia • Increased response to a stimulus that is usually painful.
• Increased sensitivity to stimulation and does not imply a painful sensation.
Hyperaesthesia
• Hyperalgesia is a special case of hyperesthesia.
Hypoaesthesia • Decreased sensitivity to stimulation.
Hypoalgesia • A special case of hypoesthesia in which pain response to normally painful stimuli is diminished.
Paraesthesia • Abnormal (but not unpleasant) sensation.
Dysesthesia • Unpleasant abnormal sensation (AIIMS MAY 2012)
• It is a painful syndrome with increased reaction to a stimulus and an increased threshold; faulty
Hyperpathia identificat-ion and localization of stimuls; delayed and radiating sensations and after-sensation may be
present.
It is a syndrome of:
• Burning pain
Causalgia
• Allodynia and
• Hyperpathia after a traumatic nerve lesion
• Described as perceptions that an individual experiences relating to a limb or an organ that is not
physically part of the body, for example Limb loss as a result of either removal by amputation or
Phantom pain congenital limb deficiency.
• Phantom limb pain is the feeling of pain in an absent limb or a portion of a limb.

The various orofacial pains of vascular origin are:


• Cluster headache
• Temporal arteritis (involves extracranial carotid circulation)
• Chronic paroxysmal hemicrania

37. 'D' [Shafer 7th ed 868,869]


Myositis ossificans progressive is a disease of unknown etiology characterised by formation of fibrous tissue and bone within
interstitial tissues of muscles, tendons, fascia, ligaments, aponeurosis and even the skin. Most cases of myositis ossificans occur
as a result of trauma, and thus the main demographic is young adults.

38. 'A' [Shafer 7th ed 857]


Paroxysmal lacrimation (crocodile tears) is seen in patients exhibiting profuse lacrimation when food is eaten, particularly
hot and spicy foods. It generally follows facial paralysis, either Bell's palsy type or the result of herpes zoster, head injury
or intracranial operative trauma. This is thought to be due to the degeneration of autonomic nerve, when adjacent normal
autonomic fibers will give sprouts that connect up with appropriate cholinergic or adrenergic endings, resulting in crocodile tears.

39. 'A' [Shafer 7th ed 851, Burkets 12th ed 315]


Trigeminal neuralgia is an archetype of orofacial neuralgias that follows the anatomic distribution of fifth nerve that exhibits a
trigger zone, stimulation of which initiates a paroxysm of pain. It is characterised by paroxysmal, excruciating pain in trigeminal
dermatomes. Latency (refers to the short time period between stimulation of a trigger area and pain onset) and refractory period
(occurs following an attack and during this time pain may not be initiated) are other 2 attack related phenomena related to TN.
_____ r ORAL PATHOLOGY & MEDICINE

10. SKIN AND VESICULOBULLOUS LESIONS


1. A flat, circumscribed discolouration of skin or mucosa d) Pemphigus, CHF, diabetes
that may vary in size and shape is referred to as (COMEDK -03, 06, KAR-04)
a) Epulis b) Macule 11. A fluid filled elevated lesion of skin is called
c) Nodule d) Papule a) Bulla b) Macule
(MAN -94,95; AP- 08) c) Papule d) Nodule
2. A twenty-one-year-old woman complains that regular, (AP -98)
gentle brushing of her teeth is painful besides causing 12. Which of the following is not a type of lichen planus?
profuse bleeding. Oral examination reveals the loss of a) Atrophic b) Hypertrophic
epithelium from the attached gingival of both arches. c) Verrucous d) Erosive
Which of the following dermatological problems is this (KAR -03)
patient most likely to have? 13. Unusual extensibility of the tongue is a characteristic
a) Benign mucous membrane pemphigiod feature of:
b) Chronic discoid lupus erythematosus a) rpidermolysis bullosa b) Syphilis
c) Pemphigus d) Psoriasis c) Darier-White disease d) Ehlers-Danlos syndrome
(MAN -95) (KAR -03)
3. Which of the following is an oral manifestation of lichen 14. Tzanck smear test is used in the diagnosis of
planus? a) Pemphigus b) ANUG
a) Dentinogenesis imperfecta c) Apthous disease d) Lichen planus
b) Fordyce spots (KAR -02}
c) White, chalky enamel surface 15. Koebner's phenomenon is seen with
d) White radiating lines on the buccal mucosa a) Erythema multiforme b) Pemphigoid
(MAN -95) c) Psoriasis d) Impetigo
4. Antinuclear antibodies are seen in (KAR -02)
a) SLE b) Systemic sclerosis 16. Darier's disease is associated with:
c) Morphea d) All of the above a) Pernicious anaemia
(MAN -98, KCET -07) b) Rickets with involvement of teeth and bones
5. Which of the following diseases of the skin is the most c) Vitamin A deficiency and involvement of oral epithelium
likely to be associated with partial anodontia? and skin
a) erythema multiformae d) Diffuse tender ulceration on the palate predominantly
b) hereditary ectodermal dysplasia (AIPG -04}
c) keratosis follicularils d) lichen planus 17. Erythema multiformae is:
(MAN -98, KAR -03) a) An acute self limiting disease, of skin and oral mucous
6. Which of the following is absent in CREST syndrome membrane
a) calcinosis cutis b) Raynaud's phenomenon b) Painless vesicular self limiting disease
c) Telagiectasis d) Endocrine disorders c) A viral disease d) Bacterial infection
(KAR -03, PGI -99, MAN -2K) (KAR -97)
7. Nikolsky's sign is positive in 18. Scleroderma involves:
a) bullous pemphigus b) eipdermolysis bullosa a) Tightening of oral mucosa a11d periodontal involvement
c) herpes simplex d) erythema multiforme b) Multiple palmar keratosis
(KAR -03, COMEDK -03) c) Raynaud's phenomenon d) All of the above
8. MONRO's abscess are seen in (KAR -97)
a) Pemphigus b) Lichen planus 19. In ectodermal dysplasia all of the following structures
c) Leukoplakia d) Psoriasis are affected except:
(MAN -2K, KAR -97,98) a) Hair b) Nails
9. In lichen planus the basal cells which are shrunken c) Teeth d) Salivary glands
with an eosinophilic cytoplasm and with a pyknotic and (AIPG -94)
fragmented nuclei are called 20. Pemphigus is characterized by:
a) Tzanck cells b) Civatte bodies a) Acanthosis b) Acantholysis
c) Donovan bodies d) Rushton bodies c) Hyperorthokeratosis d) Hyperparakeratosis
(MAN -01) (AIPG -94, 06)
10. Grinspan syndrome is associated with: 21. Histopathologic study of lichen planus shows:
a) Hypertension, diabetes, lichen planus a) Mixed cellular inflammatory infiltrate
b) Oral, ocular, genital lesions b) Presence of T-lymphocytes predominantly
c) Hypertension with oral lesions c) Antiepithelial antibodies

1) B 2) A 3) D 4) A 5) B 6) D 7) A 8) D 9) B 10) A 11) A 12) C 13) D


14) A 15) C 16) C 17) A 18) D 19) D 20) B 21) B
Dental 9'ul-6e
d) Scattered infiltrate with ill-defined lower border c) Necrotic pulp d) Chronic gingivitis
(AIIMS -99,94; KAR -98) (KAR -98)
22. Which of the following is inherited as an autosomal 33. Which of the following are seen in ectodermal dysplasia?
dominant trait? a) Hyperpyrexia
a) Lichen planus b) Bullous pemphigoid b) Protuberant lips and frontal bossing
c) Pemphigus yulgaris d) White sponge nevus c) Defective or absence of sweat glands
(KAR -98,AIIMS -94} d) Any of the above
23. LE. Cell phenomenon in peripheral blood is seen in: (AIIMS -90)
a) Rheumatic heart disease 34. Intra-epithelial bulla are found in
b) Infective endocarditis a) Pemphigus b) Bullous pemphigoid
c) Ischemic heart disease c) Bullous lichen planus d) Pemphigoid
d) Systemic Lupus Erythematosus (AIIMS-98}
(KAR -2K} 35. Lichen planus:
24. Lichenoid reactions are mainly due to: a) Can undergo malignant change
a) intake of certain drugs b) betel nut chewing b) Treated only by medication
c) cigarette smoking d) intake of alcohol c) Must be excised d) Is an idiosyncrasy reaction
(AP-99} (KAR -02)
25. Intraepithelial vacuolation with formation of vesicle 36. Oral lesions are not seen in:
or bulla intraepithelially above the basal layer is a) Psoriasis b) Pemphigoid
character istically seen in: c) Stevens-Johnson syndrome
a) Candida albicans b) Bullous pemphigoid d) Candidiasis
c) Pemphigus d) Lichen planus (AP -04)
(AIPG -95) 37. Oral, ocular and genital lesions are seen in:
26. A 40 year old woman reports with the complaint of a) Erythema multiforma
burning sensation in the mouth. Clinical examination b) Stevens-Johnson syndrome
reveals lesions consisting of radiating white striations c) SLE d) None of the above
in a retiform arrangement affecting buccal mucosa, (AP -04}
tongue, lips & gingiva bilaterally. An incisional biopsy is 38. Wickham's striae are seen in
suggestive of lichen planus. The following are different a) Lichen planus b) Leukoplakia
clinical forms of Lichen Planus except c) Leukoedema d) Erythema multiformae
a) Atrophic Lichen Planus b) Hypertrophic Lichen Planus (AP -03}
c) Bullous Lichen Planus d) Verrucous Lichen Planus 39. In which of the following disorders a circulating
(COMEDK-2011} antibody directed to intercellular cementing substance
27. lmmunoflourescence test is positive in: of stratified squamous epithelium is observed:
a) Psoriasis b) Pemphigus vulgaris a) Lichen planus b) Verrucous vulgaris
c) Lupus erythematosus d) Scleroderma c) Bullous pemphigoid d) Pemphigus vulgaris
(AIPG -95} (AIIMS, KAR -03}
28. Immunoflourescence is seen at basement membrane as 40. White radiating lines can be observed in case of lesions of
patchy distribution in a) Lichen planus b) Erythema multiforme
a) Lichen Planus b) Pemphigus c) Pemphigus d) Leukoplakia
c) Pemphigoid d) Lupus erythematosus (AP -2K, AIIMS MAY -2012}
(KAR -97} 41. Oral diagnostic features of scleroderma include all of the
29. Erosive lichen planus resembles which of the following following, except:
a) Monilial gingivitis b) Desquamative gingivitis a) A hard and a rigid tongue
c) Herpetic gingivitis d) Acute ulcerative gingivitis b) Widening of the oral aperture
(AIIMS -2K} c) Pseudo ankylosis of the T.M. Joint
30. A 60-year-old has got severe bulla and target lesion d) Difficulty in swallowing
which erythema around halo and genital lesions: (KAR -2K}
a) Stevens Jhonson syndrome 42. Primary lesion in lichen planus is:
b) Herpes zoster a) Macule b) Pa.pule
c) Herpes simplex d) Herpangina c) Vesicle d) Bulla
(AIIMS -2K} (KAR -97)
31. Target lesions are observed in case of 43. Bullae formation after striking an intact skin/mucosa!
a) Erythema multiforme b} Lichenplanus surface is known as:
c) Pemphigus vulgaris d} Psoriasis a) Tinel's sign b) Bablnski's sign
(COMEDK -03, 06; AP -2K} c) Nikolsky's sign d) Chovstek's sign
32. fine Needle aspiration biopsy is indicated to diagnose: (KAR -ZK}
a) Traumatic ulcer b) Pemphigus

22} D 23) D 24) A 25) C 26} D 27) B&C 28) C 29) B 30) A 31) A 32) B 33) D 34} A
35) A 36) A 37) B 38) A 39) D 40) A 41) B 42) B 43) C
44. Lupus erythematosus is
r ORAL PATHOLOGY & MEDICINE
55. Ectodermal dysplasia is:
a) Reactive lesion b) Degenerative condition a) Autosomal recessive b) Autosomal dominant
c) Autoimmune disorder d) Neoplastic condition c) X-linked dominant d) X-linked recessive
(KAR -03) (AIPG-05)
45. Psoriasis is associated with: 56. Pathologic calcification is seen in -
a) Geographic tongue a) Scleroderma b) Lichen planus
b) Benign median rhomboid glossitis c) Dystrophic epidermolysis bullosa
c) Lupus erythematosus d) Lupus vulgaris d) Lupus erythematosus
(PGI -95) (COMEDK-09)
46. Formation of multiple pinpoint bleeding spots on 57. Joint erosions are not a feature of:
scratching the skin is characteristic of: a) Rhematoid arthritis b) Psoriasis
a) Pemphigus vulgaris b) Lupus erythematosus c) Multicentric reticulohistiocytosis
c) Psoriasis d) Herpangina d) Systemis lupus eythematosus
(KAR -2K) (AIPG-06)
47. Mucocutaneous lesion associated with neoplasia 58. Multiple pulp stones are seen in
a) Pemphigus vegetans b) Parapemphigus a) Down's syndrome b) Ehler's Dantos syndrome
c) Paraneoplastic pemphigus c) Marfan Syndrome d) Apert's syndrome
d) Familial benign pemphigus (COMEDK-10, 11)
(COMEDK-2011) 59. Grinspan syndrome is associated with
48. Steven-Johnson syndrome involves : a) Leukoplakia b) Lichen planus
a) Type I hypersensiitivity reaction c) Aphthous ulcer d) Oral submucous fibrosis
b) Type II hypersensitivity reaction (KAR-04)
c) Type III hypersensitivity reaction 60. Psoralane Ultra Violet A (PUVA) therapy is advised in
d) Type IV hypersensitivity reaction a) Pemphigus vulgaris b) Apthous ulcers
(AIPG -98) c) Carcinoma in situ d) ANUG
49. Which is a degeneration disorder characterized by (COMEDK-05)
atrophic changes of the deeper structures (e.g. fat, 61. In Cicatrial pemphigoid, which antigen is bound by IgG
muscle, cartilage & bone) involving one side of the face: on the epidermal side of the salt split skin technique:
a) Scleroderma b) Parry Romberg syndrome a) XVII collagen b) epiligrin
c) Miescher's syndrome d) Peutz-Jeghers syndrome c) Laminin 5 d) BP antigen 1 & 2
(AIPG -04, 05) (COMEDK-07)
50. A 3-year-old patient has extensive vesicles on lip, 62. Subepithelial vesicles are characteristic all of the
tongue, oral mucous membrane. After 2-4 days vesicles following EXCEPT
rupture at followed by pseudomembrane formation and a) Bullous pemphigoid b) Cicatricial pemphigoid
also some dermal lesions seen what will be the diagnosis c) Pemphigus
a) Herpetic stomatitis b) EM d) Epidermolysis bullosa acquisita
c) ANUG d) Steven-Johnson syndrome (KCET-08)
(AP -04) 63. All are diseases of skin except
51. The primary cause of acantholysis in pemphigus vulgaris IS a) Erythema multiforme b) Keratosis follicularis
a) auto immunity b) intercellular oedema c) Erythema migrans d) Psoriasis form lesion
c) intra epithelial oedema d) chronic alcoholism (AIIMS-09)
(KAR -98) 64. Hydropic degeneration of the basal cell of the stratum
52. Which sites are characteristically affected in Stevens­ germinativum is a feature of?
Johnson syndrome? a) Leukoplakia b) Lichen Planus
a) Liver, spleen, pancreas c) Syphilis d) Pemphigus
b) Conjunctiva, genitalia, oral mucosa (PGI-08)
c) Oral mucosa, lacrimal apparatus, ears 65. Butterfly rash is typically seen in
d) Parotid gland, pa.late, conjunctive a) Herpes simplex b) Systemic lupus erythematosus
(APPSC -99) c) Scleroderma d) None of the above
53. Histological clefts in lichen planus are - (KCET-10}
a) Civatte bodies b) Wickham's Striae 66. Cafe au lait macules are seen in
c) Max - Joseph spaces d) Auspitz's sign. a) Von Reklinghausen's neurofibromatosis
(COMEDK-09) b) Albright's syndrome and Blo,om's syndrome
54. The swollen degenerating epithelial cell due to c) All of the above d) None of the above
acantholysis is (KCET-10)
a) Anitschow cell b) Tzanck cell
c) Ghost cell d) Prickle cell
(KAR-04)

44) C 45) A 46) C 47) C 48) C 49) 8 50) 8 51) A 52) 8 53) C 54) 8 55) D 56) A
57) D 58) 8 59) 8 60) A 61) A 62) C 63) C 64) 8 65) 8 66) C
Dental 9'ul-6e
67. Ehlers Dantos syndrome is? 75. False about mucous-membrane pemphigoid
a) Autosomal Dominant b) Autosomal recessive a) Twice in female
c) X-Linked Dominant d) X-Linked recessive b) Autoantibodies towards basement membrane protein
(AIPG-09) c) Disease associated antigen are most frequently present
68. Oral lesion associated with ulcerative colitis in lamina densa
a) Lichen planus b) Pyostomatitis Vegetans d) Oral mucosa is most frequently involved
c) Sarcoidosis d) Dermatitis herpetiformis (PGI JUNE-2014)
(COMEDK-10) 76. Which of the following individual show susceptibility to
69. Xeroderma pigmentosum is characterized by dental caries?
a) Autosomal dominant inheritance a) Hereditary fructose intolerance
b) Inability to repair sunlight induced damage to DNA b) Down syndrome
c) Irregular accumulation of melanin in the basal cell layer c) Pierre robin syndrome d) Epidermolysis bullosa
d) Acanthosis of epithelium with elongation of rete ridges (PGI D EC-2013)
(COMED-2012) 77. Autoantibodies Ani Ro and Anti La are completely absent
70. All the following are inherited disorders of connective in
tissue EXCEPT a) Systemic lupus erythematosus
a) Alport syndrome b) Ehlers-Danlos syndrome b) Sjogren's syndrome
c) Marfan syndrome d) McArdle's disease c) Diffuse Scleroderma d) Rheumatoid arthritis
(KAR- 2013) (COMEDK-15)
71. Fish Net pattern in pemphigus vulgaris is seen in which
of the following tests?
a) Direct immunofluorescence
b) Tzanck smear
c) FNAC d) Histopathology
(KAR- 2013)
72. 60 year old diabetic female presented with burning
sensation to spicy food. Intraoral examination revealed
multiple periodontal abscess and keratotic area in a lace
pattern with occasional erosive areas inside the lace
pattern.

A.Provisional diagnosis for this lady will be


a)Oral hairy leukoplakia b) Oral lichen planus
c)Oral squamous cell carcinoma
d)Oral pemphigus
(COMEDK-14)
B. Syndrome associated with this disease is
a) Down's syndrome b) Grinspan syndrome
c) Peutzjeghers syndrome d) Sjogren's syndrome
(COMEDK-14)
C. Histological feature will be
a) Elongated rete ridges b) Flattened rete ridges
c) Saw tooth rete ridges d) Bulbous rete ridges
(COMEDK-14)
73. Which one of the following is NOT a lichenoid reaction?
a) Graft versus Host disease
b) Fixed drug eruptioin
c) Pemphigus vulgaris d) Secondary syphilis
(COMEDK-14)
74. A typical example of an immunologically mediated
collagen vascular/connective tissue disorder is
a) Lichen planus b) Pemphigus vulgaris
c) Lupus erythematous d) Epidermolysis bullosa
(MCET-14)

67) A 68) B 69) B 70) D 71) A 72A) B 72B) B 72C) C 73) A 74) C 75) C 76) D 77) D
_____ r ORAL PATHOLOGY & MEDICINE

10. SKIN AND VESICULOBULLOUS LESIONS - ANSWERS


1. 'B' [Burket's 11th ed 42/ 10th ed 50] Because of degeneration of basal keratinocytes, the
epithelial-connective tissue interface is weakened resulting
2. 'A' [Shafer 7th ed 830/ 6th ed 822] in histologic clefts known as Max-Joseph spaces.

3. 'D' [Shafer 7th ed 809/ 6th ed 800] 10. 'A' [Shafer 7th ed 808/ 6th ed 800]

4. 'A' [Shafer 7th ed 837/ 6th ed 829] 11. 'A' [Burket's 11th ed 42/ 10th ed 50]
Well circumscribed, flat lesions that are
5. 'B' [Shafer 7th ed 806/ 6th ed 798] Macules
inflammed or pigmented.
Ectodermal dysplasia is characterized by congenital dysplasia
of ectodermal structures, manifested as hypohidrosis (partial Solid lesion of< 1cm in diameter that are raised
or complete absence of sweat glands) hypotrichosis and Papules above skin. Seen in Erythema multiformae,
hypodontia or partial anodontia. Rubella, L. E, Sarcoidosis, and Darriers disease
Plaques Large papules >lcm in diameter
Two types Elevated blister like lesions that contain clear
Vesicles
• Hypohidrotic (Christ-Siemens-Touraine syndrome) most fluid and <lcm in diameter
common type and includes dental manifestations. Bullae Large vesicles that are >1cm in diameter
• Hidrotic (Clouston syndrome)- no specific dental defects
Pustules Elevated lesions containing purulent material.
are seen.
12. 'C' [Shafer 7th ed 809/ 6th ed 801]
6. 'D' [Shafer 7th ed 839/ 6th ed 831]
CREST Syndrome is acronym of Five major findings:
13. 'D' [Shafer 6th ed 833]
• Calcinosis cutis, EHLERS -DANLOS syndrome is a connective tissue disorder
• Raynaud's phenomenon, characterized by hyper elasticity of skin, hyper extensibility
• Esophageal dysfunction, of joints (Rubber man) and fragility of skin and oral mucosa)
• Sclerodoctyly and
Hyper mobility of TMJ, resulting in "repeated dislocations"
• Telengiectasiasis of Jaw; lack of normal scallo·ping of the dentinoenamel
junction, passage of many dentinal tubules into the enamel,
7. 'A' [Shafer 7th ed 825/ 6th ed 817] with a tendency to form multiple pulp stones (COMEDK-1O)
The loss of epithelium occasioned by rubbing apparently are oral manifestations of EHLERS -DAN LOS syndrome
unaffected skin is termed Nikolsky's sign. In pemphigus, it
is caused by prevesicular edema which disrupts the dermal - 14. 'K [Shafer 7th ed 826/ 6th ed 818]
epidermal junction. Tzanck test is also used for diagnosis of viral diseases like
Herpes Zoster.
"Nikolsky's sign" is positive in:
• Pemphigus 15. 'C' [Burket's 10th ed 298]
• Familial benign chronic pemphigus (hailey-hailey disease)
16. 'C' [Shafer 4th ed 820]
• Recessive dystrophic epidermolysis bullosa [ Epidermolysis
Darrie(s disease shows benign dyskeratosis, which is
bullosa occurs due to alteration in structure of type VII
characterized by typical dysplastic cells called "corps, ronds
collagen. (COMED K-1O)]
and grains".
8. 'D' [Shafer 7th ed 813/ 6th ed 804]
17. 'A' [Shafer 7th ed 814/ 6th ed 806]
MONRO'S abscesses are seen in psoriasis form of lesions,
which include, Psoriasis, Reiter Syndrome, Benign migratory
18. 'D' [Shafer 7th ed 839/ 6th ed 831]
glossitis and ectopic geographic tongue.
[Note: MONRO'S abscesses with elongated rete ridges are 19. 'D' [Shafer 7th ed 806/ 6th ed 798]
seen in Benign migratory glossitis while the rete ridges are (Note:- Option 'D' is correct according to shafer 4th edition.
not elongated in Reite(s syndrome] Actually all the 4 options are involved in ectodermal
hypoplasia. Hypoplasia of salivary glands with resultant
9. 'B' [Shafer 7th ed 810/ 6th ed 802] xerostomia is also seen in ectodermal dysplasia.)
Colloid bodies , also known as civatte bodies or hyaline
bodies or fibrillar bodies are often present in spinous and 20. 'B' [Shafer 7th ed 826/ 6th ed 818]
basal layers. They appear as round eosinophilic globules In pemphigus there is formation of vesicle or bulla
representing degenerated epithelial cells. intraepithelially, just above the basal layer producing the
Dental 9'ul-6e
distinctive suprabasilar split. The intercellular bridges 27. 'B & C' [Shafer 6th ed 818)
in supra basal layers disappear due to edema resulting Type of
in acantholysis. Because of this, clumps of degenerating Disease Reacts with
Flourescence
epithelial cells are found in the vesicular space, which are
known as Tzanck cells. • Intercellular
deposition of
21. 'B' [Shafer 7th ed 810/ 6th ed 802) IgG
IgG; sometimes C3,
HistoLogic features of Lichen pLanus include: Pemphigus • "Granular"
IgA, IgM
• Saw, tooth appearance of retepegs intercellular
• Liquefaction degeneration of basal layer which is space
replaced by thin band of eosinophilic coagulum fluorescence
• Presence of civatte bodies Fluorescence along
the basement
• Characteristic band like subepithelial mononuclear
infiltrate consisting of T- lymphocytes and histiocytes membrane zone
Lichen planus Antifibrinogen
with numerous
sharply limited to papillary and most superficial portion
�xl�rt�iun� inlu
of the reticular layers of connective tissue
the lamina propria
• Hyperparakeratosis or hyperorthokeratosis
• C3 (97%), IgG Patchy Li near
• Thickening of granular layer (51%), IgM pattern along the
Cicatricial (36%), IgA basement
22. 'D' [Shafer 7th ed 821/ 6th ed 813)
Pemphigoid (33%) membrane
Lichen planus, pemphigoid, and pemphigus are autoimmune
diseases whereas white sponge nevus is inherited as • C3 > lgG > lg M
autosomal dominant trait. > IgA
Tissue-bound
23. 'D' [Shafer 6th ed 829) anti-basement
Bullous
The addition of blood serum from a person under suspicion IgG membrane zone
pemphigoid
to the buffy coat of normal blood is the basis for LE .cell antibodies/
phenomenon. If the patient is L.E. +ve, then typical "L.E" patchy Linear
Cell develop. The cell consists of rosette of neutrophils Erythema Patchy Linear
------
surrounding a pale nuclear mass, derived from a lymphocyte. multiformae
24. 'A' [Shafer 7th ed 680/ Burket's 10th ed 110)
"Speckled" or
Discoid Lupus
Lichenoid reactions are drug induced and exhibit "Particulate"
erythematosus IgG, IgM, IgA, C3
histopathological features similar to lichen planus. The pattern at
(Only lesional & fibrinogen
common drugs implicated in oral lichenoid reactions are the basement
mucosa & skin)
- antihypertensives, NSAIDS, pencillamine followed by membrane
dapsone, ketocanazole, streptomycin, sulfamethoxazole, Fluorescence
tetracycline, chloroquine, and oral hypoglycemic agents, etc. Systemic Lupus at basement
erythematosus membrane zone,
Drug induced lichenoid reactions may resolve promptly (both normal IgG, IgM, IgA with numerous
when the offending drug is eliminated. Disease exhibiting & lesional extensions into
lichenoid reactions include lichen planus, erythema mucosa & skin) the superficial
multiformae, secondary syphilis and lupus erythematosus lamina propria
[Note: Oral lichen planus like reaction (lichenoid reaction)
appears as striae radiating from a central erythematous area 28. 'C' [Shafer 7th ed 810/ 6th ed 803]
at the site of placement of the betel quid. Refer Shafer 5 th
edition page 136] 29. 'B' [Shafer 7th ed 809/ 6th ed 801)
Desquamative gingivitis represents oral manifestations of
25. 'C' [Shafer 7th ed 826/ 6th ed 818) various diseases like
a) Lichen planus (Erosive or atrophic form),
26. 'D' [Shafer 7th ed 809/ 6th ed 801] b) Cicatrial pemphigoid or Benign mucous membrane pemphigoid
The different forms of Lichen planus are reticular form, c) pemphigus.
bullous form, atrophic form, hypertrophic and erosive forms.
Verrucous variant is seen in leukoplakias. Cicatrial pemphigoid is the synonym for [AP-2012)
a) mucous membrane pemphigoid
b) pemphigus vulgaris
c) Hailey-Hailey disease
d) paraneoplastic pemphigus
___ r ORAL PATHOLOGY & MEDICINE
Scleroderma can be circumscribed (Morphea) or linear. The
Most common lesion in Mucous membrane pemphigoid is
a) Esophageal ulcer b) Symblepharon linear bands are often called as En cuop de sabre, since
c) Ulcer of Soft palate d) Desquamative gingivitis they resembles the mark produced by the blow of a saber.
(COM ED K-14)
"En coup de sabre" is most likely to be associated with
30. 'A' [Shafer 7th ed 815/ 6th ed 806] which type of scleroderma? (KAR- 2013)
Stevens - Johnson syndrome is very severe bullous form of Ans: Linear
erythema multiformae and is characterized by oral, ocular
and genital lesions. 42. 'B' [Shafer 7th ed 808/ 6th ed 800]

31. 'A' [Shafer 7th ed 815/ 6th ed 806] 43. 'C' [Burket's 11th ed 63/ Shafer 6th ed 817]
"Target" or "Iris" or "Bull's eye" lesions on hands, wrists,
and ankles are chara,cteristic of erythema multiformae. 44. 'C' [Shafer 7th ed 835/ 6th ed 829]

32. 'B' [Shafer 4th ed 832] 45. 'A' [Shafer 7th ed 812/ 6th ed 804]

33. 'D' [Shafer 7th ed 806/ 6th ed 798] 46. 'C' [Shafer 7th ed 812/ 6th ed 803]
Hyper pyrexia due to hypohidrosis, hypotrichosis,
hypodontia, protuberant lips and depressed nasal and 47. 'C' [Burket's 11th ed 65/ Shafer 7th ed 829]
supraorbital bridges are important features of ectodermal Paraneoplastic pemphigus is a severe variant of pemphigus,
dysplasia. associated with an underlying neoplasm - most frequently
non Hodgkin's lymphoma, chronic lymphocytic leukemia,
34. 'K [Shafer 7th ed 826/ 6th ed 818] thymoma, and castleman dis.ease. Tumor antigens are
Intraepithelial bulla Subepithelial bulla hypothesized to evoke an immune response that leads to
the development of an autoimmune response to intercellular
• Herpes simplex • Pemphigoid adhesins (plakins). This autoantibody response leads to
• Herpes zoster • Bullous pemphigoid blistering in mucosa and other epithelia.
• Chicken pox (most common
subepithelial blistering
• Pemphigus Desmoplakin is the target antigen in [AP-2012]
disease)
• Familial benign a) Pemphigus vulgaris
• Bullous lichenplanus b) Paraneoplastic pemphigus
pemphigus (Hailey-Hailey
disease) • Dermatitis herpeti- c) Drug induced pemphigus
formis d) Pemphigus foliaceous
• Epidermolysis bullosa­
dystrophic recessive form • Epidermolysis bullosa
48. 'C' [Burket's 10h ed 57]
• Oral lesions of erythema • Skin lesions of erythema
Both erythema multiformae and steven Johnson syndrome
multiforme multiformae
are due to deposition of immune complexes in the superficial
microvasculature of skin and mucosa. These immune complex
35. 'K [Shafer 7th ed 811/ 6th ed 803] diseases are mediated through type III hypersensitivity
reaction.
36. 'A' [Shafer 7th ed 812/ 6th ed 804]
If the deep scales in psoriasis are removed, one or more tiny 49. 'B' [Shafer 7th ed 15/ 6th ed 14]
bleeding points are seen. This feature is known as Auspitz's
sign and this is characteristic of psoriasis. 50. 'B' [Shafer 7th ed 815/ 6th ed 806]
• Option 'D' steven-johnson syndrome is characterized by
37. 'B' [Shafer 7th ed 815/ 6th ed 806]
oral, genital and ocular lesions.
38. 'A' [Shafer 7th ed 809/ 6th ed 800] • Option 't,:. herpetic gingivostomatitis usually occurs in
children and young adults.
39. 'D' [Shafer 7th ed 826-27/ 6th ed 816] • Option 'C' though pseudomembrane formation is seen in
ANUG, no dermal lesions are seen.
40. 'A' [Shafer 7th ed 809/ 6th ed 800]
51. 'K [Shafer 7th ed 826/ 6th ed 816]
41. 'B' [Shafer 7th ed 839-40/ 6th ed 832]
Important features of scleroderma: 52. 'B' [Shafer 7th ed 815/ 6th ed 806]
• Stiff and board like tongue
53. 'C' [Shafer 7th ed 810/ 6th ed 802]
• The lips become rigid and fixed microstomia
In Lichen Planus, there is degeneration of keratinocytes of
• Dysphasia, inability to open and close the mouth stratum basale (stratum germinativum) and disruption of
• Extreme widening of periodontal ligament. anchoring elements of the epithelial basement membrane
Dental 9'ul-6e
and basal keratinocytes (eg: Hemi desmosomes, filaments, 58. 'B' [Shafer 7th ed 843/ 6th ed 835]
fibrils). This weakens the epithelial-connective tissue Ehler Dantos syndromes:
interface resulting in formation of histologic clefts (i.e., Includes more than 10 different disorders involving a genetic
Max-Joseph spaces). defect in collagen and connective tissue synthesis.
• Classic variety
54. 'B' [Shafer 7th ed 826/ 6th ed 818)
Types 1 & 2 • Causative mutations may involve the
55. 'D' [Shafer 7th ed 80 6/ 6th ed 798] COLSAl, COL5A2, and tenascin-X genes
Type 4
Decreased amount of type III collagen
56. 'A' [Shafer 7th ed 839/ 6th ed 831) (dangerous)
Systemic sclerosis: Characterized by deficiencies in
• Characterized by ultimate induration and atrophy of hydroxylase and lysyl oxidase, (post-
Types 5 & 6
skin and fixation of epidermis to deeper subcutaneous translational modifying enzymes in
tissues. The skin becomes hardened and atrophic and collagen biosynthesis)
cannot be wrinkled or picked up because of its firm Due to deficiency of amino-terminal pro-
fixation to the deep connective tissue. This contracture Type 7
collagen peptidase
of skin gives a mask-like appearance to the face of claw­
like appearance to hands. Type 9 Due to abnormal copper metabolism
• Scleroderma can be circumscribed (morphea) or linear Due to non-functioning plasma
Type 10
(Coup de sabre) (KCET-11). fibronectin
• The tongue becomes stiff and board like, causing the
patient difficulty i in eating and speaking. The characteristic clinical features of this disease are the
• Reduced opening of mouth and fixation of jaw as a result hyperelasticity of skin, hyperextensibility of the joints
(Rubberman) and fragility of the skin and blood vessels
of involvement of the peritemporo mandibular joint
resulting in excessive bruising as well as defective healing.
tissues making dental care very difficult.
• Some times deposition of calcium in affected areas is
Oral manifestations:
also found. • Oral mucosa is very fragile and bruised easily.
• Radiographic features:
• Gingival hyperplasia with fibrous nodules.
- Extreme widening of PDL, two to four times normal
thickness is diagnostic. • Hypermobility of TMJ, resulting in repeated dislocation
- Bone resorption of angle of mandibular ramus and of the jaw.
partial or complete resorption of condyles and/or • Lack of normal scalloping of the DEJ.
coronoid processes of the mandible are also seen. • Passage of many dentinal tubules into the enamel.
• Overall, increased collagen in tissues is characteristic • Formation of irregular dentin.
feature of systemic sclerosis or scleroderma. • Increased tendency to form multiple pulp stones.
• Hypoplasia of enamel may also be seen and the teeth
57. 'D' [Burket 11th ed 444/ 10th ed 486 - 87]
have been reported to be fragile and have a tendency for
SLE is an multisystem autoimmune inflammatory diorder and
fracture.
is known as the great mimic (since it virtually affects every
organ system).
Multiple pulp stones are also seen in type II or coronal
Systems Manefestations dentin dysplasia.
• Butterfly rash on malar area
Cutaneous • Papulosquamous Annular- 59. 'B' [Shafer 7th ed 808/ Burket 10th ed 1103]
&
polycystic eruptions
60. 'A' [Burket 10th ed 70]
Proteinuria; glomerulonephritis & According to Rook and Colleagues the extra corporeal
Renal
renal failure photochemotherapy for drug resistant pemphigus vulgaris
• Arthralgia include administration of 8-methoxypsoralen followed by
• Non erosive symmetric arthritis of exposure of peripheral blood to ultraviolet radiation. It is
Musculoskeletal also used for vitiligo and graft versus host disease.
hands, wrists, knees, TMJ (60%)
• Myalgia & myositis
61. 'A' [Burket 11th ed 68/ 10th ed 72]
CNS Psychosis, stroke, seizures Majority cases of mucous membrane pemphigoid or cicatrial
Vasculitis, pericarditis, endocordial pemphigoid demonstrate I1 G directed against antigens on
CVS the epidermal side of the salt-split skin, which have been
damage, myocarditis
identified as BP 180 or XVII collagen. The antigen present
Defects in coagulation pathways
on the dermal side is epiligrin (Laminin 5)
Miscellaneous fatigue, depression, pancreatitis,
hepatitis, premature ovarian failure
62. 'C' [Shafer 7th ed 826/ 6th ed 818/ Also refer synopsis]
r
63. 'C' [Shafer 6th ed 31/ Burket's 9th ed 258)
ORAL PATHOLOGY & MEDICINE
the vesicular space, known as Tzanck cells. These changes
Option 'C' erythema migrans is other name of ectopic are seen in cytological smears also known as the Tzanck
geographic tongue / annulus migrans in which lesions are smear or Tzanck test. The fluid in most vesicles has scarce
not always restricted to the tongue, and similar irregular or inflammatory cell infiltration.
circinate lesions occurring elsewhere in the oral cavity.
Direct Immunofluorescence predominantly demonstrates the
64. 'B' [Shafer 7th ed 810/ 6th ed 802) presence of IgG in combination with IgM, IgA, C3. Direct
Refer Q No. 53 Immunofluorescence reveals a typical fish net pattern or
chicken wire pattern of IgG antibodies against desmoglein
65. 'B' [Shafer 7th ed 835/ 6th ed 827) 1 or 3 even in the perilesional skin.
The cutaneous lesions of SLE consist of erythematous patches
on the face that coalesce to form a roughly symmetrical 72A. 'B' [Shafer 7th ed 808)
pattern over the cheeks and across the bridge of the nose Oral lichen planus is a common mucocutaneous disease,
in form of 'Butterfly' distribution. When SLE involve the characterised by bilateral white striations, papules, plaques
kidney, the glomerular capillaries shown fibrinoid thickening on the buccal mucosa, tongue and gingiva. Erosive form
and produces characteristic 'wire loops', which may be may present as erosions or frankly ulcerated lesions, with
sufficient to result in renal insufficiency. characteristic radiating striae on the periphery of the
individual lesions. Grinspan's syndrome is an interesting
66. 'C' [Shafer 7th ed 202, 714/ 6th ed 198, 710) association of lichen planus, diabetes mellitus and vascular
'Bloom syndrome' is characterized by a high frequency hypertension.
of breaks and rearrangements in an affected person's
chromosomes. The affected person characteristically 72B. 'B' [Shafer 7th ed 808)
develops a facial rash after exposure to sun. Other clinical Grinspan's syndrome is an interesting association of lichen
features include a high pitched voice, distinct facial features planus, diabetes mellitus and vascular hypertension.
such as long, narrow face, micrognathism of mandible, and
pigmentation changes of skin with cafe aulait spots. 72C. 'C' [Shafer 7th ed 810)
The histological features include hyperparakeratosis or
67. 'A' [Shafer 7th ed 842 Table 19-1/ 6th ed 834) hyperorthokeratosis,with thickening of granular layer,
Ehler-Danlos syndrome is classified into 8 types and more acanthosis with intracellular edema, saw tooth appearance
than 5 types show autosomal dominant transmission. of rete ridges, band like subepithelial mononuclear infiltrate,
degenerating basal keratinocytes that form colloid known as
68. 'B' [Shafer 7th ed 336/ 6th ed 330) Civatte, hyaline, cytoid bodies.
In pyostomatitis vegetans, the oral lesions are one part
of the syndrome, in which the patients also manifest 73. 'A' [Burket 11th ed 89, 95)
concomitantly ulcerative colitis or other gastrointestinal Lichenoid reactions represent a family of lesions with
disturbances. different etiologies with a common clinical & histological
appearance. They include Llchenoid contact reactions,
The oral lesions consist of large numbers of broad-based Lichenoid drug eruptions, Lichenoid reaction of graft versus
papillary projections, tiny abscesses or vegetations host disease.
developing in areas of intense erythema. These lesions
may occur in any area of the oral cavity, although tongue 74. 'C' [Shafer 7th ed 835]
involvement appears to be uncommon. Systemic Lupus Erythematosus is a multisystem autoimmune
disease characterised by autoantibodies, immune complex
69. 'B' [Shafer's 7th ed 127/ 6th ed 121) formation and immune dysregulation resulting in damage to
Xeroderma pigmentosum is a genetically determined disorder, kidney, skin, blood vessels, CNS. Pemphigus, Lichen planus,
in which defective DNA repair mechanisms lead to chronic epidermolysis bullosa are mucocutaneous disorders.
UV damage and subsequent development of different sun
related skin tumors, including melanoma, in sun exposed 75. 'C' [Burkets 12th ed 83]
areas. Mucous membrane pemphigoid is a chronic autoimmune
subepithelial disease in which the autoantibodies are
70. 'D' [Shafer 6th ed 833, 696) directed against proteins in basement membrane zone, more
McArdle's disease is also known as glycogenosis type V, frequently against antigens in lamina lucida portion of
which is an inherited glycogen storage disorder due to basement membrane zone.
deficiency of myophosphorylase deficiency.
76. 'D' [Shafer 7th ed 833)
71. 'A' [Ref: Immunofluorescence of the outer root sheath in Junctional epidermolysis bullosa and recessive variant
angen and telogen hair: an aid to diagnosis in Pemphigus­ Epidermolysis bullosa ctystrophic variant manifest severe
International J of Trichology2009: 1(2):138-9] disturbances in enamel and dentin formation. The recessive
Pemphigus vulgaris causes intraepithelial bullae formation form may also manifest rudimentary teeth, congenitally
and results in loss of cohesiveness or acantholysis. Due absent teeth, hypoplastic teeth, crowns denuded of enamel.
to this, clumps of epithelial cells are found lying free in
Dental 9'ul-6e � - - - - - -

77.' D' [Burkets 12th ed 501, Table 20-2]


Decreasing order of frequency of finding Anti - Ro (or SSa)
& Anti- La (or SSb) antibodies is Sjogren's syndrome, SLE,
Systemic sclerosis. SSa antibodies may be occasionally found
but SSb antibodies may be totally absent in rheumatoid
arthritis.
_____ r ORAL PATHOLOGY & MEDICINE

11. BACTERIAL INFECTIONS


1. Which of the following is Not characteristic of congenital 12. Hansen's disease is another name of:
Syphilis? a) Leprosy b) Tuberculosis
a) Ghon complex b) Interstitial keratitis c) Diabetes d) Lichen planus
c) Mulberry molars d) Notched incisors (AIPG -97)
(MAN -94,95) 13. Cancerous involvement is seen with
2. A bacterial disease with oral manifestations is a) Chancre b) Mucous patch
a) Herpes b) measles c) Syphilitic glossitis d) Gumma
c) Diphtheria d) leishmaniasis (AP -2K)
(MAN -98) 14. Hard swelling at the angle of mandible with numerous
3. Leutic glossitis is an intra-oral manifestation caused by draining sinuses is most likely:
a) Mycobacterium tuberculosis a) Actinomycosis b) Ludwig's angina
b) Actinomycosis c) Mucormycosis d) Cellulitis
c) Trepenoma pallidium d) Streptococcus (AIPG -94, AIIMS -94)
(MAN -98) 15. Which diseases are infectious but not communicable?
4. Syphilis becomes seropositive in a) Measles b) Mumps
a) Chancre (primary syphilis) c) Scarlet fever d) Tetanus
b) Muco patches (secondary syphilis) (AIIMS -95)
c) Gumma (tertiary syphilis) 16. Oral lesions are rarely seen in:
d) Congenital syphilis a) AIDS b) Tuberculosis
(MAN -2K) c) Syphilis d) Leukemia
5. Sulfur granules in pus are seen in (KAR -97)
a) Candida albicans b) Actinomyces israelii 17. Spasm of masseter muscles occur in:
c) Nocarda braziliances d) Histoplasma capsulatum a) Tetanus b) Trigeminal neuraligia
(COMEDK -04) c) Tuberculous meningitis d) Rabies.
6. Multi nucleated giant cells are least likely to be found in (KAR -98)
which of the following disorders? 18. The oral lesion of syphilis that is highly infective is a:
a) Blastomycosis b) Cat scratch fever a) Gumma b) Koplik spot
c) Sarcoidosis d) Streptococcus pneumonia c) Mucous patch d) Tabes dorsalis
(KAR- 2013) (AIPG -03)
7. A case presenting with a gray coloured pseudomembrane 19. Lesions of oral tuberculosis are associated with all the
whose removal is difficult and painful, can be following except:
a) ANUG a) Central caseation
b) Diphtheritic lesion b) Hyaline degeneration
c) Secondary stage of syphilis c) Giant cells in the center
d) Desquamative gingivitis d) Presence of epithelioid cells
(KAR -98) (AIIMS -96, 99)
8. Oral ulcers that are painless are associated with 20. "Strawberry Tongue" is associated with
a) Secondary herpes b) Primary syphilis a) syphilis b) measles
c) Tuberculosis d) Primary herpes c) scarlet fever d) typhoid
(KAR -98) (AIIMS -2K ;AP -05)
9. Rigidity of facial muscles 'Risus sardonicus' is associated 21. Hutchinson's triad includes:
with: a) Hypoplasia of incisor and molar, VIII nerve motor
a) Tetany b) Tetanus paralysis and peg laterals.
c) Leprosy d) Actinomycosis b) Hypoplasia of incisor and molar, V nerve motor paralysis
(AIPG -96, COMEDK-06) interstitial keratitis
10. Complications of Vincent's angina is: c) Hypoplasia of incisor and molar, VIII nerve deafness and
a) Actinomycosis b) Noma interstitial keratitis
c) Systemic candidiasis d) Cellulites d) None of the above
(KAR -03) (PGI -99, APPG- 2013)
11. The bacteria observed as a causative organism in case of 22. Which of the following oral structures are not effected in
Recurrent apthous ulcers is leprosy
a) Streptococcus sangius b) Streptococcus mutans a) Gingiva b) Tongue
c) Borellia vincentii d) Staphylococci albus c) Hard Palate d) Soft Palate
(AP -2K) (AP -98)

1) A 2) C 3) C 4) B 5) B 6) D 7) B 8) B 9) B 10) B 11) A 12) A 13) C


14) A 15) D 16) B 17) A 18) C 19) B 20) C 21) C 22) A
Dental 9'ul-6e
23. Gumma occurs in: b) life long immunity results
a) Primary stage of Syphilis c) vesicles occur with both diseases
b) Secondary stage of Syphilis d) intranuclear inclusion bodies are present
c) Tertiary stage of Syphilis (APPSC -99)
d) Primary tuberculosis 35. Secondary syphilis occurs after
(KAR -2K) a) 6 weeks b) 9 weeks
24. All the following can give rise to membrane on the c) 13 weeks d) 1 week
pharynx except (KAR -98, AP -02)
a) Staphylococcus aureus b) Corynebacterium 36. Kissing disease is also known as
c) Candida d) Vincent's angina a) Scarlet fever
(KAR -99) b) Acute herpetic gingivostomatitis
25. Actinomycosis is a c) Glandular fever d) Rubella
a) Bacterial infection b) Fungal infection (KCET-2011)
c) Viral infection d) Parasitic infection 37. Which of the following ulcers are characteristically painless
(KAR -03, COMEDK-06) a) Trauma b) Tuberculosis
26. The causative agent for "Oculoglandular syndrome of c) Primary syphilis d) Herpes
Parinoud" is (KAR -98)
a) Arachnia propionica b) Bartonella henselae 38. The best laboratory test to use in the diagnosis of Lupus
c) Bifidobacterium dentium d) Mycobacterium leprae vulgaris in the oral cavity is:
(KCET-2011) a) Bacterial smear b) Blood studies
27. Clinical features of which of the following include c) Biopsy d) Blood chemistry
conjunctivitis, urethritis muco-cutaneous lesions and (AIPG-05)
arthritis? 39. Most common site of tuberculous lesion in the oral cavity is
a) Behcet's syndrome b) Hodgkin's disease a) Buccal mucosa b) Lips
c) Grinspan syndrome d) Ehler-Danlos syndrome c) Tongue d) Palate
(AIPG -95) (COMEDK-06)
28. Oral ulcers which occurs in groups, persist for about 6 40. 'Lock jaw' indicates
weeks and leave scars on healing are: a) Ankylosis b) Spasm of masseter muscle
a) Recurrent aphthous major c) Inflammatory trismus d) Fracture of condyle
b) Recurrent aphthous minor (COMEDK-05)
c) Recurrent herpetiform ulcers 41. Early tongue changes seen in scarlet fever is
d) Acute herpetic gingivostomatitis a) Raspbery tongue b) Strawberry tongue
(AIPG -95) c) Auspitz's sign d) Nikolsky's sign
29. Splitted papule at corner of mouth (AIPG-05)
a) Recurrent herpes labialis 42. 'Canker sore' is otherwise commonly known as
b) Recurrent herpetic stomatitis a) Recurrent apthous ulcer
c) Increased vertical dimension b) Recurrent herpetic gingivitis
d) Secondary syphilis c) Acute necrotizing ulcerative gingivitis
(PGI -2003) d) Recurrent herpes labialis
30. +ve pathergy test is seen in (APPG-2013)
a) Sarcoidosis b) Histoplasmosis 43. The oral lesion called mucous patches is usually multiple,
c) candidiasis d) Behcet's disease grayish white plaque associated with
(AP -02, 2013) a) Pain b) No pain
31. Trismus is due to infection by c) Itching d) Burning sensation
a) Staphylococci b) Streptococci (KCET-07)
c) Diphtheria d) Clostridium Tetani 44. The explosive and widespread form of secondary syphilis
(AP -02) in immuno compromised individual is known as
32. The first consideration in the differential diagnosis a) Condylomata lata b) Mucous patches
of a painless palatal perforation would be c) Lues maligna d) Lupus vulgaris
a) Syphilis b) Histoplasmosis (COMEDK-07)
c) Scrofuloderma d) Actinomycosis 45. Tuberculous Ulcer of Oral cavity is usually
(APPSC -99) a) painless b) painful
33. Noma is c) itching d) Asymptomatic
a) pyogenic granuloma b) fibrous dysplasia (AP-09)
c) Vincent's disease d) cancrum oris 46. Crohn's disease
(KAR -98) a) has oral ulcerations similar to major apthous ulcerations
34. Recurrent aphthae resemble recurrent herpes in that b) is a self limiting lesion
a) symptoms are similar c) is commonly seen among Indian population

23) C 24) A 25) A 26) B 27) A 28) A 29) D 30) D 31) D 32) A 33) D 34) A 35) A
36) C 37) C 38) C 39) C 40) B 41) B 42) A 43) B 44) C 45) B 46) A
r
d) is usually treated with Erythromycin
ORAL PATHOLOGY & MEDICINE

(AP-09)
47. Botryomycosis is a ................. disease.
a) Fungal b) Bacterial
c) Viral d) Parasitic
(KCET-10)
48. Kveim Slitzbach test is useful in the diagnosis of-
a) Tuberculosis b) Sarcoidosis
c) Leprosy d) Cat scratch disease
(COMEDK-09)
49. Oral lesions of secondary syphilis includes all EXCEPT -
a) Snail track ulcers b) Mucous patches
c) Chancre of tongue d) Hutchinson's wart
(KCET-09)
50. A patient has asymptomatic painless burrowing ulcer in
the palate. He might be suffering from
a) Syphilis b) Tuberculosis
c) Histoplasmosis d) Actinomycosis
(AP-10)
51. Primary antibody deficiencies are characterized by
a) Recurrent allergic reactions
b) Recurrent bacterial infections
c) Implicit allergic reactions
d) Implicit bacterial infections
(BHU-2012)
52. Apthous like ulcers are seen in:
a) Bechets syndrome b) Sweet syndrome
c) PFAPA (periodic fever acute pharyngitis apthous stomatitis)
d) All of the above
(PGI-2011)
53. Which of the following is a painful ulcer?
a) syphilitic ulcer b) trophic ulcer
c) oral chancre d) all the above
(KAR -01)
54. True about syphilis is
a) Oral lesions are seen at any stage
b) Clinical manifestations occur after few months of
exposure
c) VDRL test is positive in early days of primary syphilis
d) Penicillins are ineffective
(PGI DEC-2011)

47) B 48) B 49) C 50) A 51) B 52) D 53) C 54) A


Dental 9'ul-6e � - - - - - -

11. BACTERIAL INFECTIONS - ANSWERS


1. 'A' [Shafer 7th ed 330/ 6th ed 324) Blastomycosis is a mycotic infection caused by Blastomyces
The pathognomic of congenital syphilis is Hutchinson's dermatitidis. It may occur in a cutaneous form or a systemic
triad, which include - form involving bones, lungs, liver, subcutaneous tissues.
a) Mulberry molars and notched incisors. The oral manifestations are ulcers or tiny abcesses similar
to actinomycosis. The microscopic features are similar to
b) Interstetial keratitis
chronic granulomatous infections showing occasional giant
c) 8th Nerve deafness. cells and macrophages.

Congenital or prenatal syphilis is characterized by: frontal Cat-scratch disease is caused by Bartonella henselae, a
bossing, short maxilla, high palatal vault, saddle nose, gram-negative bacillus. The disease arises after a traumatic
higoumenaki's sign, rhagades and saber skin apart from break in the skin due to scratch or bite of a household cat
hutchinson's traid. and is characterised by papule, pustule, and vesicle at the
site of injury followed by regional lymphadenitis. Epitheloid
2. 'C' [Shafer 7th ed 319/ 6th ed 312) cells and multinucleated giant cells are occasionally
Oral manifestations of diphtheria includes "diptheric seen.
membrane" which usually begins on tonsils. The patients
will have peculiar nasal twang and may exhibit nasal 7. 'B' [Shafer 7th ed 319/ 6th ed 312)
regurgitation of liquids. There is temporary paralysis of soft Pseudomembrane formation is seen in:
palate during 3 rd to 5t11 weeks of disease. • ANUG - Pseudomembrane can be easily removable
3. 'C' [Shafer 7th ed 330/ 6th ed 324) • Syphilis - Pseudomembrane is undetachable
Leutic glossitis or syphilitic glossitis is atrophic or interstitial • Diphtheria - Difficult to remove and painful
glossitis due to tongue involvement by treponema. The • Vincent's angina
importance of leutic glossitis is the chance of undergoing • Candidiasis - Pseudomembrane can be removable
malignant transformation. This type of glossitis is most
characteristic and is due to endarteritis obliterans. It is
8. 'B' [Shafer 7th ed 329/ 6th ed 323)
seen in tertiary syphitis.
9. 'B' [Shafer 7th ed 328/ 6th ed 322)
(Note: Osteoradionecrosis is also due to endarteritis
obliterans )
10. 'B' [Shafer 7th ed 399/ 6th ed 327)
4. 'B' [Shafer 7th ed 329/ 6th ed 323)
11. 'A' [Shafer 6th ed 662)
5. 'B' [Shafer 7th ed 325/ 6th ed 319)
12. 'A' [Shafer 7th ed 323/ 6th ed 317)
Actinomycosis commonly involves cervicofacial region. There
The oral lesions of tuberculosis are called "lepromas",
is development of multiple abscess which drain through skin
which develop on tongue, lip and hard palate. They show a
and the pus contains typical sulfur granules. Histologically,
tendency to breakdown and ulcerate.
the colonies shows peripheral radiating filaments because of
which Actinomycosis is also known as "Ray fungus".
13. 'C' [Shafer 7th ed 330/ 6th ed 324]

Which of the following is a classical feature of 14. 'A' [Shafer 7th ed 325/ 6th ed 319)
actinomycosis? (KAR- 2013)
a) Extensive induration of lower jaw 15. 'D' [Shafer 7th ed 327/ 6th ed 321]
b) Multiple subcutaneous nodules
c) Discharging sinuses 16. 'B' [Shafer 7th ed 320/ 6th ed 314)
d) Sulphur granules in the discharge
17. 'A' [Shafer 7th ed 328/ 6th ed 322)
6. 'D' [Shafer 7th ed 672, 368, 334/ 6th ed 669,359,327)
Sarcoidosis is a multisystem granulomatous disease of 18. 'C' [Shafer 7th ed 329/ 6th ed 323]
unknown origin characterised by the formation of non­
caseating epitheloid granulomas. The chief histologic 19. 'B' [Shafer 7th ed 322/ 6th ed 316)
features of Sarcoidosi s are presence of nests of epitheloid The tuberculous lesions of oral cavity usually appear
cells with multinucleated giant cells in non caseating secondary to pulmonary TB. They consists of:
granulomas. The granuloma transforms into a solid, • Irregular, deep, painful ulcer on tongue, palate, lips,
amorphous, eosinophilic hyaline mass as it ages. buccal mucosa.
• Tuberculous osteomyelitis

r ORAL PATHOLOGY & MEDICINE
Primary oral tuberculosis: Gingiva become diffuse, 30. 'D' [Burket's 11th ed 61/ 10th ed 66]
hyperemic and nodular Positive pathergy is an inflammatory reaction within 24
hours of needle puncture, scratch or saline injection.
Tuberculous lesions in the mouth have the same characteristic
histopathological appearance of caseating granulomas 31. 'D' [Shafer 7th ed 327-28/ 6th ed 322]
(foci of caseation necrosis, surrounded by epitheloid cells, The exotoxin of clostridium (tetanospasmin) is a neurotoxin
lymphocytes, multinucleated giant cells.) that causes spinal inhibition and this results in rigidity of
facial muscles producing the typical "risus sardonicus",
20. 'C' [Shafer 7th ed 318/ 6th ed 311]
Scarlet fever in children is caused by -hemolytic type of "Cephalic tetanus" occurs in association with seventh
streptococci. Appearance of bright scarlet skin rash is cranial nerve palsy.
characteristic. Small papules of normal colour erupt through
these rashes giving a characteristic sand paper feel to the 32. 'K [Shafer 7th ed 330/ 6th ed 324]
skin. This rash that is particularly prominent in the areas of
skin folds is called pasta lines. 33. 'D' [Shafer 7th ed 333/ 6th ed 326]
Noma is rapidly spreading gangrene of oro facial region, and
The oral manifestations are known as Stomatitis scarlatina. is caused by Vincent's organisms.
The tongue exhibits a white coating and the fungiform
papilla are hyperemic, projecting above the surface. This is 34. 'A' [Shafer 7th ed 667-69/ 6th ed 665]
known as Strawberry tongue or white Strawberry tongue Recurrent herpetiform ulcers are a type of aphthous ulcers,
(MCET-10). Later, the coating is lost and the tongue becomes characterised by crops of multiple, small, shallow ulcers often
red and smooth except for swollen hyperemic papilla. This is upto 100 in number. Cooke pointed the clinical similarities
known as Raspberry tongue or red Strawberry tongue. between both the diseases though histopathologically they
are different.
21. 'C' [Shafer 7th ed 330/ 6th ed 324]
35. 'A' [Shafer 7th ed 329/ 6th ed 323]
22. 'A' [Shafer 7th ed 3 24/ 6th ed 318]
Lepromas are common on tongue, lips and hard palate. 36. 'C' [Shafer 7th ed 781/ 6th ed 774]
Infectious mononucleosis / 1Glandular fever / Kissing
23. 'C' [Shafer 7th ed 330/ 6th ed 324] disease is caused by Epstein Barr virus, and is transmitted
through oropharyngeal secretions through deep kissing. The
24. 'A' [Shafer 7th ed 319 for option B, oral excretion of EBV may continue for as long 18 months
3 72 for option C, following the onset of disease.
397 for option DJ
37. 'C' [Shafer 7th ed 329/ 6th ed 323]
25. 'A' [Shafer 7th ed 3 24/ 6th ed 318] The ulcers may become painful because of secondary
infection.
26. 'B' [Shafer 7th ed 334/ 6th ed 327, 328]
0culoglandular syndrome of Parinoud is unusual 38. 'C' [Shafer 7th ed 322]
manifestation of cat scratch disease, which is caused by Lupus vulgaris Primary tuberculosis of skin (MCET-10)
Bartonella henselae, a gram negative bacillus. It consists
Tuberculous infection of submaxillary
of localised granuloma of the eye with preauricular Scrofula
and cervical lymph nodes (AIIMS-og)
lymphadenopathy. The lymph nodes gradually become soft
and fluctuant, owing to necrosis and suppuration. If the • Blood borne spread of primary T. B.
Miliary T.B. results in miliary T.B.
preauricular, submaxillary, or cervical chain of nodes is (Extrapulmonary
involved, the dentist may be consulted to rule out dental TB) • It is characterized by wide spread
disease as etiologic factor. involvement of many organs in body.
Tuberculous infection resulting in
27. 'A' [Shafer 7th ed 670/ 6th ed 667] Potts disease
spinal curvature (Kyphosis)
Behcet's syndrome should have at least two of classic triad
of the disease i.e., recurrent oral ulcers, recurrent genital 39. 'C' [Shafer 7th ed 321/ 6th ed 314]
ulcers and ocular innammation. (KCET-10)
Most common site of tuberculous lesion Tongue
28. 'A' [Shafer 7th ed 667/ 6th ed 665] Most common site of hemangioma tumor Lip
Apthous major is also known as periadenitis necrotica Most common site of Basal cell carcinoma Upper lip
recurrens or Sutton's disease or Mikulicis scarring apthae.
Most common site for mucoceles Lower lip
29. 'D' [Burket's 9th ed 646] Most common site of peripheral giant cell
Gingiva
Mucous patches, split papules, condyloma latum are oral granuloma
manifestations of secondary syphilis.
Dental 9'ul-6e

Common site of melanoma on the Various viral diseases with oral manifestations are:
Matar region
orofacial skin • Hand, foot, mouth disease
• Most common site for ectopic salivary • Molluscum contagiosum
gland tumor • Condyloma acuminatum
Palate
• Site of occurrence of necrotising • Chikungunya
sialometaplasia
48. 'B' [Shafer 7th ed 672/ 6th ed 669)
40. 'B' [Shafer 7th ed 328/ 6th ed 322) Lip lesions of sarcoidosis resemble herpetic lesions or 'fever
blisters'. Kviem-Slitzbach intracutaneous test is an important
41. 'B' [Shafer 7th ed 318/ 6th ed 311) aid in the early and accurate diagnosis of the disease.

42. 'A' [Shafer 7th ed 665/ 6th ed 662) 49. 'C' [Shafer 7th ed 329/ 6th ed 323]
Chancre lesions are characteristic of primary stage of
43. 'B' [Shafer 7th ed 329/ 6th ed 323) syphilis.

44. 'C' [Shafer 7th ed 330/ 6th ed 323] 50. 'A' [Shafer 7th ed 328-30/ 6th ed 323)
The asymptomatic, painless, burrowing ulcer in the palate is
45. 'B' [Shafer 7th ed 321/ 6th ed 315) chancre. Chancre is manifestation of primary syphilis which
Tuberculosis can be primary (usually seen in children) is usually painless unless secondarily infected.
or secondary or military. Primary tuberculosis of skin is
called Lupus vulgaris. Primary oral tuberculosis is usually 51. 'B' [Burket's 11th ed 439/ 10th ed 481)
associated with regional lymphadenopathy, mostly involving Selective Immunoglobulin deficiencies are a group of
submandibular and cervical lymphnodes. Scrofula is other disorders characterized by an abnormality of the B cell
name for tuberculous Lymphadenitis. antibody system. The primary adult deficiencies rarely
become apparent until the third decade of life. The most
Primary oral tuberculosis usually involve gingiva and present common symptoms include recurrent gram- positive
as diffuse, nodular or papillary proliferation of gingival bacterial infections, esp. of the respiratory tract, followed
tissues. by the joints, GI tract and skin.

Lesions of secondary oral tuberculosis may occur at any site


Type of recurrent
on oral mucous membrane, but the tongue is most commonly Deficiency
infection
affected, followed by palate. The usual presentation is in
form of superficial or deep, irregular painful ulcer. Innate immunity (phagocyte
Bacterial & fungal
defects)
46. 'A' [Burket's 11th ed 355) Abaptive immunity
Bacteria, viruses, fungi
Crohn's disease is an inflammatory disease of the small (combined B & T-cell defects)
or large intestine. Recurrent apthous ulcers are the
B-Cell deficiencies
most common oral manifestations of Crohn's disease.
Certainly, pyostomatitis vegetans, Cobble Stone mucosa[ Decrease in number Bacterial
architecture, and mi nor salivary grand duct pathology Bacterial infection,
represent granulomatous changes that constitute the hall Immunoglobulin deficiency may develop
mark of Crohn's diseas,e. Less often, Crohn's disease patients autoimmune disease
develop diffuse swelling of the lips and face, inflammatory
hyperplasias of the oral mucosa with a Cobble Stone pattern T-Cell deficiencies
and indurated polypoid tag-Like lesions in the vestibule Decrease in Number Viruses, fungi
and retro-molar pad area.
Qualitative T-cell defects Fungi, viruses
47. 'B' [Shafer 7th ed 326/ 6th ed 320, 328)
52. 'D' [Check Explanation Below]
Various Bacterial diseases of oral cavity are:
Sweet's syndrome or acute febrile neutrophilic dermatosis
• Actinomycosis is a skin disease characterized .by sudden onset of fever,
• Botryomycosis leukocytosis, and tender erythematous well demarcated
• Tularemia (Rabbit fever) papules and plaques which show dense infiltrates by neutrophil
granulocytes on histologic examination and recurrent oral
• Melioidosis
ulceration. PFAPA (Periodic fever, accompanied by aphthous
• Rhinoscleroma stomatitis, pharyngitis and cervical adenitis syndrome) is
• Cat-Scratch disease a condition of periodic fever characterized by abrupt fever,
malaise, aphthous stomatitis, tonsillitis, pharyngitis and
cervical lymphadenopathy. It usually begins before the age of
5 years and gets resolved before the age of 10 years.
r
53. 'C' [Shafer 7th ed 3 29/ 6th ed 3 23]
ORAL PATHOLOGY & MEDICINE

Chancre at site of inoculation


Oral chancre is usually painless but may bewme painful because of
Primary stage i.e., commonly occurs on penis
secondary infection.
and on the vulva
• Mucous Patches on tongue, • Painless. The serological reaction is always positive. The mucous
Secondary stage or patches on the palate and
gingiva and are highly
metastatic stage
infectious. • tonsil are described as snail- track ulcers. (KCET-11)
(develops 6 wks
• Split Papules, Condyloma • In immunocompr-omised patients, syphilis can present as an
after primary lesion)
latum are other features. explosive and widespread form known as lues maligna.
Tertiary stage (CNS Gumma; commonly involves • More common than primary or secondary.
+ CVS are involved) tongue, palate. • Causes perforation of palate.

Transmitted to • Hutchinsons triad.


Congenital
offspring by • Pseudorrhagades
syphilis
infected mother • underdeveloped premaxilla.

54. 'A' [Shafer 7th ed 3 28]


Acquired syphilis has three distinctive stages, namely primary, secondary and tertiary forms. The primary oral lesions of primary,
secondary and tertiary syphilis are chancre, mucous patches and gumma respectively. Congenital syphilis is characterised by
Hutchinson's incisors and Fournier's molars. The primary stage may manifest approximately after 3- 90 days after exposure. Not
every patient with primary syphilis exhibits a positive serologic reaction despite the presence of spirochaete. Serologic reactions
are however positive in secondary syphilis. Primary, secondary and tertiary syphilis are treated with a single dose of 2.4 million
units of benzathine penicillin G.
Dental 9'ul-6e � - - - - - -

12. VIRAL INFECTIONS


1. The feature that distinguishes herpes zoster from other 11. The reason why most patients su ffering from recurrent
vesiculo bullous eruptions is herpes labialis rarely give a history of having acute form
a) Unilateral occurrence b) Severe burning pain of the herpetic gingivastomatitis is because:
c) Prominent crusting vesicles a) Etiological agents differ
d) Sub epidermal bullous formation b) The acute form occursonlyin severelyim munocompromised
(AIPG -97,AIIMS -2K) individuals
2. A 3 year old child has a fever of 102 degrees F; and c) The primary infection was subclinical
following upper respiratory tract infection discrete d) The patient has received antibodies during intrauterine
vesicles and ulcers on the soft plate and pharynx are life and the antibodies have persisted
noted. The most probable diagnosis is (AP -97)
a) Herpangina b) Scarlet fever 12. In herpes primary lesion is
c) Rubella d) Herpetic gingivostomatitis a) ulcer b) papule
(AIIMS -2K) c) vesicle d) Bullae
3. Intra nuclear inclusions detected during the course of (AP -04)
herpes simplex virus infection are called 13. Which of the following medications shortens the recovery
a) Bacteriophages b) Lipschutz bodies period of primary Herpetic gingivostomatitis?
c) Negri bodies d) Donavan bodies a) Acyclovir b) Ziduvidine
(AIIMS -2K) c) Kenalog in orabase d) All of the above
4. Coxsackie virus causes (KAR -01, 04)
a) infectious mononucleosis b) lymphoma 14. Ramsay Hunt syndrome the cranial nerve involved is:
c) herpangina d) herpes a) Trigeminal. b) Facial
(AIIMS -99,AP -97) c) Glossopharyngeal. d) Occulomotor nerve.
5. Koplik spots (KAR -98)
a) First manifestation of measles 15. All about herpangina are false except:
b) Rarely seen in measles a) May lead to high grade fever
c) Are seen 2-3 days after cutaneous rashes b) May cause Dysphagia
d) Is first manifestation but seldom seen c) May lead to vesicles formation in Anterior part of mouth
(PGI -98,2K) around 20-30 in number
6. Recurrent ulcers occurring on gingiva and palate are d) Anterior Portion of mouth have only minor vesicles.
most probably: (AIPG -02)
a) Aphthous ulcers b) Herpes simplex 16. Herpetiformis vesicles, which rupture and leave areas of
c) Koplick spots d) Lesions of Behcet's syndrome superficial intraoral ulcers, are caused by:
(AIPG -95) a) Varicella Zoster virus
7. Which of the following is associated with HIV infection b) Herpes zoster virus
a) Hairy leukoplakia b) Erythroplakia c) Coxsackie virus
c) Oral Lichen planus d) Bullous pemphigoid d) None of the above
(AIIMS -2004) (AP- 2013)
8. Primary herpetic lesions involving the gingiva are most 17. Prodromal symptoms precede 1 to 2 days before the
likely to occur during ages: onset of disease in
a) 1-5 years a) viral fever b) erythema multiforme
b) 6-12 years c) pemphigus d) pemphigoid
c) 13-16 years {AP -99)
d) They are likely to occur equally at any age 18. The commonest opportunistic infection in AIDS cases in
(AIPG -97) India
9. Inflammation of the dorsal root ganglion and vesicular a) Pneumocystics carinii. b) Cryptococcal meningitis.
eruption of the skin and mucous membrane in area supplied c) Toxaplasmosis. d) Tuberculosis.
by a sensory nerve that is affected is characteristic of: (KAR -98)
a) Herpes zoster b) Herpes simplex 19. Which of the following is a difference between
c) Uveoparotid fever d) Aphthous stomatitis herpangina and primary herpetic stomatitis:
(AIPG -91) a) It is preceded by prodromal symptoms
10. Mumps is caused by: b) It is unilateral in nature
a) Orthomyxo virus b) Paramyxo virus c) Ulcers relationship seen on the anterior faucial pillars
c) Rheno virus d) EB virus d) Viral etiology
(AIPG -96) (AIPG -01)

1) A 2) A 3) B 4) C 5) D 6) B 7) A 8) A 9) A 10) B 11) C 12) C 13) A


14) B 15) B 16) D 17) A 18) D 19) C
20.
r ORAL PATHOLOGY & MEDICINE
Cytological smear showing multinucleated giant cells, d) Routine cytology
synctium and ballooning degeneration of the nucleus is (COMEDK-05)
a characteristic of 31. Herpangina is caused by
a) Herpes simplex virus infection a) Herpes simplex virus b) Coxsackie virus
b) Erythema multiforme c) Measles virus d) Varicella zoster virus
c) Recurrent apthous stomatitis (COMEDK-05)
d) Coxsackie virus infection 32. Rubeola refers to
(KAR -99) a) German measles b) Measles
21. Reactivation of varicella virus in a posterior root ganglion c) Small pox d) Chicken pox
results in: (COMEDK-06)
a) Chicken pox. b) Herpes zoster. 33. Which of the following is NOT TRUE about primary HSV
c) Herpes simplex. d) Poliomyelitis. infections?
(KAR -98) a) Primarily affects the anterior portion of the mouth
22. Shingles occurs b) Causes acute gingivits
a) primary infection b) is unilateral c) Occurs as epidemic d) Shows prodromal symptoms
c) occurs on movable tissues (COMEDK-05)
d) is bilateral 34. The lesion which is erythematous, recurrent and
(AP -99) distributed along with the sensory nerve trunk is:
23. Site specificity is seen in: a) Herpes zoster b) Erythema multiformae
a) Syphilis b) Recurrent herpes labialis c) Herpetic gingivo stomatitis
c) Carcinoma d) Traumatic aphthous ulcer d) Recurrent aphthous
(AIIMS -90) (AIPG-07)
24. Steroids are indicated in all of the following conditions except 35. Herpes simplex is seen in
a) oedema following extractions a) < 10 yrs. of age b) 12-15 yrs. of age
b) oral ulcers in AIDS patients c) 25-30 yrs. of age d) 55-60 yrs. of age
c) TMJ arthritis d) Angioneurotic oedema (AIPG-05)
(KAR -99) 36. The virus which causes Herpes Zoster is same as that
25. Recurrent herpes occurs due to: which causes
a) Virus in oral mucosa a) Herpes simplex b) Herpangina
b) Latent virus in skin supplying the area c) Chicken pox d) Measles
c) Latent virus in nerve ganglia (COMEDK-10)
d) None of the above 3 7. Which virus given below is not a teratogenic virus?
(AIPG -93) a) Rubella b) Cytomegalovirus
26. Acute herpetic ging·ivostomatitis c) Herpes simplex d) Measles
a) occur in children and young adults (COMEDK-10)
b) occur only in yoU1ng children 38. The most common reported oral malignancy in HIV infection is
c) occur in only adUllts d) all of the above a) Squamous cell carcinoma
(PGI -2003) b) Verrucous carcinoma
27. Drug used in case of Herpetic lesions c) Multiple myeloma d) Kaposi sarcoma
a) Acyclovir b) Penicillin (COMEDK-10)
c) Tetracycline d) Ciprofloxacin 39. In HIV, which cells are affected
(AP -02) a) CD4 b) CD8
28. A 40-year old lady presents with unilateral dermatonic c) Monocytes d) Lymphocytes
distribution of vesicular eruptions. associated with (AIPG-10)
severe pain. The most likely diagnosis is 40. A 60 year old male presents with a 2 day history of a
a) Herpes zoster b) Chicken pox severe left ear ache with a burning sensation in the
c) Recurrent herpes simplex infection ear and loss of taste. There is left sided weakness of
d) Infectious mononucleosis both upper and lower facial muscles. Facial sensation is
(UPSC -01) normal. A few vesicles are seen in the pharynx. What is
29. Coxsackie virus is implicated in the most likely diagnosis?
a) Herpes zoster b) Measles a) Bell's palsy b) Acoustic neuroma
c) Small pox d) Hand, Mouth & foot disease c) Diphtheria d) Ramsay Hunt syndrome
(KAR -98) (KCET-2011)
30. Which is best for diagnosis of primary herpes simplex 41. Which of the following is not associated with gingival
infection? lesions?
a) Smear stained with Giemsa stain a) Herpes b) Recurrent apthae
b) Smear stained with Wrist's stain c) Pemphigus d) Pyogenic granuloma
c) Flurorescent staining of cytology smear (AIIMS-2011, 12)

20) A 21) B 22) B 23) B 24) B 25) C 26) A 27) A 28) A 29) D 30) C 31) B 32) B
33) C 34) A 35) A 36) C 37) D 38) D 39) A 40) D 41) B
Dental 9'ul-6e � - - - - - -

42. Papulovesicular oral lesions are seen in


a) Measles b) Herpangina
c) Rubella d) Hand foot and mouth disease
(KCET-2012)
43. Which of the following does not cause oral cancer in
children?
a) Herpes simplex b) Epstein bar
c) Cytomegalovirus d) Varicella zoster
(NEET-2013)
44. Which of the following is false regarding measles?
a) Koplik's spots b) Maculo papular skin rash
c) Fever and malaise d) Nikolsky's sign
(KAR- 2013)
45. Which of the following would be seen in late HIV cases?
a) Kaposi's Sarcoma b) Oral hairy leukoplakia
c) Gingivitis/periodontitis d) All of the above
(KAR- 13)
46. A 16 year old female patient reported with chief
complaint of small ulcers on the cheek and tongue since
once week. Mild rise in temperature was noted on general
examination. On oral examination, multiple ulcers were
noted on the buccal and labial mucosa. She also reported
that she is having her board exams in a week.

A. The likely diagnosis is


a) Bullous Pemphigoid b) Minor aphthous ulcers
c) Traumatic ulcer d) Tuberculoid ulcer
(COMEDK -14)
B. The probable etiology is
a) Emotional disturbances
b) Vitamin deficiency
c) Hormonal disturbances d) Any of the above
(COMEDK -14)
C. The management of such a case would be
a) Antibiotics and analgesics
b) Symptomatic treatment, topical steroids and relaxation
c) Anti-fungal medications
d) Systemic steroids
(COMEDK -14)
47. True about HPV 6, 11, 16 and 18?
a) Associated with verruca vulgaris
b) A vaccine targeted against HPV types 6, 11, 16 and
18 has been intmduced recently for the prevention of
cervical cancer and genital wart
c) Associated with Squamous papilloma
d) Associated with Condylomalata

48. Forcheimer sign is seen in


a) Infectious monorncleosis
b) Scarlet fever
c) Rubella d) Rubeola
(PG! JUNE-14)

42} B 43} D 44} D 45} D 46A} B 46B} D 46C} B 47) B 48} C


ORAL PATHOLOGY & MEDICINE

12. VIRAL INFECTIONS - ANSWERS


1. 'A' [Shafer 7th ed 3 51/ 6th ed 345] cavity and oropharynx and pain of external ear are important
In herpes zoster the most sites involved are the areas features (COMEDK-08).
innervated by spinal cord segments 03 to LZ and the
trigeminal nerve, particularly its ophthalmic branch. 15. 'B' [Shafer 7th ed 345/ Burket's 9th ed 17]
Herpangina or apthous pharyngitis is caused by Coxsackie
2. 'A' [Shafer 7th ed 345/ 6th ed 339] virus. The clinical features are mild and short duration (1
Herpangina or apthous pharyngitis is caused by coxsackie week). Vesicles are more common on pharynx and posterior
virus, commonly affecting the children. Small ulcers on oral mucosa and are less frequent on tongue, buccal mucosa
anterior faucial pillars, palate and pharynx that are preceded and palate. The patients will have difficult in swallowing.
by vesicle formation are important features of Herpangina.
16. 'D' [Shafer 7th ed 834/ 6th ed 826, 339, 344; Clin
3. 'B' [Shafer 7th ed 344/ 6th ed 337] Dermatol. 2011 Sep-Oct; 29(5):509-10. Shape and
In herpes simplex, there is ballooning degeneration ot cells configuration of skin Lesions: grouped herpetiform]
contain intranuclear inclusions called Lipschutz bodies. These Dermatitis herpetiformis is a rare, benign, chronic, recurrent,
are eosinophilic, ovoid structures within the nucleus and immune-mediated blistering dermatologic disease with
displace the nucleolus and chromatin material peripherally. associated gluten - sensitive enteropathy. Oral lesions
include herpetiformis vesicles and bullae which rupture
4. 'C' [Shafer 7th ed 345/ 6th ed 339] rapidly to leave areas of superficial ulceration at any intra
oral site.
Coxsackie group A virus Herpangina
17. 'K [Burket's 11th ed 43]
Coxsackie virus AlO Acute lymphonodular pharyngitis
Coxsackie virus A16 Hand, Foot & Mouth disease 18. 'D' [Check Explanation Below]
Pneumocystis carini is the mos.t opportunistic infection in
5. 'D' [Shafer 6th ed 341] world, but in India T. B is the most common opportunistic
In measles (Rubeola) the oral lesions are prodromal occurs infection.
2-3 days before the cutaneous rash. These lesions are
pathognomic of the disease and are known as "Koplik's 19. 'C' [Shafer 6th ed 339/ Burket's 11th ed 51]
spots". These spots occur in 97% of cases, but seldom
noticed unless the parents or dentist watches carefully and 20. 'A' [Burket's 11th ed 45-46/ Shafer 6th ed 338]
disappear after the onset of rash.
21. 'B' [Shafer 7th ed 351/ 6th ed 345]
6. 'B' [Shafer 7th ed 344/ 6th ed 338]
Apthous ulcers are recurrent and appears mostly on 22. 'B' [Shafer 7th ed 351/ 6th ed 345]
lining mucosa whereas herpes lesions appears mostly on
masticatory mucosa i.e., gingiva, palate etc. 23. 'B' [Shafer 7th ed 343/ 6th ed 337-38]

7. 'A' [Burket's 10th ed 93/ Shafer 7th ed 360] 24. 'B' [Shafer 7th ed 361]

8. 'A' [Shafer 5th ed 468] 25. 'C' [Shafer 7th ed 343/ 6th ed 339]

9. 'A' [Burket's 11th ed 47/ Shafer 6th ed 345] 26. 'A' [Shafer 7th ed 342/ 6th ed 336]

10. 'B' [Burket's 11th ed 205/ 10th ed 249] 27. 'K [Shafer 7th ed 344/ 6th ed 339]

11. 'C' [Shafer 7th ed 341] 28. 'A' [Shafer 7th ed 350/ 6th ed 345]

12. 'C' [Shafer 7th ed 342] 29. 'D' [Shafer 7th ed 346/ 6th ed 340]
Herpetic whitlow occurs in fingers due to inoculation.
Herpes gladiatorum occurs in wrestlers. 30. 'C' [Burket's 11th ed 45]
Cytologic smears from base of herpes lesions stained with
13. 'A' [Burket's 11th ed 46/ Shafer 6th ed 339] Wright, Giemsa or Papanicolaou stain demonstrate ballooning
degeneration, syncytium, multinucleated giant cells or
14. 'B' [Shafer 7th ed 3 51/ 6th ed 346] intranuclear inclusions. This does not distinguish HSV & VZV.
JAMES RAMSAY HUNT's syndrome is a special form of Cytologic smear with direct fluorescent antigen detection
zoster infection of geniculate ganglion, external ear and test using monoclonal antibody against HSV conjugated to
oral mucosa. Facial paralysis, vesicular eruptions in oral fluorscein. This is more specific and accurate for HSV.
Dental 9'ul-6e
31. 'B' [Shafer 7th ed 345/ 6th ed 339) 38. 'D' [Burket's 11th ed 176)
HIV infection that leads to immunosuppression increases the
32. 'B' [Shafer 7th ed 347/ 6th ed 341) risk of development of neoplastic disease. Kaposi sarcoma is
Rubeola refers to measles while rubella refers to German the most common neoplastic disease of AIDS (COMEDK-14).
measles. Non-Hodgkin's lymphoma is the most rapidly increasing
malignant disease of AIDS.
33. 'C' [Shafer 7th ed 343/ 6th ed 333]
39. 'A' [Shafer 7th ed 357/ 6th ed 351)
34. 'A' [Shafer 7th ed 350/ 6th ed 344-45] The two major targets of HIV infection are the immune system
and central nervous system. A characteristic feature of AIDS in
35. 'A' [Burket's 11th ed 43/ Shafer 7th ed 343] profound immunosuppression of cell mediated immunity. After
Newborn upto 6 months of age are protected from HSV gaining entry into the host, the first step in virus infection
infections by placentally transferred antibodies. After 6 involves binding of the virion to the surface of a target cell.
months of age, the incidence of primary HSV1 infection This is mediated by binding of the envelope gp120 to the CD4
increases and reaches a peak between 2 to 3 years of age. surface protein found on most helper T cells.

36. 'C' [Shafer 7th ed 350-51/ 6th ed 345) 40. 'D' [Shafer 7th ed 351/ 6th ed 346]
Both Chicken pox (varicella) and Herpes Zoster are caused Herpes zoster infection of the geniculate ganglion, with
by Varicella Zoster. involvement of the external ear and oral mucosa is called
James Ramsay - Hunt syndrome. The clinical manifestations
Option 'A' Herpes simplex is caused by Herpes simplex include facial paralysis, pain of the external auditory meatus
type-I virus [BHU-2012]. Option 'B' Herpangina is caused and pinna of the ear. Vesicular eruptions also occur in the
by Coxsackie group A virus. Option 'D' measles (Rubeola) is oral mucosa and oropharynx, with hoarseness, tinnitus,
caused by paramyxo virus. vertigo and other disturbances.

37. 'D' [Shafer 7th ed 342-355/ 6th ed 341] 41. 'B' [Shafer's 7th ed 667/ 6th ed 665, 338)
The clinical features of Rubeola/Measles include fever, Aphthous ulcers are commonly seen in all locations of labile
cough, conjunctivitis, photophobia, lacrimation and mucosa, not bound to the periosteum like the labial, buccal
eruptive lesions of tlhe skin and oral mucosa. Intra oral mucosa, soft palate, pharynx, tongue etc. Pemphigus affects
lesions in form of Koplik's spots occur in most of the the gingivae with chronic desquamative gingivitis, pyogenic
patients. Histopathologic features show pathognomanic granuloma occurs usually on the labial gingivae and intraoral
multinucleated giant cells called Warthin-Finkeldey giant herpes characteristically involves the keratinized mucosa
cells. (COMEDK-11) (hard palate and the gingivae)

Complications of option 'K rubella occurs in women during 42. 'B' [Burket's 10th ed 54]
the first trimester of pregnancy. In such cases, the offspring Herpangina is caused by Coxsackie virus and causes a
has high incidence of congenital defects such as blindness, prodromal phase of fever, chills, anorexia, sore throat and
deafness, and cardiovascular abnormalities. dysphagia. Oral lesions start as punctuate macules, quickly
evolve into papules and vesicles on the posterior pharynx,
Transplacental infection by herpes simplex virus has been tonsils, faucial pillars and soft palate.
reported. The infants manifest wide variety of signs &
symptoms by 4th - 7th day of life and die on 9th - 12th Hand, foot and mouth disease is characterized by low grade
day of life. Surviving infants have residual neurologic fever, oral vesicles and ulcers, non pruritic macules, papules
involvement. and vesicles on the extensor surfaces of the hands and
feet. Measles gives rise to Koplik's spots, whereas Rubella
Other teratogenic viruses are: causes retarded tooth development, eruption and enamel
• Cytomegalo virus hypoplasia, if compatible with life.
• Herpes virus
• Parvovirus B-19 43. 'D' [Shafer 7th ed 184, 341, 351, 355/ 6th ed
• Rubella (German measles) 349,344,180]
• Varicella Zoster HSV has been possibly associated with hit and run mechanism
• Venezuelan equine· encephalitis (VEE) in oral cancer causation. Epstein barr virus causes African
jaw lymphoma, CMV possibly is associated with Kaposi's
Other teratogenic infections: sarcoma. Hence Herpes zoster is the best option.
• Syphilis - Bacterial infection
• Toxoplasmosis - Protozoan infection 44. 'D' [Shafer 7th ed 347/ 6th ed 341]
• Listeriosis - Bacterial infection Measles is an acute, contagious, dermatotropic viral
infection, characterised by fever, malaise, Koplik's spots
Rubella is more life threatening if it occurs in first trimester (Prodromal sign), cough, maculopapular rashes. Koplik's
of pregnancy. spots are prodromal and disappear after the onset of the
rash. They usually occur on the buccal mucosa as small,
r ORAL PATHOLOGY & MEDICINE
irregular bluish white flecks surrounded by a bright red
margin. Nikolsky's sign is seen in pemphigus, pemphigoid,
epidermolysis bullosa etc. The histopathological features
show pathognomonic multinucleated giant cells known as
Warthin - Finkeldey giant cells.

45. 'D' [Shafer 6th ed 3 52, www.aidsetc.org]


All the above lesions are classified as lesions strongly
associated with HIV infection.

46A. 'B' [Shafer 7th ed 665)


RAS is a common disease characterised by painful, multiple
ulcerations of the oral mucosa. The proposed etiologic
features include bacterial infection, immunological
abnormalities, iron, B12, folate deficiency and genetic
factors. The precipitating factors include trauma. endocrine
conditions, stress & psychic factors, allergic factors or some
systemic diseases.

46B. 'D' [Shafer 7th ed 666)

46C. 'B' [Shafer 7th ed 669)


The various formulations that have been tried in RAS
include tetracycline mouth wash, 1.5% cortisone acetate,
antihistaminics, levamisole, multivitamin, stress reduction
etc.

4 7. 'B' [Shafer 7th ed 82, Burket's 11th ed 15 7)


H PV 6 & 11 are commonly associated with squamous
papillomas, failed to be demonstrated in oral malignancies,
as well as potentially malignant disorders. H PV 16 & 18 are
considered to be oncogenic, being implicated in anogenital
and cervical dys.plasia, carcinoma-in-situ, invasive
carcinoma, apart from Oral squamous cell carcinomas.

48. 'C' [Check Explanation Below]


Forchheimer's sign refers to an enanthem of red macules
or petechiae confined to the soft palate in patients with
rubella. The sign presents in up to 20% of patients during
the prodromal period or on the first day of the exanthema.
Dental 9'ul-6e �- - - - - -

13. MYCOTIC INFECTIONS


1. All of the following statements about thrush are true b) Geographic tongue
EXCEPT c) Oral thrush
a) It is caused by a gram-negative fungus d) Denture stomatitis
b) It is characterized by a plaque of proliferating epithelial (PGI -98)
and other cells 10. White raised painless areas when on peeling exhibit
c) It is a complication of immunosuppression or systemic painful erythematous areas in case of
disease a) Oral thrush b) Pemphigus vulagaris
d) It can affect neonates in an epidemic fashion c) Leukoplakia d) Erythroplakia
(MAN -94) (AP -2K)
2. A thirty-two-years-old adult male presents with a chronic 11. Denture sore mouth is caused by
suppurative lesion on the angle of the jaw. Microscopic a) Actinomyces b) Candida albicans
examination shows small abscesses immediate beneath c) Blastomyces d) None of the above
the epidermis with moderate growths of epithelial cords. (AP -01)
In the abscess there are oval, unicellular organisms 12. Clinical diagnosis of candidiasis is confirmed by
measuring 20 um in diameter and having a thick, double­ a) Characteristic odour
refractive cell wall and cytoplasm containing refractive b) Demonstration of mycelia and spores in scrapping
granules and vacuoles. Which of the following is the c) Response to injection of vitamin B 12
most likely diagnosis? d) Demonstration of ray fungus in granules
a) Blastomycosis b) Foreign body reaction (AIIMS -94, 98)
c) Syphilis d) Tuberculosis 13. Thrush:
(MAN -95) a) Occurs only in elderly
3. White patch is seen on the buccal mucosa consisting of b) Is caused by a gram -ve bacteria
pseudomycelium and chalmydospores with desquamated c) Is seen clinically as a proliferative lesion
epithelium adjacent to it, the patient is suffering from d) Responds to nystatin
a) histoplasmosis b) cryptococcosis (PGI-99, AP-09)
c) candidiasis d) coccidiomycosis 14. The microorganism most commonly cultured from a
(AIIMS -99, AIIMS - 2012) chronic bilateral ulcer at the corner of mouth:
4. Id reaction is associated with a) Mucor b) Candida
a) apthous ulcer b) herpetic stomatitis c) Treponema d) Aspergillus
c) syphilis d) candidiasis (AIIMS -01)
(MAN-2K) 15. A patient is having severe mycotic infection diabetes
5. Candidiasis is a complication of all of the following except mellitus and cellulitis is suffering with:
a) Prolonged antibiotic therapy a) Mucormycosis b) Histoplasmosis
b) Corticosteroid therapy c) Candidiasis d) None of the above
c) Immuno compromised disease (AIPG -2K)
d) Aspiration pneumonia 16. Drug used to treat oral thrush
(MAN -02) a) Clobetasol b) Co-trimoxozole
6. Thrush refers to c) Miconazole d) Penicillin
a) Acute atrophic candidiasis (AP -03)
b) Acute pseudomembranous candidiasis 17. Oral Thrush develops in infants at
c) Chronic atrophic candidiasis a) 6 days b) 2-6 weeks
d) Chronic hyperplastic candidiasis c) 10-12 weeks d) 18 months
(MAN -01) (AP -03 )
7. Phycomycosis is also called 18. Candida infection is involved in
a) Botryomycosis b) Coccidiomycosis a) migrating geographic tongue
c) Phytomycosis d) Mucormycosis b) median rhomboidal glossitis
(AP -99, 03) c) prolonged tetracycline therapy
8. A deficiency of which of the following cells can predispose d) all of the above
to candidiasis (PGI -97)
a) Eosinophils b) Macropages 19. Prolonged use of antibiotics in children can result in
c) Plasma cells d) T Lymphocytes a) Necrotising ulcerative gingivtis
(KAR -2K) b) Candidiasis
9. Candid infection is not seen in: c) Actinomycosis d) Apthous ulcers
a) Median rhomboid glossitis (KAR-04)

1) A 2) A 3) C 4) D 5) D 6) B 7) D 8) D 9) B 10) A 11) B 12) B 13) D


14) B 15) A 16) C 17) B 18) B 19) B
20.
r
Candidiasis associated with dentures is:
ORAL PATHOLOGY & MEDICINE

a) Acute Pseudo Membranous


b) Acute Atrophic
c) Chronic Hyperplastic d) Chronic atrophic

21. Darlings disease is


a) Histoplasmosis b) Phycomycosis
c) Actinomycosis d) Bleomycosis
(AP-06)
22. Which type of candidiasis is associated with leukoplakia
a) Acute pseudomembranous candidiasis
b) Acute atrophic candidiasis
c) Chronic atrophic candidiasis
d) Chronic hyperplastic candidiasis
(COMEDK-09, AP-2014)
23. Vascular involvement and thrombosis is seen in
a) Coccidiomycosis b) Aspergillosis
c) Mucormycosis d) Histoplasmosis
(C0MEDK-07, 15)
24. The yeast which shows thick gelatinous capsule and
positive for mucicarmine is
a) Cryptococcus neoformans
b) Histoplasmosis
c) Blastomycosis d) Paracoccidiomycosis
(KCET-10)
25. Non-septate hyphae with a tendency to branch at 90
degree angle is characteristic of -
a) Mucor b) Aspergillosis
c) Cryptococcus neoformans
d) Coccidioides immitis
(KCET-09)
26. Oral candidiasis is divided into primary and
a) Secondary infections b) Subsidiary infections
c) Subclinical infections d) All of the above
(BHU-2012)
27. Antibiotic stomatitis is the other name for
a) Acute atrophic Candidiasis
b) Chronic atrophic Candidiasis
c) Chronic hyperplastic Candidiasis
d) Acute pseudomembranous Candidiasis
(COMEDK -14)
28. 'Mickey Mouse Ears' is a histological feature of
a) Candidiasis b) Paracoccidioidomycosis
c) Leptospirosis d) Tuberculosis
(KERALA-2015)

20) D 21) A 22) D 23) C 24) A 25) A 26) A 27) A 28) B


Dental 9'ul-6e � - - - - - -

13. MYCOTIC INFECTIONS - ANSWERS


1. 'A' [Burket's 11th ed 82/ Shafer 7th ed 371) Cryptococcosis Torulosis
Thrush is acute pseudomembranous form of candidiasis
caused by yeast like fungus, candida a/bic The oral lesions Candidiasis Monialiasis or Thrush
resemble white plaques resembling milk curds which can be Mucormycosis or
wiped away with a gauze. Phycomycosis
zygomycosis

8. 'D' [Burket's 11th ed 439/ Shafer 6th ed 364]


Type Disease
Acute • Thrush 9. 'B' [Shafer 7th ed 30, 372-74/ 6th ed 29, 364-66)
Pseudomem • Most commonly occurs in
branous immunocompromised patients 10. 'A' [Shafer 7th ed 372/ 6th ed 364)
Acute • Only type of candidiasis which is 11. 'B' [Shafer 7th ed 374/ 6th ed 365)
atrophic consistently painful
• Leukoplakic type of candidiasis 12. 'B' [Burket's 11th ed 82/ Shafer 6th ed 367)
Chronic
• Considered as premalignant condition
hyperplastic
because of its association with oral 13. 'D' [Shafer 7th ed 372/ 6th ed 3 67)
candidiasis
epidermoid carcinoma. Thrush may occur at any age, but is prone to occur in
Chronic • Denture candidiasis or denture atrophic debilitated or chronically ill patients or infants.
candida sore mouth candidiasis
associated • Angular cheilitis 14. 'B' [Burket's 11th ed 81/ Shafer 6th ed 917)
lesions • Medi an rhomboid glossitis The condition is angular cheilitis or Perleche and is due to
chronic atrophic candidiasis or riboflavin deficiency.
Candida can be identified by PAS stained smear from an
15. 'A' [Shafer 7th ed 376/ 6th ed 367-68)
infected area. It can also be cultured on Sabourad agar /
In mucormycosis the infection is characterized by
Pagano - Levin Agar (to differentiate between species). classical syndrome of uncontrolled diabetes, cellulitis,
ophthalmoplegia and meningo encephalitis. Reddish black
2. 'A' [Shafer 7th ed 368/ 6th ed 359) nasal turbinate and nasal septum with nasal discharge is
North american blastomycosis is called Gilchrist's disease one of manifestations of this disease. The fungus has an
and South american blastomycosis is called Lutz's disease. apparent predilection for blood vessels; it penetrates their
Oral manifestations of blastomycosis are similar to walls and thereby produces thromboses.
actinomycosis. Histopathology reveals microabscess and
pseudoepitheliomatous hyperplasia.
Ulceration of the palate in a poorly controlled diabetic
3. 'C' [Shafer 7th ed 371/ 6th ed 366-67) patient on biopsy showed PAS +ve organism invading
Candida exists in 3 forms: Pseudohyphae, yeast and arteries and causing the thrombosis. Most likely
chlamydospore forms. diagnosis is (KAR- 2013)
a) Syphilis
4. 'D' [Burket's 9th ed 68/ Shafer 7th ed 375) b) Mucormycosis
Id reaction / monilids are due to allergic response to candida c) Midline lethal granuloma
antigens. These patients will develop vesiculo popular rash d) Candidiasis
due to allergy and the lesions will resolve with treatment of
candida infection. 16. 'C' [Burket's 11th ed 83/ Shafer 6th ed 367)

5. 'D' [Shafer 7th ed 37 2/ 5th ed 363] 17. 'B'

6. 'B' [Shafer 7th ed 37 2/ 5th ed 506] 18. 'B' [Shafer 7th ed 30/ 6th ed 29)

7. 'D' [Shafer 7th ed 37 5/ 5th ed 510) 19. 'B' [Shafer 7th ed 372/ 6th ed 365)

20. 'D' [Shafer 6th ed 365]


Disease Other name
South American Blastomycosis Lutz's disease 21. 'A' [Shafer 7th ed 369/ 6th ed 360)

North American Blastomycosis Gilchrists disease 22. 'D' [Shafer 7th ed 374/ 6th ed 3 65)
Histoplasmosis Darling's disease
23. 'C' [Shafer 7th ed 376/ 6th ed 368)
Coccydomycosis Valley fever
24. 'A' [Shafer 7th ed 3 71/ 6th ed 362]
r ORAL PATHOLOGY & MEDICINE

Cryptococcosis is a chronic fungal infection caused by


cryptococus neoformans (Torula histolytica) and cryptococus
bacillispora. The oral lesions resembles leukemic ulcers.

Histologically, the causative organism is a gram +ve,


budding, yeast like cell with an extremely thick, gelatinous
capsule. The cryptococcus in tissue sections, appears as a
small organism with a large clear halo; sometimes described
as "tissue microcyst".

25. 'A' [Shafer 7th ed 3 77/ 6th ed 368]


Options 'B', 'C' & 'D' show septate hyphae. So only option
left is option 'K mucor. According to many recent journals,
the hyphae in mycormycosis are broad, septate and branch
at 90 ° degrees. But according to Shafer 6 th ed Pg 368,
mucormycosis histologically shows large, non septate
hyphae with branching at obtuse angles.

26. 'A' [Shafer's 7th ed 372 Table 7-1/ 6th ed 364]


Oral candidiasis is classified into primary oral candidiasis
and secondary oral candidiasis. Oral manifestations of
systemic mucocutaneous candidiasis as a result of diseases
like thymic aplasia and candidiasis endocrinopathy syndrome
is called Secondary oral candidiasis.

27. 'A' [Shafer 7th ed 3 72]


The current nomenclature used to describe acute atrophic
candidiasis is acute erythematous candidiasis. It is also
known as antibiotic sore mouth since it usually occurs as
a sequela to a course of broad spectrum antibiotics. It is
characterised by central papillary atrophy of the tongue.

28. 'B' [Check Explanation Below]


Paracoccidioidomycosis or South American Blastomycosis
is caused by dimorphic fungi Paracoccidioides brasiliensis.
Microscopically, numerous large, 20-60 um, round, narrow
base budding yeast cells are present. Single and multiple
budding occurs, the latter are thick-walled cells that form the
classical steering wheel or Mickey mouse structures that
are diagnostic for this fungus, especially in methenamine
silver stained tissue sections.
Dental 9'ul-6e � - - - - - -

14. ORAL ASPECTS OF METABOLIC DISEASE


1. Delayed eruption of at least part of the dentition is a 10. A progressive increase in mandibular length and in
recognized feature of all of the following EXCEPT mandibular interdental spacing in an adult patient is
a) Rickets b) Congenital hyperthyroidism characteristic of:
a) Cleidocranial dysplasia b) Cherubism a) Periodontosis b) Hypothyroidism
(MAN -94) c) Hyperpituitarism d) Hypoadrenalism
2. A five-year-old child presents with chronic bed wetting (AIPG -03)
and bilateral loose deciduous first molars. His mother 11. Which of the following deficiencies are associated
says that she want to drink of water several times during with the disorders of hyperplasia of salivary gland and
each night. Which of the following is the most likely keratinisation of the salivary gland
diagnosis? a) Vit -A b) Vit -B
a) Hand-Schuller-Christian disease c) Vit -C d) Vit -K
b) Marble bone disease (AP -99)
c) Nieman-Pick disease d) Polyostotic fibrous dysplasia 12. Oral lesions on the tongue and other mucosal surfaces of
(MAN -95) the oral cavity is caused by the deficiency of
3. Which of the following is related to an enzyme deficiency a) Vit -Bl b) Niacin
and involves periodontal destruction around primary c) Vit-C d) Vit-K
teeth? (AP -99)
a) Hypophosphatasia b) Cyclic neutropenia 13. Deficiency of which of the following will cause enamel
c) Juvenile periodontitis d) Papillion-Lefevre syndrome hypoplasia?
(MAN -97) a) Vitamin B and C b) Vitamin A, C and D
4. Osteomalacia is c) Calcium d) Vitamin C
a) defective osteoid+ normal mineralization (AIIMS -95, KCET-08)
b) normal osteoid + defective mineralization 14. Which of the following may be a feature of acromegaly?
c) abnormal osteoid + abnormal mineralization a) Large tongue b) Micrognathia
d) normal osteoid an demineralization c) Hypoglycemia d) Crowded teeth
(MAN -99) (AIIMS-02, AIPG-11)
5. The deficiency of which of the following vitamins does 15. Gingiva is most commonly affected by deficiency of:
not effect on tooth development a) Vitamin A b) Vitamin D
a) Vit - A b) Vit - D c) Vitamin C d) Vitamin B
c) Vit - C d) Vit - K (AIIMS -95)
(MAN -02) 16. Magenta tongue and cracks at corner of mouth are seen
6. Which of the following tooth structure during formation in deficiency of:
is most effected due to Vit -A deficiency a) Vitamin B 1 b) Niacin
a) Enamel b) Dentin c) Riboflavin d) Pantothenic acid
c) Cementum d) Periodontal Ligament (AIIMS -95)
(AP -98, AIIMS -99) 17. Diabetes insipidus, bone lesions and exophthalmos is
7. Addison's disease is related to:- seen in
a) Adrenal medulla b) Adrenal cortex a) Sickle cell anemia b) Niemen pick disease
c) Post pituitary d) Parathyroid gland c) Littere Siewe Disease
(AIIMS -98) d) Hand-Schuller-Christian disease
8. Which of the following is least affected in Vit -C (AIIMS -2K, AIIMS -97)
deficiency 18. The histiocytosis X is a spectrum of disorders, which
a) Gingival Fibres b) Periodontal ligament include the following condition.
c) Blood Vessels of the gingiva a) Eosinophilic granuloma
d) Epithelial lining of the mucosa b) Hand Schuller - Christian disease
(AP -98) c) Letterer - Siwe disease d) All of the above
9. Dinesh, a 24-year-old male, complains of loose teeth (KAR -03)
in a single quadrant. His radiograph shows irregular 19. An etiological factor for macroglossia is:
bone loss and histopathology reveals eosinophils and a) Atrophic glossitis b) Oral submucous fibrosis
histiocytes. The most probable diagnosis is: c) Syphilitic glossitis d) Hurler's syndrome
a) Hand-Schuller-Chriistian disease (KAR -03)
b) Paget's disease 20. A disease which only affects the formation and eruption
c) Osteoclastoma d) Albright's syndrome of tooth but does not cause hypoplasia is:
(AIIMS - 99, AIIMS 94, AIPG -89,92) a) Hypoparathyroidism b) Hyperthyroidism

1) B 2) A 3) A 4) B 5) D 6) A 7) B 8) D 9) A 10) C 11) A 12) B 13) B


14) A 15) C 16) C 17) 0 18) D 19) D 20) B
c) Rickets
r
d) Syphilis
ORAL PATHOLOGY & MEDICINE
32. Beefy red and painful tongue is characteristic of
(PGI -99, KAR -99, AP- 06) a) Vitamin A deficiency b) Any periodontal disease
21. Premature exfoliation of deciduous teeth is seen in c) Acute nicotinic acid deficiency
a) Hypophosphatasia b) Hypophosphatemia d) Ascorbic acid deficiency
c) Hyper phosphtasia d) Hyperparathryroidism (TNPSC -99)
(AP -2K) 33. Letterer-seiwe disease is a disturbance of
22. Bone changes in hyperparathyroidism include: a) Protein metabolism b) Histiocytic disorder.
a) Generalized demineralization c) Mucopolysaccharide metabolism
b) Brown tumours d) Carbohydrate metabolism
c) Cystic changes d) All of the above (KAR -98)
(KAR -03) 34. Macrodontia is associated with
23. Eosinophilic granuloma results from the proliferation of a) Acromegaly b) Pituitary gigantism
a) Histiocytes b) eosinophils c) Hypoparathyroidism d) Hyperthyroidism
c) Lymphocytes d) fibroblast (KAR -98)
(AP -99) 35. Red fluorescent fluid is seen in
24. Acrodermatitis enteropathica is due to deficiency of a) Pemphigus b) erythema multiforme
a) Mercury b) Zinc c) lichen planus d) porphyria
c) Lead d) Bismuth (PGI -02, BHU-07)
(AP -99, 03) 36. Hypogonadism, developmental delay, loss of taste and
25. Vitamin "D" deficiency causes all except: smell is due to deficiency of:
a) Widening of predentin b) Defective calcification a) Cu b) Zn
c) Microdontia c) K d) Cr
d) Inter globular dentin formation (AIIMS-06, 2012)
(AP -01) 37. Swollen joint, anemic, loose teeth & dentin dysplasia are
26. An abnormal resorption pattern in primary teeth, delayed because of deficiency of:
eruption of permanent teeth and a large tongue are the a) Vitamin C b) Vitamin D
features of: c) Vitamin B1 d) Vitamin E & D
a) Addison's disease b) Hypothyroidism (AIIMS-06)
c) Hyperthyroidism d) Von-Recklinghausen disease 38. Intestional absorption of Calcium is decreased by
(AIIMS -01) a) Proteins b) Lactose
27. Lamina Dura is lost or partially lost in: c) Phytic Acid d) Acidity
a) hypothyroidism b) hyperthyroidism (COMEDK-06)
c) hypoparathyroidi sm d) hyperparathyroidism 39. Brown tumours are seen in:
(KAR -98) a) Hyperparathyroidism
28. Which of the following is associated with a low b) Pigmented villonodular synovitis
concentration of ionized calcium in the serum? c) Osteomalacia
a) Hypothyroidism b) Osteogenesis imperfecta d) Neurofibromatosis
c) Paget's disease of the bone (AIPG-06)
d) Tetany 40. 65 year old man who is anaemic, complaints of back
(AIIMS -03) pain, multiple radiolucencies in panaromic view
29. A 50-year-old obese man complains of several recent a) Multiple myeloma b) Osteosarcoma
abscesses in the gingiva with loosening of teeth. He c) Giant cell granuloma d) Eosinophilic
also suffers from itching of skin and polyuria. The most (AP-06)
probable etiology is: 41. Normal serum calcium level is
a) Scurvy b) Myxoedema a) 5 to 7 mg% b) 7 to 9 mg%
c) Diabetes mellitus d) Vitamin A deficiency c) 9 to 11 mg% d) 11 to 13 mg%
(AIPG -89, MAN -2K) (KAR-04)
30. Consider the following statements; giant cells are a 42. Pigmentation occurs in oral cavity and skin in all of the
characteristic histopathologic finding in following except:
a) Apthous ulcers b) Keratocyst a) Peutz-jeghers syndrome b) Addison's syndrome
c) Brown tumor of hyper parathyroidism c) Cushing syndrome d) Albright syndrome
d) Dentigerous cyst (AIPG-07)
(UPSC -01) 43. Hyperparathyroidism radiological features seen are
31. Bone Pain, bone, cysts, fractures and renal stones are a) Loss of laminadura
characteristics of b) Osteitis fibrosa cystica
a) Hyperparathyroidism b) Cushing's syndrome c) Erosion below the duramater of skull
c) Multiple myeloma d) Marfan's syndrome d) All of the above
(KAR -98) (AP-07)

21) A 22) D 23) A 24) B 25) C 26) B 27) D 28) D 29) C 30) C 31) A 32) C 33) B
34) B 35) D 36) B 37) A 38) C 39) A 40) A 41) C 42) C 43) D
Dental 9'ul-6e
44. Which of the following vitamin is associated with 55. A one year old child, who is earlier normal, is suffering
manifestations of neurological problem from copious nasal discharge, coarse facial features,
a) Vit A b) VitK large tongue, bulging head and flattened nose. He is
c) Folic acid d) Cyanacobalamin also having enlarged spleen and liver. What may be the
(AP-07) cause?
45. All the following affects absorption of calcium, EXCEPT a) Hypothyroidism b) Beckwith widman syndrome
a) Citric acid b) Retinoic acid c) Proteus syndrome d) Hurler syndrome
c) Phytates d) Oxalates (AIIMS NOV-14)
(MCET-07)
46. A 9 year old child has increased Horizontal anterior
bone loss, less cementum and on test shows excretion of
phosphoethanolamine in the urine. The child is suffering
from.
a) Hypophosphatasia b) Vit. D resistant Rickets
c) Juvenile periodontitis d) Osteomalacia
(AIIMS-08}
47. Which of the following may be a feature of acromegaly?
a) Large tongue b) Micrognathia
c) Hypoglycaemia d) Crowded teeth
(AIPG-09)
48. Wernick's encephalopathy is caused by deficiency of
a) Thiamine b) Cyanocobalamine
c) Niacin d) Riboflavin
(AIPG-10)
49. Hepatolenticular degeneration is seen with deposition of
a) cadmium b) lead
c) aluminium d) copper
(AIPG-10)
50. Increase in height of mandible with increase in
interdental spaces in elderly man
a) Hyperpituitarism b) Hyperthyroidism
c) Hypopituitarism d) Hyperparathyroidism
(AIIMS-09)
51. Reilly bodies are inclusion bodies seen in hurler's
disease within
a) Lymphocytes b) Fibroblast
c) RBC d) WBC
(AIPG-10)
52. Hyperpigmentation is seen in all except
a) peutz jeghers syndrome b) addison's disease
c) cushing's sydrome d) albright syndrome
(AIPG-10)
53. A histologic evidence of widespread formation of
globular hypocalcified dentin and pulp horns reaching
the dentinoenamel junction, absence of lamina dura
around the tooth in radiograph are the characteristic
features of -
a) Vitamin-□ resistant rickets
b) Hypophosphatasia
c) Hypervitaminosis-A d) Vitamin-A deficiency
(KCET-09)
54. Addison's disease typically
a) Causes hypertension b) Causes hypopigmentation
c) Is an autoimmune disease
d) Steroids are contraindicated
(AP-2012)

44} D 45) B 46} A 47) A 48} A 49} D 50) A 51} A 52} C OR NONE 53) A 54) C 55) D
_____ r ORAL PATHOLOGY & MEDICINE

14. ORAL ASPECTS OF METOBOLIC DISEASE - ANSWERS


1. 'B' [Shafer 7th ed 652/ 6th ed 650] with an increased tendency towards fracture. Radiologically
Delayed eruption is seen in conditions like: there is severe asymmetric deformities of all stress bearing
• Rickets bones like long bones of legs, spine and pelvis.
• Down syndrome
Osteomalacia is failure of mineralization of newly formed
• Hypothyroidism organic bone matrix (Osteoid reminerealization] at sites
• Cleidocranial dysplasia of bone turnover; whereas Rickets is due to failure of
• Achondroplasia mineraliztion of endochondrial new bone formed at the
growth plates of children. Bone matrix is normal in both.
• Cherubism
5. 'D' [Shafer 7th ed 635-642/ 6th ed 640-41]
In hyperthyroidism, premature shedding of deciduous teeth
The three vitamins, which participates in tooth development
and accelerated eruption of permanent teeth is seen.
and calcification areA, C, D. Deficiency of vitaminA causes
failure of histodifferentiation and morphodifferentiation of
2. 'A' [Shafer 7th ed 7 50/ 6th ed 7 44]
the odontogenic epithelium. Enamel matrix formation is
arrested or enamel hypoplasia occurs. Vitamin D deficiency
Non-Lipid Lipid causes meneralization deficit in bone and teeth. Such teeth
reti culoendothelioses reticuloendothetioses have wide pre-dentin zone and much interglobular dentin,
• Letterer -Siwe disease • Gaucher's disease eruption is retarded. Deficiency of vitaminC causes atrophy
and disorganization of odontoblasts causing irregularly laid
• Hand-Schuller-Christian • Niemann - Pick disease
down dentin.
disease (multifocal
eosinophic granuloma)
Vitamin K is coagulation vitamin, and is useful in synthesis
• Eosinophilic granuloma of some clotting factors like 2,7,9,10. Gingival bleeding is
the most common oral manifestation of vitamin K deficiency.
Hand-Schuller Christian disease is characterized by
classic triad of:
Which vitamin does not play an important role in
• Single or multiple areas of regular "punched - out" tooth development? (NEET-2013}
bone destructions in skull. a)VitA b)VitC
• Unilateral or bilateral exopthalmos and c)Vit D d) Vit E
• Diabetes insipidus.
6. 'K [Shafer 7th ed 635/ 6th ed 634]
Gingivitis, loose and sore teeth with precocious exfoliation
of teeth, failure of healing of sockets, and loss of supporting 7. 'B' [Shafer 7th ed 655/ 6th ed 654]
alveolar bone mimicking advanced periodontal disease are Addison's disease is due to insufficiency of adrenal cortex
important oral manifestations. hormones.Clinically the disease is manifested as pale brown
to deep chocolate pigmentation of oral mucosa, spreading
3. 'A' [Shafer 7th ed 640/ 6th ed 639] over buccal mucosa, gingiva, tongue and lips and bronzing
Hypophosphatasia is due to deficiency of enzyme alkaline of skin. Biopsy of the lesions shows acanthosis with silver­
phosphatase in serum or tissues and excretion of phospho positive granules in the cells of stratum germinativum.
ethanol amine in the urine. Hypotension and gastrointestinal disturbances are other
features.
Loosening and premature loss of deciduous teeth. Chiefly
the incisors, irregular qualifications of metaphyses of long 8. 'D' [Shafer 7th ed 642-43/ 6th ed 641]
bones are important features. The teeth characteristically
shows absence of cementum as a result of failure of 9. 'A' [Shafer 7th ed 750/ 6th ed 744]
cementogenesis.So, there is no sound functional attachment
of the tooth to bone by periodontal ligament and this 10. 'C' [Shafer 7th ed 650/ 6th ed 649]
accounts for early loss of deciduous teeth. Gigantism in children and acromegaly in adults are
manifestations of hyperpituitarism due to excessive
4. 'B' [Burket's 11th ed 534/ Shafer 7th ed 637/ 6th ed growth hormone secretion. Macrodontia, hypercementosis,
636] mandibular prognathism and macroglossia causing spaced
Osteomalacia is due to deficiency of Vitamin-D in adults. denliliun due tu labial Lipping uf mandibular leelh are
The disease is mostly seen in Post-menopausal females with important oral manifestations of hyperpitutarism.
a history of low calcium intake and little exposure to U.V.
light. Remodeling of bone occurs in absence of calcium and
this results in a softening and distortion of the skeleton
Dental 9'ul-6e
11. 'A' [Shafer 7th ed 635-36/ 6th ed Pg633-34] 19. 'D' [Shafer 7th ed 631/ 6th ed 630)
Important manifestations of Vit. A deficiency: Hurler's syndrome or mucopolysaccharidosis is characterized
• Keratinising metaplasia of epithelium. by elevated mucopolysaccharide level in urine and there is
an excessive intracellular accumulation of both chondroitin
• Occlusion of salivary gland ducts with keratin.
sulfate B and heparin sulfate. "Claw hands", broadening of
• Enamel hypoplasia, atypical dentin formation and mandible with a wide intergonial distance, localized areas
epithelial invasion of pulpal tissue are important features. of bone destruction, spaced dentition, gingival hyperplasia
• Enamel is more severely affected than dentin. and macroglossia are important features.

12. 'B' [Shafer 7th ed 646/ 6th ed 644-45] "Hurler cells" or "gargoyle cells" are important
histological findings. In addition, metachromatic granules
13. 'B' [Shafer 7th ed 52, 53/ 6th ed 50] or Reilly bodies are often demonstrated in the cytoplasm of
Deficiency of vitamins A, C and D causes enamel hypoplasia circulating lymphocytes (AIPG-1O).
(KCET-07). Though hypocalcemia also causes enamel
hypoplasia, the causes for hypocalcemia are many and is not 20. 'B' [Shafer 7th ed 651/ 6th ed 650]
due to deficiency of calcuim alone.
21. 'A' [Shafer 7th ed 640/ 6th ed 639]
14. 'A' [Shafer 7th ed 650/ 6th ed 649] Option 'K Hypophosphatasia is characterized by deficiency
Macroglossia is due to: of alkaline phosphatase enzyme in serum or tissues and
• Beckwith-wiedmann syndrome (97.5% cases) excretion of phosphoethanolamine in urine.
• Amyloidosis
Option 'B' Hypophosphatemia is other name for vitamin D
• Congenital macroglossia due to over development of resistant rickets. Oral manifestations of Hypophosphatemia
musculature. includes
• Lymphangioma
• Formation of globular, hypocalcified dentin in region of
• Hemangioma pulp horns
• Neurofibromatosis • The pulp horns are elongated and extend high, reaching
• Blockage of lymphatic vessels in cases of malignant nearly to the dentinoenamel junction. So pulp exposure
neoplasms in tongue. is common in apparently normal tooth without
• Cretinism or congenital hypothyroidism. demonstrable destruction of the tubular matrix.
• Acromegaly or hyperpituitarism. • In addition to abnormal cementum, the laminadura
• Hurler's syndrome around the teeth is frequently absent.
• Scurvy
• Down syndrome 22. 'D' [Shafer 7th ed 652-53/ 6th ed 651-52]

23. 'A' [Shafer 7th ed 751/ 6th ed 745-46]


15. 'C' [Shafer 7th ed 643/ 6th ed 641]
24. 'B' [Shafer 7th ed 624/ 6th ed 622]
16. 'C' [Shafer 7th ed 645/ 6th ed 644]
Raw beefy tongue and bald tongue of 25. 'C' (Shafer 7th ed 637/ 6th ed 636]
Niacin
sandwith is seen in deficiency of Delayed eruption of teeth, developmental abnormalities
Magenta coloured tongue is seen in of dentin and enamel, malalignment of the teeth in the
Riboflavin jaws are features of Vitamin D d,eficiency. The teeth shows
deficiency of
defective calcification, wide predentin zone and increased
Moellers glossitis or Hunter's glossitis is interglobular dentin formation.
Vitamin 8 12
seen in deficiency of
26. 'B' [Shafer 7th ed 651/ 6th ed 650]
17. 'D' (Shafer 7th ed 750/ 6th ed 744] Maxillary prognathism, delayed eruption of teeth, prolonged
retention of primary teeth due to abnormal resorption
18. 'D' (Shafer 7th ed 749/ 6th ed 743] pattern and malocclusion due to macroglossia are features
The old term histiocytosis X is replaced by new term called of hypothyroidism.
Langerhans cell histiocytois (LCH). The presence of
birbeck granules is characteristic finding in LCH. 27. 'D' [Shafer 7th ed 653/ 6th ed 651-52]

By electron microscopic examination the presence of 28. 'D' [Shafer 7th ed 617/ 6th ed 653]
"Birbeck granules" in the cytoplasm of the cells in one
of the following conditions is characteristic of (AP -14) 29. 'C' (Shafer 7th ed 658/ 6th ed 656]
a) Adenolymphoma b) Plasmacytoma
c) Burkitt's lymphoma 30. 'C' [Shafer 7th ed 653/ 6th ed 652]
d) Langerhans cell histiocytosis
31. 'A' [Shafer 7th ed 652/ 6th ed 652]
r ORAL PATHOLOGY & MEDICINE
Citrates reduce the pH of intestinal tract, which in turn
increases the absorption of calcium and its deposition in
32. 'C' [Shafer 7th ed 646/ 6th ed 645] bone. Lactose or milk sugar also increases the calcium
Casal's necklace in form of characteristic skin rash absorption.
particularly in the areas exposed to sunlight especially in
the neck region is important feature of niacin deficiency. 46. 'A' [Shafer 7th ed 640/ 6th ed 699]
Patients with hypophosphatasia have defects in
33. 'B' [Shafer 7th ed 634 , 750/ 6th ed 744] mineralization of bone due to TNSALP (Tissue non-specific
alkaline phosphatase) deficiency. The earliest manifestation
34. 'B' [Shafer 7th ed 650/ Shafer 6th ed 649, 39] of the disease may be loosening and premature loss of
deciduous teeth, chiefly the incisors.
35. 'D' [Shafer 7th ed 629/ 6th ed 627]
Porphyria is due to inborn errors of porphyrin metabolism, 47. 'A' [Shafer 7th ed 650/ 6th ed 649]
characterized by overproduction of uroporphyrin. Excretion An increase in the number of granules in the acidophilic
of red urine containing much uroporphyrin, photo sensitivity, cells or an adenoma of the anterior lobe of the pituitary is
vesicular eruptions containing serous fluid that exhibits associated with gigantism or acromegaly. If the increase in
red fluorescence, staining of deciduous and permanent granules occurs before the epiphyses of the long bones are
dentitions are important features. closed, gigantism results. If the increase occurs later in life
i.e., after epiphyseal closure, acromegaly develops.
The deciduous and permanent teeth may show a red or
brownish discoloration and exhibit red fluorescence. In acromegaly, the lips become thick and negroid. The
Deposition of porphyrin in bone and teeth is due to its tongue also become enlarged and shows indentations on the
physical affinity for calcium phosphate. sides from pressure against the teeth. Also, the mandible
becomes large and macrognathic because of accelerated
36. 'B' [Shafer 7th ed 624/ 6th ed 622] condylar growth. The teeth in the mandible are usually
tipped to the buccal or labial side (spaced dentition), owing
37. 'A' [Shafer 7th ed 642-44/ 6th ed 641-42] to enlargement of tongue.

38. 'C' (Shafer 7th ed 616/ 6th ed 614] 48. 'K [Shafer 7th ed 645/ 6th ed 643]
Vit. D and citrates increase the absorption of calcium from Alcoholic patients with chronic thiamin deficiency shows
intestine. Phytic acid, which is found in cereals, forms an CNS manifestations known as Wernicke's encephalopathy,
insoluble calcium phytate with ingested calcium and renders which consists of horizontal nystgmus, opthalmoplegia,
it non available. Fat aids in calcium absorption except in cerebral ataxia and mental impairment.
sprue/idiopathic steatorrhea.
Along with the above-mentioned symptoms, if there is loss
39. 'A' [Shafer 7th ed 654/ 6th ed 653) of memory and confabulatory psychosis, it is known as
Wernicke-Korsa Koff Syndrome.
40. 'A' [Shafer 7th ed 188-89/ 6th ed 184]
49. 'D' [Shafer 7th ed 623/ 6th ed 621]
41. 'C' (Shafer 7th ed 617/ 6th ed 614] Copper is necessary for normal erythropoiesis as well as
for iron absorption. Copper deficiency produces microcytic
42. 'C' (Shafer 7th ed 656/ 6th ed 655] hypochromic anemia, due to impairment of erythropoiesis
Note: and decrease in erythrocyte survival time, which cannot be
Cushing's syndrome is caused by various endogenous corrected by administration of iron.
etiologies including an activating pituitary tumor (cushing's
disease), adrenal pathology (hyperadrenocorticism), ectopic Human copper deficiency diseases of importance are
secretion of steroids, ACTH or corticotropin - releasing hepatolenticular degeneratio111 (Wilson's disease) and
hormone by various neoplasms. Menke's syndrome (Steely-or-Kinky hair syndrome).

Cushing's syndrome produced by ACTH - secreting tumors, 50. 'A' [Shafer 7th ed 650/ 6th ed 649]
also produces diffuse mucocutaneous pigmentation similar
to adrenal insufficiency. 51. 'A' [Shafer 7th ed 631/ 6th ed 869]
In Hurler's syndrome, histologically, abnormal deposits are
43. 'D' [Shafer 6th ed 651-52/ White & Pharoah 6th ed 456] found in fibroblasts assuming the ppearance of 'Hurler
cells' or 'gargole cells' in gingival tissues of affected. Also,
44. 'D' [Shafer 7th ed 652-53/ Burket's 11th ed 119/ 10th presence of reilly bodies or metachromatic granules in
ed 891) the cytoplasm of circulating lymphocytes is an important
feature.
45. 'B' [Shafer 7th ed 616/ 6th ed 614]
Oxalates and phylates form insoluble calcium oxalate and
calcium phytate complexe s and renders it non-available.
Dental 9'ul-6e
52. 'C' or 'None' [Shafer 7th ed 656/ 6th ed 655] 55. 'D' [Shafer 7th ed 630, 27)
In Option 'P< Peutz Jeghers syndrome, oral pigmentation is Hurler syndrome is a disturbance of mucopolysaccharide
distinctive and is usually pathognomonic. Multiple, focal, metabolism that becomes apparent within the first
melanotic brown macules are concentrated about the Lips, two years of life. The disease progresses during early
while the remaining facial skin is less strikingly involved. childhood and is characterised by prominent forehead,
saddle nose, hypertelorism, thick lips, nasal congestion,
In Option 'B' Addison's disease, there is increased secretion of hepatosplenomegaly, and short neck and flexion
melanocyte stimulating hormone and which in turn induces contractures. 97.5% of patients with option 'B' Beckwith­
the deposition of melanin in the skin and mucous membrane. Wiedemann syndrome have macroglossia, but other features
The patient shows non-specific multiple focal dark spots or are absent.
generalized diffuse streaks of hyperpigmentation which may
be macular; flat, brown and variable in shape.

In Option 'D' Albright syndrome, the pigmentation is in form


of Cafe-au-lait pigmentation.

Option 'C' Cushing's syndrome is due to excess secretion


of hormones of adrenal cortex. There is no evidence of
pigmentation in case of Cushing's syndrome (According to
Shafer 6th ed 655 and Harrison 16th ed Pg 302)

But according to Burket's 9 th ed Pg 129, Bronzing of skin


and patchy melanosis of the oral mucosa are signs of Addison
disease and pituitary-based Cushing's syndrome. In both of
these endocrine disorders, the cause of hyperpigmentation
is over secretion of adreno cortico tropic hormone (ACTH),
a hormone with melanocyte stimulating properties. In both
cases, the skin may appeared tanned, and the gingival,
palate and buccal mucosa may be blotchy. These changes
in pigmentation are due to accumulation of melanin
granules as a consequence of increased hormone-dependent
melanogenesis.

So, the correct answer in option 'C' or none. But according


to me, option 'C' Cushing's syndrome is more appropriate.

53. 'A' [Shafer 7th ed 638)


Vitamin □-resistant rickets has marked effects on the
teeth and supporting structures. Characteristically, there is
histological evidence of wide spread formation of globular,
hypocalcified dentin, with clefts and tubular defects
occurring in the region of the pulp horns. In addition,
these pulp horns are elongated and extend high, often
reaching nearly to the dentinoenamel junction. In addition
to abnormal cementum, the lamina dura around the teeth is
also reported to be frequently absent or poorly defined on
the roentgenogram, and also the alveolar bone pattern is
often abnormal.

54. 'C' [Shafer's 7th ed 655/ 6th ed 654)


Chronic insufficiency of the adrenal cortex or Addison's disease
usually develops following the autoimmune destruction of
adrenal glands and frequently occurs in conjunction with
other autoimmune disorders. It is characterized by lethargy,
fatigue, muscular weakness, hyperpigmentation, severe
anemia, feeble heart action.
_____ r ORAL PATHOLOGY & MEDICINE

15. TONGUE DISORDERS


1. A blue nodular mass on the lateral border of the tongue d) Cytoplasm of the striated muscle cells
is soft, smooth and blanches upon pressure. It is most (AIPG -03)
Iikely to be: 11. Glossodynia is
a) Lymphoma b) Hemangioma a) Pain in the tongue b) Burning of the tongue
c) Epulis fissuratum d) Epithelioma c) Swelling of the tongue d) White patch on tongue
(AIPG -92) (COMEDK -03}
2. Leutic glossitis can occur in connection with: 12. "Hairy Tongue" is a condition in which certain structures
a) Vitamin B deficiency b) Vitamin C deficiency become enlarged.
c) Iron deficiency d) Syphilis a) Fungiform papillae b) Filiform papillae
(PG I -99) c) Circumvallate papillae d) Taste buds
3. Median rhomboid glossitis is due to: (APPSC -99)
a) Inflammation of the tongue 13. Magenta tongue is found in the deficiency of the vitamin
b) Persistence of tuberculum impar a) Riboflavin b) thiamine
c) Hypertrophy of fi Ii form papillae c) nicotinic acid d) pyridoxine
d) Atrophy of filiform papillae (TNPSC -99)
(AIIMS -91) 14. Squamous cell carcinoma on tongue most common site is
4. Median rhomboid glossitis is associated with: a) Apex b) Base
a) Burning mouth syndrome c) Lateral borders d) Oorsum
b) Fungal infection (KAR -98)
c) Bacterial infection d) Absence of filiform papillae 15. The syndrome of geographic tongue should be treated by:
(AIIMS -96,97) a) Excision of the lesion b) Penicillin therapy.
5. Burning sensation of the tongue is called: c) Topical application of Nystatin
a) Glossopyrosis b) Glossodynia d) Routine observation at recall time
c) Glossitis d) Glossolgia (AIPG -04,AIPG -95)
(KAR -98) 16. Bald tongue is not seen in
6. A median round lesion in front of circumvallate papillae, a) Pyogenic stomatitis b) Reiter Disease
with epithelial hyperplasia diagnosis is: c) Atrophic lichen planus d) Psoriasis
a) Median rhomboid glossitis (PGI JUNE-2014)
b) Erythema migrans 17. All the following conditions can lead to depapillation of
c) Apthous ulcer d) Chemical burn the tongue EXCEPT
(AIPG -2K) a) Plummer-Vinson syndrome
7. Following a general examination and a biopsy of a b) Amyloidosis
firm, pale nodule in the tongue, a diagnosis of primary c) Pernicious anemia d) Luetic glossitis
amyloidosis has been reached. What underlying disease (MHCET-15)
is this patient likely to have 18. Riga fede disease is the ulceration seen on
a) Leprosy b) Syphilis a) Attached gingiva b) Buccal mucosa
c) Tuberculosis d) None of the above c) Lower lip d) Tongue
(AIIMS -93) (APPG-15)
8. Which of the following occurs most commonly on tongue?
a) Lymphangioma aind granular cell myoblastoma
b) Lipoma and fibroima
c) Neuroblastoma and Lipoma
d) Lymphangioma aind fibroma
(AIIMS -95)
9. Oral hairy leukoplakia is seen in AIDS patients. The most
likely site of appearance is:
a) Lateral borders of tongue b) Sublingual mucosa
c) Soft palate d) Buccal mucosa
(AIIMS -90)
10. In amyloidosis of the tongue, the amyloid is deposited
primarily in the:
a) Stromal connective tissue
b) Cells of the surface epithelium
c) Nuclei of the striated muscle cells

1} B 2) D 3) B 4) D 5) A 6) A 7) D 8) A 9) A 10) A 11) A 12) B 13) A


14) C 15) D 16) A 17) B 18) D
Dental 9'ul-6e � - - - - - -

15. TONGUE DISORDERS - ANSWERS


1. 'B' [Shafer 7th ed 145-46/ Burket's 10th ed 127) Secondary or Type A contains AA amyloid. Secondary or
reactive amyloidosis occurs typically as a complication
2. 'D' [Shafer 7th ed 330/ 6th ed 324] of chronic infectious (Eg: TB, osteomyletis, syphilis),
The atrophic or interstitial glossitis, characteristic of syphilis non-infectious chronic inflammatory conditions such as
is known as leutic glossitis. The incidence of carcinomatous rheumatoid arthritis, and in familial Mediterranean fever.
transformation is more with leutic glossitis.
Type C amyloid includes amyloid of aging and amyloid
3. 'B' [Shafer 7th ed 30/ 6th ed 29) adjacent to APUD tumors i.e. pheochromocytoma.

4. 'D' (Shafer 7th ed 30/ 6th ed 29] 8. 'A' [Shafer 7th ed 151, 193/ 6th ed 147, 189)
Tongue is the most common site for both lymphangioma and
Condition Papilla involved
granular cell myoblastoma.
Benign migratory glossitis Multiple areas of
or migratory glossitis desquamation of filiform 9. 'A' [Burket's 10th ed 551/ Shafer 7th ed 360)
papillae Oral hairy leukoplakia, Burkitt's lymphoma, infectious
Median rhomboid glossitis mononucleosis and nasopharyngeal carcinoma are
(Posterior midline Filiform papillae are absent manifestations of EBV virus (COMEDK-14) in AIDS
atrophic candidiasis) patients. Oral hairy leukoplakia manifests as an symptomatic
Hairy tongue, Acanthosis Hypertrophy of hyperkeratotic striae that are usually seen on lateral borders
filiform
of tongue. Because of its clinical characteristics, differential
nigricans. papillae
diagnosis should include hyperplastic candidiasis. Candida
Riboflavin deficiency Atrophy of filiform papillae albicans may be present in more than 50% of oral hairy
with normal or engorged leukoplakia lesions.
fungiform papillae
Strawberry tongue Swollen, hyperemic, fungiform 10. 'A' [Shafer 7th ed 628/ 6th ed 626)
(scarlet fever) papillae
Familial dysautonomia Absence of fungi form and 11. 'A' (Shafer 7th ed 856/ 6th ed 847)
circumvallate papillae Both Glossopyrosis and glossodynia are seen in orolingual
paresthesia and pernicious anemia.
Red, painful tongue with
atrophy of filiform papillae 12. 'B' [Burket's 9th ed 29/ Shafer 7th ed 32)
Iron deficiency anemia
and later the fungiform Lingua nigra or lingua villosa is the other name for hairy
papillae. tongue.

5. 'A' [Shafer 7th ed 856/ 6th ed 847) 13. 'A' (Shafer 7th ed 645/ 6th ed 645)
Glossodynia Painful tongue
Glossopyrosis Bumi ng tongue
14. 'C' [Shafer 7th ed 116/ 6th ed 111]
Lateral margins followed by ventral surface are common sites
Glossoplegia Paralysis of tongue of squamous cell carcinoma of tongue. Dorsum of tongue
is usually involved in patients with a history of syphilitic
6. 'A' [Shafer 7th ed 30/ 6th ed 29) glossitis or leutic glossitis. (KCET-09)
Median rhomboid glossitis is due to failure of tuberculum
impar to retract before the fusion of the lateral halves of 15. 'D' (Shafer 7th ed 32]
tongue, so that a structure devoid of papillae is interposed No treatment is usually necessary for geographic tongue.
between them. It appears clinically as an ovoid, diamond Symptomatic leasions should be treated with topical
or rhomboid shaped reddish patch on dorsal surface of prednisolone and / or systemic antifungal medication.
tongue immediately anterior to the circumvallate papillae.
Histologically it may show pseudoepitheliomatous 16. 'A' (Shafer 7th ed 31, 809)
hyperplasia. Psoriasis and Reiters syndrome are characterised by
depapillated areas involving the tongue as geographic
7. 'D' (Shafer 7th ed 628/ 6th ed 626] tongue. Atrophic lichen planus is characterised by smooth,
The condition is Primary or Type B amyloidosis. It mainly red poorly defined areas, usually involving the tongue.
contains AL type fibril proteins and is thought to be of
immune in origin. Commonly seen in multiple myeloma and 17. 'B' [Shafer 7th ed 762,771]
waldenstrom's macroglobulinemia. Clinically the patient is Amyloidosis causes macroglossia but not bald tongue.
asymptomatic.
18. 'D' [Burket's 12th ed 674]
r ORAL PATHOLOGY & MEDICINE

Riga Fede's disease is characterised by ulceration of the


sublingual aspect of tongue due to trauma from natal/
neonatal teeth in infants.
Dental 9'ul-6e � - - - - - -

16. PIGMENTATION
1. The pigment associated with hemochromatosis is d) Bechet's syndrome
a) Bilirubin b) Haemosiderin (AIIMS -01)
c) Methemoglobin d) myoglobin 12. Green discoloration of teeth is seen in:
(MAN -94, AIPG -94) a) Tetracycline therapy b) Fluorosis
2. The most common intraoral location for a pigmented c) Erythroblastosis fetalis d) None of the above
nevi is the (AIIMS -98)
a) Hard palate b) Soft palate 13. All of the following are seen in lead poisoning except:
c) Buccal mucosa d) Floor of mouth a) Hallucinations b) GIT disturbances
(AIIMS -94) c) Peripheral neuritis d) Encephalitis
3. Yellowish discoloration of teeth is seen in children fed (AIPG -99}
on: 14. Cafe-au-lait spots on the skin are characteristic of
a) High-protein diet b) Tetracyclines a) Addision's disease b) Peutz-Jeghers syndrome
c) Penicillins d) Erythromycin c) Von recklinghausen disease
(AIPG -98) d) Hyper pituitarism
4. Disease which increases oral melanin pigmentation: (AIPG-05)
a) addison's disease b) Hyperthyroidism 15. Yellowish discolouration of oral mucous membrane,
c) Nephritis d) All of the above skin and sclera of eye is
(KAR -2K) a) Pernicious anemia b) Sickle cell anemia
5. Patient reports with discolored teeth bearing brown c) Chloromycin therapy d) Ca.rotinemia
stains. The teeth glow fluorescent in UV light. The most (KAR -98)
likely diagnosis is: 16. Pink's disease is due to
a) Porphyria b) Amelogenesis imperfecta a) Toxicity of silver b) Toxicity of Mercury
c) Hutchinson's teeth c) Toxicity of Lead d) Toxicity of Tetracycline
d) Tetracycline staining of teeth (KAR -98)
(AP -01, AIPG -95) 17. The common site of melanoma on the orofacial skin is
6. Green stains which occur frequently in children are due to: a) Lower lip b) Malar region
a) Enamel deficiency b) Dentin deficiency c) Forehead d) Upper lip
c) Material alba d) Chromogenic bacteria (COMEDK-06}
(PG! -01, AIIMS -93) 18. Acquired, symmetric hyper pigmentation of the sun
7. Port wine stains are seen in: exposed skin of the face & neck which is strongly
a) Nevus b) Haemangioma associated with pregnancy & use of oral contraceptives
c) Melanoma d) All the above is called as
(AIIMS -96) a) Melanoma b) Ca.fe-au-lait-spots
8. A patient showing brownish pigmentation and with c) Freckle d) Melasma
normal laboratory findings may be suffering from: (COMEDK-14, AIPG-10}
a) Addison's disease b) Fibrous dysplasia 19. Melanin pigmentation in pregnancy is known as
c) Neuro fibromatosis d) None of the above a) Melasma b) Melanoma
(AIPG -02) c) Epulis d) Melanosis
9. Tetracycline stains appear as: (AIPG-10)
a) Yellow and brown stains in enamel and dentin 20. Pigmented lesions of oral mucosa do occur because of
b) Yellow and brown stains only in enamel the consumption of following drug EXCEPT
c) Yellow and brown stains only in dentin a) Nifedipine b) Zidovudine
d) Only yellow stain in enamel c) Minocycline d) Oral contraceptives
(PG! -2K) (MHCET-15)
10. In Peutz-Jeghers syndrome, the oral lesions are
a) Ulcerations of oral mucosa
b) Sebaceous glands of oral mucosa
c) Silver pigmentatio in of oral mucosa
d) None of the above
(AP -04)
11. Mucocutaneous circumoral pigmentation is found in:
a) Peutz-Jeghers syndrome
b} Plummer-Vinson syndrome
c) Lead poisoning

1) B 2) A 3} B 4) A 5) D 6) D 7) B 8) C 9) A 10) D 11) A 12) C 13} A


14) C 15) D 16) B 17) B 18) D 19) A 20) A
_____ r ORAL PATHOLOGY & MEDICINE

16. PIGMENTATION - ANSWERS


1. 'B' [Burket's 11th ed 123/ 10th ed 134] 12. 'C' [Shafer 7th ed 770/ 6th ed 764]
Hemochromatosis is a brown heme associated lesion In erythroblastosis fetalis there is deposition of bilirubin
characterized by brown to gray diffuse macules that occurs pigment in both enamel and dentin giving green, blue or
in palate and gingiva. The pigmentation is due to iron brown hue to the teeth.
deposition in the submucosa.
13. 'A' [Shafer 7th ed 558/ 6th ed 557]
2. 'A' [Burket's 11th ed 112/ 10th ed 132]
14. 'C' [Burket's 11th ed 120]
3. 'B' [Shafer 7th ed 560-61/ 6th ed 560] "Cafe-au-lait" pigmentation is seen in neurofibro-matosis or
Tetracycline can cross the placental barrier and causes von recklinghausen's disease of skin and in polyostotic form
discoloration of both permanent and deciduous dentition. of fibrous dysplasia.
TOOTH CRITICAL PERIOD
15. 'D'
4 months in utero to 3 months
Deciduous incisors
postpartum 16. 'B' [Shafer 7th ed 559/ 6th ed 556]
5 months in utero to 9 months
Deciduous canines
postpartum 17. 'B' [Burket 11th ed 113/ 10th ed 131]
Permanent incisors 3-5 months postpartum to 7 years of
and canines age 18. 'D' [Burket 11th ed 117/ 10th ed 132, Fig. 6-7]
The term melasma or cloasma is used to describe the
pigmentary changes associated with pregnancy or ingestion
Pigmentation of the permanent teeth may develop, if of contraceptive hormones. In most cases, it is the
tetracyclines are given between the ages of combination of estrogen and progesterone that induces
(COMEDK- 2013) the pigment. Estrogen replacement therapy alone without
a) 5 and 7 years progesterone does not precipitate melasma. The affected
b) 0.2 and 5 years patients show generalized facial pigmentation.
c) 6 and 10 years
d) 10 and 12 years 19. 'A' [Burkefs 11th ed 117, 118]

4. 'A' [Shafer 7th ed 656/ 6th ed 655] 20. 'A' [Burket's 12th ed 857, 858, Table 6-5]
Drug induced melanosis is caused by the soft tissue deposition
5. 'D' [Shafer 7th ed 561/ 6th ed 560] of drug metabolites or complexes and is commonly seen with
the use of antimalarials, (chloroquine, hydroxychloroquine,
6. 'D' [Burkefs 10th ed. 127 Table 6-2] quinacrine), phenothiazines, (chlorpromazine), oral
contraceptives, cyclophosphamide, busulfan, minocycline,
7. 'B' [Shafer 7th ed 145/ Burket's 10th ed 127] zidovudine, azidothymidine etc. Option 'I( Nifedipine
causes gingival hyperplasia.
8. 'C' [Shafer 7th ed 656, 711, 202/ Burkefs 11th ed. 120]
Addisons disease is characterized by an increased ACTH level.
Fibrous dysplasia is characterized by elevation of alkaline
phosphatase levels. So the best option is Neurofibromatosis,
characterized by cafe-au-lait pigmentation without any
biochemical abnormality.

9. 'A' [Shafer 7th ed 560/ 6th ed 560]


Dentin is more heavily stained than enamel.

10. 'D' [Shafer 7th ed 23/ Burket's 10th ed 171]


In Peutz-Jeghers syndrome, circum oral pigmentation is
distinctive and is usually pathognomonic. Multiple focal
melanotic brown macules of < 0.5 cm in diameter that
are concentrated about the lips, perioral and lip freckling,
patchy brown oral mucosa[ pigmentation, and true polyps
of gastrointestinal mucosa are other features.

11. 'A' [Shafer 7th ed 23/ Burkefs 10th ed 133]


Dental 9'ul-6e � - - - - - -

17. DISEASES OF BLOOD


1. All of the following statements about idiopathic c) Platelet deficiency d) Vit C deficiency
thrombocytopenic purpura are true EXCEPT (AIPG -99)
a) It is associated with platelet-specific auto-antibodies 11. Paul -Bunnell test is positive in:
b) It causes a prolonged bleeding time a) Infectious mononucleosis
c) It is often controllable by immunosuppressive treatment b) Multiple myeloma
d) It causes more prolonged hemorrhage than hemophilia c) Malignant nerves d) Rubella
(MAN -94, AIIMS -93) (AIPG -94)
2. The oral findings in erythroblastosis fetalis include 12. A young patient is hospitalized with petechiae of oral
a) Dentinal dysplasia b) Hypoplastic teeth mucous membrane, marginal gingival hemorrhage and
c) Pigmented teeth d) All of the above with a platelet count of 45,000/ cc. The BT and Clot
(AIIMS -94,99} Retraction time are increased, RBC and TLC are normal.
3. The red blood cells in beta thalassemia are typically He is suffering from:
a) Macrocytic and normochromic a) Infectious mononucleosis
b) Microcytic and Hypochromic b) Thrombocytopenic purpura
c) Normocytic and hypochromic c) Leukemia d) Hemophilia
d) Normocytic and normochromic (AIPG -01)
(AIIMS -95} 13. Pinpoint hemorrhages of< 1cm diameter are known as
4. "Hair-on-end" appearance in a skull roentgenogram is a) Petechiae b) Ecchymoses
seen in : c) Purpura d) Pustules
a) Fibrous dysplasia b) Thalassemia (AP -02)
c) Garre's Osteomyelitis d) Pagets disease 14. Which of the following agents is of value in the
(KAR -98, AIIMS - 2012) postoperative care of the hemo philic patient?
5. Which of the following is not a finding in classical a) Vitamin K b) Monsel's solution
hemophilia (hemophilia A): c) Aminocaproic acid d) Factor 8 cryoprecipitate
a) Bleeding into soft tissues, muscles, and joints (AP -91)
b) Decreased factor VIII 15. Pernicious anemia is
c) Increase Prothrombin Time a) insufficient production of red cells
d) Increase Partial Thromboplastin Time b) improper maturation of red cells
(AIPG -01} c) can be corrected by tablets of folic acid alone
6. Neurological symptoms and premature graying of hair is d) can be corrected by iron supplement
associated with: (AIIMS-2K)
a) Folic acid deficiency b) Pernicious anaemia 16. Which of the following is not associated with
c) Plummer-Vinson syndrome haemorrhage?
d) Paterson-Kelly syndrome a) Ecchymosis b) Petechiae
(AIPG -96) c) Melanosis d) Purpura
7. Which of the following is seen in idiopathic (AIIMS -95)
thrombocytopenic purpura: 17. Necrotising ragged ulceration with no apparent
a) Thrombocytosis b) Increased prothrombin time inflammatory response is indicative of:
c) Increased bleeding time d) Increased clotting time a) Leucocytosis b) Polycythemia vera
(AIPG -02) c) Sickle cell anemia d) Agranulocytosis
8. Which of the following diseases is known as the "kissing (AIIMS-91, PGI-99)
disease": 18. Leucocytopenia is seen in
a) Acquired immunodeficiency syndrome (AIDS) a) influenza b) agranulocytosis
b) Infectious mononucleosis c) liver cirrhosis d) all of the above
c) Primary syphilis (AIPG-02)
d) Recurrent apthous stomatitis 19. Bleeding time is prolonged in:
(AIPG -94) a) Haemophilia b) Von Willebrand's disease
9. To prevent excessive bleeding during surgery a patient c) Henoch Schenolein purpura
with hemophilia A may be given: d) Telangiectasia
a) Whole blood b) Fresh frozen plasma (COMEDK -03,08; KAR -03)
c) Factor VIII concentrate d) Factor IX concentrate 20. Clinical features of infectious mononucleosis
(AIPG -01) a) Glandular involvement b) Febrile
10. Hemophilia B is due to: c) Palatine petechiae d) All of the above
a) Factor VII deficiency b) Factor IX deficiency (AIPG-02)

1) D 2) C 3) B 4) B 5) C 6) B 7) C 8) B 9) C 10) B 11) A 12) B 13) A


14) C 15) B 16) C 17) D 18) D 19) B 20) D
21. Petechial hemorrhage is seen in
r ORAL PATHOLOGY & MEDICINE
d) It is usually of the lymphoblastic variety
a) cyclic neutropeni'a b) agranulocytosis (AIIMS -93)
c) pernicious anemia d) thrombocytopenic purpura 32. Which of the following is the most serious and life­
(AIIMS -2K) threatening blood dyscrasias caused with a drug:
22. Christmas disease is due to deficiency of a) Aplastic anemia b) Megaloblastic anemia
a) Hageman Factor b) Platelets c) Thrombocytopenia d) Hemolytic anemia
c) Plasma thromboplastin antecedent (AIPG -01)
d) Plasma thromboplastin component 33. Monospot test is used to diagnose:
(AIIMS -2K) a) Pernicious anemia b) Sickle cell anemia
23. A boy complains of bleeding gums, swollen, Joints with c) Infectious mononucleosis
hemorrhage into joints. His paternal and maternal uncle d) Leukemia
complains of same problem. It is due to deficiency of (AIPG -01)
factor: 34. Precancerous potential in Plummer- Vinson's syndrome
a) VIII b) IX may be due to change in the epithelium like:
c) X d) VI a) Atrophy b) Hypertrophy
(AIIMS -97) c) Acanthosis d) All the above
24. Common oral change seen with nutritional anemia is: (KAR -97)
a) Enlarged tongue b) Atrophic glossitis 35. Infectious mononucleosis has:
c) Generalized osteolysis d) Focal marrow expansion a) Multiple draining sinuses
(PGI-99, 01) b) Ulcers which bruise easily
25. Hypopigmentation, gray streaks of hair, degranulation c) Palatal perforation d) Alveolar bone loss
defect of neutrophils and neuropathy are seen in (PGI -2K}
a) aleukemic leukemia 36. Chemotherapy can be successful during treatment of:
b) chronic granulocytic leukemia a) Ameloblastoma b) leukemia
c) lazy leukocyte syndrome d) Chediak higashi syndrome c) Fibrosarcoma d) Basal cell carcinoma
(KAR -97, 99) (AIPG -90)
26. Chronic granulocytic leukemia is due to: 37. Virus responsible for infectious mononucleosis is
a) Chromosomal deletion b) Chromosomal mutation a) RNA paramyxo virus b) Varicella zoster virus
c) Chromosomal translocation c) Epstein Barr virus d) Coxsackie virus A 16
d) None of the above (KAR-04)
(AIPG -99) 38. Oral manifestations of infectious mononucleosis is most
27. Bleeding joints is a characteristic feature of commonly:
a) Vit -C deficiency b) Heamophilia a) Bluish red spots opposite maxillary molar
c) Vit-K deficiency d) Thrombocytopenia b) Pseudomembrane on gingiva
(AP -98) c) Pinpoint petechiae on the palate
28. Plummer- Vinson syndrome: d) Gingival hyperplasia
a) Is due to folic acid deficiency (AIPG-95)
b) Common in males 39. Megaloblastic anaemia occurs due to
c) Not associated with oral premalignancy a) Iron deficiency b) Folate deficiency
d) Strong association with post-cricoid carcinoma c) Vitamin C deficiency d) Protein deficiency
(PGI -01, AP -05) (COMEDK-04)
29. Patient giving history of thrombocytopenic purpura 40. One of the following syndrome is characterized by an
reports for extraction. What could be the most common esophageal web with resulting dysphagia, atrophic
postoperative complication? changes in the mucous membranes of the mouth and a
a) Oedema b) Haemorrhage hypochromic microcytic anemia
c) Infection spreading through tissue spaces a) Marfan's syndrome b) Plummer-vinson syndrome
d) Dry socket c) Meckels syndrome d) Sjogren's syndrome
(AIIMS -93) (TNPSC-99)
30. For extraction in a leukemic patient: 41. Aplastic anaemia is common with
a) Consult physician b) Obtain WBC count a) Chloramphenicol b) Cephalosporin
c) Obtain platelet count d) All of the above c) Tetracycline d) Penicillin
(AIPG -93) (TNPSC-99)
31. All of the following statements about acute leukemia in 42. Hemophilia is associated with:
children are true except: a) Normal bleeding time normal clotting time
a) It characteristica.lly causes gross gingival swelling b) Normal bleeding time prolonged clotting time
b) It may be manifested by mucosa[ pallor c) Prolonged bleeding time normal clotting time
c) It can cause obvious purpura d) Prolonged bleeding time prolonged clotting time
(AIPG-96)

21) D 22) D 23) A 24) 8 25) D 26) C 27) 8 28) D 29) 8 30) D 31) A 32) A 33) C
34) A 35) 8 36) 8 37) C 38) C 39) 8 40) 8 41) A 42) 8
Dental 9'ul-6e
43. Which of the following is sex linked disorder? 53. The most reliable criteria in Gustafson's method of
a) Thalassemia b) Klinefelter's syndrome identification is
c) Hemophilia d) Neurofibromatosis a) Cementum apposition b) Transparency of root
(COMEDK -04) c) Attrition d) Root resorption
44. A hair on end' appearance of the skull is seen in all of (AIPG-05)
the following expect 54. Which of the following blood disease has a racial
a) Thalassemia b) Sickle anemia predilection?
c) Cooley's anaemia d) Pagets disease a) Purpura b) Hemophilia
(KAR-98) c) Polycythemia d) Thalassemia
45. A patient on warfarin sodium following myocardial (COMED K-10)
infarction reports for an oral surgical procedure. Which 55. All the following are TRUE in Immune thrombocytopenic
one of the following laboratory tests should be preferred Purpura (ITP} EXCEPT
to ascertain the fitness a) Chronic ITP commonly occur in adult women
a) Prothrombin time b) Tourniquet time b) Associated with normal bleeding time
c) Clotting time d) Bleeding ti me c) Prothrombin Time (PT) & Partial Thromboplastin Time
(UPSC-01} (PTT) are normal
46. Which one of the following is not true about thalassemia? d) Increased megakaryocytes in bone marrow
a) Increase in number of globulin chain (AP-09)
b) There is erythrocyte fragility and hemolysis 56. Chediak- Higashi syndrome is inherited as
c) Hypochromic microcytic anaemia is present a) X-linked dominant trait b) Autosomal dominant
d) There is severe anaemia and thrombocytopenia c) Autosomal recessive d) X-linked recessive
(AIPG-96) (COMEDK-10)
47. Erythroblastosis fetalis can be prevented if the mother is 57. Commonest mode of inheritance of Von Willebrand's
injected at parturition, with an antibody called disease is
a) Blocking antibody b) Rh (D) immunoglobulin a) Codominant b) Autosomal dominant
c) Antilymphocyte globulin c) Autosomal recessive d) X-linked recessive
d) Antithymocyte serum (COMEDK-09)
(COMEDK-06) 58. Patient with increased PT, APTT & TT and normal
48. Deficiency of all the three components of coagulation fibrinogen and platelet counts, diagnosis is?
factor VIII result in: a) DIC b) Fae. VIII def.
a) Von willebrand's disease c) Liver disease d) Vitamin K deficiency
b) Haemophilia - A (AIIMS MAY-13)
c) Parahemophilia d) Haemophilia - B 59. Agent responsi hie for increasing factor VIII activity in
(AP-OS) haemophilia is
49. Cooley's anemia is also known as a) Epsilon amino caproic acid
a) Erythroblastosis fetalis b) Aplatic anemia b) Tranexamic acid
c) Thalassemia d) Pernicious anemia c) Avitene
(COMEDK-06) d) Deamino D Arginine vasopressin
50. The most striking haematological finding in (COMEDK-14)
agranulocytosis is 60. A 24 year old male complained of recurrent attacks of sore
a) Decreased absolute neutrophil count throat since 2 years. The total leucocyte count was 3000/
b) Increased absolute eosinophil count µl. A differential count revealed severe neutropenia. The
c) Decreased absolute basophil count diagnosis is
d) Increased absolute monocyte count a) Subleukemicleukemia
(COMEDK-06) b) Agranulocytosis
51. The most common coagulation disorders haemophilia A c) Infectious mononucleosis
and Von Willebrand's disease are due to - d) Leukoerythroblasticanemia
a) Factor IX deficiency b) Vitamin K deficiency (COMEDK-15)
c) Factor X deficiency d) Factor VIII deficiency 61. Mycosis fungoides is a
(COMEDK-09) a) Papillitis b) Fungal infection
52. In Radionuclide imaging the most useful radio c) T cell lymphoma d) Paracytic infection
pharmaceutical for skeletal imaging is: (APPG-15)
a) Gallium 67 (67 Ga) 62. The deficiency of ADAMTS 13 enzyme is seen in
b) Technetium-99m (99m Tc-Sc) a) Idiopathic thrombocytopenic purpura
c) Technetium-99m (99m Tc) b) Thrombotic thrombocytopenic purpura
d) Technetium-99m linked to Methylene disphosonate (99m c) Hemolyhticuraemic syndrome
Tc-MDP) d) Von-Willebrand's disease
(AIPG-05) (COMEDK-15)

43} C 44} D 45) A 46) A 47) B 48} A 49} C 50) A 51} D 52} D 53) B 54) D 55) B
56} C 57) B 58} D 59} D 60} B 61} C 62} B
_____ r ORAL PATHOLOGY & MEDICINE

17. DISEASES OF BLOOD - ANSWERS


1. 'D' [Burket's 11th ed 418/ Shafer 6th ed 781-82] Infectious mononucleosis is caused by Epstein-Barr virus
Idiopathic or immune thrombocytopenia is due to antiplatelet and is transmitted through deep kissing or oral exchange of
antibodies and is characterized by spontaneous hemorrhagic saliva. For this reason, it is called as "kissing disease"
lesions, epistaxis, malena or hematemesis and intracranial
hemorrhage that results in hemiplegia . The platelet count 9. 'C' [Burket's 11th ed 422/ 10th ed 464,465]
is below 60,000/mm3 causing prolonged bleeding time.
10. 'B' [Shafer 7th ed 792/ 6th ed 785]
2. 'C' [Shafer 7th ed 770/ 6th ed 763-64] Disease Clotting factor deficiency
In erythroblastosis fetalis, there is deposition of bilirubin
pigment in the enamel and dentin of developing teeth Plasma thromboplastinogen
Hemophilia A
giving them a green, brown or blue hue. (anti Hemophilic globulin) or
(Classic hemophilia)
factor VIII
In some cases enamel hypoplasia involving the incisal edges Hemophilia B Plasma thromboplastin
of anterior teeth and middle 3 rd of deciduous cuspid and (CHRTSTMAS disease) component (PTC) or factor IX
pt molar may be seen resulting in a characteristic ring like Plasma thromboplastin
defect known as "Rh hump". Hemophilia C
Antecedent (PTA) or Factor XI
No, treatment for tooth pigmentation is necessary since it Von wille brand's disease
Deficiency of von Willebrand
affects the deciduous teeth and presents only a temporary (Pseudohemophilia or
factor
cosmetic problem. vascular hemophilia)

3. 'B' [Shafer 7th ed 768/ 6th ed 761] 11. 'K [Shafer 7th ed 781/ 6th ed 774]
a and p Thalassemia are due to deficient synthesis of a - In Infectious mononucleosis the titer of agglutinin and
chain and p - chain respectively. hemolysins in blood against sheep red blood cells is raised
from 1:8 to 1:4096. this is known as positive "Paul Bunnel
Macrocytic Pernicious anemia. Test". Agglutination of horse RBCs on exposure to EBV
Anemias due to infections and heterophilic antibodies (monospot test) is highly specific
Microcytic
inflammations for infectious mononucleosis.
Hypochromic
Iron deficiency anemia and thalassemia. 12. 'B' [Shafer 7th ed 788-89/ 6th ed 781-82]
microcytic
Normocytic Anemias due to hemorrhage, hemolysis etc. 13. 'A' [Burket's 11th ed 42/ 10th ed 51)

4. 'B' [Shafer 7th ed 768/ 6th ed 761-62] 14. 'C' [Burket's 11th ed 422/ 10th ed 471]
"Crew - cut" or "Hair - on - end" appearance of skull is
characteristic of both sickle cell anemia and thalassemia. 15. 'B' [Shafer 7th ed 762/ 6th ed 756-57)
In thalassemia radiographs of maxilla and mandible shows Pernicious anemia is due to failure of secretion of "Intrinsic
characteristic apparent coarsening of some trabeculae and factor" which is responsible for intestinal absorption of
the blurring and disappearance of others, resulting in "salt "extrinsic factor" (vitamin B 12 ) or "Erythrocyte- maturing
and pepper" effect. factor". Tear drop or pear-shaped erythrocytes are
characteristic of pernicious anemia.
Other important features of thalassemia:
• The presence of fessas inclusion bodies in erythrocytes 16. 'C' [Burket's 11th ed 108 Table 1/ 10th ed 132]
that lead to increased hemolysis.
• Rodent facies with mangoloid features. 17. 'D' [Shafer 7th ed 776/ 6th ed 769]
• Rib-within-a-rib radiological feature Agranulocytosis involves W.B.C, and the W.B.C count is
below 2000 cells /mm3 • All the systems in the body are
'C' [Shafer 7th ed 792 table 18-13/ infected due to low W.B.C count. There is rapid destruction
5.
Shafer 6th ed 766]
of gingiva and supporting structures.

6. 'B' [Shafer 7th ed 7 63/ 6th ed 755-56]


The most characteristic feature of the disease is presence
Option 'D' Paterson-Kelly syndrome is another name of of infection in the oral cavity, GIT, genitourinary tract,
plummer-vinson syndrome. respiratory tract and skin. The lesions appear as ragged
necrotic ulcers covered by a gray or even black membrane.
7. 'C' [Burket's 11th ed 418/ Shafer 6th ed 782]
There is no apparent inflammatory cell infiltration around
the periphery of the lesions.
8. 'B' [Shafer 7th ed 781/ 6th ed 774]
18. 'D' [Shafer 7th ed 775 Tab 18-4/Shafer 6th ed 768)
Dental 9'ul-6e
19. 'B' [Shafer 7th ed 793/ 6th ed 786) 32. 'A' [Shafer 7th ed 766/ 6th ed 759)
Von Wille brand's disease or pseudohemophilia is a Aplastic anemia is characterized by general lack of bone
characterized by normal platelet count, normal clotting marrow activity. It affects the R.B.C, W.B.C and platelets
time, normal serum fibrinogen and normal prothrombin resulting in pancytopenia.
time. Only bleeding time is prolonged
33. 'C' [Burket's 9th ed 665/ Shafer 7th ed 781]
IN option 'C' Henoch Schenolein purpura, non­
thrombocytopenic purpuric lesions are seen due to 34. 'A' [Shafer 7th ed 772/ 6th ed 764-65)
dermal leukocytoclastic vasculitis with IgA in vessel walls Integrity of epithelium depends on adequate serum iron
associated with joint pain and swelling, colic, and occurs levels. Iron deficiency is characterized by atrophy of
characteristically in young children. epithelium with a predisposition for the development of
carcinoma in the upper alimentary tract. The incidence of
vWD is classified into three types all intraoral carcinoma in women is high because of high
• Type 1: Characterized by a partial quantitative decrease incidence of preexisting plummer-vinson syndrome.
of vWF and factor VIII.
• Type 2: Characterized by qualitative defects of vWF The characteristic triad of Plummer-Vinson syndrome
include:
• Type 3: Most severe and rarest. Characterized by marked
• Iron deficiency anemia
deficiencies of both vWF and factor VIII.
• Koilonychia (spoon shaped nails)
20. 'D' [Shafer 7th ed 781/ 6th ed 774] • Dysphagia due to eosophageal webs/strictures.

21. 'D' (Shafer 7th ed 788/ 6th ed 785] The other features or lemon tinted palor of skin and
Petechiae on palate in the form of tiny grouped clusters of predisposition for the development of carcinoma in the
red spots are seen in thrombocytopenic purpura. upper alimentary tract (post-cricoid carcinoma).

22. 'D' [Shafer 7th ed 792/ 6th ed 774] 35. 'B' [Shafer 7th ed 781/ 6th ed 774-75)

23. 'A' (Shafer 7th ed 792/ 6th ed 785-86] 36. 'B' [Shafer 7th ed 786/ 6th ed 779)
In Hemophilia the defect is carried by X-chromosome and
is transmitted as sex-linked mendelian recessive trait; 37. 'C' (Shafer 7th ed 787/ 6th ed 774]
thus hemophilia occUJrs in males only. The occurrence of
hemophilia is theoretically possible in homozygous female. 38. 'C' (Shafer 7th ed 781/ 6th ed 774]
Petechial hemorrhages at the junction of soft and hard
24. 'B' [Shafer 7th ed 763/ 6th ed 756) palate is early manifestation of infectious mononucleosis.
Addison's or Pernicious anemia characteristically shows
glossitis, glossodynia, glossopyrosis and atrophy of tongue 39. 'B' [Burket's 11th ed 394-95/ 10th ed 436)
papilla resulting in "bald tongue", referred to as "Hunter's Vitamin B 12 deficiency and folic acid deficiency are the major
glossitis or Moeller's glossitis" and is similar to "bald causes of megaloblastic anemia.
tongue of sandwith" seen in pellagra.
Megaloblastic anemia along with neurological signs
25. 'D' [Shafer 7th ed 778/ 6th ed 772] caused by vitamin B 12 deficiency do not improve on giving
Chediak-Higashi syndrome is disease of leukocytes folate supplements but improve on giving vitamine B12
characterized by lymphomas, nystagmus, oculocutaneous supplements.
albinisms, photopholbia and recurrent infections. The
patients exhibit abnormal granules in the peripheral 40. 'B' [Shafer 7th ed 772/ 6th ed 764-65)
circulating leukocytes, which are hallmark of the syndrome.
41. 'A' [Shafer 7th ed 776/ 6th ed 759)
26. 'C' [Burket's 10th ed 445)
42. 'B' [Shafer 7th ed 793/ 6th ed 785)
27. 'B' [Shafer 7th ed 792/ 6th ed 785) Prolonged bleeding time and normal clotting time are
seen in "thrombocytopenic purpura" and "von Willebrand's
28. 'D' (Shafer 7th ed 771/ 6th ed 764-65] disease"

29. 'B' [Shafer 7th ed 788/ 6th ed 781) 43. 'C' (Shafer 7th ed 791/ 6th ed 785]

30. 'D' [Burket's 11th ed 400-403/ 10th ed 444,446] 44. 'D' (Shafer 7th ed 768 for option "A'
769 for option "B'
31. 'A' [Shafer 7th ed 785/ Burket's 10th ed 443) 733 for option "D']
Gingival hyperplasia is common with AML, when compared
to ALL. 45. 'A' [Burket's 11th ed 336/10th ed 463)
46.
r
'A' [Shafer 7th ed 767/ 6th ed 760-61]
ORAL PATHOLOGY & MEDICINE
54. 'D' [Shafer 7th ed 767/ 6th ed 760)
Thalassemia (cooley's anemia or Mediterranean anemia or
47. 'B' [Shafer 7th ed 771/ 6th ed 764] erythroblastic anemia) is seen primarily in Mediterranean
populations, in races bordering the eastern Mediterranean
48. 'A' [Shafer 7th ed 753/ 6th ed 785] sea or in families originating from these areas. (Thalassa
means "Sea" in Greek)
49. 'C' [Shafer 7th ed 769/ 6th ed 760]
Also known as Biermer anemia or 55. 'B' [Burket 11th ed 418/ Shafer 6th ed 783]
Pernicious Thrombocytopenia is a disease in which there is an abnormal
Addisonian anemia or lederar anemia
anaemia reduction in the number of circulating blood platelets. When
or B 12 deficiency anemia.
this occurs, the patient develops focal hemorrhages into
Cooley's anemia or Mediterranean various tissues and organs, including the skin and mucous
Thalassemia
anemia or erythroblastic anemia membranes.
Plummer Vinson
Patterson-Brown Kelly syndrome
syndrome In the thrombocytopenia, the platelet count is usually below
10,000 platelets per cubic millimeter. As a consequence,
50. 'A' [Shafer 7th ed 776/ 6th ed 767] the bleeding time is prolonged, often to one hour or
more. The coagulation time, prothrombin time and partial
51. 'D' [Shafer 7th ed 792-93/ 6th ed 785] thromboplastin time are normal.
Hemophilia (Bleeder's disease or the disease of kings) is due
to deficiency of Factory 8. Von Willebrand disease is due to 56. 'C' [Shafer 7th ed 778/ 6th ed 772]
an abnormality; either quantitative or qualitative, of the Chediak-Higashi syndrome is an autosomal recessive
VWF, which is a large multimeric glycoprotein that functions immunodeficiency disorder characterized by abnormal
as the carrier protein for factors 8. VWF also is required intracellular protein transport. The disease is characterized
for normal platelet adhesion. It functions in both primary by immune deficiency, easy bruisability and bleeding as a
(involving platelet adhesion) and secondary (involving result of deficient platelet dense bodies; recurrent infections
factor VIII) hemostasis. with neutropenia, and abnormal natural killer (NK) cell
function. Ulcerations of the oral mucosa, severe gingivitis
52. 'D' [White & Pharoah 6th ed 218/ Burket 10th ed 44) and glossitis, and periodontal breakdown due to defective
leukocyte function are important oral manifestations.
53. 'B' [Shafer 7th ed 894/ 6th ed 885]
Gustafson method is a method for age estimation based on 57. 'B' [Shafer 7th ed 793/ 6th ed 786]
morphological and histological changes of the teeth. This Modes of inheritance of various blood disorders
method assess regressive changes like:
• Voh Willebrand's disease
• Amount of occlusal attrition (A)
• Vascular hemophilia Autosomal dominant
• Coronal secondary dentine deposition (S) • Disfibrinogenemia
• Loss of periodontal attachment (P)
• Chediak-Higashi syndrome
• Cementum apposition at the root apex (C) • Familial thrombesthenia
• Resorption at the apex (R) (Qualitative defect in platelets)
• Dentine translucency (T) • Fibrinogen deficiency Autosomal recessive
• Factor 5, 7, 10, 12 deficiency
For each of these aggressive changes or variables, different • Fibrin stabilizing factor
scores ranging from 0-3 were assigned. This meant attrition deficiency
could have any one of four scores (AO, Al, A2, A3) and
Hemophilia A & B Sex-linked recessive
similar scores for the other variables. Adding the allotted
score for each variable (Eg: A2 +51 + P2 +C2+R2 +Tl= X), Mendelian dominant,
Sickle cell anemia
a total score was obtained. It was found that an increase in non-gender linked
the total score (X) corresponds to an increase in age. Age X-linked genetic
was estimated using the formula: Age= 11.43 +4.56X. Wiskott-Aldrich syndrome
defect
PTA deficiency Incomplete recessive
Maples and Rice found that there was a miscalculation in
the above formula, and proposed a correction: Age = 13.45
58. 'D' [Burket 11th ed 414, 421]
+ 4.26X. However, the improvements made by Johanson are
Clinical tests to evaluate primary hemostasis are platelet
widely accepted. Instead of the original four grades (0 - 3),
count and bleeding time. Tests to evaluate the status of other
he proposed seven grades (0, 0.5, 1, 1.5, 2, 2.5, and 3).
aspects of hemostasis are prothrombin time/ International
Using these seven grades, the formula age was calculated.
normalised ratio, activated partial thromboplastin time,
Johanson recognized Dentin translucency or root
thrombin time, Fibrin degradation products, specific
translucency is the best and extensively used parameter for
coagulation assays and coagulation factor inhibitor
determining the age among Gustafson's six criteria. As the
screening tests.
age increases, translucency of dentin increases.
Dental 9'ul-6e � - - - - - -

Vitamin K deficiency is associated with the production of


poorly functioning vit K dependent factors II, VII, IX, X. A
rapid fall in factor Vii leads to an initial elevation in INR
and a subsequent prolongation of aPTT. Liver disease causes
impaired protein synthesis, which reduces the synthesis of
important factors and inhibitors of clotting and fibrinolytic
system. Acute or chronic hepatocellular disease displays
decreased Vit K dependent factor levels and abnormal
fibrinogen molecules.

59. 'D' [Burket 11th ed 422)


Therapy for haemophilia A & B is usually replacement
therapy. Commercially prepared factors VIII, IX complex
concentrates desmopressin acetate (DDAVP - de amino-8-
arginine vasopressin) and to a lesser extent, cryoprecipitate
& fresh frozen plasma are the replacement options.

60. 'B' (Shafer 7th ed 77 4]


Agranulocytosis is ch.aracterised by decreasing number of
circulating granulocytes, with WBC count less than 2000
cells/mm3, characterised by high fever with chills and sore
throat. Presence of infection is the most characteristic
feature.

61. 'C' [Shafer 7th ed 179, Table 2-17)


Mycosis fungoides, also known as Alibert-Bazin syndrome
or granuloma fungoides, is the most common form of
cutaneous T-cell lymphoma. It generally affects the skin,
but may progress internally over time. Symptoms include
rash, tumors, skin lesions, and itchy skin.

62. 'B' [Robbins pathology 9th ed 660)


Thrombotic thrombocytopenic purpura (TTP or Moschcowitz
syndrome) is a rare disorder of the blood-coagulation
system, causing extensive microscopic clots in the small
blood vessels throughout the body. TTP is characterised
by pentad of fever, thrombocytopenia, microangiopathic
hemolytic anemia, transient neurologic deficits and renal
failure.

Most cases of TTP arise from inhibition of the enzyme


ADAMTS13 (also known as von Willebrand factor-cleaving
protease), a metalloprotease responsible for cleaving large
multimers of von Willebrand factor into smaller units. In
its absence, these multimers accumulate in plasma and
tend to promote platelet activation and aggregation.
Superimposition of endothelial cell injury (caused by other
conditions) may further promote the formation of platelet
aggregates, thus initiating exacerbating clinically evident
TTP.
_____ r ORAL PATHOLOGY & MEDICINE

18. MISCELLANEOUS
1. Forensic Identification utilizes giving one month history of using any new teething gel
a) Lip prints b) Lip Schutz bodies available in market. The child is suffering from.
c) Lip pits d) Lip reading a) Acrodynia b) Pemphigus vulgaris
(KCET-08) c) Epidermolysis Bullosa d) Erosive lichen planus
2. Strength of collagen is due to: (AIIMS-08)
a) Hydroxy glycine b) Glycine 11. Endocarditis prophylaxis is recommended during
c) Proline d) Hydroxyproline following dental procedures except:
(AIIMS -07) a) Dental extractions
3. Tubular (Canalicular) adenoma occurs on: b) Initial placement of orthodontic brackets
a) Upper lip b) Palate c) Intracanal endodontic treatment
c) Lower lip d) Gingiva d) Periodontal procedures
(MCET-07) (IGNOU-10)
4. Bruxism is characterized by: 12. Characteristic 'Tram - Line' calcifications in skull
a) Increased mobility of the teeth radiographs is observed in:
b) Radiographic widening of the pdl a) Cleidocranial dysostoses b) Sturge - weber syndrome
c) Morning pain in muscles d) All the above c) Paget's disease d) Mc Cline - Albright syndrome
(MCET-07) (IGNOU-10)
5. Alveolitis sicca dolorosa is otherwise known as 13. Disease characterized by insidious onset and by an
a) Trigeminal neurallgia b) Sicca Syndrome absolute increase in number of circulating RBCs and in
c) Dry socket d) Myospherulosis total blood volume?
(KAR-02) a) Leukopenia b) Osler's disease
6. Strawberry gingivitis is seen in: c) Mediterranean disease d) Aplastic anaemia
a) Wegener's Granulomatosis (AIPG-09)
b) Scorbutic Gingivitis 14. Perception of taste even in absence of stimuli is known as
c) Plasma Cell Gingivitis d) Leukemic Gingivitis a) Ageusia b) Oysguesia
(MCET-07) c) Cocoguesia d) Phantoguesia
7. Radiographs of a 40 year old female revealed radiolucent (AIIMS-09)
areas around several of her mandibular teeth, all which 15. Biopsy of a clinically suspicious lesion is negative. The
tested vital on the electric pulp tester. These areas most appropriate treatment is
represent. a) Tell patient no malignancy
a) Multiple granulomas b) Periapical osteotibroses b) Repeat the biopsy
c) Chronic periapical abscesses c) Observe the patient for twelve months
d) Bone hypoplasias associated with opalescent teeth d) Observe the patient for thre,e months
(AIIMS-09) (AIIMS-09)
8. What is the term for the radio opaque area found at the 16. Enzymes which play an important role in calcification are
root apex of young permanent teeth involved with a a) Enolase & calcitonin
chronic pulpitis? b) Alkaline phosphatase & catalase
a) Apical cyst c) Alkaline phosphatase & pyrophosphatase
b) Apical condensing osteitis d) Pyrophosphatase & carbonic anhydrase
c) Chronic apical periodontitis (COMEDK-10)
d) Stage one apical osteotibroses 17. Cardiac condition requiring Antibiotic Prophylaxis for
(AIIMS-08) Infective Endocarditis?
9. A radiograph of the mandibular anterior teeth in a a) Coronary Heart Disease b) Rheumatic Heart Disease
patient reveals radiolucencies above the apices of right c) Cardiac Pacemakers d) Hypertensive Heart Disease
lateral and central incisors. No restorations or cavities (PGI-08)
are present. There is no pain or swelling and the pulps 18. Antischkow cells are present in all of the following
are vital. The diagnosis is conditions except
a) Periapical granuloma b) Cementoblastoma a) Sickle cell anaemia b) Iron deficiency anaemia
c) Radicular cyst d) Chronic abscess c) Apthous ulcer d) herpes simplex
(AIIMS-08) (AIIMS-09, AIIMS MAY- 2012)
10. A 9 year old child's mother comes to dental clinic with the 19. Which of the following is a virus induced epithelial
complaint of oral ulceration, fever and shedding of skin hyperplasia?
of palms and soles; she is giving history of premature a) Molluscum contagiosum b) Focal epithelial hyperplasia
shedding of teeth and increased sweating, she is also

1) A 2) D 3) A 4) D 5) C 6) A 7) B 8) B 9) B 10) A 11) B 12) B 13) B


14) D 15) B 16) C 17) B 18) D 19) D
,� -+-1'rv-l Dental 9'ul-6e
c) Squamous papilloma d) All of the above 29. For primary herpes simplex, the diagnosis is made by
(KCET-10) which gene
20. Difference between epithelium of oral cavity and cavity a) Culture with giemsa stain
lining of cyst is? b) Culture with wright strain
a) Stratum corneurn b) Stratum lucidum c) Routine cytology
c) Stratum germinativum d) Stratum spinosum d) Flourescent stain for cytology
(AIPG-09) (AIIMS-2011, 2012)
21. Differential diagnosis of hypercementosis includes all of 30. HLA-B27 histocompatibility antigen is seen in
these EXCEPT - a) Sjogren's disease b) Ankylosing spondylitis
a) Cementa[ dysplasia b) Cemental aplasia c) Felty's syndrome d) Scleroderma
c) Condensing osteitis (COMED-2012)
d) Focal periapical osteopetrosis 31. Tooth discolouration due to high bilirubin secretion is
(KCET-09) seen in the
22. Which is the most preferred route for drug administration a) Pink tooth of mummery
in the management of chronic pain b) Ochronosis
a) Intrathecal b) Oral c) Chlorodontia d) Leong teeth
c) Subderrnal d) Intravenous (AIPG-2012)
(AIIMS-09) 32. Burning tongue might not be associated with
23. Feature of acanthosis nigricans is a) Ranula b) Diabetes rnellitus
a) Insulinoma, obesity & cutaneous hypopigmentation c) Pernicious anaemia d) Local irritation
b) Insulin resistance, obesity, cutaneous hyperpigmentation (AIIMS-09)
c) Thickening of spinous layer, insulin resistance, obesity 33. Which of the following is a non-destructive method of
d) Thickening of spinous layer, insulin resistance, lean age estimation in adults?
(AIPG-10) a) Assessment of root dentin translucency
24. The sensitive period for tetracycline induced b) Amino acid racernisation
discolouration in the permanent maxillary mandibular c) Evaluation of tooth cementum annulations
incisors and canines is - d) Pulp to tooth ratio of canines
a) 3 months postpartum to 7th year of life (KAR- 2013)
b) 4 months in utero to 3 months postpartum 34. Herpetic Whitlow is seen in
c) 5 months in utero to 9 months postpartum a) Fingers b) Lymph nodes
d) Birth to 7th year c) Bone d) Tongue
(KCET-09) (KCET-10)
25. Perimolysis is - 35. In biopsy true about formalin as fixative is all except
a) Tooth wear due to gastric secretion a)
To prevent autolysis
b) Tooth wear due to bruxism b)
To make tissue rigid
c) Tooth wear due to dentifrices c)
To kill micro organisms d) 2% formaline is used
d) Peripheral blood cell destruction (AIIMS MAY-13)
(COMEDK-09) 36. To identify bite mark in case of child abuse, which
26. Which is a T cell tum or? impression material should be used to record bite marks
a) Burkitt's lymphoma b) Mycosis fungiodes a) Alginate
c) Mantel cell leukemia d) Hairy cell leukemia b) Impression plaster
(AIPG-10) c) Polyvinyl siloxane d) Silicone putty
27. Out of Syphilitic glossitis, Plummer Vinson syndrome, (AIIMS MAY-13)
Mikulicz's syndrome and hepatitis A; which of these 37. Saliva buffer system having low importance in stimulated
predispose to squamous cell carcinoma saliva
a) Syphilitic glossitis and plummer Vinson syndrome a) Arnino acids b) Carbonic acids
b) Syphilitic glossitis and Mikulicz's syndrome c) Phosphates d) None
c) Plummer Vinson disease and hepatitis A (AIIMS NOV-13)
d) hepatitis A and Mi kulicz's Syndrome 38. Antibiotic prophylaxis is not recommended in
(AIIMS-09) a) Making of impressions b) Dental extraction
28. The disorder characterized by craniosynostoses, c) Replantation of tooth d) Gingivectomy
craniofacial anomalies, severe symmetrical syndactyly (COMEDK-14)
(cutaneous and bony fusion) of hands and feet along with 39. Koilocytes are seen in all of the following EXCEPT
preaxial syndactyly and variable soft form syndactyly? a) Condylomaacuminatum
a) carpenter syndrome b) Crounzon syndrome b) Actinic keratosis
c) Apert syndrome d) Down's syndrome c) Verruca vulgaris
(AIPG-2011) d) Squamous papilloma
(MCET-14)

20) C 21) B 22) B 23) B 24) A 25) A 26) B 27) A 28) A 29) D 30) B 31) C 32) A
33) D 34) A 35) D 36) C 37) A 38) A 39) B
40.
r ORAL PATHOLOGY & MEDICINE
A patient who is on regimen of steroid therapy and has 50. A technique used to map a gene responsible for a trait to
no need for extraction of chronically infected teeth a specific location on a chromosome is
requires premedication with? a) Familial aggregation b) Genome wide analysis
a) Atropine to reduce the hazard for vagal stimulation and c) Segregation analysis d) Linkage analysis
cardiac arrest
b) Antihypertensive to combat tendency towards shock 51. The cytoprotective agent used for the prevention of
c) Antihistaminic to prevent allergy xerostomia in patients undergoing head and neck
d) Antibiotics radiotherapy is
(AIPG-14) a) Valacyclovir b) Amifostine
41. In bone biopsy decalcification and fixation is done c) Glycopyrrolate d) Naproxen
simultaneously by? (KERALA-2015)
a) Nitric acid and formalin b) Formic acid and Formalin 52. Which one of the following is a metric dental feature?
c) EDTA and Formic acid d) Nitric acid and EDTA a) Shovelling b) Caravelli's trait
(AIPG-14) c) Mandibular molar groove pattern
42. Amino acid that is high in root dentin used for age d) Root length of maxillary canine
estimation is (KERALA-2015)
a) Tyrosine b) Serine
c) Aspartic acid d) Glycine
(KERALA-2015)
43. Isotope in the enamel used for age estimation is
a) Strontium 84 b) Iodine 131
c) Carbon 14 d) Phosphorus 32
(KERALA-2015)
44. Oral dyskinesia may be caused by
a) Complete loss of teeth
b) TMJ ankylosis
c) Herpes simplex infection
d) Pemphigus vulgaris
(KERALA-2015)
45. The study of tooth prints is also called as
a) Dermatoglyphics b) Ameloglyphics
c) Dentinoglyphics d) Cementoglyphics
(KERALA-2015)
46. A critical factor in the regulation of craniofacial
development
a) Sonic hedgehog (SHH)
b) Fibroblast Growth factor (FGFz)
c) Wingless Drosophila melanogaster segment - Polarity
gene (Wnt)
d) Bone Morphogenic Protein (BMP)
(COMEDK-15)
47. Dental profiling is also called as
a) Comparative identification
b) Reconstructive identification
c) Positive identification
d) Oral autopsy
(KERALA-2015)
48. The diagnostic procedure of applying pressure to a
suspected lesion to visualize the evacuation of coloration
of the lesion is
a) Fluoroscopy b) Diascopy
c) Arthroscopy d) Endoscopy
(APPG-15)
49. Mouth opening is considered to be restricted when the
distance is
a) Less than 40 mm b) Less than 50 mm
c) Less than 53 mm d) Less than 58 mm
(APPG-15)

40) D 41) B 42) C 43) C 44) A 45) B 46) A 47) B 48) B 49) A 50) D 51) B 52) D
Dental 9'ul-6e � - - - - - -

18. MISCELLANEOUS - ANSWERS


1. 'A' [Shafer 7th ed 903/ 6th ed 896] Note: The codition that is confusing with periapical cemental
The lips contain wrinkles and grooves visible on lips as dysplasia (cementoblastoma) is Benign cementoblastoma,
"Sulci labiorum ruborum". The imprint produced by these which is a true neoplasm of functional cementoblasts which
grooves is termed as "lip print". Examination of lip print is form a large mass of cementum or cementum like tissue
known as "Cheiloscopy". surrounding the tooth root. The Benign cementoblastoma
occurs most frequently under the age of 25 years, with
2. 'D' [Shafer 7th ed 1005/ 6th ed 729) no significant gender predilection. The mandibular first
Hydroxyproline is a product of collegen breakdown. Due to permanent molar is the most frequently affected tooth.
stereochemical properties of pyrrotidine ring, hydroxyproline
gives added stability to collagen by the formation of extra 10. 'A' [Shafer 7th ed 559/ 6th ed 557-58)
hydrogen bonds.
11. 'B' [Burket's 11th ed 32)
3. 'A' [Shafer 7th ed 232/ 6th ed 228) A) Dental procedures that require endocarditis
The canalicular adenoma is a rare neoplasm of columnar prophylaxis include:
epithelial cells of salivary gland tissue. It originates primarily • All dental procedures that involve manipulation of
in the intraoral accessory salivary glands and in majority of
gingival tissue or the periapical region of teeth or
cases it occurs in the upper tip.
perforation of the oral mucosa.
4. 'D' [Shafer 7th ed 526/ 6th ed 524-25)
B) Dental procedures that do not require endocarditis
5. 'C' [Shafer 7th ed 601/ 6th ed 601) prophylaxis are:
Alveolitis sicca dolorosa, alveolalgia and postoperative osteitis • Routine anesthetic injections through non-infected
are other names for dry socket. tissue.

6. 'A' [Shafer 7th ed 674/ 6th ed 670) • Taking dental radiographs.


Wegener's graulomatosis is a disease of unknown etiology, which • Placement of removable prosthodontic or orthodontic
basically involves the vascular, renal and respiratory systems. appliances.
Oral involvement is very rare. In reported cases, involvement of
gingiva is most common resulting in characteristic strawberry • Adjustment of orthodontic appliances.
gingivitis. Oral biopsy specimens show pseudoepitheliomatous • Placement of orthodontic brackets.
hyperplasia and subepithelial abscesses.
• Shredding of deciduous teeth and bleeding from
7. 'B' [White & Pharoah 6th ed 432-33) trauma to lips or oral mucosa.
Option 'A' granulomas and Option 'C' chronic periapical
abscesses are associated with non-vital teeth. 12. 'B' [Shafer 7th ed 150/ Burket's 9th ed 123)

8. 'B' [Shafer 7th ed 495/ 6th ed 494) 13. 'B' [Shafer 7th ed 773/6th ed 766)
Condensing osteitis or chronic focal sclerosing osteitis Option 'B' Osler's disease is other name polycythemia vera,
usually involves mandibular first molars in young adults which is chronic stem cell disorcler with an insidious onset
before 20 years of age. It is usually asymptomatic other characterized as a panhyperplastic malignant and neoplastic
than mild pain associated with an infected pulp. The marrow disorders. The most prominent feature is an absolute
periapical radiograph demonstrates the pathognomonic, well increase in the number of circulating RBCs and in the total blood
circumscribed radiopaque mass of sclerotic bone surrounding volume because of uncontrolled red blood cell production.
and extending below the apex of one or both roots.
14. 'D' [Ann Otol Rhinollaryngol 2012;121 (2):113-8)
9. 'B' [White & Pharoah 6th ed 43 2-34/ Shafer 6th ed 297- Phantogeusia is an spontaneous abnormal taste sensation
298) without any external stimulus. The patients with
Options A, C, and D are associated with non-vital teeth. phantogeusia have shown increased expression of T2R taste
Cementoblastoma (periapical osteofibrosis or periapical receptor genes in their tongues.
fibrous dysplasia or periapical cemental fibrous dysplasia)
usually involves people of over 20 years of age and women 15. 'B' [Shafer 7th ed 595)
appear to be affected for more than men.
16. 'C' [Current opinion in orthopedics 2007; 18: 444-48]
The lesion occurs in and near the periodontal ligament
around the apex of a tooth, usually a mandibular incisor. 17. 'B' [Burket's 9th ed 462)
In most cases, the lesions involve the apices of several
mandibular anterior teeth or bicuspids.
ORAL PATHOLOGY & MEDICINE 729
V
18. 'D' [Shafer 7th ed 669/ 6th ed 666] 29. 'D' [Burket 11th ed 45/ 10th ed 53]
Anitschkow cells are seen in: Cytology:
• Recurrent apthous ulcers For cytology, a fresh vesicle can be opened and a scraping
made from the base of the lesion and placed on a microscope
• Sickle cell disease
slide. The slide may be strained with Giemsa, Wright's, or
• Megaloblastic anemias Papanicolaou's strain and searched for multinucleated giant
• Iron-deficiency anemia cells, syncytium, and ballooning degeneration of the nucleus.
• Children receiving chemotherapy for cancer Flouroscent staining of cytology smears has been shown to
be more sensitive (83%) compared with routine cytology
• Even in normal people
(54%); it is the cytologic test of choice, when available.
19. 'D' [Burket's 11th ed 137-139]
30. 'B' [Shafer's 7th ed 250, 839/ 5th ed 247, 831]
Sjogren's disease is associated with HLA- DR3, HLA- B8.
20. 'C'
Scleroderma is associated with HLA- B8, H LA- DRS, HLA -
DR3, HLA-DR52, HLA-DQB2. Felty's syndrome is characterized
21. 'B' [Shafer 6th ed 474 for Option 'A'
by rheumatoid arthritis, enlarged spleen and neutropenia.
494 for Option 'C'
The cause of Felty's syndrome is not known.
699 for Option 'D']
31. 'C' [Oral and maxillofacial pathology by Nevillee 2nd ed
22. 'B' [Burket's 11th ed 266]
65]
Staining of the teeth in children has been well documented
23. 'B' [Shafer 7th ed 323/ 6th ed 814]
from various causes - the most recognized being tetracycline
Acanthosis is an unusual dermatosis of hyperpigmentation.
administration. Green dental pigmentation (ch lorodontia)
The tongue and lips appear to be most frequently involved.
is relatively uncommon. Hyperbilirubinemia causes
There is hypertrophy of the filliform papillae producing a
accumulation of bilirubin pigment within the skin and
shaggy, papillomatous surface to the dorsal tongue. In
mucous membranes that manifests as yellow discoloration.
majority of patients, acanthosis nigricans is associated with
In the newborn, jaundice becomes apparent on the face
obesity and insulin resistance.
at a serum bilirubin level of about 5 mg/dL; it progresses
caudally as the level increases.
24. 'A' [Shafer 7th ed 560/ 6th ed 560]
Ref. Q.No. 3 in Chapter 'Pigmentation'.
When bilirubin levels are elevated for several months
bilirubin pigments deposit throughout the body, includin�
25. 'A' [Check Explanatin Below]
the teeth. In the soft tissues, the pigment is removed
Acidic dental erosion is irreversible loss in tooth structure
over time. However, in the hard dental tissues, the
due to chemical by acids not of bacterial origin. Dental
pigment becomes trapped because of the lack of metabolic
erosion can occur from extrinsic or intrinsic sources. Various
activity and thus results in permanent discoloration.
extrinsic sources include fruit juices, carbonated drinks, etc.
Histological evaluation of green-stained deciduous teeth
(CHLORODONTIA) from patients with hyperbilirubinemia
Intrinsic dental erosion is known as perimolysis, whereby
has shown bilirubin deposits. The other reason for green
gastric acid from the stomach comes into contact with
teeth is tetracycline staining.
the teeth. People with diseases such as anorexia nervosa
bulimia and gastroesophageal reflux disease often suffe;
Alkaptonuria is a metabolic disorder characterized by a
from perimolysis.
triad of homogentisic aciduria, arthritis and ochronosis
(Option A). It is due to the deficiency of the enzyme
26. 'B' [Shafer 7th ed 179 Table 2-17/ 6th ed 174 Table
homogentisic acid oxidase which catalyzes the conversion
2-16]
of homogentisic acid to maleylacetoacetic acid in the
Mycosis fungoides is also known as cutaneous T-cell
catabolism of tyrosine leading to brown discoloration of
lymphoma.
permanent dentition, buccal mucosa and in the nails or the
skin, giving these areas a dusty coloration.
27. 'A' [Shafer 7th ed 330, 771/ 6th ed 109 & 765]
32. 'Pr [Shafer 7th ed 856/ 6th ed 847]
28. 'A' [Ghai Essential paediatrics 602]
Etiology of glossodynia (painful tongue) and
Carpenter syndrome:
glossopyrosis (burning tongue) include
It is genetically determined disorder with autosomal
recessive inheritance. In this craniosynostosis is associated • Deficiency states such as pernicious anemia and pellagra
with mental retardation and preaxial polysyndactyly of the • Diabetes mellitus
feet. Soft tissue syndactyly of hands is also present. Patella • Gastric disturbances such as hyperacidity or hypoacidity
is displaced. These children have short suture, are obese and • Psychogenic factors
often suffer from congenital heart disease. • Trigeminal neuralgia
• Periodontal disease
• Xerostomia
Dental 9'ul-6e
• Hypothyroidism buffered with a pH as low as 5 .3 in unstimulated saliva,
• Referred pain from abscessed teeth or tonsils whereas it becomes well buffered at high flow rates (during
• Angioneurotic edema stimulation) to a pH as high as 7 .8.
• Mercurailism
38. 'A' [Burket 11th ed 340)
• Moeller's glossitis
All dental procedures that involve manipulation of gingival
• Oral habits such as excessive use of tobacco, spices, etc. tissue or the periapical region of teeth or perforation of the
• Antibiotic therapy oral mucosa need antibiotic prophylaxis to prevent Bacterial
• Local dental cause.s such as dentures, irritating clasps or endocarditis. Procedures such as anesthetic injections
new fixed bridges through non infected areas, taking dental radiographs,
• TMJ disturbances placement of removable prosthodontic/orthodontic
• Electrogalvanic discharge occurring between dissimilar appliances, adjustment of ortho appliances, placement of
metallic dental restorations. brackets, shedding of deciduous teeth, bleeding from trauma
to lips or oral mucosa do not need antibiotic prophylaxis.
Burning sensation of tongue can be seen in all except
a) Diabetes mellitus b) Pernicious anemia Cardiac conditions requiring prophylaxis include prosthetic
c) Ranula d) Local irritation cardiac valve, previous infective endocarditis, congenital
(AIPG-14) heart disease, cardiac transplantation recipients who
develop cardiac valvulopathy. They may be given oral
33. 'D' (Shafer 7th ed 894-96/ 6th ed 887, 888) Amoxicillin 2 gm/ Azithromycin 500mg/ Cephalexin 2gm/
Root dentin translucency, aminoacid racemisation and Clindamycin 600mg (OR) Ampicillin 2 gm IM or IV/ Cefazolin
evaluation of cementum annulations necessitate tooth or Ceftriaxone 1 gm IM or IV/ Clindamycin phosphate 600mg
extraction, sectioning and destruction. A non destructive IM or IV.
method of age estimation includes Kvaals radiographic
technique that measures six teeth on IOPAs including pulp­ 39. 'B' [Burket 11th ed 85)
tooth length, pulp tooth length, pulp -root width at CEJ and Koilocytosis is a feature of viral infections, characterised by
midpoint etc. edematous epithelial cells and pyknotic nuclei. The complex
chromatin arrangements may mirror viral replication in the
34. 'A' [Shafer 6th ed 336) nuclei of koilocytic epithelial cells. Actinic keratosis is
Herpetic Whitlow is a painful infection of herpes simplex caused due to exposure to UV light, hence koilocytes are
and typically affects the fingers or thumbs. Herpes Whitlow not found.
can be caused by HSV-1 or HSV-2. Herpetic Whitlow lesions
are commonly seen in dental workers and medical workers 40. 'D' [Burket 11th ed 527)
exposed to oral secretions. It is also observed in thumb­ Patients on long term moderate to high dose glucocorticoid
sucking children with primary HSV-1 oral infection (Auto therapy are considered to be immunocompromised and more
inoculation). susceptible to infections. Antil>iotic coverage should be
considered for dentoalveolar infections or any scheduled oral
35. 'D' (Shafer 7th ed 594) surgery, but this decision should be based on the underlying
The fixative used for biopsy specimens is either 10% formalin oral conditions and not solely on glucocorticoid therapy.
or 70% alcohol. Formalin fixes the tissues and makes
them suitable for further processing. The goal of fixation 41. 'B' [Shafer 7th ed 936)
is to block all lytic enzyme activity as well as activity of The most common solutions used for hard tissue processing
bacteria or other infectious agents in order to preserve the (bone, tooth) include 10-30% formalin, glutaraldehyde,
constituents of a tissue as they were in the living state. paraformaldehyde and alcohol based solutions. The standard
solution used is 10% neutral buffered formalin.
36. 'C' [Shafer 7th ed 901)
Bite mark analysis should include careful visual examination 42. 'C' (Shafer 7th ed 896)
to assess the type of injury, orient the injury, assess the • Dental age estimation is done by different methods
colour, size, shape of the bite, contour, texture, elasticity for different age groups - prenatal, neonatal and early
of the bite site etc. Cllose-up photographs should be made, postnatal period; children & adolescents; adults.
followed by collection of saliva with a swab (saliva may
have WBC, sloughed epithelial cells - source of DNA). Then • Age estimation in children & adolescents is done by
impressions of the bite area should be made. The material stages of tooth calcification by Schour & massler's
of choice is Vinyl Polysiloxane. The impression material may method; Demirjan's method.
be reinforced with dental stone, autopolymerizing acrylic or • Adult age estimation is done by Gustafson's method,
impression compound to prevent dimensional change. dentin translucency, incremental lines of cementum and
pulp- tooth area ratio. Biochemical indicators of age
3 7. 'A' [Shafer 7th ed 434] include aminoacid racemisation and 14C levels.
In saliva, the chief l>uffers are bicarbonate carbonic acid • Relationship between extent of aspartic acid
and phosphate. Variations in bicarbonate concentration racemisation in dentine and age estimation had been
are the chief determinants of salivary pH. Saliva is poorly suggested by Helfman and Ba.da.
r ORAL PATHOLOGY & MEDICINE
• Age estimation lby 14C levels is done as there is no 50. 'D' [Carranza 11th ed 275]
turnover of enamel after it is laid down and 14C levels Linkage analysis is a technique used to map a single
reflects that in the atmosphere at the time of enamel gene responsible for a trait to a specific location on a
formation. Data for 14C levels are available for previous chromosome. This technique has been highly successful
60 years, which can be used as a reference. in finding molecular defects for simple genetic diseases
(caused by mutation in a single gene), such as cystic fibrosis
43. 'C' (Shafer 7th ed 896) and Hunington's disease.
Check Q. No 43
51. 'B' [Burket 12th ed 42, 11th ed 180)
44. 'A' [Shafer 7th ed 860) Amifostine is a radioprotective agent that has indications
0rofacial dyskinesia is thought to arise as an extrapyramidal for both acute and chronic radiation induced xerostomia.
disorder or complication of phenothiazine therapy. It is The usual adult dose is 200mg/m2 as a 3 min IV infusion
most common in edentulous patients wearing upper 15-30 min before each sitting of radiotherapy. It is a
and lower dentures in gross malocclusion, who exhibit sulfhydryl compound that acts through scavenging free
involuntary movements. There is either a marked diminution radicals generated in tissues exposed to radiation and
or total disappearance of symptoms when dentures with promotes repair of damaged DNA. It had been shown to
proper physiologic craniomandibular relationships were protect mucosa, cardiac tissue, renal tissue, bone marrow,
constructed. 0rofacial dyskinesia is characterised by severe, salivary glands and prevents oto-and-neurotoxicity. There is
involuntary, dystonic movements of facial, oral & cervical reduced uptake of amifostine in tumor and there is no tumor
musculature like lip- smacking, lip- licking, pouting, protection.
protrusion of tongue & mandible with uncoordinated
movements etc. 52. 'D' [Shafer 6th ed 880)
Dental features are broadly categorised as metric (tooth size)
45. 'B' (Shafer 7th ed 884) and non-metric (tooth shape). Metric features are based
Tooth prints are the pattern formed by the enamel rod on measurements, and non-metric in terms of presence or
ends at the crown surface of the tooth and are called as absence of a particular feature, Eg., whether carabelli's cusp
ameloglyphics. is present or not.

46. 'A' [Check Explanation Below] More than 30 non-metric features of the tooth crown
Sonic hedgehog is a protein that in humans is encoded by and root have been described by Scott and Turner. A few
the SHH ("sonic hedgehog") gene. SHH plays very important examples are:
role in organogenesis and most importantly craniofacial • Shovelling
development.
• carabelli's trait
4 7. 'B' [Shafer 7th ed 880) • Three cusped maxillary second molar
Dental identification is of 2 forms - comparative • Mandibular molar groove pattern
identification (attempts conclusive identification by
comparing dead individual's teeth with dental records of
presumed individual) or reconstructive identification or
dental profiling (attempts to elicit the population affinity
or race, sex, age and occupation of the dead individual).
Dental profiling is undertaken when virtually no clue exists
about the identity of the decedent.

48. 'B' [Burket 12th ed 157)


• Diascopy is the technique of applying pressure to a
suspected vascular lesion to visualize the evacuation of
colouration and may facilitate the differentiation of a
small, vascular lesion from a pigmented lesion.
• Dermascopy, also known as epiluminescence microscopy,
is another clinical test useful in the diagnosis of
melanocytic lesions. This non-invasive technique
is performed through the use of handheld surface
microscope and may be used in the evaluation of labial
or anterior lingual pigmentation.

49. 'A' [Burket 12th ed 286)


Normal maximum mouth opening is > 40mm. (Average
maximum mouth opening recorded in males is 52.8mm for
men and 48.3 mm for women, 44.8 mm in children aged 6
years). Normal lateral and protrusive movements are> 7mm.
Dental 9'ul-6e � - - - - - -

ORAL PATHOLOGY & MEDICINE - SYNOPSIS


I. SYNDROMES:
SYNDROME IMPORTANT FEATURES
1. DEVELOPMENTAL DISTURBANCES
PARRY ROMBERG SYNDROME Facial hemiatrophy
VANDER WOUDE'S SYNDROME Pits of lower lip and cleft of palate
ASCHER'S SYNDROME Acquired double lip+ Blepharochalasis+ Non toxic thyroid enlargement.
ORO FACIAL DIGITAL SYNDROME Cleft tongue and clefting of mandibular alveolar process.
MEDIAN CLEFT FACE SYNDROME Hypertelorism+ Median cleft of premaxilla, palate+ Cranium bifidurn occultum.
MEISCHER'S SYNDROME Cheilitis granulomatosa.
MELKERSON-ROSENTHAL SYNDROME Cheilitis granulomatosa+ Facial paralysis+ Scrotal tongue
PEUTZ-JEGHERS SYNDROME Multiple intestinal polyposis+ Pigmentation on face, oral cavity and hands.
BECKWITH'S HYPOGLYCEMIC SYNDROME OR Macroglossia+ Neonatal hypoglycemia+ Microcephaly + Fetal viscerornegaly
BECKWITH-WIEDEMAN SYNDROME
Talons cusp+ developmental retardation + broad thumbs+ great toes+ delayed
RUBINSTEIN-TAYBI SYNDROME
or incomplete descent of testes.
KLENEFELTER SYNDROME (Trisomy 46) Taurodontism, XXY genetic constitution.
Multiple polyposis of large intestine + osteomas of bone + multiple sebaceous
GARDNER'S SYNDROME
cysts+ impacted supernumerary teeth.
• Lacrimal apparatus involvement with lacrimal sac inflammation and lacrimal
gland aplasia
LADD syndrome • Auricles are deformed with the ear having a cup shaped appearance
• Dental involvement like peg shaped teeth, hypodontia, and enamel hypoplasia
• Digital deformities like deviation of fingers medially or laterally (clinodactyly)
2. BENIGN AND MALIGNANT TUMORS
Oral papillomatous lesions+ Facial trichilemmomas associated with GIT, thyroid,
COWDEN'S SYNDROME
CNS abnormalities.
B-K MOLE SYNDROME Large pigmented and high risk for development of melanoma.
MEN-I (MULTIPLE ENDOCRINE NEOPLASIA Hyperplasia of pituitary, parathyroid, adrenal cortex, pancreatic islets.
SYNDROME)
MENS-II (SIPPLE SYNDROME) Parathyroid hypoplasia+ Pheochromocytoma + medullary carcinoma of thyroid.
3. SALIVARY GLAND TUMORS
SJOGRENS SYNDROME Keratoconjunctivitis sicca+ Xerostomia+ Rheumatoid arthritis.
4. CYSTS AND TUMORS OF ODONTOGENIC ORIGIN
GORLIN-GOLTZ SYNDROME or JAW CYST - CELL Multiple odontogenic keratocysts + basal cell carcinoma + bifid basal rib +
NEVUS - BIFID RIB SYNDROME neurologic+ ophthalmologic + sexual abnormalities.
5. BACTERIAL, VIRAL AND MYCOTIC INFECTIONS
HEERFORDT SYNDROME Uveoparotid fever.
Behcets syndrome is characterized by triad of recurrent oral ulcers, recurrent
BEHCET'S SYNDROME
genital ulcers and ocular inflammation
REITERS SYNDROME (COMEDK-10) Tetrad of Urethritis+ arthritis+ conjunctivitis+ oral ulcers
Herpes zoster infection of the geniculate ganglion, with involvement of the
RAMSAY HUNT'S SYNDROME
external ear and oral mucosa
6. DISEASES OF BONES AND JOINTS
CROUZON SYNDROME/ Prognathic mandible, hypoplastic maxilla, high arched palate, OR parrot beak
CRANIOFACIAL DYSOSTOSIS appearance and hypertelorism.
ORAL PATHOLOGY & MEDICINE SYNOPSIS

Syndactyly involving hands and feet (known as mitten hands and sock feet)
APERT SYNDROME
with features similar to Crouzon syndrome.
Hypoplasia of mandible and malar bone, macrostomia, OR malformation of
TREACHER-COLLINS SYNDROME/ MANDIBULOFACIAL external ear, high arched palate, bird or fish like face and antimongoloid slant
DYSOSTOSIS (FRANSCHETTI SYNDROME) with coloboma of lower eyelids.

PIERRE ROBIN SYNDROME Cleft palate+ Micrognathia+ Glossoptosis


Long thin extremities, hyper extensibility of joints, spidery fingers,
MARFAN SYNDROME arachnodactyly, bifid uvula and CVS complications.
Hypermobility, macroglossia, flat face, large anterior fontanelle, sexual
DOWN SYNDROME (Trisomy 21, ongolism) underdevelopment and cardiac abnormalities.
VAN BUCHEM SYNDROME Generalised cortical hyperostosis.
GORHAM SYNDROME Massive osteolysis or vanishing bone.
ALBRIGHT SYNDROME Precocious puberty+ Polyostotic fibrous dysplasia+ (afe-au-lait pigmentation.
Infantile hyperostosis, dysphagia, hyperirritability, increased alkaline
CAFFEY-SILVERMAN SYNDROME
phosphatase.
COSTEN SYNDROME Tinnitus, otalgia, headache, dizziness, burning tongue and throat.
Masticatory muscle tenderness + pain + limitation of motion + clicking or
MYOFACIAL PAIN DYSFUNCTION SYNDROME
propping noise.
7. DISEASES OF BLOOD
FANCONI SYNDROME Aplastic anaemia, microcephaly, hypogenitalism and olive brown pigmentation.
Characterised by triad of Iron deficiency anemia, carcinoma of hypopharynx
PLUMMER VINSON SYNDROME
(post cricoid carcinoma) and koilonychia (spoon shaped nails) .
ALDRICH SYNDROME Thrombocytopenic purpura, eczema and increased susceptibility to infection.
Malignant lymphomas, Albinism, nystagmus, recurrent infections, photophobia,
CHEDIAK-HIGASHI SYNDROME
gingivitis, glossitis.
KOSTMANN SYNDROME Severe congenital neutropinia.
8. DISEASES OF PERIODONTIUM
PAPILLON-LEFEVRE SYNDROME Juvenile periodontitis+ Palmar plantar keratosis+ Calcification of falxcerebri
9. SKIN DISEASES
Severe bullous form of erythema multiforme involving skin, eyes, oral cavity and
STEVEN-JOHNSON SYNDROME
genitalia.
Calcinosis cutis, Raynauds phenomenon, Esophageal dysfunction, Sclerodactyly
CREST SYNDROME
and Telangiectasia.
EHLER DANLOS SYNDROME Hyperextension of joints, hyper mobility of TMJ (Rubberman)
GOLTZ GORLIN SYNDROME (Focal dermal hypoplasia) Multi papillomas+ Atrophy of skin+ Polydactyly+ Sunken eye+ Sparse hair.
Characterised by triad of Lichen planus + vascular hypertension + diabetes
GRINSPAN SYNDROME mellitus.
10. DISEASES OF NERVES AND MUSCLES
READER'S SYNDROME (Paratrigeminal syndrome) Severe headache+ signs of ocular sympathetic paralysis.
AURICULOTEMPORAL SYNDROME (FREYS SYNDROME) Gustatory sweating.
HORNER'S SYNDROME Ptosis+ Miosis+ Anhidrosis+ Vasodilation
Ptosis + rapid elevation of the ptotic eyelid occurring on movement of the
JAW-WINKING SYNDROME
mandible to the contralateral side.
Tumors of nasopharynx producing pain similar to trigeminal neuralgia and
TROTTERS SYNDROME
middle ear deafness.
Dysphagia (pain during mandibular movement and the pain is subsided when the
EAGLES SYNDROME
jaws are closed), sore throat, otalgia and glossodynia.
FLOPPY INFANT SYNDROME Generalised weakness due to hypotonia, inability to sit, stand and walk.
Dental 9'ul-6e � - - ---

Partial / complete facial paralysis, drooling of saliva, difficulty in mastication


MOBIUS SYNDROME
and congenital facial diplegia.
HORTON'S SYNDROME Vascular headache.
11. ORAL ASPECTS OF METABOLIC DISEASE
CUSHING'S SYNDROME Hyperadrenocorticism (excessive secretion of ACTH hormone)
HURLERS SYNDROME OR GORGOYLISM Carbohydrate storage disease, Mucopolysaccharidosis (MPS)
HUNTER SYNDROME MPS-II
WATERHOUSE-FRID ERICHSEN SYNDROME Acute adrenal insufficiency in connection with acute septicemia

II. SOME CONFUSING AND IMPORTANT POINTS


• Seen in pemphigus, familial benign chronic pemphigus and recessive form of epidermolysis bullosa.
Nikolsky's sign
• Loss of epithelium due to rubbing resulting in raw, sensitive surface.
• s��n in µsuriasis.
Auspitz's sign
• If the deep scales are removed, one or more tiny bleeding points are disclosed.
• Seen in congenital syphilis [KCET-2012)
Higoumenaki's sign
• Characterized by irregular thickening of sternoclavicular portion of clavicle.
• Seen in tetany (Hypoparathyroidism)
Trousseau's sign • Trousseau's sign is the appearance of carpal spasm after application of pressure on the arm by inflation
of the sphygmomanometer cuff.
• Seen in tetany.
Chvostek's sign • Tapping at the angle of saw stimulates the facial nerve and causes twitching of the muscles of face on
the same side.
• Seen in subcondylar fractures and fractures involving base of skull
Battle's sign
• Characterized by ecchymosis in the post auricular region over the mastoid process.
• Seen in Lefort-I fracture.
Guiren's sign
• Seen as ecchymosis near greater palatine foramen.
• It is a useful feature indicative of basilar skull fracture.
Racoon sign • The condition is seen as periorbital ecchymosis from subconjunctival hemorrhage, which occurs secondary
to blood dissecting from the disrupted skull cortex to the soft tissue of the periorbital region
• Indicates orbital floor fracture.
"Hanging drop sign"
• It is due to herniation of inferior oblique and inferior rectus muscles in to antrum.
in radiograph
• Best demonstrated in waters projection.
Crowe's sign
• Von Recklinghausen's neurofibramatosis (Elephant Man Syndrome)
( Axillary Freckling)
Murphy's sign • Seen in cholecystitis.
• Used earlier as an indication of the start of nerve regeneration
Tinel's sign • It is elicited by percussion over the divided nerve, which results in tingling sensation in the part supplied
by the peripheral section
• Asboe-Hansen sign refers to the ability to enlarge a blister in the direction of the periphery by applying
Asboe-Hansen sign
mechanical pressure on the roof of the intact blister spreading of a blister into a clinically normal skin
(blisterspread sign)
when lateral pressure is applied on the edge of a blister
• also termed string of pearls or rosettes sign
Cluster of jewels sign • It refers to an early stage of chronic bullous disease of childhood when new lesions appear at the margin
of older ones resembling a cluster of jewels
Dubois' sign • shortening of the little finger associated with congenital syphilis
• Flag sign refers to horizontal alternating bands of discoloration in the hair shafts corresponding to
periods of normal and abnormal hair growth.
Flag sign • The flag sign may be seen in patients with nutritional deficiencies such as kwashiorkor and with certain
medications such as intermittent high dosage of methotrexate or following chemotherapy. Patients with
ulcerative colitis may also manifest the flag sign.
' ORAL PATHOLOGY & MEDICINE SYNOPSIS

• refers to an enanthem of red macules or petechiae confined to the soft palate in patients with rubella. The
Forchheimer's sign
sign presents in up to 20% of patients during the prodromal period or on the first day of the exanthema.
• It is seen in patients with Ehlers-Danlos syndrome. It is used to describe the ability to touch the tip of
Gorlin's sign
the nose with the extended tongue
• Hoagland's sign is early and transient bilateral upper lid edema occurring in patients with infectious
Hoagland's sign mononucleosis. The sign is usually present only for the first few days of the clinical presentation of the
illness
Hutchinson's nose • Hutchinson's nose sign refers to the presence of vesicles occurring on the tip of the nose in patients
sign with herpes zoster.
• refers to eyelid pigmentation occasionally seen in hyperthyroidism.
Jellinek's sign • The hyperpigmentation is secondary to increased corticotrophin levels and may also occur on other areas
of the face but usually spares the buccal mucosa
• refers to hyperpigmentation occurring on the neck owing to pellagra. The 'necklace' can extend as a
Necklace of Casal sign
broad collar-like band around the entire circumference of the neck

• It is seen in patients with bulimia nervosa owing to repeated contact of the incisor teeth with the skin
Russell's sign
and during self-induced vomiting

• It is seen in patients with dermatomyositis and is characterized by confluent, symmetric, macular


Shawl sign
violaceous erythema on the posterior shoulders and neck, giving a distinctive shawl-like appearanc

• Slapped cheek sign is seen in children with fifth disease (Erythema infectiosum by parvovirus B19} as
Slapped cheek sign
confluent, erythematous, edematous plaques on the cheeks.
Tin-tack sign • also known as carpet tack sign, is a useful clinical feature in diagnosing discoid lupus erythematosus

2.
Pink spot Internal resorption.
Koplik spot Initial lesions in measles (Rubeola)
Roth spot Subacute bacterial endocarditis, typhoid fever.
Bitot's spot Seen as white plaque in the conjunctiva of vitamin A deficient children.
Herald spot Primary lesions on skin seen in pitryiasis rosea.
Cafe-au-lait spots Brownish pigmentation seen in neurofibromatosis, fibrous dysplasia, and Peutz-jeghers syndrome.
Sore spots Traumatic ulcers from denture irritation.

3.
Shick test, Elek's test Diphtheria
Dick test, Scultz-Charlton test Scarlet fever
Schimmers test, Split lamp test, Rose Bengal test Eye (lacrimal gland function) tests for Sjogren's syndrome
Schillers test Carcinoma of cervix
Schillings test B12 deficiency
Rosewaller test Rheumatoid arthritis
Kveims test / Kveim Slitzbach test) Sarcoidosis (COMEDK-09}
Gordons biological test Hodgkin's disease
Tzanck test Pemphigus, Herpes simplex
Freis test Lymphogranuloma venereum
Figlus excretion test Folic acid absorption test
Monospot test, Paul Bunnel test Infectious mononucleosis
Rumpel leede phenomenon or Torniquet test Test for capillary fragility
Montoux test Tuberculosis
Dental 9'ul-6e � - - - - - -

Sweat test Cystic fibrosis


Prick skin test or scratch skin test Test for allergic skin sensitivities
ELISA, Western blot tests AIDS
Kahn, VDRL, TPI, FTA-Abs Syphilis
Weil-Felix test Rickettsial infection

4.
Different types of Cells PATHOLOGY
Downey Cells Kissing disease or infectious mononucleosis.
Raquet cells/ Ribbon cells (modified spindle cells) Rhabdomyosarcoma
Benign dyskeratotic cells (Corps, ronds and grains) Dyskeratosis follicularis
Safety pin cells (Normoblasts), Target cells Thalassemia
Hyaline cells (modified myoepithelial cells) Pleomorphic adenoma
• Herpes
Tzanck cells (multinucleated giant cells of epithelial origin)
• Pemphigus
• Sickle cell disease
Anitschow cells (modified epithelial cells with elongated nuclei • Apthous ulcer
and linear bar of chromatin with radiating process of chromatin) • Iron deficiency anemia
• Rhematic heart disease (AIIMS-09)
Arbiskov cells (modified monocytes) • Myeloblastoma
Angulate body cells • Granular cell myoblastoma
• Craniopharyngioma (Rathke's pouch tumour)
Ghost cells (Epithelial cells with eosinophilic cytoplasm and • Odontomas
without nucleus) • Ameloblastic fibrodontoma
• Calcifying odontogenic cyst (Gorlin- Gold cyst)
LE Cell (Consists of a rosette of neutrophils surrounding a pale
• Systemic lupus erythematosus
nuclear mass apparently derived from a lymphocyte )
Naevus cells • Pigmented mole
• Acinic cell carcinoma
Glycogen free clear cells
• Mucoepidermoid carcinoma
• Lateral periodontal cyst
Glycogen rich clear cell
• Gingival cysts of adults
• Hurler syndrome
Hurler cells or clear cells or Gorgoyle cells
• Mucopolysaccharidosis
Pericyte of Zimmer man • Glomus tumour
Acantholytic cells • Pemphigus
Reed-Sternberg cell or Dorothy Reed cell
• Hodgkin's lymphoma
(modified B-lymphocytes or macrophage-monocyte derivative)
• Histologic variants of Reed-Sternberg cells.
Lacunar cells
• Seen in Nodular sclerosis type of Hodgkin's lymphoma.
Pale cells and dark cells • Odontogenic myxoma.
• Pale call containing eccentric nucleus and a wrinkled or
Gaucher cells "crumpled silk" cytoplasm
• Seen in Gaucher's disease
• Lipid laden cells distributed through out reticuloendothelial
Niemann - pick disease system
• Seen in Niemann - pick disease
Lepra cells (vacuolated macrophages or vacuolated histiocytes) • Lepromatous leprosy.
ORAL PATHOLOGY & MEDICINE SYNOPSIS

Foam cells • Lipid laden cells seen in periapical granuloma.


• H PV altered epithelial cells with perinuclear clear spaces and
Koilocytes
nuclear pyknosis.
• A variety of lymphocytic and histiocytic cells seen in Nodular
Popcorn cells
lymphocyte-pred ominant type of Hodgkin's lymphoma
Touton- type multinucleated giant cells • Fibrous histiocytoma
Centrocytes and centroblasts • Follicular type of non-Hodgkin's Lymphoma

5. DIFFERENT TYPES OF INCLUSION BODIES


Verocay bodies (Hyaline in nature) Neurilemmoma / Schwannoma
Dohle bodies Chediak -Higashi syndrome
Asteroid bodies Sporotrichosis
Reilly bodies or metachromatic granules in lymphocytes Hurler syndrome
Howell-Jolly bodies and Cabot's rings Pernicious anemia
Henderson-Paterson inclusions or molluscum bodies
Molluscum contagiosum(Caused by pox virus group)
(eosinophilic intra cytoplasmic inclusion bodies)
Lipschutz bodies (eosinophilic intranuclear inclusions which tend
Primary herpetic stomatitis
to displace the nucleolus and nuclear chromatin peripherally)
• Vaccinia virus
Gaurnier bodies (intracytoplasmic)
• Small pox virus
Cow dry type A (intranuclear) Herpes
Cow dry type B (intranuclear) Poliomyelitis
Negri bodies Rabies
Pappenheimer bodies Sideroblastic anemia
Heinz bodies Seen in RBC of persons suffering from G/D deficiency
Russell bodies (immature plasma cells unable to produce Seen in chronic inflammatory diseases, multiple myeloma and
antibodies) periapical granuloma.
• Apical periodontal cyst
Rushton bodies • Infected dentigerous cyst
• Gingival cysts of neonate
Civatte / Hyaline / Colloid / Cytoid bodies Lichen planus
Psammoma bodies Papillary carcinoma of thyroid
Fessas bodies Thalassemia

6. SOME INCUBATION PERIODS


Diphtheria, cholera, typhoid 1 - 5 days
Influenza 1 - 3 days
Chicken pox, Rubella, Mumps 2 - 3 weeks
Measles (Rubeola) 10 days
Tetanus 3 -21 days
Hepatitis - A 2 - 7 weeks
Hepatitis - B 7 -23 weeks

7.
Test tube retepegs Dilantin hyperplasia
Saw tooth retepegs Lichen planus
Absence of retepegs Oral submucous fibrosis
Dental 9'ul-6e � - - - - - -

8.
Rhagades Congenital syphilis
Pseudorhagades Ectodermal dysplasia

9.
• Bells palsy
Mask like face • Scleroderma
• Long facies or adenoid facies syndrome
• Bilateral TMJ ankylosis
Bird face • Treacher Collins syndrome (mandibulofacial dysostosis)
• Pierre Robin syndrome
Fish face appearance • Treacher Collins syndrome
Sorrowful appearance • Myasthenia gravis
Herculean appearance • Congenital mytonia
• Craniofacial dysostosis
Parrot beak
• Congenital syphilis
Petrified man • Generalised myositis ossificans
Rubbery man • Ehler's danlos syndrome
Hebra nose • Rhinoscleroma
Swan neck • Dystrophic myotonia
• Scleroderma
Claw like hands • Leprosy
• Hurler syndrome/ Mucopolysaccharidosis
Angelic look/ Eyes looking towards heaven • Cherubism
Sailor's skin or farmer's skin • Solar elastosis
Coup-de sabre appearance • Scleroderma
Butterfly distribution of lesions • SLE
Bumpy lips • Type III multiple endocrine neoplasia syndrome (MEN)
Spidery fingers • Marfan's syndrome
Moon face, Buffalo hump • Cushing syndrome

10.
Pink tooth Internal resorption (UPSC-09)
Pink disease Mercury poisoning (Acrodynia or Swift's disease)

11.
Hunter's/ Moeller's tongue Pernicious anaemia
Bald tongue of sandwith Niacin deficiency (Pellagra)
Magenta or Beefy red tongue Riboflavin deficiency
Strawberry (early tongue change) and Raspberry (late change) Scarlet fever
Scrotal tongue Fissure tongue
Black hairy tongue Prolonged oral use of antibiotics
Cerebriform tongue A feature of pemphigus vegetans, characterized by a pattern of
sulci and gyri on the dorsum of tongue.

12.
Lock jaw Tetanus
Lumpy jaw Actinomycosis
African jaw lymphoma Burkitts lymphoma
ORAL PATHOLOGY & MEDICINE SYNOPSIS
13.
Migratory glossitis Geographic tongue
Atrophic glossitis Iron deficiency and pernicious anaemia
Leutic glossitis Syphilis

14.
Von Recklinghausen's disease of skin Neurofibromatosis
Von Recklinghausen's disease of bone Hyperparathyroidism

15.
Bull's eye lesions are clinically seen in Erythema multiformae
Bull's eye radiological feature is diagnostic of Lingual impaction
Bull teeth Taurodontism

16.
• Macrostomia
Mandibulofacial dysostosi s • Bird like or fish like face
(Treacher Collins syndrome or bronchial arch syndrome) • Blind fistulas between angles of mouth and the angles of ear
• Hypoplasia of mandible
• Hypoplasia of maxilla and mandibular prognathism.
• Hypertelorism
Craniofacial dysostosis (Crouzon syndrome)
• Parrots beak
• High arched palate

17.
Cortical thickening or hyperostosis due to
• Infantile cortical hyperostosis (Caffey's disease)
actively proliferating periosteum
• Osteopetrosis (Marble done disease/Albers -Schonberg disease)
The medullary cavities are reduced due to
• The density of spongiosa/medullary portion is increased and
replacement by bone and the cortex is thickened
sometimes the teeth are nearly invisible on the X-ray.
Presence of both osteoblastic and osteolytic areas giving
• Paget's disease (Osteitis deformans)
"Cotton wool" appearance
• Medullary bones are rarefied
• Irregular (-shaped or Chinese character shaped trabeculae • Fibrous dysplasia
• Woven fibre bone
Bone growth in between periosteum and cortex (supracortical
• Garre's osteomyelitis
and subperiosteal)

Centrifugal growth pattern of bone • Central ossifying fibroma


(The tumour grows by expansion equally in all directions • Central cementifying fibroma
resulting in a round tumour mass) • Central cemento-ossifying fibroma

18.
Gorlin-Goltz syndrome (Bifid rib syndrome) Multiple odontogenic keratocyst + Basal cell carcinoma + Bifid rib and other
abnormalities
Goltz-Gorlin syndrome (Focal dermal hypoplasia) Multiple papillomas + Atrophy of skin + Polydactyly + Sunken eye + Sparse hair
Gorlin -gold cyst Calcifying odontogenic cyst

19.
Mickulicz's apthae (Sutton's disease) Recurrent apthous major
Salivary gland neoplasm that exhibits both inflammatory and
Mickulicz's disease
neoplastic characteristics.
Dental 9'ul-6e � - - - - - -

20.
Brachycephalic with flat occipit Down syndrome (COMEDK -04)
Brachycephalic skull with narrow foramen magnum Achondroplasia
Dolicocephalic skull, leptoprosopic facial form, and spidery Marfan's syndrome
fi ngers/arachnodactyly (Note: - Orthodontic treatment is contraindicated in Marfan's
syndrome patients)

21.
Histocytosis X (Nonlipid reticulo endothelioses) Includes Letterer Siwe disease, Hand-Schuller Christian
disease and eosinophilic granuloma
Histocytosis Y (Benign tumour of connective tissue origin) Verruciform xanthoma

22.
Regular, punched out radio lucent lesions of bones Multiple myeloma (AIIMS-06)
Irregular, punched out radiolucent lesions Eosinophilic granuloma
Punched out lesions of interdental papilla ANUG

23.
Absence of cementum as a failure of cementogenesis Hypophosphatasia
Absence of cellular cementum with no increased thickening of primary acellular cementum Cleidocranial dysplasia

24.
Risus sardonicus Rigidity of facial muscles
Risus opisthotonus Rigidity of entire body muscles

25.
Stomatitis nicotiana (Pipe smoker's palate) Seen in heavy pipe smokers and is manifested as redness and inflammation of palate.
Stomatitis mediacamentosa Allergic reaction due to systemic administration of certain drugs.
Allergic reaction due to local application or contact of certain drugs.
Stomatitis venenata
Eg: Dentifrices, mouth rinses.
Stomatitis scarltina Oral manifestation of scarlet fever.

26.
• Median anterior maxillary cyst
• Globulomaxillary cyst.
Fissural cysts
• Median palatal cyst
(non-odontogenic inclusion/ developmental cysts)
• Naso alveolar cyst
• Palatal cysts of neonate / Epstein pearls / Bohn's nodules
• Dentigerous cyst
• Primordial cyst
• Periapical cyst
Odontogenic cysts
• O.K cyst
• Lateral periodontal cyst
• Calcifying odontogenic cyst
• Stafne's cyst
Pseudo cysts • Aneurysmal bone cyst
(Note:- Pseudohorn cysts are seen in Acanthosis nigricans.) • Mucocele-extravasation type
• Hemorrhagic or traumatic bone cyst

27.
Mouse eaten furrowed ulcer is seen in Oral tuberculosis lesion.
Moth eaten appearance is seen in Chronic osteomyelitis
ORAL PATHOLOGY & MEDICINE SYNOPSIS
28.
Tram line calcifications (due to bilamellar radiopaque tracks) are seen in Sturge weber syndrome (IGNOU-1O)
Tram line effect is diagnostic of CSF rhinorrhea
Pipe stem or tram-track appearance. Calcified deposits in artery wall.

29.
Grinspan Syndrome Triad of Diabetes mellitus, Lichen planus, and Hypertension
Greenspan lesion Other name for hairy leukoplakia

30.
• Epidemic infection caused by Coxsackie A16. Primarily affects children of age 6 months to 6 yrs.
Hand, foot and mouth disease
• A sore mouth and refusal to eat is one of the most common findings
A viral disease rarely affects man and is transmitted by use of milk of affected animals or contact
Foot- and mouth disease
with the infected animals directly.

31.
Thrombocytopenic purpura / Thrombocytopenia Abnormal reduction in no of circulating blood platelets
Uncommon form of Thrombocytopenic purpura characterized by thrombocytopenia,
Thrombotic thrombocytopenic purpura
hemolytic anemia, fever, transitory neurologic dysfunction and renal failure.
A group of rare diseases in which the patient manifests a bleeding tendency due
Thrombocytopathic purpu.ra to qualitative defects in platelets. The platelet count is normal, but the platelet
aggregation is defective. The bleeding time may be normal or prolonged
Variety of diseases characterized by qualitative defect in blood platelets. The
Thrombocytasthenia
bleeding time is prolonged and clot retraction is impaired. Platelet count is normal.
Thrombocythemia Characterized by abnormal increase in circulating platellets

32.
Inflammed patient's oral mucosa due to mechanical irritation, infection, or an allergic reaction Burning mouth sensation
• Clinically healthy oral mucosa, burning sensation of oral tissues, pain that starts in the
morning and become aggravated during the day. Burning mouth syndrome (BMS)
• Local, systemic and psychogenic etiology

III. DIFFERENT DISEASES WITH SIMILAR FEATURES 2. Tzank cells are seen in
• Herpes
1. • Pemphigus
Intraepithelial bulla Subepithelial bulla
3. Multinucleated giant cells are seen in
• Herpes simplex • Pemphigoid
• Cherubism
• Herpes zoster • Bullous pemphigoid • Hyperparathyroidism
• Chicken pox • Bullous lichenplanus • Aneurysmal bone cyst
• Pemphigus • Dermatitis herpetiformis • Giant cell granulomas
• Osteoclastoma
• Familial benign pemphigus • Epidermolysis bullosa
• Osteosarcoma
(Hailey-Hailey disease) • Skin lesions of erythema • Herpes
• Epidermolysis bullosa­ multiformae
• Leprosy (Langerhans giant cells)
dystrophic recessive form
• Eosinophilic granuloma
• Oral lesions of erythema
multiformae 4. Psoriasis form of lesions with Monro's abscess are seen in
Both intraepithelial & subepithelial • Benign migratory glossitis
• Erythema multiforme • Psoriasis
• Reiter's syndrome
• Epidermolysis bullosa
(Micro-abscess formation is also seen in Vitamin A
• Paraneoplastic pemphigus deficiency)
Dental 9'ul-6e
5. Ghost cells are seen in 15. Ground glass appearance is seen in
• 0dontoma • Primary hyperparathyroidism
• Craniopharyngioma • Monostotic fibrous dysplasia
• Ameloblastic fibroodontoma • Cherubism
• Calcifying epithelial odontogenic cyst
16. Widening of periodontal ligament is seen in
6. Bence Jones protein is diagnostic of • 0steosarcoma
• Multiple myeloma (important feature)
• Scleroderma
• Polycythemia vera
• Trauma from occlusion
• Leukemia
• Solitary myeloma
17. Hypercementosis is seen in
7. Blue sclera are seen in • Pagets disease
• 0steogenesis imperfecta • Hyperpituitarism (Gigantism)
• Marfan's syndrome
18. Obliteration of periodontal Ligament space is seen in
• Cherubism
• Hypercementosis
• Ehlers-Danlos syndrome
• 0steopetrosis • Ankylosis
• Fetal rickets • Hypofunction of teeth
• Paget's disease
8. Abnormal dentino-enamel junction is seen in
• Dentinogenesis imperfecta 19. "Cotton wool" appearance is seen in
• Ehlers-Danlos syndrome • Pagets disease (Osteitis deformans)
• Chronic sclerosing diffuse osteomyelitis
9. Rushton bodies are seen in • Fibrous dysplasia
• Periapical cyst • Cemento-osseous dysplasia
• Dentigerous cyst
• Neonatal cyst 20. Multi pie unerupted teeth are seen in
• Dentin dysplasia
10. Nikolsky's sign is seen in • Cleidocranial dysplasia
• Pemphigus • Hypothyroidism
• Familial benign chronic pemphigus • Cherubism
• Epidermolysis bullosa (dystrophic recessive)
Note: Both partial anodontia and supernumerary teeth are
11. Pseudoepitheliomatous hyperplasia is seen in seen in cleft lip & palate cases.
• Granular cell myoblastoma
• Blastomycosis 21. Cafe-au-lait pigmentation is seen
• Papillary hyperplasia • Neurofibroma
• DLE • Peutz-Jegher's syndrome
• Squamous cell carcinoma • Polyostotic fibrous dysplasia
• Hypothyroidism
12. Sunray or sunburst appearance is seen in
• 0steogenic sarcoma 22. Periosteal new bone formation is seen in
• Central hemangioma • Garre's osteomyelitis
• Ewing's sarcoma • Hypervitaminosis A
• Caffey's disease or infantile cortical hyperostosis
13. Onion peel appearance is seen in
• Syphilis
• Ewing's sarcoma
• Leukemia
• Garre's osteomyelitis
• Caffey's disease • Ewing's sarcoma

23. Centrifugal growth pattern of bone tumour is seen in


14. Crew-cut or Hair-on-end appearance is seen in
• Central cementifying fibroma
• Thalassemia
• Sickle cell anemia • Central ossifying fibroma
• Central cemento ossifying fibroma
r ORAL PATHOLOGY & MEDICINE SYNOPSIS
24. Hyper mobility of joints is seen in • Chediak-Higashi syndrome
• Ehlers Dantos syndrome • Letterer-Siwe disease
• Marfan's syndrome • Eosinophilic granuloma
• Osteogenesis imperfecta • Hand-Schuller Christian disease
• Downs syndrome • Hypophosphatasia
(Note:- Hypo mobility of joints in seen in Achondroplasia) • Dentin dysplasia
• Acrodynia (mercury poisoning)
25. Palmar-Plantar keratosis is seen in
• Cherubism
• Gorlin-Goltz syndrome
(Note: Premature exfoliation of primary teeth is the most
• Papillon-Lefevre syndrome common local cause of malocclusion. Prolonged retention
• Follicularis keratosis of deciduous teeth and subsequent delay in eruption of
succedaneous teeth is seen in Cleidocranial dysplasia.)
26. Russell bodies is seen in
• Plasma cells in multiple myeloma 34. Delayed eruption of teeth is seen in

• Periapical granuloma • Rickets


• Cretinism (hypothyroidism)
27. Koilonychia or spoon shaped nails are seen in • Cleidocranial dysplasia
• Plummer-Vinson syndrome (Iron deficiency anemia) • Hemifacial atrophy
• Chondro ectodermal dysplasia ( Ellis Van CreveId • Fibrous dysplasia
syndrome)
• Dentin dysplasia
28. Juxta epithelial hyalinization is seen in • Ghost teeth
• Oral submucous fibrosis • Pituitary dwarfism
• Ameloblastic fibrnma (Note :- early eruption of permanent teeth is seen in hyper
parathyroidism)
29. Microcyst formation is seen in
35. Causes of Xerostomia
• Acinic cell carcinoma
• Psychological cases like anxiety and depression
• Mucoepidermoid carcinoma
• Duct calculi
• Neurilemmoma
• Sialadenitis or inflammation of salivary glands
• Squamous odontogenic tumour
• Drugs like anticholinergic and sympathomimetics,
30. The tumours which are common in anterior maxillary tricyclic antidepressants, bronchodilators, diuretics and
region antihistamines.
• Compound odontoma • Zyban, a drug that aid in cessation of smoking also
found to cause xerostomia
• Adenomatoid odontogenic tumour
• Salivary gland aplasia
• Squamous odontogenic tumour
• Sjogrens syndrome
31. Keratin plugging formation is seen in • Ectodermal dysplasia
• Fordyce granules • Patients under radiotherapy
• Verrucous carcinoma • Other systemic disorders like polyurea, diabetes,
• Verrucous xanthoma parkinson's disease, cystic fibrosis and sarcoidosis etc.
• Keratoacanthoma
36. Epidermoid cysts are seen in
• DLE
• Gardner syndrome
32. Absence of lamina dura • Basal cell nevus syndrome
• Hyperparathyroidism (important feature) • Pachyonychia congenita
• Pagets disease
37. Snail track ulcers are seen in
• Vitamin D resistant rickets (Hypophosphatemia)
• Secondary syphilis
33. Premature exfoliation of teeth • Pyostomatitis vegetans
• Juvenile diabetes
• Juvenile periodontitis
• Papillon-Lefvre syndrome
Dental 9'ul-6e
IV. DENTAL CARIES 13. Heredity also plays an important role in the incidence of
dental caries.
1. THEORIES OF DENTAL CARIES
Acidogenic theory or 14. Pit and fissure caries spread can be visualized as a cone
MILLER with apex at the periphei:y and base towards DEJ. Thus
Chemico-Parasitic theory
there may be a large carious lesion with only a tiny point of
Chemical or acidic thoery ROBERTSON
opening causing undermining of enamel.
Parasitic theory DUBOS
Proteolytic theory GOTTILEB Smooth surface caries spread can be visualized as cone with
apex towards DEJ and base towards the outer surface.
Proteolysis chelation theory SCHARTZ AND MARTIN (KCET-09}
Sulfatase theory PINCUS 15. Acute dental caries occurs most frequently in children and
Sucrose chelation theory BURCH AND JACKSON young adults, as their dentinal tubules are large, open and
show no sclerosis.
2. Acidogenic or Millers Chemico-Parasitic theory:
It states that dental decay is a chemico-parasitic process Chronic dental caries progresses slowly and occurs mostly in
consisting of decalcification of hard tissues followed by adults. This is due to the sclerosis of dentinal tubules and
dissolution of the softened residue. deposition of secondary dentin in response to the irritation.
3. Proteolytic theory: 16. Arrested caries occurs mostly in large open occlusal cavities
It states that organic portion of the tooth, such as enamel in which the superficially softened and decalcified dentin
lamellae and enamel rod sheaths play an important role is gradually burnished. This is called as EBURNATION OF
in the carious process since they serve as a pathway for DENTIN.
microorganisms throU!gh the enamel.
17. Maxillary arch is more frequently involved by caries than
4. Schatz Proteolysis Chelation theory: the mandibular arch. The highest incidence of dental caries
Keratinolytic microorganisms cause breakdown of protein is with mandibular first molars. Lowest incidence of dental
and keratin. This results in the formation of substances caries is with mandibular incisors.
which form soluble chelates with the mineralized component
of the tooth and thereby decalcify the enamel. 18. THE ZONES OF DENTAL CARIES

5. Initiation of dental caries is caused by streptococcus ENAMEL caries (Beginning dentinally)


mutans and propagation by lactobacilli. Root surface caries
is caused by Actinomyces species. 'Translucent zone' lies at the advancing front of the
Zone 1
lesion. It is not always present.
6. S. mutans can metabolize dietary sucrose and synthesize 'Dark zone' lies adjacent and superficial to the Zone I.
Zone 2
dextran or glucan which is the glucose moiety of sucrose. This is the positive zone and is usually present.
Glucan is an insoluble, sticky or slimy gel causes plaque to 'Body of the lesion' lies between unaffected surface
adhere tenaciously to, tooth surfaces. Zone 3 layer and dark zone. It is the area of greatest
demineralization (BHU- 07).
Levan or Flucan is the fructose moiety of sucrose.
Zone 4 'Surface zone' appears relatively unaffected.
7. Stephan's curves are the pH curves of plaques in different
individuals studied after rinsing the mouth with 10% DENTINAL caries (Beginning pulpally)
glucose or sucrose solution.
Zone 1 Zone of fatty degeneration of Tomes fibers.
8. Stralfors curves are similar to those of Stefan but in addition Zone of the dental sclerosis characterized by deposition
Zone 2
correlated the plaque pH fall after the carbohydrate rinse of calcium salts in dentinal tubules.
with the lactobacillus count. Zone of decalcification of dentin, a narrow zone
Zone 3
preceding bacterial invasion.
9. The critical pH of dental caries is 5.2
Zone of bacterial invasion of decalcified but intact
Zone 4
10. The highest incidence of dental caries in permanent teeth dentin.
is with mandibular first molars since these are the first Zone 5 Zone of decomposed dentin.
permanent teeth to erupt into the oral cavity.
19. Microorganisms found penetrating the dentinal tubules before
11. High ammonia concentration retards the formation of there is any clinical evidence of the carious process are termed
plaque. as "PIONEER BACTERIA".

12. SELENIUM causes an increase in the dental caries rates 20. "MACH BAND" is the line of optical illusion found on the images
while VANADIUM decreases the dental caries rates. of unaffected teeth below the occlusal enamel. (AIIMS-05)
r ORAL PATHOLOGY & MEDICINE SYNOPSIS
21. ANTON VAN LEEVWEN HOEK was first to observe microorganisms in association with dental caries.

22. JK clanke concluded that caries caused by microorganisms and attributed it to S. mutans

23. The scientist who discovered that caries is the most common cause of of extraction of teeth is ALLEN.

IV. Most Common Sites Of Some Important Pathologies

Condition Most common site


Enameloma Bifurcation and trifurcation of maxillary posterior teeth.
• Composite compound odontoma

• Adenomatoid odontogenic tumour Anterior maxilla

• Sriu�mous odontogenic tumour


Composite complex odontoma Posterior maxilla
Pleomorphic adenoma Parotid gland
Epstein pearls Along the median raphae of hard palate
Bohn's nodules Most numerous along junction of hard and soft palate
Static bone cyst/ Stafne's bone cyst / Developmental
Below the mandibular canal
lingual mandibular salivary gland depression
Traumatic or hemorrhagic or idiopathic bone cyst Above the mandibular canal
Torus palatinus Midline of palate
Lingual surface of mandible above the myalohyoid line,
Torus mandibularis
opposite the bicuspid teeth.
Branchial cleft cyst Lateral aspect of neck.
Thyroglossal cyst Midline of neck region.
Median anterior maxillary cyst
Between central incisors.
(Naso palatine duct cyst or incisive canal cyst)
Globulomaxillary cyst Between the roots of lateral incisor and canine.
Fordyce's granules Bilaterally on buccal mucosa opposite the molar teeth.
Leukoplakia Buccal mucosa and commisures.
• Maxillary and mandibular posterior area.
Dentigerous cyst
• Maxillary cuspid area.
• Ameloblastoma

• Pindborg tumor Mandibular molar-ramus area

• Odontogenic keratocyst
Lateral periodontal cyst Mandibular bicuspid-cuspid area
• Granular cell myoblastoma
Tongue
• Papilloma
Oral carcinoma Lower lip followed by tongue.
• Keratoacanthoma
Lips
• Hereditary hemorrhagic Telangiectasia (AP -2004)
Basal cell carcinoma Middle 3m of face
Eruption sequestrum is commonly associated with 1 st permanent molar
Dental 9'ul-6e � - - - - - -

V. MOST COMMON TEETH IN VOLVED IN SOME PATHOLOGIES


Condition Teeth commonly involved
• Most commonly present natal teeth
Primary mandibular central incisor.
• Smallest tooth in human dentition
• Ankylosed / submerged teeth / infra occlusion Deciduous mandibular second molar.(In permanent dentition, the
• Primary tooth most susceptible to caries teeth which are associated with ankylosis are 1st & 2nd molars)
• Dens in dente (Dens invaginatus)
• Teeth most commonly in abnormal relation with other teeth in
the jaws Permanent maxillary lateral incisors.
• Permanent teeth demonstrating greatest variation in tooth
mass, size and shape next to third molar
Dens evaginatus (Leongs premolar or occlusal enamel pearl) Premolars
Maxillary molars are the teeth most frequently involved, especially
Concrescence
a third molar and a supernumerary tooth
Teeth which are commonly affected with microdontia Maxillary lateral incisors followed by third molar.
Teeth commonly involved in Turner's hypoplasia Mandibular premolars followed by Permanent maxillary incisors .
Regional odontodysplasia (Ghost teeth) Permanent anteriors.
Adenomatoid odontogenic tumour Maxillary or mandibular canine.
Teeth which are commonly blocked out due to lack of space Permanent maxillary canine.
Supernumerary roots (Bifurcated roots) Mandibular canine and premolars.
Supernumerary teeth Mesiodens followed by maxillary and mandibular 4th molar.
Third molars followed by maxillary laterals and mandibular second
Most commonly missing permanent teeth
molars.
Most commonly missing deciduous teeth Deciduous maxillary and mandibular lat,eral incisors.
Teeth that are least commonlii extracted for orthodontic purpose Maxillary anteriors.
Teeth that are commonlii extracted for orthodontic purpose First premolars
• Tooth that shows greatest variation in occlusal form next to
maxillary 3rd molar.
• Tooth in the mandibular which is mostly likely to be displaced Mandibular second premolar.
due to arch size discrepancy.
• Tooth that shows greatest variation in eruption timing.
Taurodontism Permanent or primary molars.
Most commonly impacted teeth 3'd molars followed by maxillary cuspids.
• Benign cementoblastoma (true cementoma)
• Condensing osteitis (Chronic focal sclerosing osteomyelitis) Mandibular 1st molar
• Tooth most commonly lo:st from arch due to caries
Chronic hyperplastic pulpitis Deciduous molars and 1 st permanent molars.
Teeth more commonlii involved in nursing bottle caries Maxillary incisors
Teeth least involved in nursing bottle caries Mandibular incisors
Teeth more first involved in nursing bottle caries Maxillary canines.

VI IMPORTANT CLINICAL AND HISTOLOGICAL FEATURES OF SOME DISEASES·


• Wickham's striae, bilateral symmetrical distribution of lesions with female predilection.
Lichen planus
• Saw tooth retepegs, liquefaction degeneration of basal layer, colloid or civatte bodies.
• Auspitz's sign, painless non pruritic papules
Psoriasis
• No involvement of oral mucosa and Monro's abscesses.
• Psoriasis, Reiter's syndrome, benign migratory glossitis and ectopic geographic tongue. Psoriasis form lesions
• Positive Nikolsky's sign, Tzanck cells and granular immuno fluorescence. Pemphigus
ORAL PATHOLOGY & MEDICINE SYNOPSIS

• Nikolsky's sign +ve Familial benign chronic


pemphigus
• Dilapidated brick wall effect. (Hailey-Hailey disease)
• Butterfly distribution of lesions
• Carpet tack extensions
Lupus erythematosus
• LE. cell phenomenon
• speckled or particulate immunoflourescence.
• Lesions on skin are rare except on genitalia Cicatrial pemphigoid /
• Lesions of oral mucous membrane, conjunctiva and separation of the basement membrane with the Benign mucous membrane
epithelium from the underlying connective tissue. pemphigoid / Ocular
• Ocular involvement is the most serious complication pemphigus
• Gingival tissues are involved Bullous pemphigoid / Para
• Separation of the basement membrane with the connective tissue from the overlying epithelium. pemphigus
• Target or Iris or Bulls Eye Lesions
Erythema multiforme
• Recent H/O of Herpes simplex infection
• Corps rands and grains,
Keratosis follicularis
• Leafing out pattern of parabasal cells
(Darriers disease)
• Defect in demosome tonofilament complex.
• Hyperkeratosis, acanthosis with a deep groove filled with parakeratin and absence of granular layer
Porokeratosis of Mibelli
constituting the "CORNOID LAMELLA''
• Mask like face, claw like hands, Raynauds phenomenon, CREST syndrome, staff board like tongue,
Scleroderma
coup de sabre and generalized widening of the periodontal ligament.
• Hyper elasticity of skin, hyper extensibility of joints, defective healing, rubber man
Ehlers danlos syndrome
• Lack of normal scalloping of DEJ
• Ulcers occurring on the masticatory mucosa, common in children
• Prodromal symptoms are present• Primary herpetic stomatitis
• Presence of Lipschutz bodies and Ballooning degeneration is diagnostic.
• Koplik spots. Measles (Rubeola)
• Bilateral contagious parotitis, elevated serum amylase level. Mumps
• Herpetic labialis,
• Herpetic whitlow (painful herpes simplex virus infection of a finger from direct inoculation. Most
Recurrent or secondary herpes
common in physicians, dentists, and nurses as a result of exposure to the virus in a patient's
virus infection
mouth)
• Ulcers on the masticatory mucosa, Tzanck cells.
• Ulcers occurring on the labile mucosa
• Precipitated commonly in stress. Recurrent Apthous stomatitis
• No history of Prodromal symptoms
• Sore throat, ulcers on the faucial pillars, soft and hard palate,
• Occurs as an epidemic in summer Herpangina
• Caused by Coxsackie group A virus.
• UNILATERAL painful vesicles along the course of sensory nerves Tzanck cells. Herpes Zoster
• Fungal disease affecting the reticulo-endothelial system and resembles granulomatous infection. Histoplasmosis
• Fungal disease occurring in debilitated and diabetic patients and causes necrosis of the tissue. Phycomycosis (mucormycosis)
• Strawberry tongue, Ras[Pberry tongue. Scarlet fever
• Lock jaw due to muscle trismus
• Risus sardonicus (Rigidity of facial muscles) Tetanus
• Risus opisthotonus (Rigidity of entire body muscles)
• Hebra nose. Rhinoscleroma
Dental 9'ul-6e �-
-

• Rhagades formation is seen


• HUTCHINSON'S TRIAD, which include hypoplasia of the incisor and molar teeth, eight nerve Congenital syphilis
deafness and interstitial keratitis.
• Steven Johnson's syndrome
• Cicatrial Pem phigoid
• Vesiculo bullous lesions affecting skin, oral mucosa, eyes and genitals.
• Behcet's syndrome
• Reiter's syndrome
• Cyst found in place of a tooth. Primordial cyst
• Cyst causing "hollowing out" of the entire ramus Dentigerous cyst
• Multilocular variant of lateral periodontal cyst. Botryoid odontogenic cyst
• Keratin filled cysts along the midpalatine raphae of the new born. Epstein pearls
• Keratin filled cysts along the junction of the hard and soft palate in the newborn. Bohn's modules
• Cystic epithelium containing daughter or satellite cysts
Odontogenic keratocyst
• Cystic fluid containing protein content of less than 4 gm/100 ml.
Jaw cyst - Basal cell nevus
• Multiple odontogenic keratocysts, basal cell carcinoma, rib anomalies.
Bifid rib syndrome
• Multiple odontogenic cysts, excessive length of bones, hyper extensibility of joints. Marfan's syndrome
• Driven snow radiographic appearance Calcifying epithelial
• Lisegang rings, APUD amyloid odontogenic tumor
• Clear cells. (Pindborg tumor)
• Cyst developing frequently in the anterior maxilla, occurring mostly at 20 years of age, female
Adenoameloblastoma
predilection.
• Tennis racket shaped septa
• Pale cells, Dark cells histologically Odontogenic myxoma
• High content of hyaluronic acid.
• Asymptomatic lesions occurring near the apices of mandibular incisors, vital teeth, female Periapical cemental dysplasia
predilection. (cementoma)
• Neoplasm made up of different types of tissues not native to the area. Teratoma
• Excessive proliferation of chronically inflamed dental pulp Chronic hyperplastic pulpitis
• Occurs mostly in teeth with large open carious lesions especially in children, contains few nerves. (Pulp polyp)
• Decayed VITAL tooth, sclerosing bone reaction, high tissue resistance, low grade infection, bone
Condensing osteitis
scar formation
• Decayed tooth, periosteal thickening, mandible predilection, occurring mostly in children. Garre's osteomyelitis
• Extreme fragility and fractures of the bones, blue sclera, brownish or yellowish teeth with
Oste,ogenesis imperfecta
translucent or opalescent hue.
• Hyperostosis, tender, deeply placed soft tissue swellings occurring exclusively in infants. Infantile cortical hyperostosis
• Brachycephalic skull, abnormalities of clavicles
• Hypermobility of shoulder due to absence of clavicle
• Multiple unerupted supernumerary teeth, Cleidocranial dysplasia
• Absence of cellular cementum
• Presence of Wormian bones.
• Early synostosis of sutures, small maxilla, parrot beak nose, hypertelorism and triangular frontal
Craniofacial dysostosis
defect.
• Antimongoloid palpebral fissure, hypoplasia of malar bones and mandible, facial clefts, malformation
Mandibulofacial dysostosis
of ears, fish like or bird like face, deficiency of eye lashes.
• Cleft palate, micrognathia (small mandible). Glossoptosis, Bird face. Pierre Robin syndrome
• Mongoloid fades, subnormal mentality, large mandible, macroglossia, low caries incidence than in
Down syndrome
normal people and high incidence of periodontal diseases.
ORAL PATHOLOGY & MEDICINE SYNOPSIS

• Endosteal production of bone, lack of resorption of bone, medullary cavities replaced by bone,
Osteopetrosis
thickened cortex.
• Dwarfism, disturbance of endochondral bone formation, possess high strength. Achondroplasia
Massive osteolysis (vanishing
• Resorption of bone with ultimate total disappearance of bone.
bone)
• Enlargement of skull and jaw bones
• Compression of nerves in their foramens
• Both osteolytic and osteoblastic lesion,
• Cotton wool radiographic appearance Pagets disease
• Increased serum alkaline phosphatase level,
• Mosaic and Jigsaw puzzle appearance of bone histologically
• Development of osteosarcoma in involved bones is common.
• Cafe-au-lait spots, endocrinal disturbances, lesions involving more than one bone. Polyostotic fibrous dysplasia
• Mottled, ground glass or peaud orange radiographic appearance,
• Lesion involving one bone, requires cosmetic recontouring as treatment, Monostotic fibrous dysplasia
• Chinese letter shaped trabeculae
• Chubby face, angelic look or eyes towards heaven appearance
• Premature exfoliation of deciduous teeth, (which is also seen in Noonan's syndrome)
• Teeth appear to be floating in cystic spaces radiographically {floating tooth syndrome) Cherubism
• Presence of giant cells histologically
• Ground glass appearance radiographically
• Pain, muscle tenderness (mostly lateral pterygoid), clicking of TMJ,
• Limitation of jaw motion, lack of tenderness in the joint when it is palpated through external MPDS
auditory meatus.
• Hunters of Moeller's glossitis
• Acholorhydria, nervous system involvement Pernicious anemia
• Howell-Jolly bodies and Cabot's rings.
• Bald tongue of sandwith, Diarrhea, Dementia and Dermatitis. Pellagra
• General lack of bone marrow activity, anemia, thrombocytopenia and leucopenia Aplastic anemia
• Target cells
• Hair on end or crew cut appearance of skull (This appearance causes a "Step-lader" effect in PA
views of skull) Thalassemia
• Salt and Pepper appearance of IOPA
Note: Salt and pepper effect in MRI of salivary gland is seen in Sjogren's syndrome
• Sickle hemoglobin (Hbs), sickle shaped erythrocytes,
Sickle cell anemia
• Hair on end or crew cut radiographic appearance of skull.
• Purplish discoloration of deciduous teeth
• Enamel hypoplasia involving the incisal edges of anterior teeth and middle 3rd of deciduous cuspid Erythroblastosis fetalis
and 1st molar may be seen resulting in a characteristic ring like defect known as "Rh hump".
• Oral ulcerations, presence of infection,
Agranulocytosis
• Lack of inflammatory cell infiltration around the lesions.
• Also known as Kissing disease
• Palatal petechiae is important identification feature Infectious mononucleosis
• Positive Paul bunnel and Monospot tests.
• Periodic or cyclic diminution of PMNs for every 21 days. Cyclic neutropenia
• Philadelphia chromosome.
• The most consistent chromosomal abnormality is Philadelphia chromosome involving the reciprocal Chronic myeloid leukemia
translocation of parts of long arm of chromosome 22 to the long arm of chromosome 9
Dental 9'ul-6e �-
-

• Brown non cariogenic plaque line on the enamel at the cervical margin of the tooth. Mesenteric Line
• Hematoxyphilic line between regular dentin and postoperative dentin
Caciotraumatic line
• It is due to disturbance in dentin formation at the cavity preparation
• Punched out lesions of interdental papilla, pseudomembrane, fusospirochetal infection (mostly of
ANUG
intermediate sized).
• Defective neutrophil che motaxis,
• Mirror image shaped vertical bone loss around molars. LJP
Note: Mirror image shaped nuclei are seen in Hodgkin's lymphoma
• Juvenile periodontitis, Palmar plantar keratosis, calcification of falx cerebri. Papillon Lefevre syndrome
• Trigger zones, fleeting severe pain of facial muscles,
• More involvement of maxillary and mandibular divisions than ophthalmic division, relieved Trigeminal neuralgia
commonly by carbamazepine
• Alarm clock headache, vi dian nerve neuralgia, no trigger zone. Sphenopalatine neuralgia
Auriculotemporal or Frey's
• Gustatory sweating.
syndrome
• Mask like or expressionless face, inability to close eye on affected side
Bells palsy
• Mouth drawn to unaffected side on smiling.
• Trigger zone in the tonsillar area, sharp shooting pain in the pharynx, tonsil or posterior portion
Glossopharyngeal neuralgia
of tongue.
• Charcots triad (nystagmus, tremor, and scanning speech)
Multiple sclerosis
• Personality and mood deviation towards friendliness.
• Rapid elevation of the ptotic eyelid on movement of the mandible to the contra lateral side. Jaw winking syndrome
• Myopathic facies, Swan neck. Dystrnphic myotonia
• Herculean appearance. Congenital myotonia
• Sorrowful appearance. Myasthenia gravis
• Floppy infant syndrome. Hypotonia
• Petrified man. Generalized myositis
• Masseter (PGI June-13) is most commonly involved. ossificans
• Painless, slow growing parotid mass, mixed tumor, hyaline cell, predilection for women. Pleomorphic adenoma
• Bilateral chronic painless enlargement of lacrimal and salivary glands. Mikulicz's disease
• Characterized by triad of Keratoconjunctivitis sicca, xerostomia and rheumatoid arthritis
Sjogren's syndrome
• Cherry blossom or branchless fruit laden tree or salt and pepper appearance of sialogram.
• Carcinoma insitu occurring in patients who have had arsenical therapy. Bowen's disease
• The basal cells are arranged in anastomosing cords or a duct like pattern, the central portion of may
contain a mucoid material , producing the typical cibriform or Swiss cheese or honey comb pattern Adenoid cystic carcinoma
(Cylindroma)
• Perineural spread of tumor cells is seen.
• Most common malignant salivary gland tumor of children
Mucoepidermoid carcinoma
• Most common intraosseus salivary gland neoplasm.
• Exophytic growth with finger like projections, cauliflower like surface
Papilloma
• Most common benign neoplasm originating from surface epithelium.
• Benign lesion resembling epidermoid carcinoma both clinically and histologically, a self healing
Keratoacanthoma
lesion.
• Carcinoma resembling benign lesion
Verrucous carcinoma
• Parakeratin plugging is seen.
• Most common intra oral nevus. Blue nevus
• "Abtrop fung" or dropping off effect, junctional activity, premalignant condition. Junctional nevus
• Premalignant lesions. Leukoplakia, Erythroplakia
• Strap cells, racquet cells and ribbon cells. Rhabdomyosarcoma
ORAL PATHOLOGY & MEDICINE SYNOPSIS

• Intracellular edema of the spinous layer. The characteristic edematous cells are large and pale with
a reticular pattern
Leukoedema
• lesion resembles leukoplakia but is a variant of normal mucosa.
• Lesion disappears on stretching the buccal mucosa.
• Burning sensation, trisimus, absence of retepegs, reduced fibroblasts and hyalinization of collagen
Oral submucous fibrosis
bundles.
• Most common carcinoma of skin
• Men predilection
• Rolled out margins Basal cell carcinoma
• Most frequent on middle third of the face
• Direct spread, no metastasis, so known as benign carcinoma.
• Most common neoplasm of oral cavity, second most common neoplasm of skin, most common in
Epidermoid carcinoma
lower lips.
Carcinoma of tongue and
• Epidermoid carcinoma which has poor prognosis.
floor of the mouth
• 3rd most common neoplasm of skin, neoplasm of melanocytes, exists in radial and vertical growth
Malignant melanoma
phases.
• Malignant melanoma which exists solely in vertical growth phase. Nodular melanoma
• Most common benign soft tissue neoplasm of oral cavity. Fibroma
• Granular septa originating at right angles from the periphery of the lesion, indentation at the site
Central giant cell granuloma
of origin of the septa, lesion occurring anterior to the first molar usually.
• Loss of laminadura, ground glass appearance, brown tumors. Hyperparathyroidism
• Blood welling up from the tissue, eccentrically ballooned soap bubble radiographic appearance
Aneurysmal bone cyst
• Pseudocyst.
• Pumping action i.e, if the tooth in the tumor area is pushed into the socket, it is rebounded back. Central haemangioma
• Angiomas of skin in the area supplied by V nerve, intracranial calcifications, lepto-meningeal Encephalo trigeminal
angioma. angiomatosis
• Predominate malignant bone tumor in children, onion skin and sunray radiographic appearance. Ewing's sarcoma
• Most common primary bone tumor, sunray appearance, widened periodontal ligament. Osteosarcoma
• B-cell neoplasm caused by EBV virus,
Burkitt's lymphoma
• STARRY SKY histologic appearance.
• Reed-Sternberg cell, Lacunar cell (R-S cell in nodular sclerosis type). Hodgkin's Lymphoma
• BENCE JONES PROTEIN
• Plasma cells with cart wheel or checkerboard pattern of chromatin
Multiple myeloma
• Russel bodies
• Multiple sharply punched out radiolucent areas, hyperglobulinemia, anemia
Solitary plasma cell myeloma
• Bence Jones proteins in urine with no anemia and hyperglobulinemia.
( Plasmacytoma)
• Cafe au lait spots, macroglossia, cosmetic disfigurement, neoplasm arising from perineural cells
Neurofibroma
and Schwann cells
• Metastatic lesions of the jaws are mostly from Carcinoma of breast
• Metastasis from mandibular sarcoma mostly occurs to Lung
• Protein deficiency with sufficient calorie intake,
Kwashiorkor
• "Flag sign" hair.
• Starvation with overall lack of calories. Marasmus
• Photosensitivity, red urine (also seen in patients taking Rifampicin)
Porphyria
• Reddish brown discoloration of both deciduous and permanent dentition.
• Gorgoyle cells or Hurler cells, Reily bodies. Hurler syndrome
• Magenta colored tongue, angular cheilitis. Riboflavin deficiency
Dental 9'ul-6e �-
-

Hereditary fructose
• Vomiting after ingestion of fruit or cane sugar.
intolerance
• Multiple punched out bone lesions, Exophtholmos, diabetes insipidus, multifocal eosinophilic Hand-Schuller Christian
granuloma. disease
• Gaucher cells, glucosideroside deposition due to lack of an enzyme glucocerebrosidase. Gaucher's disease
• Foamy, lipid laden Niemann pick cells, storage of sphingomyclin due to lack of sphingomyelinase. Niemann-Pick disease
• Deficiency of alkaline phosphatase, excretion of phosphoenthanolamine in urine, premature loss
Hypophosphatasia
of deciduous teeth.
Waterhouse Friderichsen
• Acute adrenal cortical insufficiency.
syndrome
• Bronzing of the skin, pigmentation of oral mucosa, chronic adrenal cortical insufficiency. Addison's disease
• Pink hued crown, odontoclasts on the inner pulpal surface of dentin. Internal resorption
• Rrtdiolucency situated BE'LOW the mandibular Crtnal_ Static nr Strtfne's cyst
• Scalloped radiolucency extended between roots of teeth ABOVE the mandibular canal.
Traumatic bone cyst
• Pseudocyst.
Type III dentinogenesis
• Brandy wine type, shell teeth.
imperfecta
Type I (radicular) dentin
• Lava flowing around boulders (histologic appearance).
dysplasia
Type II (coronal) dentin
• Thistle tube pulp chamber.
dysplasia
• Ghost teeth. Regional odontodysplasia
• Disease occurring due to malabsorption of zinc. Acrodermatitis enteropathica
• Cobblestone or fissured appearance clinically, caused by HPV 13 AND 32, Club shaped wide rete
ridges, presence of viral altered mitosoid cell histologically Focal epithelial hyperplasia
Focal Epithelial hyperplasia caused by HPV 13 and 32 is also termed [COMEDK-2012) (heck's disease)
a) Coudens disease b) Hecks disease c) Acanthosis migrans d) Kerato acanthoma
Kuttner tumor or Chronic
• Benign inflammatory condition of the submandibular gland that mimics a malignant neoplasm
sclerosing sialadenitis of
clinically because of presentation as a hard mass
submandibular gland.
• Traumatic ulceration of mucosa due to missing teeth, malposed teeth, partial denture and Riga TUGSE or traumatic
Fede disease (traumatic sublingual ulcerations due to erupting teeth in infancy) followed by granuloma with stromal
proliferation of granulation tissue (granuloma) adjacent to the area of ulceration eosinophilia
EPULIS FISSURATUM or enture
• Leaf like denture fibroma that occurs beneath a maxillary denture, mucopolysaccharide keratin
epulis or inflammatory fibrous
dystrophy or plasma pooling histologic feature
hyperplasia
• Cambium layer is seen in Alveolar rhabdomyosarcoma
• Congenital epulis in newborns located on maxillary or mandibular gingiva. Neuman's tumor
• The lips characteristically develops looseness and protrusion that have been described as tapir lips.
Mild restricted muscular
• Myopathic facies
dystrophy.
• Patients are unable to whistle or smile
Malignant peripheral nerve
• Triton tumor is other name for
sheath tumor (MPNST)
• Dentinogenic ghost cell tumor is other name for Calcifying odontogenic cyst.
• A periapical granuloma without cystic transformation Bay cyst
• An inflammatory cyst found primarily on the distal or facial aspect of a vital mandibular 3rd molar Paradental cyst
• The antigen that can be detectable during the window period and the late phase of infections when
P24 antigen
the virus replicating fast
• The disease in which the causative organisms contains a clear halo and is described as 'tissue
Cryptococcosis
microcyst'
r ORAL PATHOLOGY & MEDICINE SYNOPSIS
VII. CLINICAL AND ORAL EFFECTS OF SOME VITAMINS
Vitamin General manifestations of deficiency Oral manifestations of deficiency
• Night blindness • Keratinising metaplasia of epithelium resulting
• Xeropthalmia characterized by dryness in conjunctiva and in increased keratin formation.
cornea • Occlusion of salivary gland ducts with keratin.
• Bitot spots in forms of triangular plaques in conjunctiva • Enamel hypoplasia, atypical dentin formation
Vitamin A
• If xeropthalmia persists, destruction of cornea occurs, and epithelial invasion of pulpal tissue are
causing total blindness. This condition is known as characteristic features.
keratomalacia. • Enamel is more severely affected than dentine.
• Delayed eruption of teeth
• Rickets in children • Delayed eruption of primary and permanent
• Osteomalacia in adults teeth.
• Pigeon chest is one of important feature of rickets • Mal-alignment of the teeth in the jaws.
Vitamin D • The vitamin, which acts more like a hormone, is Vitamin D. • Developmental anomalies of dentin and enamel.
(Anti rachitic • Renal rickets or renal osteodystrophy is seen in patients The teeth shows wide predentin zone with much
vitamin) with chronic renal failure. Renal rickets is mainly due to interglobular dentin.
decreased synthesis of calcitriol in kidney. • The pulphorns are elongated and extend high,
• In rickets, the plasma calcitriol is decreased and alkaline reaching the dentinoenamel junction.
phosphatase activity is elevated
• Decreased male fertility
Vitamin E
• Impaired fetal-maternal vascular relationships
(Anti sterility
• Encephalomalada
vitamin)
• Nutritional muscular dystrophy
• Deficiency is uncommon • Prothrombin levels below 35% results in gingival
Vit. K
• Brings about post-translational modification of 2,7, 9,10 bleeding after tooth brushing.
(Coagulation
blood clotting factors, particularly prothrombin. So • Spontaneous gingival hemorrhages occur, when
Vitamin)
deficiency may result in prolonged clotting time the prothrombin levels fall below 20%.
• Deficiency may result in SCURVY. It is characterized by • The pathognomic sign is the swollen and spongy
spongy and sore gums, loose teeth, anemia, swollen joints, gums, particularly the interdental papillae is
delayed wound healing, hemorrhage, osteoporosis, etc. involved producing the appearance of scurvy
• Defective collagen synthesis. buds.
Vit. C
• "Cork-screw" hair pattern, "Woody legs" with large • In severe cases, hemorrhages to periodontal
(Ascorbic acid) membranes followed by loss of bone and
spontaneous bruises in lower extremities are other
features. loosening of teeth occurs.
• "Trummer field zone" is the classic histologic picture of
bone in scurvy.
• 8 1 deficiency is seen in populations consuming polished
Vit. 8 1 rice as a staple food
(Anti beri-beri • Dry beri-beri or peripheral neuritis; Wet beri-beri or
or anti-neuritic cardiac manifestations, and cerebral or Wernickes
vitamin) encephalopathy (AIPG-10) with Korsakoff's psychosis are
features of 8 1 deficiency.
• Riboflavin deficiency is common in children who do not • Glossitis: The filiform papillae become atrophic
drink milk. while the fungiform papillae become en-gorged
• Nasolabial seborrhea or dyssabacea. and mushroom shaped, resulting rn magenta
Vit. 8 2 • Vascularisation of cornea colored tongue.
(Riboflavin) • Scrotal dermatitis • Cheilosis, ocular lesions. (Non-specific bilateral
angular cheilosis may be seen in association
with faulty dentures or in patients with reduced
vertical dimension due to attrition.)
• Pellagra (the symptoms of pellagra are referred to as three • Bald tongue of sandwith.
'D's ·i.e., dermatitis, diarrhoea, dementia and if not treated • 'Raw beefy' tongue.
Niacin may lead 4th 'D' i.e, death).
• The mucosa becomes fiery red and painful.
• Salivation is profuse.
Dental 9'ul-6e

Vit. 85 • Burning feet syndrome Pain and numbness in the toes,


(Pantothenic sleeplessness and fatigue are features
acid or chick • Pantothenic acid is one of the water-soluble vitamins that
-------------
anti-dermatitis is synthesized in the body
factor)
• Peripheral neuropathy (due to decreased synthesis of
Vit. 86 serotonin catecholamines) and demyelination of neurons.
-------------
(Pyridoxine) • Isoniazid (drug used in treatment of TB) is a antagonist
of Vitamin 86.
Biotin (Anti-egg • Biotin deficiency is uncommon since it is well distributed
white injury to foods and also supplied by the intestinal bacteria. -------------
factor) • Also known as Vitamin 8 7 or Vitamin H.
• Macrocytic anemia, glossitis. • GLOSS/TIS: The filiform papillae disappear first,
• Aminopterin and methotrexate are structural analogues but in advanced cases the fungi form papillae are
Folic acid of fol·ic acid used in treatment of many cancers including lost and the tongue becomes smooth and fiery
leukemia. These drugs block the formation of TH F and red in colour.
hence DNA synthesis is impaired.
Vit. 812 (Anti • Pernicious anemia • Beefy red tongue with glossopyrosis, glossitis
pernicious • Neurological manifestations due to degeneration of and glossodynia.
vitamin or posterior and lateral tracts of spinal cord. • Hunter's glossitis or Moellers glossitis, which
extrinsic factor • Degeneration of myelin sheath and peripheral nerves also is similar to "bald tongue of sand with" seen in
of castle) occur.s. pellagra.

VIII. CLINICAL AND GENERAL EFFECTS OF SOME HORMONES


Hormone Hypo and Hyper secretion ORAL EFFECTS
• Gigantism is due to hypersecretion of GH in childhood
before fusion of epiphysis of bone with shaft. • Large mandible
• d,cromegaly is due to hypersecretion of GH in Adults. • Extreme prognathism, giving the head typical
Enlargements of bones (especially mandible), kyphosis acromegalic appearance
Growth (bowing of spine) and bulldog scalp are important features. • Macroglossia
Hormone • Dwarfism is due to hypo secretion of GH in infancy or early • Small dental arches resU1lting in malocclusion
(GH) childhood. Stunted but proportionatedgrowth with normal
mental activity and reproductivefunctions are important • Shorter roots
feature. • Small mandible
• Acromicria is due to deficiency of GH in Adults. I t is • Retarded eruption and shedding of teeth.
characterized by atrophy of extremities of body.
a) Hyperthyroidism
• Due to ,excess secretion of thyroid hormones. • Earlier shedding of deciduous teeth
• Thyroid adenoma, Grave's autoimmune disease are common • Accelerated eruption of permanent teeth
causes. • Delayed eruption of teeth
T hyroid
hormones • 2 types - primary and secondary. • Macroglossia
b) Hypothyroidism • Macroglossia
• £retinism is due to hypothyroidism in £hildren. • Swollen lips
• Myxedema is due to hypothyroidism in adults.
• Tetany is due to hypoparathyroidism. Tetany occurs if the
plasma calcium level falls below 6 mg% from its normal • Giant cell tumor or cyst of the jaws
value of 9.4 mg%.
• Brown tumors
Parathormone • Carpopedal spasm, Trousseau's sign and Chvostek's sign are
important features of tetany. • Loss of lamina dura
• Ground glass appearance of bone
• Bone diseases like osteitis fibrosa cystica are important
features of hyperparathyrodism.
ORAL PATHOLOGY & MEDICINE SYNOPSIS

• Diabetes mellitus is due to reduced secretion of insulin by • Periodontitis


beta cells of islets of Langerhans.
Insulin • Multiple, fleeting periodontal abscesses
• Increased blood sugar level, POLYURIA, POLYDIPSIA,
POLYPHAGIA, are various features of diabetes mellitus. • Dry mouth

• Hyper aldosteronism is due to excessive secretion of


aldosterone.
• It is classified as primary and secondary types.
Mineral • Pale brown, deep chocolate pigmentation of oral
corticoids • Primary aldosteronism is also known as Conn's syndrome. It mucosa, gingiva, tongue and lips.
develops due to a tumour in Zona glomerulosa of adrenal
cortex.
• Secondary aldosteronism is due to extra adrenal causes.

IX. SOME DISEASES AND THEIR CAUSATIVE ORGANISMS.


ORGANISM DISEASE
• Predominant organism of plaque in healthy mouths.
STREPTOCOCCUS VIRIDANS(a-hemolytic)
• Subacute bacterial endocarditis.
a) Streptococcus sanguis
• Another group of streptococci to cause caries after S.mutans
• Forms longest chains
b) Streptococcus salivarius • First organism to invade the oral cavity after birth
• Other common organism of STREPTOCOCCUS VIRIDANS that causes SABE after S.sanguis.
• Initiation of smooth surface caries
c) Streptococcus mutans
• Synthesize glucan by its action on dietary carbohydrates
• Pharyngitis, Cellulitis, Scarlet fever, Quinsy, Ludwig's angina.
Streptococcus pyogenes (13 - hemolytic) • Rheumatic fever
• Most commonly isolated organism from root canals and dental infections
• Skin infections
- furuncle, folliculitis
- Stye
- Impetigo
Staphylococci
- Carbuncle
• Infection of burns and wounds
• Osteomyelitis and sialadenitis
• Toxic shock syndrome
• Cystic fibrosis
• Toxic shock syndrome
Staphylococcus aureus or
• Acute endocarditis
Staphylococcus pyogenes
• Food poisoning
• Sialadenitis
Staphylococcus albus or • Postoperative endocarditis
Staphylococcus epidermis • Stitch abscess
Lactobacillus • Progression of carious lesion
Actinomyces viscosus • Multiple abscesses with draining fistulae
Prevotella intermedia • Pregnancy gingivitis
Porphyromonas gingivitis • Generalised juvenile periodontitis
A. actinomycetam comitans • Localised juvenile periodontitis
Capnocytophaga • Pubertal gingivitis
Fusospirochaetes • ANUG (mostly by intermediate sized spirochetes)
Clostridium tetani • Tetanus, (Lock jaw).
Dental 9'ul-6e � - - ---

Clostridium perfringens • Gas gangrene and gastroenteritis


Clostridium botulinum • Botulism or food poisoning
Clostridium difficile • Acute pseudomembranous colitis
• Typhoid fever
Salmonella
• Osteomyelitis
• Rhino scleroma, which includes hebra nose, paralysis of soft palate with enlargement
Klebsiella rhinoscleromatis
of uvula.
Haemophilus influenza • Epiglottitis (AIIMS- 2004)
Vibrio cholera • Cholera.
Anthrax bacilli • Anthrax disease or malignant pustule.
T. Pallidum (spirochaetes) • Syphilis
Treponema denticola • Dry socket --
f-
Actinobacillus • Botryomycosis (bacterial disease)
• 'MUMPS', which includes
- Rubbery or elastic swelling of salivary glands frequently elevating the ear.
- Epidydimitis, Acute pancreatitis
Paramyxo virus
- Orchitis and oophoritis
• MEASLES (Rubeola)
- Presence of Koplik spots is characteristic.
• German measles (Rubella)
TOGA Virus
• Koplik spots are not seen
Coxsackie group A virus Herpangina
Coxsackie virus A10 Acute lymphonodular pharyngitis
Coxsackie virus A16 Hand, Foot & Mouth disease
PPLO (plasma pneumonia like organisms) Behcet's syndrome

X. SOME GENETICALLY TRANSMITTED DISEASES


TYPE OF INHERITANCE &
DISEASE
CHROMOSOME INVOLVED
• Thalassemia, Agnathia Autosomal recessive
• Hemophilia Sex linked recessive
Apert syndrome, Cleft lip and palate Polygenic
• Osteogenesis imperfecta
• Amelogenesis imperfecta ( Due to defective gene amelogenin,
• closely linked to the locus DXS85 at Xp22)
• Dentinogenesis imperfecta (the gene maps to chromosome #4,
• which encodes a protein called dentin sialophosphoprotein (DSSP}
• Sickle cell anemia
• Von Willebrand's disease Autosomal dominant
• White sponge nevus
• Chediak Higashi syndrome (COMEDK-09)
• Cherubism
• Achondroplasia
• Cleidocranial dysplasia
• Colour blindness
Van der woude syndrome 1
Peutz Jeughers syndrome Mutation of STK11 gene located on 19
ORAL PATHOLOGY & MEDICINE SYNOPSIS

Osteogenesis imperfecta 19
Marfan syndrome 15
Cherubism 4

XI. IMPORTANT FEATURES OF DIFFERENT ANEMIAS.


• Commonest cause of anemia.
• Iron is absorbed mainly in the duodenum and proximal jejenum.
• Ferritin and hemosiderin are the storage forms of excess iron.
• Iron is tronsported in plasma bound to transferrin, which is synthesized in liver.
• The characteristic triad of PLUMMER VINSON syndrome are
- Iron deficiency anemia.
- Koilonychia or spoon shaped nails.
Iron deficiency - Hysterical dysphagia with a predisposition for the development of carcinoma in the upper alimentary
anaemia tract.
• The mucous membranes of oral cavity and esophagus are atrophic and show loss of keratinisation.
• A smooth red painful tongue with atrophy of filiform and fungiform papillae is seen.
• RBC count of 3-4 millions/mm3, hemoglobin content of< 6g/dl, lack of reticulocyte response following
administration of Vitamin 8 12, lack of free HCl in stomach are laboratory findings.
• Anisocytosis, Poikulocytosis and microcytosis of RBC are other features.
• "Target cell", with a central round stained area and peripheral rim of hemoglobin is seen.
• Decreased serum iron and serum ferritin levels with increased total iron binding capacity are other features.
• Seen due to deficiency of Vit.8 12 or Erythrocyte maturation factor or extrinsic factor.
• Peripheral neuropathy due to degeneration of posterior and lateral tracts of the spinal cord with loss of
nerve fibres and degeneration of myelin sheath is seen.
• Tongue is "beefy red" in colour and characteristically shows glossitis, glossodynia and glossopyrosis.
Pernicious anemia • There is gradual atrophy of papillae of tongue resulting in a "bald" tongue which is often referred to as
or Addison's Hunter's g/ossitis or Moellers glossitis which is similar to the "bald tongue of sandwith" seen in pellagra.
anemia
or Biermer's • RBC count of< 1 million, macrocytosis, poikulocytosis, polychromatophilic cells, stippled cells, nucleated
anemia cells, Howell-Jolloy bodies and Cabot's ring are the laboratory findings.
• Bone marrow studies reveal immature red cells or MEGALOBLASTS. Polymacrocytes (macropolyps) are large
polymorpho nuclear leukocytes with large polylobed nuclei are also found.
• Achlorhydria or lack of HCl secretion is a constant feature of the disease.
• The oral and general manifestations of "sprue" are closely related to pernicious anemia.
• Normal adult hemoglobin (HbA) is genetically altered to produce sickle hemoglobin (HbS) by the substitution
of valine for glutamine at sixth position of the b -globin chain.

• In the heterozygous form or sickle cell trait, only 40% of hemoglobin is HbS and the individual suffers from
sickling only under conditions of severe hypoxia.

• In homozygous form, nearly all the hemoglobin is HbS and the individual suffers from sickle cell anemia.

• Triggering causes of sickle cell crises include general anesthesia, exercise, and pregnancy.
Sickle cell anemia • Systolic murmur, cardiomegaly, shortness of breath and local tissue anoxia are other features.

• Oral manifestations include osteoporosis and loss of trabeculation of the jawbones with the appearance of
large irregular spaces and morphologic alterations in nuclei of oral epithelial cells.

• "Hair-on-end pattern' (which is also seen in thalassemia patients) is one of the radiological feature.
• During deoxygenation, the RBC containing HbS change from biconcave disc shape to an elongated crescent
shaped or sickle shaped cells. These distorted cells block the blood circulation and causes repeated vaso-
occlusive episodes.
Dental 9'ul-6e � - - ---

• It is a rare linked disorder associated with defective enzyme activity of "amino levulinic acid synthetase"
required for heme synthesis.
Sideroblastic
anemia • The siderocytes contain non-heme granules known as "Pappenheimer bodies".

• Siderocytes may or may not have a defect in heme or globin synthesis.


• a-thalassemia is due to deficient synthesis of a-chain and 13-thalassemia is due to deficient synthesis of
!3-chain.

• Heterozygous form of thalassemia is called thalassemia minor or thalassemia trait.

• Homozygous form of 13-thalassemia is called homozygous 13-thalassemia or thalassemia major.

• Homozygous forms of a-thalassemia are called hemoglobin H disease and hemoglobin Bart's diseases.

• The disease usually occurs within the first two years of life. Siblings are commonly affected.
Thalassemia
• Prominent cheekbones, depression of the bridge of the nose, unusual prominence of the premaxilla,
Or Erythroblastic
protrusion of maxillary anterior teeth, are some MONGOLOID features seen in thalassemia patients.
Anemia or
Cooley's anemia • Target cells (also seen in iron deficiency anemia) "Safety-pin" cells, and normoblasts are nucleated red
blood cells are characteristically seen.

• Poikulocytosis, anisocytosis, hypochromic and microcytic anemia are other features.

• The serum bilirubin is elevated, indicating severe hemosiderosis.

• Bone marrow aspiration reveals large immature RBC, indicating maturation arrest.

• Bristle like "crew-cut" or "hair-on end" appearance of the surface of skull and peculiar trabecular pattern
of maxilla and mandible, resulting in salt and pepper effect are radiological features.
• Characterised by general lack of bone marrow activity. It affects not only the red blood cells but also the
white cells and platelets, resulting in pancytopenia.

• "Fanconi syndrome" is a form of familial and congenital aplastic anemia.


• Chemicals, which cause aplastic anemia, are acetophenitidine, amidopyrine, arsenicals, benzol,
Aplastic anemia chloramphenicol, silver, bismuth, mercury, sulfonamides and penicillin.

• Spontaneous gingival hemorrhages (due to decreased platelet count), lack of resistance to infection (due to
neutropenia), ulcerative lesions of oral mucosa and pharynx are important clinical features.

• Prolonged bleeding time, normal clotting time, positive torniquet test, reduction in hematocrit and
hemoglobin values and RBC count less than 1 million/mm3 are laboratory features.
MISCELLANEOUS • Plasma cell type - Symptomatic with fever, hemolytic
anemia and hypergamma globulinemia
1. Agnathia (hypoplasia or absence of mandible with
abnormally positioned ears) is due to failure of migration • Multi centric or generalized - Shows hepato­
of neural crest mesenchyme in to the maxillary prominence splenomegaly and considered as multicentric form of
at 4th - 5th week or gestation. plasma cell type of castleman's disease.

2. The retrocuspid papilla is seen on the lingual mucosa of According to some researches, IL-6 is considered to be
man di buler cuspids. responsible castleman tumor.

3. Some times the lingual tonsil may become inflammed and s. HIV-I and HIV-2 are genetically distinct types. HIV-I is
clinically evident. This reactive lingual tonsil has often responsible for AIDS in Central Africa and rest of the world,
been called as "foliate papillitis" or Reactive lymphoid while HIV-2 is responsible for AIDS in West Africa and India.
aggregate.
6. Some carcinogen-metabolizing enzymes
4. Lymphoid hamartoma or costleman turner is characterised • Alchohol dehydrogenase (AD H)
by non-cancerous benign growths that may develop in • Cytochrme P450
lymphnode tissue throughout the body. It is of three types.
• Glutathione transferase
• Hyaline vascular type - usually asymptomatic
r ORAL PATHOLOGY & MEDICINE SYNOPSIS
• N - acetyltransferases NATl and NATZ. 17. In primary sjogrens syndrome or sicca complex patients have
• Immune defects may predespose to OSCC, espcially lip only dry eyes and dry mouth.
cancer.
Secondary sjogrens syndrome is charcterised by dry eyes,
7. The subtypes of basal carcinoma, which are more dry mouth and other connective tissue disorders like SLE,
aggressive are Rheumatoid arthritis.
• Micromodular BCC (with well defined border)
18. Abfraction is the pathological loss of both enamel and dentin
• Morpheaform and infitrating basal cell cercinoma caused by biomechanical loading forces.
(borders are not well defined and often extends well
beyond clinical margins) 19. A test that differentiates hypoparathyroidism from
pseudohypoparathyroidism is "Ellsworth-Howard test".
8. In India, maximum number of carcinoma of uterine cervix
cases are found in Chennai. 20. The syndromes that result from genetic mutations responsible
for fibroblast growth factor receptors 2 and 3 are called
9. The isolation of lactobacilli can be done by use of selective as craniosynostosis syndromes.
ROGOSA agar medium.
21. In apert syndrome, there is syndactyly involving the hands
10. The "Clearing agent" having a refractive index of enamel and feet with partial to complete fusion of digits, often
to demonstrate "the translucent zone" in enamel caries is involving 2nd, 3rd and 4th digits. These are often termed Mitten
Quinoline. hands and Sock feet.

11. "Parulis or pus pocket" is the terminal end of fistula that 22. Taurodontism is associated with Tricho - Dento - Osseous
extends from the apical or lateral region. syndrome.

12. The small elevalion point in mucosa through which repeated


pus discharge occurs in chronic abscess is called "Gum boil"

13. The most common mucosal type of oral nevi is intramucosal


nevus. Blue nevus is the second most common type found
in oral cavity.

14. The most common variety of basal cell carcinoma is


"Nodular basal cell carcinoma".

15. The ABCDE rule that helps in diagnosis of melanoma include


• Asymmetry-in which one half does not match the other half.
• Border irregularity
• Colour irregularity
• Diameter - greater than 6mm
• Elevated surface

16. Malignant melanoma classification


Level I Tumor is confined to epidermis and is
(Tumor insitu) present above basement membrane.
• Tumor is confined to papillary
Level II epidermis
• The tumor is in radial growth phase
• Tumor cells found throughout
papillary dermis with impingement
Level Ill on the reticular dermis.
• The tumor is in vertical growth phase.
Tumor cells are seen clearly in reticular
Level IV
dermis
Tumor cells show invasion of sub-
Level V
cutaneous fat

Вам также может понравиться